英语习题
英语四八级历年真题及答案

1999年英语专业八级试题及答案

Part Ⅱ Proofreading and Error Correction  (15  min)

  The following passage contains TEN errors. Each line contains a maximum of ONE error. In each case, only ONE word is involved. You should proofread the passage and correct it in the following way. For a wrong word,      underline the wrong word and wri te the correct one in the blank provided at the end of the line. For a missing word,        mark the position of the missing word with a "∧" sign and write the word you believe to be missing in the blank provided at the end of the line. For an unnecessary word      cross out the unnecessary word with a slash "/' and put the word in the blank provided at the end of the line.

  Example

  When∧art museum wants a new exhibit,          (1) an

  it never/  buys things in finished form and hangs        (2) never

  them on the wall. When a natural history museum          

  wants an exhibition, it must often build it.            (3) exhibit

  

  The hunter-gatherer tribes that today live as our prehistoric                                              1.___

  human ancestors consume primarily a vegetable diet supplementing                                      2.___

  with animal foods. An analysis of 58 societies of modem hunter-

  gatherers, including the Kung of southern Africa, revealed that one 

  half emphasize gathering plant foods, one-third concentrate on fishing

  and only one-sixth are primarily hunters. Overall, two-thirds

  and more of the hunter-gatherer's calories come from plants. Detailed                                  3.___

  studies of the Kung by the food scientists at the University of

  London, showed that gathering is a more productive source of food

  than is hunting. An hour of hunting yields in average about 100                                            4.___

  edible calories, as an hour of gathering produces 240.                                                                5.___

  Plant foods provide for 60 percent to 80 percent of the Kung                                              6.___

  diet, and no one goes hungry when the hunt fails. Interestingly, if

  they escape fatal infections or accidents, these contemporary

  aborigines live to old ages despite of the absence of medical care.                                          7.___

  They experience no obesity, no middle-aged spread, little dental

  decay, no high blood pressure, on heart disease, and their blood

  cholesterol levels are very low( about half of the average American                                      8.___

  adult), if no one is suggesting what we return to an aboriginal life                                          9.___

  style, we certainly could use their eating habits as a model for                                                10.___

  healthier diet.

  答案与详解

  1.答案:as→like

  【详细解答】as our prehistoric human ancestors意为"作为人类史前的祖先那样",但是 根据上下文,此处应表达的意思是"像人类史前的祖先那样",故应该将as改为介词like。

  2.答案:supplementing→supplemented 

  【详细解答】本句中的分词短语supplementing with animal foods 是定语,修饰 vegetable diet,根据上下文,此处是指"素食被肉食补充",故应该用过去分词表被动。

  3.答案:and→or 

  【详细解答】根据上下文,这里的意思只能是三分之二或多于三分之二,所以不能用表示并列 关系的and,而应该用表示选择关系的or。

  4.答案:in→on 

  【详细解答】on average为固定搭配,意为"平均"。

  5.答案:as→whereas/while 

  【详细解答】根据上下文意思,这两句话表示的是一种对比关系。故应该改用连词whereas或w hile,引出相反或不同的事实。

  6.答案:for 

  【详细解答】provide是及物动词,可以直接连宾语,不需要任何介词。

  7.答案:of

  【详细解答】despite是一介词,其后可直接接名词。

  8.答案:half∧→that 

  【详细解答】在这里加上that,用来指代前面的blood cholesterol levels,句子结构才完整。

  9.答案:if→though 

  【详细解答】根据上下文,这里的主从句关系不是条件,而是让步关系,即"尽管没有人建议 我们回到土著人的生活方式,…"。

  10.答案:for ∧→a 

  【详细解答】本句中的a healthier diet表示一种更加健康的饮食。冠词a泛指"一种或一类"。

  Part Ⅲ Reading Comprehension  (40  min)

  SECTION A READING COMPREHENSION      (30 min )

  In this section there are four reading passages followed by a total of fifteen m ultiple-choice questions. Read the passages carefully and then mark your answers on your Coloured Answer Sheet.

  TEXT A

  Ricci's "Operation Columbus"

  Ricci, 45, is now striking out on perhaps his boldest venture yet. He plan s to market an English language edition of his elegant monthly art magazine, FMR , in the United States. Once again the skeptice are murmuring that the successfu l Ricci has headed for a big fall. And once again Ricci intends to prove them wr ong.

  Ricci is so confident that he has christened his quest "Operation Columbu s " and has set his sights on discovering an American readership of 300,000. That  goal may not be too far-fetched. The Italian edition of FMR - the initials, of course, stand for Franco Maria Ricci-is only 18 months old. But it is already  the second largest art magazine in the world, with a circulation of 65,000 and a profit margin of US $ 500,000. The American edition will be patterned after th e Italian version, with each 160-page issue carrying only 40 pages of ads and no  more than five articles. But the contents will often differ. The English-langua ge edition will include more American works, Ricci says, to help Americans get o ver "an inferiority complex about their art." He also hopes that the magazine will become a vehicle for a two -way cultural exchange - what he likes to think of as a marriage of brains, culture and taste from both sides of the Atlantic.

   To realize this vision, Ricci is mounting one of the most lavish, enterpris ing - and expensive-promotional campaigns in magazine - publishing history. Between November and January, eight jumbo jets will fly 8 million copies of a sample 16-page edition of FMR across the Atlantic. From a warehouse in Michigan,  6.5 million copies will be mailed to American subscribers of various cultural, art and business magazines. Some of the remaining copies will circulate as a spe cial Sunday supplement in the New York Times. The cost of launching Operation Co lumbus is a staggering US $ 5 million, but Ricci is hoping that 60% of the price  tag will be financed by Italian corporations." To land in America Columbus had  to use Spanish sponsors," reads one sentence in his promotional pamphlet. "We would like Italians."

  Like Columbus, Ricci cannot know what his reception will be on foreign shor es. In Italy he gambled - and won - on a simple concept: it is more important to show art than to write about it. Hence, one issue of FMR might feature 32 fu ll-colour pages of 17th-century tapestries, followed by 14 pages of outrageous e yeglasses. He is gambling that the concept is exportable. "I don't expect that more than 30% of my reader…… will actually read FMR," he says. "The magazine is such a visual delight that they don't have to." Still, he is lining up an impr es sive stable of writers and professors for the American edition , including Noam Chomsky, Anthony Burgess, Eric Jong and Norman Mailer. In addition, he seems to be pursuing his won eclectic vision without giving a moment's thought to such e s tablished competitors as Connosisseur and Horizon. "The Americans can do almost everything better than we can," says Rieci, "But we(the Italians)have a 2,000 year edge on them in art."

  16. Ricci intends his American edition of FMR to carry more American art works in order to___.

  A. boost Americans' confidence in their art

  B. follow the pattern set by his Italian edition

  C. help Italians understand American art better

  D. expand the readership of his magazine

  17. Ricci is compared to Columbus in the passage mainly because___.

  A. they both benefited from Italian sponsors

  B. they were explorers in their own ways

  C. they obtained overseas sponsorship

  D. they got a warm reception in America

  18. We get the impression that the American edition of FMR will probably ___.

  A. carry many academic articles of high standard

  B. follow the style of some famous existing magazines

  C. be mad by one third of American magazine readers

  D. pursue a distinctive editorial style of its own

  TEXT B

   My mother's relations were very different from the Mitfords. Her brother, Uncle Geoff, who often came to stay at Swimbrook, was a small spare man with th oughtful blue eyes and a rather silent manner. Compared to Uncle Tommy, he was a n intellectual of the highest order, and indeed his satirical pen belied his mil d demeanor. He spent most of his waking hours composing letters to The Times and  other publications in which he outlined his own particular theory of the develo pment of English history. In Uncle Geoff's view, the greatness of England had r isen and waned over the centuries in direct proportion to the use of natural man ure in fertilizing the soil. The Black Death of 1348 was caused by gradual loss of the humus fertility found under forest trees. The rise of the Elizabethans tw o centuries later was attributable to the widespread use of sheep manure. 

  Many of Uncle Geoff's letters-to-the-editor have fortunately been preserv ed in a privately printed volume called Writings of a Rebel. Of the collection, one letter best sums up his views on the relationship between manure and freedom

  . He wrote:

  Collating old records shows that our greatness rises and falls with the li ving fertility of our soil. And now, many years of exhausted and chemically murd ered soil, and of devitalized food from it, has softened our bodies and still wo rse, softened our national character. It is an actual fact that character is lar gely a product of the soil. Many years of murdered food from deadened soil has m ade us too tame. Chemicals have had their poisonous day. It is now the worm's t urn to reform the manhood of England. The only way to regain our punch, our char acter, our lost virtues, and with them the freedom natural to islanders, is to c o mpost our land so as to allow moulds, bacteria and earthworms to remake living s oil to nourish Englishmen's bodies and spirits.

  The law requiring pasteurization of milk in England was a particular targe t of Uncle Geoff's. Fond of alliteration, he dubbed it "Murdered Milk Measure ", and established the Liberty Restoration League, with headquarters at his house i n London, for the specific purpose of organizing a counteroffensive. "Freedom n o t Doctordom" was the League's proud slogan. A subsidiary, but nevertheless imp or tant, activity of the League was advocacy of a return to the "unsplit, slowly s m oked fish" and bread made with "English stone-ground flour, yeast, milk, sea s alt and raw cane-sugar."

  19. According to Uncle Geoff, national strength could only be regained by ___.

  A. reforming the manhood of England

  B. using natural manure as fertilizer

  C. eating more bacteria-free food

  D. granting more freedom to Englishmen

  20. The tone of the passage can most probably be described as___.

  A. facetious    B. serious      C. nostal gic      D. factual

  TEXT C

   Interview

  So what have they taught you at college about interviews? Some courses go t o town on it, others do very little. You may get conflicting advice. Only one th ing is certain: the key to success is preparation.

  There follow some useful suggestions from a teacher training course co-ordi nator, a head of department and a headteacher. As they appear to be in complete harmony with one another despite never having met, we may take their advice seri ously.

  Oxford Brookes University's approach to the business of application and in t erview focuses on research and rehearsal. Training course co-ordinator Brenda St evens speaks of the value of getting students "to deconstruct the advertisement , see what they can offer to that school, and that situation, and then write the letter, do their CVs and criticize each other's." Finally, they role play inte rviewer and interviewee. 

  This is sterling stuff, and Brookes students spend a couple of weeks on it.  "The better prepared students won't be thrown by nerves on the day, "says Ms St evens. "They'll have their strategies and questions worked out. " She also sa ys, a trifle disconcertingly, "the better the student, the worse the interviewee. " She believes the most capable students are less able to put themselves forward. Even if this were tree, says Ms Stevens, you must still make your own case.

  "Beware of infernality," she advises. One aspirant teacher, now a head of d epartment at a smart secondary school, failed his first job interview because he  took his jacket off while waiting for his appointment. It was hot and everyone in the staffroom was in shirtsleeves but at the end of the day they criticized h is casual attitude, which they had deduced from the fact that he took his jacket off in the staffroom, even though he put it back on for the interview.

  Incidentally, men really do have to wear a suit to the interview and women really cannot wear jeans, even if men never wear the suit again and women teach most days in jeans. Panels respond instantly to these indicators. But beware: it  will not please them any better if you are too smart.

  Find out about the people who will talk to you. In the early meetings they are likely to be heads of departments or heads of year. Often they may be concer ned with pastoral matters. It makes sense to know their priorities and let them hear the things about you that they want to hear.

  During preliminary meetings you may be seen in groups with two or three oth er applicants and you must demonstrate that you know your stuff without putting your companions down. The interviewers will be watching how you work with a team

  . But remember the warning about informality: however friendly and co-operat ive the other participants are, do not give way to the idea that you are there j ust to be friends.

  Routine questions can be rehearsed, but "don't go on too long," advises th e department head. They may well ask: "What have been your worst/best moments w h en teaching?", or want you to "talk about some good teaching you have done. " The experts agree you should recognize your weaknesses and offer a strategy for over coming them. "I know I've got to work on classroom management - I would hope fo r some help," perhaps. No one expects a new teacher to know it all, but they ho pe for an objective appraisal of capabilities.

  Be warned against inexpert questioning. You may be asked questions in such  a way that it seems impossible to present your best features. Some questions may be plain silly, asked perhaps by people on the panel who are from outside the s ituation. Do not be thrown, have ways of circumnavigating it, and never, ever le t them see that you think they have said something foolish.

  You will almost certainly be asked how you see the future and it is import ant to have a good answer prepared. Some people are put off by being asked what they expect to be doing in five or ten years' time. On your preliminary visit, s ays the department head, be sure to give them a bit of an interview of your own,  to see the direction the department is going and what you could contribute to i t.

  The headteacher offers his thoughts in a nine-point plan. Iron the application form! Then it stands out from everyone else's, which have  been folded and battered in the post. It gives an initial impression which may get your application to the top of the pile.  Ensure that your application is tailored to the particular school. Make the hea d feel you are writing directly to him or her. Put yourself at ease before you meet the interviewing panel: if you are nervous , you will talk too quickly. Before you enter the room remember that the people are human beings too; take away the mystique of their roles.  Listen. There is a danger of not hearing accurately what is being said. Make ey e contact with the speakers, and with everyone in the room. Allow your warmth and humanity to be seen. A sense of humour is very important.

   Have a portfolio of your work that can link theory to practice. Many schools wa nt you to show work. For a primary appointment, give examples from the range of the curriculum, not just art. (For this reason, taking pictures on your teaching  practice is important. )  Prepare yourself in case you are asked to give a talk. Have prompt cards ready, and don't waffle.

  Your speech must be clear and articulate, with correct grammar. This is importa nt: they want to hear you and they want to hear how well you can communicate wit h children. Believe in yourself and have confidence.

  Some of the people asking the questions don't know much about what you do. B e ready to help them.

  Thus armed, you should have no difficulty at all. Good luck and keep your jac ket on!

  21. Ms. Brenda Stevens suggests that before applying job applicants shoul d ___.

  A. go through each other's CVs

  B. rehearse their answers to questions

  C. understand thoroughly the situations

  D. go to town to attend training course

  22. Is it wise to admit some of your weaknesses relating to work?

  A. Yes, but you should have ideas for improvement in the future.

  B. Yes, because it is natural to be weak in certain aspects.

  C. No, admitting weaknesses may put you at a disadvantage.

  D. No, it will only prompt the interviewees to reject you.

  23. The best way to deal with odd questions from the interviewers is to ___.

  A. remain smiling and kindly point out the inaccuracies

  B. keep calm and try to be tactful in your answers

  C. say frankly what you think about the issues raised

  D. suggest something else to get over your nervousness

  24. The suggestions offered by the headteacher are ___.

  A. original      B. ambiguousC. practical        D. co ntroversial

  TEXT D

   Family Matters

  This month Singapore passed a bill that would give legal teeth to the moral obligation to support one's parents. Called the Maintenance of Parents Bill, i t received the backing of the Singapore Government.

  That does not mean it hasn't generated discussion. Several members of the P arliament opposed the measure as un-Asian. Others who acknowledged the problem o f the elderly poor believed it a disproportionate response. Still others believe  it will subvert relations within the family: cynics dubbed it the "Sue Your So n" law.

  Those who say that the bill does not promote filial responsibility, of cour se, are right. It has nothing to do with filial responsibility. It kicks in wher e filial responsibility fails. The law cannot legislate filial responsibility an y more than it can legislate love. All the law can do is to provide a safety net  where this morality proves insufficient. Singapore needs this bill not to repla ce morality, but to provide incentives to shore it up.

  Like many other developed nations, Singapore faces the problems of an incre asing proportion of people over 60 years of age. Demography is inexorable. In 19 80, 7.2% of the population was in this bracket. By the end of the century that fi gure will grow to 11%. By 2030, the proportion is projected to be 26%. The probl em is not old age per se. It is that the ratio of economically active people to economically inactive people will decline.

  But no amount of government exhortation or paternalism will completely elim inate the problem of old people who have insufficient means to make ends meet. S ome people will fall through the holes in any safety net.

  Traditionally, a person's insurance against poverty in his old age was his  family, lifts is not a revolutionary concept. Nor is it uniquely Asian. Care an d support for one's parents is a universal value shared by all civilized societ ies.

  The problem in Singapore is that the moral obligation to look after one's parents is unenforceable. A father can be compelled by law to maintain his child ren. A husband can be forced to support his wife. But, until now, a son or daugh ter had no legal obligation to support his or her parents.

  In 1989, an Advisory Council was set up to look into the problems of the ag ed. Its report stated with a tinge of complacency that 95% of those who did not have their own income were receiving cash contributions from relations. But what about the 5% who aren't getting relatives' support? They have several options : (a) get a job and work until they die; (b) apply for public assistance(you hav e to be destitute to apply); or(c) starve quietly. None of these options is soci ally acceptable. And what if this 5% figure grows, as it is likely to do, as soc iety ages?

  The Maintenance of Parents Bill was put forth to encourage the traditional virtues that have so far kept Asian nations from some of the breakdowns encounte red in other affluent societies. This legislation will allow a person to apply t o the court for maintenance from any or all of his children. The court would hav e the discretion to refuse to make an order if it is unjust.

  Those who deride the proposal for opening up the courts to family lawsuits miss the point. Only in extreme cases would any parent take his child to court. If it does indeed become law, the bill's effect would be far more subtle.

  First, it will reaffirm the notion that it is each individual's-not soci ety's-responsibility to look after his parents. Singapore is still conservativ e enough that most people will not object to this idea. It reinforces the tradit ional values and it doesn't hurt a society now and then to remind itself of its core values.

  Second, and more important, it will make those who are inclined to shirk th eir responsibilities think twice. Until now, if a person asked family elders, cl ergymen or the Ministry of Community Development to help get financial support f rom his children, the most they could do was to mediate. But mediators have no t eeth, and a child could simply ignore their pleas.

  But to be sued by one's parents would be a massive loss of face. It would  be a public disgrace. Few people would be so thick-skinned as to say, "Sue and  be damned". The hand of the conciliator would be immeasurably strengthened. It  is far more likely that some sort of amicable settlement would be reached if th e recalcitrant son or daughter knows that the alternative is a public trial.

  It would be nice to think Singapore doesn't need this kind of law. But th at belief ignores the clear demographic trends and the effect of affluence itsel f on traditional bends. Those of us who pushed for the bill will consider ourselv es most successful if it acts as an incentive not to have it invoked in the firs t place.

  25. The Maintenance of Parents Bill ___.

  A. received unanimous support in the Singapore Parliament

  B. was believed to solve all the problems of the elderly poor

  C. was intended to substitute for traditional values in Singapore

  D. was passed to make the young more responsible to the old

  26. By quoting the growing percentage points of the aged in the populatio n, the author seems to imply that ___.

  A. the country will face mounting problems of the old in future

  B. the social welfare system would be under great pressure

  C. young people should be given more moral education

  D. the old should be provided with means of livelihood

  27. Which of the following statements is CORRECT?

  A. Filial responsibility in Singapore is enforced by law.

  B. Fathers have legal obligations to look after their children.

  C. It is an acceptable practice for the old to continue working.

  D. The Advisory Council was dissatisfied with the problems of the old.

  28. The author seems to suggest that traditional values ___.

  A. play an insignificant role in solving social problems

  B. are helpful to the elderly when they sue their children

  C. are very important in preserving Asian uniqueness

  D. are significant in helping the Bill get approved

  29. The author thinks that if the Bill becomes law, its effect would be ___.

  A. indirect    B. unnoticed    C. apparent    D. straightforward

  30. At the end of the passage, the author seems to imply that success of the Bill depends upon ___.

  A. strict enforcement  B. public support C. government assurance        D. filial awareness

  答案与详解

  SECTION A 

  TEXT A 

  短文大意:本文介绍的是艺术月刊FMR打算在美国出版的原因以及该杂志的特点。

  16.答案:A

  【参考译文】Ricci为何计划在美国版的FMR上登载更多美国艺术品?

  【试题分析】本题为细节题。

  【详细解答】短文第二段倒数第二句说"The English-language edition will include more American works, Ricci says, to help Americans get over 'an inferiority complex about their art'."由此可知,登载更多美国艺术品是为了帮助美国人树立自信心,因 为他们对其没有太长历史的文化艺术而感到自卑。故选项A为正确答案。

  17.答案:B

  【参考译文】本文将Ricci和哥伦布相比的主要原因是什么?

  【试题分析】本题为推理题。

  【详细解答】众所周知,哥伦布是一位探险家,他发现了美洲大陆,而Ricci是想要开拓美国 的艺术杂志市场,是一位商业探索者。因此他们同是探索者,故答案选B。

  18.答案:D

  【参考译文】我们对美国杂志FMR的印象如何?

  【试题分析】本题为推理题。

  【详细解答】短文倒数第二段说"In addition, he seems to be pursuing his won eclecti c vision without giving a moment's thought to such established competitors as Co nnosisseur and Horizon."即该杂志的编辑方式不同于其他艺术杂志,故选项D"追求自己 独特的艺术风格"为正确答案。

  TEXT B 

  短文大意:本文主要介绍了Geoff舅舅对于英国发展史的看法。

  19.答案:B

  【参考译文】根据Geoff舅舅的观点,只有怎样做才能恢复国力?

  【试题分析】本题为细节题。

  【详细解答】 Geoff舅舅在写给报社的信中提到"The only way to regain our punch, our character, our lost virtues, and with them the freedom natural to islanders, is to compost our land so as to allow moulds, bacteria and earthworms to remake liv ing soil to nourish Englishmen's bodies and spirits."由此可知,Geoff舅舅认为英 国的兴衰与使用天然肥料是成正比的,故选项B"使用天然肥料"为正确答案。

  20.答案:A

  【参考译文】本文作者很可能是什么语气?

  【试题分析】本题为推理题。

  【详细解答】通读全文可知,作者在描述Geoff舅舅时用的语言幽默风趣,充满了诙谐、嘲弄 的语气。故选项A"滑稽可笑的"为正确答案。

  TEXT C  

  短文大意:本文主要介绍的是面试者如何做好面试的准备工作。

  21.答案:C

  【参考译文】Brenda Stevens女士建议求职者在求职前应该怎样?

  【试题分析】本题为细节题。

  【详细解答】短文第三段第二句说"Brenda Stevens speaks of the value of getting stud ents'to deconstruct the advertisement, see what they can offer to that school, and that situation, and then write the letter, do their CVs and criticize each o ther's.'"由此可知,面试训练班的目的是让学生对所处场面的了解,故Brenda Stevens 女士建议求职者在求职前应该是C"彻底了解所处的场面"。

  22.答案:A

  【参考译文】承认自己工作中的一些弱点是否明智?

  【试题分析】本题为推理题。

  【详细解答】短文第十段第三句说"The experts agree you should recognize your weaknes ses and offer a strategy for overcoming them."这里以专家的观点说明应聘者应该正 视自己的弱点,并且要有克服它们的策略。故答案选A。

  23.答案:B

  【参考译文】对于面试者提出的古怪问题,应聘者最好的应对方法是什么?

  【试题分析】本题为推理题。

  【详细解答】短文第十一段谈到如果面试者提出愚蠢问题时,应聘者应该"Do not be thrown , have ways of circumnavigating it, and never, ever let them see that you think they have said something foolish." 即应试者应从侧面巧妙地避开问题,并且千万不要 让面试者看出你认为他们问了愚蠢的问题。故选项B为正确答案。

  24.答案:C

  【参考译文】校长所提的建议如何?

  【试题分析】本题为归纳总结题。

  【详细解答】短文中共列出了校长提出的九点建议,均是关于具体处理问题的办法,故选项C "实用的"为最佳答案。

  TEXT D  

  短文大意:本文主要对新加坡的"赡养父母议案"的讨论。

  25.答案:D

  【参考译文】"赡养父母议案"如何?

  【试题分析】本题为细节理解题。

  【详细解答】短文首句说"This month Singapore passed a bill that would give legal t eeth to the moral obligation to support one's parents."由此可知,选项D"该议案 的通过是为了让年轻人对老年人更尽义务"为正确答案。

  26.答案:A

  【参考译文】作者引用老年人比例不断上升,似乎是在暗示什么?

  【试题分析】本题为推理题。

  【详细解答】短文通过具体数字告诉我们,在新加坡确实存在老龄人口比例增大的问题,这所 带来的影响是:"It is that the ratio of economically active people to economical ly inactive people will decline."即国家将面临严重的经济问题。故选项A为正确答案 。

  27.答案:B

  【参考译文】下列哪一种说法是正确的?

  【试题分析】本题为细节题。

  【详细解答】短文第七段第二句说"A father can be compelled by law to maintain his c hildren."这与B的说法一致,故选项B为正确答案。

  28.答案:C

  【参考译文】作者似乎在暗示传统价值观怎样?

  【试题分析】本题为细节题。

  【详细解答】短文第九段首句说"The Maintenance of Parents Bill was put forth to enc ourage the traditional virtues that have so far kept Asian nations from some of the breakdowns encountered in other affluent societies."这里是说亚洲国家的传统 美德使得他们有别于那些富有的国家,这表明传统美德在保持亚洲国家的特色方面起着重要 作用,故答案选C。

  29.答案:A

  【参考译文】作者认为如果这项议案成为法律的话,它将会产生怎样的效果?

  【试题分析】本题为推理题。

  【详细解答】短文第十段最后一句说"If it does indeed become law, the bill's effect would be far more subtle." 在接下来的几段里,作者讲到赡养父母是个人的责任,并非 社会的责任。而有些人碍于情面,不得不重新考虑赡养父母的问题。因此该议案起到了间接 的作用。故选项A为正确答案。

  30.答案:D

  【参考译文】在文章的结尾部分,作者似乎在暗示该议案的成功有赖于什么?

  【试题分析】本题为总结题。

  【详细解答】文章的最后一句说"Those of us who pushed for the bill will consider ou rselves most successful if it acts as an incentive not to have it invoked in the first place." 这句话表明该议案是否成功要看它能否激励人们维护传统道德观念,而不 是首先考虑对簿公堂,故选项D"孝顺意识"为正确答案。

  SECTION B SKIMMING AND SCANNING    (10  min)

  In this section there are seven passages with ten multiple-choice questions. Ski m or scan them as required and then mark your answers on your Coloured Answer Sheet.

  TEXT E

  First read the question.

  31. The primary purpose of the letter is to ___.

  A. illustrate the World Bank's efforts in poverty-relief programmes

  B. call for further efforts by nations in sustainable development

  C. provide evidence for the World Bank's aid to the private sectors

  D. clear up some misunderstanding about the World Bank

  Now go through TEXT E quickly to answer question 31.

  August 18th 199 

  Dear Sir,

  In your July 28th article you noted that the Bank's own internal analysis r ated one third of the projects completed in 1991 as unsatisfactory. But that sta tement fails to take account of the Bank's criteria for 'success', which are exc eptionally strict. For instance, before a project can be considered successful, it must have at least a 10% rate of return. This rate is far higher than the min imum demanded by many bilateral aid donors, many of which require a return of on ly 5% or 6%. Thus, projects rated unsatisfactory under the Bank's standards sti ll yield many benefits.

  You imply that, because it deals mainly with governments, the Bank does not sufficiently support private sector development. Here are the facts. The World

  Bank has:

  supported reforms in mere than 80 countries aimed at opening up trade, making p rices realistic and dismantling state monopolies which stifle individual enterpr ise nvested in infrastructure to facilitate business activity; assisted and advised over 200 privatization-related operations involving nearly US $ 25 billion in loans;  provided mere than US $ 12 billion through an affiliate, the International Fina nce Corp. over the last 30 years to mere than 1,000 private companies in the dev eloping world; and through another affiliate, the Multi lateral Investment Guara ntee Agency, offered insurance against non-commercial risk to encourage foreign investment in poor countries.

  The record shows that, over the past generation, more progress has been mad e in reducing poverty and raising living standards than during any other compara ble period in history. In the developing countries:  life expectancy has been increased from 40 to 63 years; infant mortality has been reduced by 50% ;and per capita income has doubled.

  The World Bank consistently stresses that most of the credit for these adva nces should go to the countries themselves. Nevertheless, the Bank and organizat ions with which it collaborates-bilateral and international agencies and non-gov ernmental organizations-have played a valuable role in this progress. In the fut ure the Bank will continue to do its utmost to support its member countries in t heir efforts to achieve sustainable development.

  (LEANDRO V. CORONEL  

  Public Affairs

  The Worm Bank

  Washington)

  

  TEXT F

  First read the question.

  32. The author's main argument is that ___.

  A. most farmers in developing countries face unemployment

  B. developing countries need agricultural aid to boost economy

  C. agricultural aid hints the economy in developing countries

  D. a well-developed agricultural sector provides a domestic market

  Now go through TEXT F quickly to answer question 32.

  Ours is an agrarian economy. We must become serf-sufficient in food to feed a rapidly growing population at an annual growth rate of more than 3 million pe ople. A well-developed agricultural sector would offset the need for food import and play an important role in the development process by providing a home marke t for the products of the industrial sector. This implies that the rate of indus trialization itself depends upon how fast agricultural incomes are rising. Devel opment in the agricultural sector in our country means a rise in the income leve l of 70 percent of the population who are related to this sector. Their increase d income in turn will give us mere voluntary savings and investment and thus a s ource of revenue through taxation and potential capital formation by the governm ent plus reduction in income inequalities between the urban population and rural masses. In this sense, aid received in the form of agricultural commodities hur ts the developing countries and benefits developed countries mere than proportio nately. Because most of the farmers in developing countries are already at a mer e subsistence level with a high rate of unemployment, disguised-unemployment and  underemployment.

  The Chinese experience with rural development has demonstrated that agricu ltural modernization via labour-intensive techniques is a highly promising way t o create extra jobs without extensive geographic displacement of the farmers. Re garding the impact of transfer of agricultural commodities on the long-term grow th rate in the recipient country, it can be said that transfer of agricultural c ommodities under confessional terms may resuit in an ultimate lowering of the re cipient countries long-term growth rate.

  TEXT G

  First read the question.

  33. The passage is most probably from ___.

  A. a review of a book on cowboys

  B. a study of cowboy work culture

  C. a novel about cowboy life and culture

  D. a school textbook on the cowboy history

  Now go through TEXT G quickly to answer question 33.

  A cowboy is defined by the work that he does. Any man can lay claim to that name if he lives on a ranch and works—— drives, brands, castrates, or murmurs ——a cattleman's herd. In addition, working accounts for ways in which cowboy s  portray themselves in their art: in 19th-century poems that they orally compose d and sang on the ranch, in 20th-century poems that they write, in books that th ey publish, and in art objects that they fashion, cowboys always represent thems elves as engaging in some form of labour. This book's three fold purpose is, fi r st, to look at art that cowboys produce——art, that has never been studied befo re——and, second, to demonstrate that cowboy art values historically document l abour routines that cowboys have traditionally acted out in their work culture.

  I use the term work culture not only to suggest that cowboys are defined b y the work that they do, but also to argue that they are serf-represented in cul ture by poems, prose, and art that ail reveal cowboys to be men who are cultural ly unified by engaging in labour routines that they think of as cowboy work. Art deals with cowboy work, as well as with concerns about economics, gender, relig ion, and literature, even though these thoughts sometimes express themselves as concerns about cattle branding, livestock castration, and other tasks. The book ' s third and most important function is, therefore, to show that artistic self-re presentations of labour also formulate systems of thought which cowboys use as a metaphor for discussing economies, gender, religion, and literature, sometimes equating branding with religious salvation, at other t imes defining spur making as freedom, and so on.

  TEXT H

  First read the question.

  34. The writer of this letter attempts to ___ the views in the editorial.

  A. refute    B. illustrate    C. support    D. substantiate

  Now go through TEXT H quickly to answer question 34.

  October 3rd 199  

  Dear Sir,

  In your editorial on August 31st, there seems to be some confused thinking in attempting to establish a direct relationship between the desire of the OAA airlines to negotiate more equitable agreements with the United States for air-t raffic rights and the cost of air travel for the public.

  It is simply untrue that the Asian carriers are not looking for increased access to the U.S. market, including its domestic market; they are, as part of b alanced agreements that provide equality of opportunity. So long as the U. S. ta kes the inequitable arrangements enshrined in current agreements as a starting p oint for negotiation, however, there is no chance that U.S. carriers will be gra nted more regional rights which further unbalance the economic opportunities ava ilable to each side. Most importantly from the consumer viewpoint, it has yet to be demonstrated that in those regional sectors where U.S. carriers currently op erate-such as Hong Kong/Tokyo-they have added anything in terms of price, qualit y of service, innovation or seat availability in peak seasons.

  Turning to cost, I am not sure to which Merrill Larrych study you are referri ng, but it would be simplistic to compare seat-mile costs of narrow-body operati on over U. S. domestic sectors with wide-body operation over international secto rs; comparative studies of seat-mile costs are valid only if they compare simila r aircraft operating over identical sectors. On this basis, International Civil Aviation Organization figures show that Asian carriers are highly competitive. O f course, given its operating environment Japan Air Lines will have high seat-mi le costs, while a carrier based in Southeast Asia, such as Singapore Airlines, w ill have relatively low costs. But it is a fallacy to assume this means 'higher  ticket prices or higher taxes' for the 'hapless Asian air traveller' if he travels on JAL.

  The Japanese carriers have to compete in the Asian marketplace with others, and costs cannot simply be passed on to the consumer or taxpayer. The people wh o really pay the price or reap the reward of differing cost levels are the share holders.

  (RICHARD. T. STIRLAND

  Director General

  Orient Airlines Association

  TEXT I

  First read the questions.

  35. Today's computers can process data ___ times faster than the 1952 model, ILLIAC.

  A. 4      B. 100      C. 200      D. 4, 000

  36. NCSA aims to develop ___.

  A. a new Internet browser

  B. a more powerful national system

  C. human-computer intelligence interaction

  D. a new global network

  Now go through TEXT I quickly to answer questions 35 and 36.

  URBANA, Illinois. Welcome to Cyber City, USA, where scientists are developi ng the next-generation Internet and leading ground-breaking research in artifici al intelligence. The University of Illinois at Urbana, which has a student body of 36,100, has a proud computing tradition. In 1952, it became the first educational institution to build and own its own computer.

  That computer, ILLIAC, was four metres tall, four metres long and sixty cen timetres deep. Its processing speed was about 50 kilohertz compared with 200 meg ahertz-that's 200,000 kilohertz for today's computers.

  At the state-of-the-art Beckman Institute for Advanced Science and Technolo gy, researchers from disciplines as far-ranging as psychology, computer science and biochemistry are focusing on biological intelligence and human-computer inte lligence interaction.

  Beckman also houses the National Centre for Supercomputing Application (NCS A), which played a key role in the development of the Internet global network. I t was NCSA that developed Mosaic, the graphically driven programme that first ma de surfing on the Internet possible.

  Mosaic, introduced in 1992, has been replaced by much more powerful Interne t browsers such as its successor Netscape or Microsoft's Internet Explorer.

  NCSA officials say they are now trying to bring more advanced computing and communication to research scientists, engineers and ultimately the public.

  "What we're looking for is a national system in which the networks are 10 0 times greater than the Internet today, and the supercomputers are 100 times more  powerful," said NCSA Director Larry Smart.

  A proposed joint project would develop a prototype or demonstration model f or the "21st century national information infrastructure" in line with an init iative announced by President Bill Clinton last October.

  If funded by the National Science Foundation, the new structure would take effect on October 1st.

  NCSA, one of the four operational federal supercomputer centres in the coun try, is awaiting a decision from the Foundation's board late this month on a co mpetition for US $ 16 million in continued annual federal funding.

  NCSA, which employs 200 people and has a yearly budget of US $ 31 million, is expected to be one of two winners along with its counterpart in San Diego.

  "The University has put a great deal of effort into this competition. We r emain hopeful about the outcome, but we will have no comment until the National Science Foundation Board's decision," Smart said.

  TEXT J

  First read the questions.

  37. In Japanese the work depato refers to ___.

  A. traditional Japanese stores

  B. modern stores in cities

  C. special clothing stores

  D. railway stores

  38. During the Meiji era depato was regarded by Japanese customers as a(n ) ___ shopping place.

  A. cheap      B. traditional      C. fashionable      D. attractive

  Now go through TEXT J quickly to answer questions 37 and 38.

  The Japanese have two words for the modern department stores that abound in large urban areas. The older word, hyakkaten, which is seldom used in daily spee ch, can usually be found engraved in ideographs in a building cornerstone, and i t is part of a store's official rifle. Literally "a store with one hundred ite ms ," this word was coined during the late Meiji era( 1868 - 1912), when clothing s tores began to expand their product lines and railroads began to build shops at major train crossings. The more recent and more commonly used word is depato (fr om the English 'department store' ). 

  These words reflect the dual nature of Japanese department stores. Words wr itten in ideographs can impart an aura of antiquity and tradition. Frequently, a s in the case of the word hyakkaten, they suggest indigenous origin. In contrast , foreign borrowed words often give a feeling of modernity and foreignness. Many  Japanese department stores actually originated in Japan several hundred years a go as dry goods stores that later patterned themselves after foreign department stores. Even the trendiest and most avant-garde of these stores practise pattern s of merchandising and retain forms of prepaid credit, customer service, and spe cial relationships with suppliers characteristic of merchandising during the Tok ygawa era (1600 - 1868). To many Japanese these large urban stores may seem lik e a direct import from the West, but like the word depato, they have undergone a  transformation in the process of becoming Japanese.

  Throughout the Tokygawa era, Japan was closed by decree to foreign influen ces. During the Meiji era, however, Japan reopened to the western world; concurr ently, depato emerged as large-scale merchandisers in Japan. The Meiji depato we re soon perceived by Japanese customers as glamorous places to shop because of t heir Western imports, which the Japanese were eager to see and buy. Depato also sold Japanese goods but often followed practices that people of the time conside red foreign, such as letting customers wear their shoes while shopping in the st ore.

  A representative of the Japan Department Store Association told me that th roughout their history depato have played on the Japanese interest in foreign pl aces, cultures and objects, and that to a great extent these were introduced to Japan through department stores. I suggest that in addition to this role of cult ural importer depato have also been involved in the creation of domestic cultura l meanings. They have made foreign customs, ideas and merchandise familiar by gi ving them meanings consistent with Japanese cultural practice.

  TEXT K

  First read the questions.

  39. The Agency for International Development is a ___ organization.

  A. new    B. regional  C. UN  D. US

  40. According to NDS's statistics, the number of babies the average Phil ipino woman bears dropped by ___ between 1960 and 1993.

  A.4.1            B.6.4          C.2.3      D.2.9

  Now go through TEXT K quickly to answer questions 39 and 40.

  When representatives from 170 nations gather in Cairo next month for the th ird International Conference on Population and Development, they will vote on th e largest population-control plan in history. It is ambitious. Not only does it call for a host of "reproductive fights" and aim to freeze world population at 7 2 billion people by 2050; it also calls for billions of dollars in new governme nt spending on the issue-US $ 13.2 billion by the end of the century.

  Some of the plan's provisions have already aroused opposition, most notabl y from Pope John Paul II. All this has been gleefully covered by the newspapers. Yet scant attention has been paid to many of the dubious social and economic ass umptions that underlie the plan. In particular, it is interesting to see how the se programmes are being sold in places like the Philippines, on the front lines of the population debate. For the way the proponents of population control have gone about pushing their programmes raises serious doubts about the integrity of  their studies, their ultimate value to development, and the role of foreign-aid  groups.

  Although population-control measures in the Philippines never reached the coercive levels they did in India, they were not popular. This time, proponents have learned their lesson. For the past few years, they have been quietly laying the groundwork for Cairo. Rather than attack the issue head-on, it has been red efined in terms of a host of new"reproductive rights"to which the solution is invariably a government-funded initiative.

  We have just had a good taste of this in the Philippines. The National Sta tistics Office recently published the results of the 1993 National Demographic S urvey(NDS),which happens to have been funded by the U.S. Agency for Internationa l Development. It is probably mere coincidence, but the NDS report, published on  the eve of the Cairo meeting, nicely supports the thrust of the Cairo Declarati on. That is, it has found a connection between mothers' and children's health an d fertility behaviour. The implication is that large-scale government family-pla nning programmes are essential if health issues are to be addressed. 

  But the demographic survey seems to have been selective about what facts i t would report and connections it would make. Take the health issue. The documen t concludes that the high risk of infant, child and maternal mortality is associ ated with pregnancies where mothers are too young, too old, or have already had several children. But a discussion of poverty is missing from the list of factor s related to health. It would be difficult to deny that poverty, lack of access to safe water, poor housing, poor hygiene and unsanitary conditions all have a s trong bearing on the health of the mother and child. Although the NDS collected data on housing characteristics, it did not include any data on income.

  A closer look at the fertility behaviour of the poor is important because of the extensive literature on the "replacement effect" of high infant mortali ty . Statistical studies in various countries show high fertility among the poor as a rational desire to have children who will survive into adulthood to help take care of them. This helps to explain why many poor women have babies at such sho rt intervals. The 1993 NDS would have been a good opportunity to verify the vali dity of this behaviour in the Philippines. 

  The NDS avoided collecting data on socio-economic variables that would have a serious effect on these health issues. But, in one area, it made painstaking efforts to quantify fertility preference to derive figures for planned and unpla nned pregnancies. It concluded that "if all unwanted births were avoided, the t o tal fertility rate would be 2.9 children, which is almost 30% less than the obse rved rate. "This, too, was used to establish an "unmet" need requiring a gove rnment programme.

  Yet the NDS's own numbers suggest that Filipinos are aware of their option s . The total fertility rote——the number of babies the average woman bears over her lifetime——has dropped to 4.1 in 1993 from 6.4 in 1960. Some 61% used contr aceptives, just a few percentage points short of the 65-80% rate prevailing in E urope, North America and most of East Asia. The delay of marriage by Filipinos t o the age of 23 years represents a reduction of the risk of pregnancy by 19% giv en the 35 years of their reproductive life.

  In short, the Philippines has its problems but its people are not as ignor ant as the population-control lobby would suppose. Unfortunately, this lobby has  development dollars, organizational muscle and support of the media. "We've b ui lt a consensus about population as a global issue and family planning as a healt h issue," says the UN's Naris Sadik, host of the conference. Yes, they have. A nd now we know how.

  

  答案与详解

  TEXT E  

  短文大意:这是一封回信,主要是对世界银行工作中存在的问题进行辩解。

  31.答案:D

  【参考译文】写这封信的目的是什么?

  【试题分析】本题为主旨题。

  【详细解答】信的第二句说"But that statement fails to take account of the Bank's c riteria for 'success', which are exceptionally strict."指出了写信人没有考虑到银 行的成功标准,接着列举一些具体数字来阐述这一观点。在第二段展开了更加深入的论述。 因此,该信的目的是为了澄清对世界银行的误解。故答案选D。

  TEXT F  

  短文大意:本文主要论述的是发达国家对发展中国家的农业援助的危害性。

  32.答案:C

  【参考译文】作者的主要论点是什么?

  【试题分析】本题为主旨题。

  【详细解答】短文第一段倒数第二句说"In this sense, aid received in the form of agr icultural commodities hurts the developing countries and benefits developed coun tries mere than proportionately."由此可知,接受发达国家以农产品形式的援助,危害 的是发展中国家的利益,受益的是发达国家。接着在第二段以中国为例,证明了该论点,故 选项C"农产品形式的援助不利于发展中国家的经济"为正确答案。

  TEXT G 

  短文大意:本文旨在说明写作"牛仔工作文化"一书的目的。

  33.答案:B

  【参考译文】这篇文章最有可能源自何处?

  【试题分析】本题为推理题。

  【详细解答】本文分别阐述了写作该书的三个目的,用的是第一人称。由此可知这是对自己所 写的"牛仔工作文化"一书的评论。故选项B为正确答案。

  TEXT H  

  短文大意:这是一封回信,东方航空公司总裁在信中驳斥了某报社对该公司的评论。

  34.答案:A

  【参考译文】写信者对报社的评论持什么态度?

  【试题分析】本题为推理题。

  【详细解答】信的开头便说"In your editorial on August 31st, there seems to be some confused thinking in …"表明作者不同意报社的评论,接着作者对报社的观点一一进行 了反驳。故选项A"反驳"为正确答案。

  TEXT I  

  短文大意:本文主要介绍的是美国科学家计算机领域的最新进展情况。

  35.答案:D

  【参考译文】目前的计算机比1952年的ILLIAC型计算机的运算速度快多少?

  【试题分析】本题为推理题。

  【详细解答】短文第三段第二句说"Its processing speed was about 50 kilohertz compar ed with 200 megahertz-that's 200,000 kilohertz for today's computers."由此句可 知,当时的速度为50千赫兹,现在的是200,000千赫兹,那么现在的速度是当时的4,000倍 。故答案选D。

  36.答案:B

  【参考译文】NCSA(全国超级计算机应用中心)的发展目标是什么?

  【试题分析】本题为细节题。

  【详细解答】在短文的第八段,NCSA公司的董事长Larry Smart说"What we're looking for is a national system in which the networks are 100 times greater than the Intern et today, and the supercomputers are 100 times more powerful"即我们所期望的是一 种全国性的网络系统,其运作速度比目前的因特网要快100倍,故选项B"一个更加强大的全 国系统"为正确答案。

  TEXT J  

  短文大意:本文通过介绍日语中两个词的起源,说明了日本百货商店受到本国文化和外国文 化的双重影响。

  37.答案:B

  【参考译文】depato在日本指的是什么?

  【试题分析】本题为细节题。

  【详细解答】短文第一段说"The Japanese have two words for the modern department sto res that abound in large urban areas. The older word, hyakkaten, which is seldom  used in daily speech, …The more recent and more commonly used word is depato ( from the English 'department store' )."由此可知,在日本有两个词用来表示大城市中 的百货商店,hyakkaten是过去用的,最近常用的是depato,因此选项B为正确答案。

  38.答案:D

  【参考译文】在梅奇时代,日本顾客认为depato是怎样的购物场所?

  【试题分析】本题为细节题。

  【详细解答】短文第三段第三句说"The Meiji depato were soon perceived by Japanese c ustomers as glamorous places to shop because of their Western imports, which the Japanese were eager to see and buy."由此可知,当时的日本顾客把depato看成是富有 魅力的场所,故选项D"有吸引力的"为正确答案。

  TEXT K  

  短文大意:本文主要介绍的是在开罗召开的第三届国际人口与发展会议的一些情况。

  39.答案:D

  【参考译文】国际发展署是一个什么样的组织?

  【试题分析】本题为细节题。

  【详细解答】短文第四段第二句说"The National Statistics Office recently published the results of the 1993 National Demographic Survey(NDS), which happens to have been funded by the U.S. Agency for International Development." 这句话告诉我们说 "NDS的基金是由美国的国际发展署提供的",由此可知,国际发展署是一个美国的一个组 织,故答案选D。

  40.答案:C

  【参考译文】根据全国人口普查的统计数据,从1960年到1993年,菲律宾妇女生育小孩的平均 数量下降了多少?

  【试题分析】本题为细节题?

  【详细解答】短文倒数第二段第二句说"The total fertility rote——the number of babi es the average woman bears over her lifetime——has dropped to 4.1 in 1993 from 6.4 in 1960."由此可知,从1960年到1993年,菲律宾妇女生育小孩的减少率由6.4减少到4 1,其差为2.3,故选项C为正确答案。


1999年英语专业四级试题及答案


Part Ⅵ READING COMPREHENSION  [30 MIN.]

  SECTION A READING COMPREHENSION    [25 MIN.]

  In this section there are four passages followed by questions or unfinished statements, each with four suggested answers marked A, B, C and D. Choose the one th at you think is the correct answer. Mark your choice on your answer sheet.

  TEXT A

  Surprisingly, no one knows how many children receive education in English hospitals, still less the content or quality of that education. Proper records are jus t not kept. We know that more than 850.000 children go through hospital each year, and that every child of school age has a legal right to continue to receive education while in hospital. We also know there is only one hospital teacher to every 1,000 children in hospital.

  Little wonder the latest survey concludes that the extent and type of hospital teaching available differ a great deal across the country. It is found that half the hospitals in England which admit children have no teacher. A further quarter have only a part-time teacher. The special children's hospitals in major cities do best; general hospitals in the country and holiday areas are worst off. From this survey, one can estimate that fewer than one in five children have some contact with a hospital teacher-and that contact may be as little as two hour s a day. Most children interviewed were surprised to find a teacher in hospital at all. They had not been prepared for it by parents or their own school. If the re was a teacher they were much more likely to read books and do math or number work; without a teacher they would only play games.

  Reasons for hospital teaching range from preventing a child falling behind and m aintaining the habit of school to keeping a child occupied, and the latter is of ten all the teacher can do. The position and influence of many teachers was summ ed up when parents referred to them as "the library lady" or just "the helper". Children tend to rely on concerned school friends to keep in touch with school w ork. Several parents spoke of requests for work being ignored or refused by the school. Once back at school children rarely get extra teaching, and are told to catch up as best they can.

  Many short-stay child-patients catch up quickly. But schools do very little to e ase the anxiety about falling behind expressed by many of the children interview ed.

  66. The author points out at the beginning that___.

  A.  every child in hospital receives some teaching

  B.    not enough is known about hospital teaching

  C.    hospital teaching is of poor quality

  D.  the special children's hospitals are worst off 

  67.    It can be inferred from the latest survey that___.

  A.hospital teaching across the country is similar

  B.    each hospital has at least one part-time teacher

  C. all hospitals surveyed offer education to children

  D.only one-fourth of the hospitals have full-time teachers 

  68.    Children in hospital usual1y turn to___in order to catch up with  the ir school work.

  A.    hospital teachers    B.    schoolmates C.    parents    D.    school teachers 

  69.  We can conclude from the passage that the author is___.

  A.    unfavourable towards children receiving education in hospitals

  B.    in favour of the present state of teaching in hospitals

  C.    unsatisfied with the present state of hospital teaching

  D.    satisfied with the results of the latest survey 

  TEXT B

  Computer people talk a lot about the need for other people to become "computer-l iterate", in other words, to learn to understand computers and what makes them t ick. Not all experts agree, however, that is a good idea.

  One pioneer, in particular. who disagrees is David Tebbutt, the founder of Computertown UK. Although many people see this as a successful attempt to bring people closer to the computer, David does not see it that way. He says that Computertown UK was formed for just the opposite reason, to bring computers to the people and make them "people-literate".

  David first got the idea when he visited one of America's best-known computer "guru" figure, Bob Albrecht,in the small university town of Palo Alto in Northern California. Albrecht had started a project called Computertown USA in the local library, and the local children used to call round every Wednesday to borrow so me time on the computers there, instead of borrowing library books. Albrecht was always on hand to answer any questions and to help the children discover about computers in their own way.

  Over here, in Britain,Computertowns have taken off in a big way,and there are now about 40 scattered over the country. David Tebbutt thinks they are most successful when tied to a computer club. He insists there is a vast and important difference between the two, although they complement each other. The clubs cater f or the enthusiasts, with some computer knowledge already, who get together arid eventually form an expert computer group. This frightens away non-experts, who a re happier going to Computertowns where there are computers available for them t o experiment on, with experts available to encourage them and answer any questions; they are not told what to do, they find out.

  David Tehbutt finds it interesting to see the two different approaches working side by side. The computer experts have to learn not to tell people about computers, but have to be able to explain the answers to the questions that people really want to know. In some Computertowns there are question sessions, rather like radio phone-ins, where the experts listen to a lot of questions and then try to work out some structure to answer them. People are not having to learn computer jargons, but the experts are having to translate computer mysteries into easily understood terms; the computers are becoming "people-literate".

  70.    According to David Tebbutt, the purpose of Computertown UK is to___

  A.    train people to understand how computers work

  B.    make more computers available to people

  C.    enable more people to fix computers themselves

  D.    help people find out more about computers 

  71.    We Learn from the passage that Computertown USA was a ___.

  A. town      B.  project      C.  library      D.  school 

  72.  Which of the following statements is INCORRECT?

  A.    Computertowns in the UK have become popular.

  B.    Computertowns and clubs cater for different people.

  C.    Computertowns are more successful than clubs.

  D.    It's better that computertowns and clubs work together. 

  73.    Which of the following is NOT an advantage of computertowns?

  A.    Experts give lectures and talks on computers.

  B.    Experts are on hand to answer people's questions.

  C. People are left to discover computers on their own.

  D.    There are computers around for people to practise on. 

  TEXT C

  There must be few questions on which responsible opinion is so utterly divided a s on that of how much sleep we ought to have. There are some who think we can le ave the body to regulate these matters for itself. "The answer is easy," says Dr . A. Burton. "With the right amount of sleep you should wake up fresh and alert five minutes before the alarm rings." If he is right many people must be undersl eeping, including myself. But we must remember that some people have a greater i nertia than others.This is not meant rudely. They switch on slowly, and they a re reluctant to switch off. They are alert at bedtime and sleepy when it is time  to get up, and this may have nothing to do with how fatigued their bodies are, or how much sleep they must take to lose their fatigue.

  Other people feel sure that the present trend is towards too little sleep. To qu ote one medical opinion, thousands of people drift through life suffering from the effects of too little sleep; the reason is not that they can't sleep. Like a dvancing colonists, we do seem to be grasping ever more of the land of sleep for our waking needs, pushing the boundary back and reaching, apparently, for a poi nt in our evolution where we will sleep no more. This in itself, of course, need  not be a bad thing. What could be disastrous, however, is that we should press too quickly towards this goal, sacrificing sleep only to gain more time in which  to jeopardize our civilization by actions and decisions made weak by fatigue. Then, to complete the picture, there are those who believe that most people are persuaded to sleep too much. Dr H. Roberts, writing in Every Man in Health, asse rts: "It may safely be stated that, just as the majority eat too much, so the ma jority sleep too much." One can see the point of this also. It would be a pity t o retard our development by holding back those people who are gifted enough to w ork and play well with less than the average amount of sleep, if indeed it does them no harm. If one of the trends of evolution is that more of the life span i s to be spent in gainful waking activity, then surely these people are in the va n of this advance.

  74.    The author seems to indicate that___.

  A.    there are many controversial issues like the right amount of sleep

  B.    among many issues the right amount of sleep is the least controversial

  C.    people are now moving towards solving many controversial issues

  D.    the right amount of sleep is a topic of much controversy among doctors

  75.    The author disagrees with Dr. Burton because___.

  A.    few people can wake up feeling fresh and alert

  B.    some people still feel tired with enough sleep

  C.    some people still feel sleepy with enough sleep

  D.    some people go to bed very late at night 

  76.    In the last paragraph the author points out that___.

  A.    sleeping less is good for human development

  B.    people ought to be persuaded to sleep less than before

  C.    it is incorrect to say that people sleep too little

  D.    those who can sleep less should be encouraged 

  77.    We learn from the passage that the author___.

  A.    comments on three different opinions

  B.    favours one of the three opinions

  C.    explains an opinion of his own

  D.    revises someone else's opinion 

  TEXT D

  Migration is usually defined as "permanent or semipermanent change of residence. " This broad definition, of course, would include a move across the street or ac ross a city. Our concern is with movement between nations, not with internal mig ration within nations, although such movements often exceed international moveme nts in volume. Today, the motives of people who move short distances are very si milar to those of international migrants.

  Students of human migration speak of "push" and "pull" factors, which influence an individual's decision to move from one place to another. Push factors are ass ociated with the place of origin. A push factor can be as simple and mild a matt er as difficulty in finding a suitable job. or as traumatic as war, or severe fa mine. Obviously, refugees who leave their homes with guns pointed at their heads  are motivated almost entirely by push factors (although pull factors do influen ce their choice of destination).

  Pull factors are those associated with the place of destination. Most often thes e are economic,such as better job opportunities or the availability of good lan d to farm. The latter was an important factor in attracting settlers to the Unit ed States during the 19th century. In general, pull factors add up to an apparen tly better chance for a good life and material well-being than is offered by the place of origin. When there is a choice between several attractive potential de stinations, the deciding factor might be a non-economic consideration such as th e presence of relatives, friends, or at least fellow countrymen already establis hed in the new place who are willing to help the newcomers settle in. Considerat ions of this sort cad to the development of migration flow.

  Besides push and pull actors, there are what the sociologists call "intervening obstacles" Even if push and(or) pull factors are very strong they still may be ou tweighed by intervening obstacles, such as the distance of the move, the trouble  and cost of moving, the difficulty of entering the new country, and the problem s likely to be encountered on arrival. The decision to move is also influenced by "personal factors" of the potential m igrant. The same push-pull factors and obstacles operate differently on differen t people, sometimes because they are at different stages of their lives, or just  because of their varying abilities and personalities. The prospect of packing u p everything and moving to a new and perhaps very strange environment may appear  interesting and challenging to an unmarried young man and appallingly difficult to a slightly older man with a wife and small kids. Similarly, the need to lear n a new language and customs may excite one person and frighten another. Regardless of why people move, migration of large numbers of people causes confl ict. The United States and other western countries have experienced adjustment p roblems with each new wave of immigrants. The newest arrivals are usually given the lowest-paid jobs and are resented by native people who may have to compete w ith them for those jobs. It has usually taken several decades for each group to be accepted into the mainstream of society in the host country.

  78.    The author thinks that pull factors___.

  A.    are all related to economic considerations

  B.    are not as decisive as push factors

  C.    include a range of considerations

  D.    are more important than push factors 

  79.    People's decisions to migrate might be influenced by all the follow ing EXC EPT___.

  A.    personalities.                        B. education. C.    marital status.                        D. abilities. 

  80.    The purpose of the passage is to discuss___.

  A.    the problems of international migrants

  B.    the motives of international migrants

  C.    migration inside the country

  D. migration between countries

  答案与详解

  TEXT A  

  短文大意:这篇文章通过一次调查,反映了当孩子们生病住院时,医院里的教学问题。

  66.答案:B

  【参考译文】作者在文章开头指出了什么?

  【试题分析】本题为细节题。

  【详细解答】在第一段中,作者告诉我们说"没有人知道有多少孩子在医院里能接受到教育,更少有人知道教育的内容和教学质量",这与选项B"not enough is known about hospital teaching"的意思一致。

  67.答案:D

  【参考译文】由最后一次调查可以推知什么?

  【试题分析】本题为细节理解题。

  【详细解答】短文第三段第二、三句说"It is found that half the hospitals in England which admit children have no teacher. A further quarter have only a part-time teacher."由此可知,一半的医院没有老师,四分之一的医院有兼职老师,那么剩余的四分之一的医院就有全职老师了。

  68. 答案:B

  【参考译文】为了赶上学校的功课,住院的孩子常常求助于谁?

  【试题分析】本题为细节理解题。

  【详细解答】短文第六段第一句说"Children tend to rely on concerned school friends to keep in touch with school work",由此可知,他们求助的是同学们。

  69. 答案:C

  【参考译文】我们可以推知作者是何态度?

  【试题分析】本题为概括题。

  【详细解答】 作者一开头就用Surprisingly一词,清楚表达了对医院教育现状的态度。然后通过具体的调查结果,表明了对医院里儿童教学现状的不满。

  TEXT B  

  短文大意:本文介绍了"英国电脑城"的特点和功用。

  70.答案:B

  【参考译文】根据David Tebbutt的观点,"英国电脑城"的目的是什么?

  【试题分析】本题为细节题。

  【详细解答】短文第二段最后一句说"…Computertown UK was formed for just the opposite reason, to bring computers to the people and make them'people-literate'",由此可知,其目的是使得电脑"能理解人"(people-literate),使电脑更好地为人们服务,故选项B 为正确答案。

  71. 答案:B

  【参考译文】由该短文可知,美国的电脑城是什么?

  【试题分析】本题为细节题。

  【详细解答】短文第三段中第二句说"Albrecht had started a project called Computertown USA in the local library...",由此可直接选出答案B。

  72. 答案:C

  【参考译文】下列哪种说法是错误的?

  【试题分析】本题为细节题。

  【详细解答】短文第四段第二、三句说"…they are most successful when tied to a computer club. He insists there is a vast and important difference between the two, although they complement each other."它告诉了我们,computertowns和computerclubs是有区别而互补的,由此可知,选项C的说法是错误的,故答案应选C。

  73.答案:A

  【参考译文】下列哪一项不是computertowns的优点?

  【试题分析】本题为细节题。

  【详细解答】短文第五段第二句说"The computer experts have to learn not to tell people about computers,but have to be able to explain the answers to the questions that people really want to know."这与选项A "give lectures and talks on computers"相矛盾,而其它几项优点在本段都提到了,故选项A 为正确答案。 

  TEXT C  

  短文大意:本文讨论的是关于人们睡眠长短的问题。

  74. 答案:D

  【参考译文】作者似乎暗示什么?

  【试题分析】本题为细节理解题。

  【详细解答】短文首句说"There must be few questions on which responsible opinionis so utterly divided as on that of how much sleep we ought to have."意思是说"很少有问题像睡眠多长时间为适量这一问题那样存在如此大的意见分歧",由此可以推知选项D为正确答案。

  75. 答案:A

  【参考译文】作者为何不同意Dr.Burton的意见?

  【试题分析】本题为细节理解题。

  【详细解答】由短文第一段第四句说"If he is right many people must be undersleeping, including myself."要是按照Dr.Burton的说法,很多人就一定是睡眠不足,由此可知,作者认为实际情况并非如此,也就是说很少的人如此,故答案选A。

  76. 答案:D

  【参考译文】在文章的最后一段,作者指出了什么?

  【试题分析】本题为细节题。

  【详细解答】短文第三段第四句说"It would be a pity to retard our development by holding back those people who are gifted enough to work and play well with less than the average amount of sleep, if indeed it does them no harm." 由此可知,作者认为那些生来就可以少睡觉而且不误工作和玩耍的人不应受到约束,故答案选D。

  77. 答案:A

  【参考译文】从短文可知作者怎样?

  【试题分析】本题为概括题。

  【详细解答】通读全文可知,作者先列出了有关睡眠量问题的三种不同观点,继而对每种观点都做了评论,说明了自己的看法,但并没有提出支持哪一观点,也没有提出自己独创的新观或修正别人的观点,故选项A为正确答案。

  TEXT D  

  短文大意:本文谈论的是国际间移民的原因及其影响等问题。

  78. 答案:C

  【参考译文】作者认为"pull factors"如何?

  【试题分析】本题为细节题,可用排除法解答。

  【详细解答】短文第三段说"Most often these are economic,such as better job opportunities or the availability of good land to farm. …When there is a choice between several attractive potential destinations, the deciding factor might be a non-economic consideration such as the presence of relatives, friends, or at leastfellow countrymen already established in the new place who are willing to help the newcomers settle in." 由此可排除选项A;文中没有谈到与选项B和D相关的信息,故正确答案只有选项C。

  79. 答案:B

  【参考译文】下列哪一项不是影响人们决定移民的因素?

  【试题分析】本题为细节题。

  【详细解答】短文第五段第二句说"The same push-pull factors and obstacles operatedifferently on different people…or just because of their varying abilities and personalities."这里提到了abilities和personalities;本段第三句中提到了marital status。文中没有谈到与education相关的信息,故选项B为正确答案。

  80. 答案:B

  【参考译文】这篇文章的论题是什么?

  【试题分析】本题为主旨题。

  【详细解答】综观全文,作者在文中主要讨论了导致人员迁徙的四种不同动机,故答案选B

  SECTION B SKIMMING AND SCANNING [5 min]

  In this section there are seven passages with a total often multiple-choice questions. Skim or scan them as required and then mark your answers on your answer sheet.

  TEXT E

  First read the following question.

  81.    This is a letter of___.

  A.    inquiry            B. complaint        C.explanation    D introduction 

  Now, go through TEXT E quickly and answer question 81.

  Flat 24

  Park Mansions

  Newbury Road

  The Manager

  Reliable Motors Ltd.

  876 Meadow Street 14th May 2002

  Dear Sir,

  I am writing to you concerning the Bernster Special 150 SE that I bought from yo u two weeks ago. The car has now developed the following faults:

  1.    The steering wheel is loose.

  2.    The hand brake does not work.

  3.    Oil is leaking from the engine.

  4.    The driver's door-does not close properly.

  Will you please telephone me and we can arrange for you to collect the car. Unle ss you can put the car in perfect working order, 1 am afraid I shall report your  company to the Consumers' Association.

  Yours faithfully,

  Tony Lockwood

  TEXT F

  First read the following question.

  82.    The passage is mainly about___.

  A.    Christmas sales B.    retailing business C.    Internet population D.    online shopping 

  Now, go through TEXT F quickly and answer question 82.Online shopping (网上购物) has become a major force in retailing this year with more than US$1 billion in Christmas season sales, industry analysts say.The figure for the holiday tops the total for Internet shopping in all of 1996. Online shopping for the holiday season remained just a drop in the ocean of the estimated US$450 bilion spent by US consumers. But the figure is growing rapidly.International Data Corporation, a market research group, predicts the World Wide  Web (万维网) population will reach almost 100 million by 1998 and that online c ommerce will grow to more than US$20 billion.

  TEXT G

  First read the following question

  83.    The passage discusses the aim of___.

  A.    the organization        B.    blood centres in the USA C.    FDA new rules      D. AIDS prevention 

  Now, go through TEXT G quick and answer question 83.

  The US Food and Drug Administration (FDA) is taking steps to protect the country's blood supply. People give blood to the centres, where it is kept until it is needed for medical purposes. The FDA has provided new rules for the blood centres. The government agency says  new rules are designed to improve the blood supply system. The new rules call for blood centres to develop more ways to make sure their wor k is done correctly.These rules are another way to help keep the blood supply pure. A leading concer n is that someone with AIDS virus might give blood to a blood centre. For this r eason, there are tests to find out if blood contains viruses that cause AIDS and other diseases.

  TEXT H

  First read the following question.

  84.    The passage advertises overseas___.

  A.    jobs  B.  studies      C.  travel    D.      aid

  Now, go through TEXT H quickly and answer question 84.

  NEW HORIZONS 

  Are you looking for something interesting to do? Then why not work abroad for a year or two? We have jobs in most parts of the world -including Europe, Africa and Southeast  Asia.We have jobs for teachers, doctors, nurses, engineers, car mechanics and many ot hers. Why not see the world? We cannot offer you very much money, but the work is interesting. You can learn another language and work with people, too.For more information write to us at:

  NEW HORIZONS JOB CENTER,

  110 Spring Gardens,

  London SW ! 7BC

  Please send your personal information (date of birth, educational qualifications , interests. experiences, etc.)

  TEXT I

  First read the following questions.

  85.    Which number would you dial for home nursing?

  A.    0734 442456 B.    0734 442675 C.    08675 559478 D.    08675 564499 

  86.    Blood donors are advised to contact___.

  A.    Berkshire county office B.    St. John Ambulance C.    John Radcliffe Hospital D.    Royal Berkshire Hospital 

  Now, go through TEXT I quickly and answer questions 85 and 86.

  HEALTH

  AMBULANCE SERVICE

  In All Emergencies Dial 999

  St John Ambulance:

  Berkshire County Office - ST John Centre Church Rd, Woodley. Reading, RG5 4QN.0734 442456.

  For details of first aid at work unit contact 0734 442675.

  Oxfordshire - St John House High St, Kidlington, Oxford. 0X5 2DN 08675 559478: Association; 08675 564499 Brigade, ambulance, home nursing, loan of medical equipment and first aid at work unit.

  BLOOD DONORS

  For information contact-

  Oxford Regional Blood Transfusion Service: John Radcliffe Hospital Headley Way,Headington, Oxford, 0X3 9DU 0865 642831

  HOSPITALS

  Royal Berkshire Hospital: London Rd. Reading, RGI SAN. 0734 875111. ( Accident and emergency patients to South Wing).

  TEXT J

  First read the following questions.

  87.    If you leave for Hong Kong on March l6th, you pay___.

  A.    $799  B.  $849    C.  $829    D. $969 

  88.    What is NOT included in the price?

  A.    Local dept tax.      B.  Return flights. C.    Transfers.        D.    Accommodation. 

  Now, go through TEXT J quickly and answer questions 87 and 88.

  SPECIAL OFFER

  from $799

  HONG KONG

  Enjoy 5 nights in Hong Kong from $799

  New World Harbour View:  Superior 1St class hotel in good location on Hong Kong Island.

  Superb swimming pool, tennis courts, restaurants, bars. Rooms with harbour view and 

  Airconditioning, tea/coffee making facilities, minibar. TV, bath and shower.

  Departure dates:  Price:

  02,10 Mar $799

  09,16 Mar $849

  17 Mar $829

  15 Apr $969

  27 Apr, 04, 18 May $999

  The price includes: Return flights. 5 nts accom (no meals). Transfers. Prices are per person sharing a twin room.

  Not included: UK dept tax. Local dept tax. Optional insurance: $30.

  To book, telephone: (open daily inc Sat/Sun)

  Tel:  01306 774300

  Fax: 01306 740328

  TEXT K

  First read the following questions.

  89.    On hearing the fire alarm, those in class should go to___.

  A.    A Block      B.  B Block C.    C Block      D.  the Assembly Area 

  90.    What should you do during the fire alarm?

  A.    Make use of the lift.

  B.    Collect personal things.

  C.    Move along without noise.

  D.    Overtake others on the way. 

  Now, go through TEXT K quickly and answer questions 89 and 90.

  FIRE INSTRUCTIONS

  THE PERSON DISCOVERING A FIRE WILL:

  1.  OPERATE THE NEAREST FIRE ALARM.

  2.    ATTACK THE FIRE WITH AVAILABLE EQUIPMENT, IF IT IS SAFE TO DO SO. 

  FIRE ALARM BELLS

  The Fire Alarm Bells will ring either in the area of A Block or in the area of B

  Block and C Block. Those in the area where the Alarm Bells are ringing should t ake action as indicated below. Others should continue with their work.

  ON HEARING YOUR FIRE ALARM:

  1. Those in class: will go to the Assembly Area under instructions given by the teacher.

  2.  Those elsewhere: will go to the Assembly Area by the most sensible route, and stay near the Head of their Department. 

  ASSEMBLY AREA

  The Assembly Area is the playing field which is south of the Sports Hall. Here n ames will be checked.

  PROCEDURE

  1 .Move quietly.

  2.Do NOT stop to collect your personal belongings.

  3.Do NOT attempt to pass others on your way to the Assembly Area.

  4.Do NOT use the lift.

  FIRE ALARMS

  Fire Alarms are situated as follows:

  1.    A Block

  At the Reception Desk; at east end of connecting corridor; outside the kitchen d oor. 

  2.    B Block

  At the bottom of both stairways and on each landing. 

  3.  C Block

  Inside entrance lobby of Sports Hall.

  答案与详解

  TEXT E  

  短文大意:这是一封消费者向汽车生产厂家提出的投诉信。

  81. 答案:B

  【参考译文】这是一封什么信?

  【试题分析】本题为主旨题。

  【详细解答】信的最后一句说"I am afraid I shall report your company to the Consumers'Association."即我恐怕要将你们公司告到消费者协会,由此可见这是一封投诉信。

  TEXT F  

  短文大意:这篇短文介绍的是网上购物的发展趋势。

  82.答案:D

  【参考译文】这篇文章主要是关于什么的?

  【试题分析】本题为主旨题。

  【详细解答】短文首句说"Online shopping (网上购物) has become a major force in retailing this year…",由此可知本文主题为Online shopping。

  TEXT G  

  短文大意:本文介绍的是美国食品和药品管理委员会为保证血液供应质量而颁布的新条例。

  83. 答案:C

  【参考译文】本文讨论的是下列哪一组织的目标?

  【试题分析】本题为主旨题。

  【详细解答】本文第一段说"(FDA) is taking steps to protect the country's blood supply."第二段说"The FDA has prodded new rules...new rules are designed to …",第三、四段是说新条例的措施和目的。因此本题正确答案为选项C。

  TEXT H  

  短文大意:这是一则有关招聘海外劳工的广告。

  84. 答案:A

  【参考译文】这是一则招聘什么人的广告?

  【试题分析】本题为主旨题。

  【详细解答】短文多处提及与工作相关的句子,如"We have jobs in most parts of theworld …","We have jobs for teachers, doctors, nurses, engineers, car mechanics and many others."由此可知,这是一则招聘去海外工作的广告。

  TEXT I  

  短文大意:这篇短文列出了一些不同类别的服务机构的以及他们的服务项目、地址和联系电话。

  85. 答案:D

  【参考译文】呀请家政人员应该拨哪个电话?

  【试题分析】本题为细节题。

  【详细解答】快速在HEALTH AMBULANCE SERVICE一栏找到home nursing,其对应的电话号码是08675。

  86.答案:C 

  【参考译文】建议献血的人与哪儿联系?

  【试题分析】本题为细节题。

  【详细解答】快速查找BLOOD DONORS一栏,其相应的机构为John Radcliffe Hospital。

  TEXT J  

  短文大意:这是一则酒店的广告。

  87. 答案:B

  【参考译文】如果你三月16日去香港,你应付多少钱?

  【试题分析】本题为细节题。

  【详细解答】在Departure dates栏里迅速找到09,16 Mar,可知其对应的价格为$ 849。

  88. 答案:A

  【参考译文】价格里面不包括什么?

  【试题分析】本题为细节题。

  【详细解答】在Not included栏里可以找到local dept tax,故价格里不包括选项A。

  TEXT K 

  短文大意:这是一则有关学校里的火警的小册子。

  89.答案:D

  【参考译文】听到火警,教室里的人应去哪儿?

  【试题分析】本题为细节题。

  【详细解答】迅速在ON HEARING YOUR FIRE ALARM栏里找到Those in class:will go to the Assembly Area…,可以确定答案选D。

  90. 答案:C

  【参考译文】火警期间,你该做什么?

  【试题分析】本题为细节题。

  【详细解答】在PROCEDURE一栏中可以查寻到Move quietly,故答案选C。

  本套真题测试的语言重点

  (重点单词)

  endow  天生具有,赋予

  exclusively  独占地;唯一地

  descend  由……传下来

  radically  完全地,本质地

  jargon  术语,行话

  tick  [口]开始起作用

  traumatic  使人不快的,令人痛苦难忘的

  jeopardize  使……处于危险中

  (重点词组)

  hand down  留传下来,把……传下去

  be guilty of  犯有……罪的

  in the van of  处于……的领先地位

  It is not so much…as…  与其说是……不如说是……

  beyond the reach of  在……范围之外

  be doomed to sth.  注定……

  TIME LIMIT: 140 MIN.

  Part Ⅰ WRITING  [45 MIN.]

  SECTION A    COMPOSITION  

  It is now generally accepted that vehicles (cars, trucks, etc.) are a major source of air pollution in cities. You are to suggest only ONE way to solve the problem.

  Write on ANSWER SHEET ONE a composition of about 150 words on the following topic:

                                    ONE WAY TO SOLVE THE PROBLEM

  You are to write in three paragraphs.

  In the first paragraph, state what your suggested way is.

  In the second paragraph, state one or two advantages of your suggestion on.

  In the last paragraph, bring what you have written to a natural conclusion.

  Marks will he awarded for content, organization, grammar and appropriacy. Failure to follow the instructions may result in a loss of marks.

  SECTION B  NOTE-WRITING  [10 MIN.]

  Write on ANSWER SHEET ONE a note of about 50-60 words based on the following situation:

  You are Mark or Sally. You have got a ticket to a computer fair, but you now find that you are unable to go. Write a note to your friend, George, explaining why you are sending the enclosed ticket to him and telling him briefly how to get there. Marks will be awarded for content, organization, grammar and appropriacy.

  答案与详解

  PART I WRITING

  SECTION A 

  [参考范文]

  One Way to Solve the Problem

  We all know that vehicles are the major source of air pollution in cities. To solve the headache, people have taken a variety of steps, but the result is not satisfying. Here, one way to solve the problem is to set gas thrifter in vehicle.

  Setting gas thrifter in vehicle not only decreases the consumption of gas which is a major source of air pollution but also slows down the cost of possessing a vehicle which makes millaye price lower. Thereby, all vehicles set gas thrifter will make air have a lower carbon dioxide content deriving from burning gas and coal. The less carbon dioxide, the less air pollution. Therefore, through setting gas thrifter, vehicles will bum lower gases, and the problem will partly be solved.

  Despite other ways to solve the problem, I think the best way is to set gas thrifter in vehicle for both effect and practice.

  SECTION B

  [参考范文]

  Dear George,

  I have got a ticket to a computer fair. Though I would like to go there, I am unable to, because I have an important date on that day. As we all know, you am computer crazy; so I enclose the ticket to you. You can get to the fair by taking the No. 6 bus.

  Wish you would enjoy yourself there.

  All the best,

  Yours,

2000年英语专业八级试题及答案

Part Ⅰ Listening Comprehension (40 min)

  In Sections A, B and C you will hear everything ONCE ONLY. Listen carefully and then answer the questions that follow. Mark the correct answer to each question on your Coloured Answer Sheet.

  SECTION A TALK

  Questions 1 to 5 refer to the talk in this section .At the end of the talk you w ill be given 15 seconds to answer each of the following five questions. Now list en to the talk. 

  1. The rules for the first private library in the US were drawn up by ___.

  A. the legislature B. the librarian C. John Harvard D. the faculty members

  2. The earliest public library was also called a subscription library because books ___.

  A. could be lent to everyone

  B. could be lent by book stores

  C. were lent to students and the faculty

  D. were lent on a membership basis

  3. Which of the following is NOT stated as one of the purposes of free public libraries?

  A. To provide readers with comfortable reading rooms.

  B. To provide adults with opportunities of further education.

  C. To serve the community's cultural and recreational needs.

  D. To supply technical literature on specialized subjects.

  4. The major difference between modem private and public libraries lies i n ___.

  A. readership B. content C. service  D. function

  5. The main purpose of the talk is ___.

  A. to introduce categories of books in US libraries

  B. to demonstrate the importance of US libraries

  C. to explain the roles of different US libraries

  D. to define the circulation system of US libraries

  SECTION B INTERVIEW

  Questions 6 to 10 are based on an interview. At the end of the interview you wil l be given 15 seconds to answer each of the following five questions. Now listen to the interview.

  6. Nancy became a taxi driver because ___.

  A. she owned a car

  B. she drove well

  C. she liked drivers' uniforms

  D. it was her childhood dream

  7. According to her, what was the most difficult about becoming a taxi dr iver?

  A. The right sense of direction.

  B. The sense of judgment.

  C. The skill of maneuvering.

  D. The size of vehicles.

  8. What does Nancy like best about her job?

  A. Seeing interesting buildings in the city.

  B. Being able to enjoy the world of nature.

  C. Driving in unsettled weather.

  D. Taking long drives outside the city.

  9. It can be inferred from the interview that Nancy in a(n) ___ moth er.

  A. uncaring B. strict C. affectionate D. perm issive

  10. The people Nancy meets are

  A. rather difficult to please

  B. rude to women drivers

  C. talkative and generous with tips

  D. different in personality

  SECTION C NEWS BROADCAST

  Question 11 is based on the following news. At the end of the news item, you wil l be given 15 seconds to answer the question. Now listen to the news.

  11. The primary purpose of the US anti-smoking legislation is ___.

  A. to tighten control on tobacco advertising

  B. to impose penalties on tobacco companies

  C. to start a national anti-smoking campaign

  D. to ensure the health of American children

  Questions 12 and 13 are based on the following news. At the end of the news item , you will be given 30 seconds to answer the questions. Now listen to the news.

  12. The French President's visit to Japan aims at ___.

  A. making more investments in Japan

  B. stimulating Japanese businesses in France

  C. helping boost the Japanese economy

  D. launching a film festival in Japan

  13. This is Jacques Chirac's ___ visit to Japan.

  A. second B. fourteenth C. fortieth D. forty-first

  Questions 14 and 15 are based on the following news. At the end of the news item , you will be given 30 seconds to answer the questions. Now listen to the news.

  14. Afghan people are suffering from starvation because ___.

  A. melting snow begins to block the mountain paths

  B. the Taliban have destroyed existing food stocks

  C. the Taliban are hindering food deliveries

  D. an emergency air-lift of food was cancelled

  15. people in Afghanistan are facing starvation.

  A. 160,000 B. 16,000 C. 1,000,000 D. 100 ,000

  SECTION D NOTE-TAKING AND GAP-FILLING

  Fill each of gaps with ONE word. You may refer to your notes. Make sure the word you fill in is both grammatically and semantically acceptable.

  On Public Speaking

  When people are asked to give a speech in public for the first time, they usually feel terrified no matter how well they speak in informal situations. In fact, public speaking is the same as any other form of (1)___ 1.___ that people are usually engaged in. Public speaking is a way for a speaker to (2)___ his thoughts with the audience. Moreover, the speaker is free 2.___ to decide on the (3)___ of his speech. 3.___ Two key points to achieve success in public speaking: -(4)___ of the subject matter. 4.___ -good preparation of the speech. To facilitate their understanding, inform your audience beforehand of the (5)___ of your speech, and end it with a summary. 5.___ Other key points to bear in mind: -be aware of your audience through eye contact. -vary the speed of (6)___ 6.___ -use the microphone skillfully to (7)___ yourself in speech. 7.___ -be brief in speech; always try to make your message (8)___ 8.___ Example: the best remembered inaugural speeches of the US presidents are the (9)___ ones. 9.___ Therefore, brevity is essential to the (10)___ of a speech. 10.___

  Part Ⅱ Proofreading and Error Correction (15 min)

  The following passage contains TEN errors. Each line contains a maximum of ONE error. In each case, only ONE word is involved. You should proofread the passage and correct it in the following way. For a wrong word, underline the wrong word and wri te the correct one in the blank provided at the end of the line. For a missing word, mark the position of the missing word with a "∧" sign and write the word you believe to be missing in the blank provided at the end of the line. For an unnecessary word cross out the unnecessary word with a slash "/' and put the word in the blank provided at the end of the line.

  Example

  When∧art museum wants a new exhibit, (1) an

  it never/ buys things in finished form and hangs (2) never

  them on the wall. When a natural history museum 

  wants an exhibition, it must often build it. (3) exhibit

   The grammatical words which play so large a part in English

  grammar are for the most part sharply and obviously different 1.___

  from the lexical words. A rough and ready difference which may

  seem the most obvious is that grammatical words have "less

  meaning", but in fact some grammarians have called them 2.___

  "empty" words as opposed in the "full" words of vocabulary. 3.___

  But this is a rather misled way of expressing the distinction. 4.___

  Although a word like the is not the name of something as man is,

  it is very far away from being meaningless; there is a sharp 5.___

  difference in meaning between "man is vile" and "the man is

  vile", yet the is the single vehicle of this difference in meaning. 6.___

  Moreover, grammatical words differ considerably among

  themselves as the amount of meaning they have, even in the 7.___

  lexical sense. Another name for the grammatical words has been

  "little words". But size is by no mean a good criterion for 8.___

  distinguishing the grammatical words of English, when we

  consider that we have lexical words as go, man, say, car. Apart 9.___

  from this, however, there is a good deal of truth in what some

  people say: we certainly do create a great number of obscurity 10.___

  when we omit them. This is illustrated not only in the poetry of

  Robert Browning but in the prose of telegrams and newspaper headlines.

  Part Ⅲ Reading Comprehension (40 min)

  SECTION A READING COMPREHENSION (30 min)

  In this section there are four reading passages followed by a total of fifteen multiple-choice questions. Read the passages and then mark your answers on your Coloured Answer Sheet.

  TEXT A

  Despite Denmark's manifest virtues, Danes never talk about how proud they a re to be Danes. This would sound weird in Danish. When Danes talk to foreigners about Denmark, they always begin by commenting on its tininess, its unimportance , the difficulty of its language, the general small-mindedness and self-indulgen ce of their countrymen and the high taxes. No Dane would look you in the eye and say, "Denmark is a great country." You're supposed to figure this out for yo urself.

  It is the land of the silk safety net, where almost half the national budg et goes toward smoothing out life's inequalities, and there is plenty of money f or schools, day care, retraining programmes, job seminars-Danes love seminars: t hree days at a study centre hearing about waste management is almost as good as a ski trip. It is a culture bombarded by English, in advertising, pop music, the Internet, and despite all the English that Danish absorbs-there is no Danish Academy to defend against it -old dialects persist in Jutland that can barel y be understood by Copenhageners. It is the land where, as the saying goes," Fe w have too much and fewer have too little, "and a foreigner is struck by the swe e t egalitarianism that prevails, where the lowliest clerk gives you a level gaze, where Sir and Madame have disappeared from common usage, even Mr. and Mrs. It' s a nation of recyclers-about 55 % of Danish garbage gets made into something new- and no nuclear power plants. It's a nation of tireless planner. Trains run on time. Things operate well in general.

  Such a nation of overachievers - a brochure from the Ministry of Busines s and Industry says, "Denmark is one of the world's cleanest and most organize d countries, with virtually no pollution, crime, or poverty. Denmark is the most c orruption-free society in the Northern Hemisphere. "So, of course, one's heart l ifts at any sighting of Danish sleaze: skinhead graffiti on buildings("Foreigne r s Out of Denmark! "), broken beer bottles in the gutters, drunken teenagers slu mped in the park. 

  Nonetheless, it is an orderly land. You drive through a Danish town, it co mes to an end at a stone wall, and on the other side is a field of barley, a nic e clean line: town here, country there. It is not a nation of jay-walkers. Peopl e stand on the curb and wait for the red light to change, even if it's 2 a.m. a n d there's not a car in sight. However, Danes don' t think of themselves as a w ai nting-at-2-a.m.-for-the-green-light people——that's how they see Swedes and Ge r mans. Danes see themselves as jazzy people, improvisers, more free spirited than Swedes, but the truth is( though one should not say it)that Danes are very much like Germans and Swedes. Orderliness is a main selling point. Denmark has few n atural resources, limited manufacturing capability; its future in Europe will be as a broker, banker, and distributor of goods. You send your goods by container ship to Copenhagen, and these bright, young, English-speaking, utterly honest, highly disciplined people will get your goods around to Scandinavia, the Baltic States, and Russia. Airports, seaports, highways, and rail lines are ultramodern and well-maintained.

  The orderliness of the society doesn't mean that Danish lives are less me s sy or lonely than yours or mine, and no Dane would tell you so. You can hear ple nty about bitter family feuds and the sorrows of alcoholism and about perfectly sensible people who went off one day and killed themselves. An orderly society c an not exempt its members from the hazards of life.

  But there is a sense of entitlement and security that Danes grow up with. Certain things are yours by virtue of citizenship, and you shouldn't feel bad f o r taking what you're entitled to, you're as good as anyone else. The rules of th e welfare system are clear to everyone, the benefits you get if you lose your jo b, the steps you take to get a new one; and the orderliness of the system makes it possible for the country to weather high unemployment and social unrest witho ut a sense of crisis.

  16. The author thinks that Danes adopt a ___ attitude towards their country.

  A. boastful B. modest C. deprecating D. mysterious

  17. Which of the following is NOT a Danish characteristic cited in the pa ssage?

  A. Fondness of foreign culture. B. Equality in society. C. Linguistic tolerance. D. Persistent planning.

  18. The author's reaction to the statement by the Ministry of Business a nd Industry is ___.

  A. disapproving B. approving C. noncommittal D. doubtful

  19. According to the passage, Danish orderliness ___.

  A. sets the people apart from Germans and Swedes

  B. spares Danes social troubles besetting other people

  C. is considered economically essential to the country

  D. prevents Danes from acknowledging existing troubles

  20. At the end of the passage the author states all the following EXCEPT that ___.

  A. Danes are clearly informed of their social benefits

  B. Danes take for granted what is given to them

  C. the open system helps to tide the country over

  D. orderliness has alleviated unemployment

  TEXT B

  But if language habits do not represent classes, a social stratification in to something as bygone as "aristocracy" and "commons", they do still of cour se s erve to identify social groups. This is something that seems fundamental in the use of language. As we see in relation to political and national movements, lang uage is used as a badge or a barrier depending on which way we look at it. The n ew boy at school feels out of it at first because he does not know the fight wor ds for things, and awe-inspiring pundits of six or seven look down on him for no t being aware that racksy means "dilapidated", or hairy "out first ball". Th e mi ner takes a certain pride in being "one up on the visitor or novice who calls t h e cage a "lift" or who thinks that men working in a warm seam are in their "u nde rpants" when anyone ought to know that the garments are called hoggers. The "i ns ider" is seldom displeased that his language distinguishes him from the "outsi der".

  Quite apart from specialized terms of this kind in groups, trades and profe ssions, there are all kinds of standards of correctness at which mast of us feel more or less obliged to aim, because we know that certain kinds of English invi te irritation or downright condemnation. On the other hand, we know that other k inds convey some kind of prestige and bear a welcome cachet.

  In relation to the social aspects of language, it may well be suggested tha t English speakers fall into three categories: the assured, the anxious and the in different. At one end of this scale, we have the people who have "position" an d "status", and who therefore do not feel they need worry much about their use o f English. Their education and occupation make them confident of speaking an uni mpeachable form of English: no fear of being criticized or corrected is likely t o cross their minds, and this gives their speech that characteristically unself c onscious and easy flow which is often envied. 

  At the other end of the scale, we have an equally imperturbable band, speak ing with a similar degree of careless ease, because even if they are aware that their English is condemned by others, they are supremely indifferent to the fact . The Mrs Mops of this world have active and efficient tongues in their heads, a nd if we happened not to like the/r ways of saying things, well, we "can lump i t ". That is their attitude. Curiously enough, writers are inclined to represent t he speech of both these extreme parties with -in' for ing. On the one hand, "w e're goin' huntin', my dear sir"; on the other, "we're goin' racin' , ma te."

  In between, according to this view, we have a far less fortunate group, th e anxious. These actively try to suppress what they believe to be bad English an d assiduously cultivate what they hope to be good English. They live their lives in some degree of nervousness over their grammar, their pronunciation, and thei r choice of words: sensitive, and fearful of betraying themselves. Keeping up wi th the Joneses is measured not only in houses, furniture, refrigerators, cars, a nd clothes, but also in speech.

  And the misfortune of the "anxious" does not end with their inner anxiet y. Their lot is also the open or veiled contempt of the "assured" on one side of them and of the "indifferent" on the other.

  It is all too easy to raise an unworthy laugh at the anxious. The people t hus uncomfortably stilted on linguistic high heels so often form part of what is, in many ways, the most admirable section of any society: the ambitious, tense, inner-driven people, who are bent on" going places and doing things". The grea te r the pity, then, if a disproportionate amount of their energy goes into what Mr Sharpless called" this shabby obsession" with variant forms of English- espe ci ally if the net result is(as so often)merely to sound affected and ridiculous. " Here", according to Bacon, "is the first distemper of learning, when men study w ords and not matter …. It seems to me that Pygmalion' s frenzy is a good emble m …of this vanity: for words axe but the images of matter; and except they have l ife of reason and invention, to fall in love with them is to fall in love with a picture."

  21. The attitude held by the assured towards language is ___.

  A. critical B. anxious C. self-conscious D. nonchalant

  22. The anxious are considered a less fortunate group because ___.

  A. they feel they are socially looked down upon

  B. they suffer from internal anxiety and external attack

  C. they are inherently nervous and anxious people

  D. they are unable to meet standards of correctness

  23. The author thinks that the efforts made by the anxious to cultivate w hat they believe is good English are ___.

  A. worthwhile B. meaningless C. praiseworthy D. irrational

  TEXT C

  Fred Cooke of Salford turned 90 two days ago and the world has been beating a path to his door. If you haven't noticed, the backstreet boy educated at Bla c kpool grammar styles himself more grandly as Alastair Cooke, broadcaster extraor dinaire. An honorable KBE, he would be Sir Alastair if he had not taken American citizenship more than half a century ago.

  If it sounds snobbish to draw attention to his humble origins, it should be reflected that the real snob is Cooke himself, who has spent a lifetime disguis ing them. But the fact that he opted to renounce his British passport in 1941 - just when his country needed all the wartime help it could get-is hardly a ma tter for congratulation.

  Cooke has made a fortune out of his love affair with America, entrancing l isteners with a weekly monologue that has won Radio 4 many devoted adherents. Pa rt of the pull is the developed drawl. This is the man who gave the world "mida tlantic", the language of the disc jockey and public relations man.

  He sounds American to us and English to them, while in reality he has for decades belonged to neither. Cooke's world is an America that exists largely in the imagination. He took ages to acknowledge the disaster that was Vietnam and e ven longer to wake up to Watergate. His politics have drifted to the right with age, and most of his opinions have been acquired on the golf course with fellow celebrities.

  He chased after stars on arrival in America, Fixing up an interview with Ch arlie Chaplin and briefly becoming his friend. He told Cooke he could turn him i nto a fine light comedian; instead he is an impressionist's dream.

  Cooke liked the sound of his first wife's name almost as much as he admir e d her good looks. But he found bringing up baby difficult and left her for the w ife of his landlord. Women listeners were unimpressed when, in 1996, he declared on air that th e fact that 4% of women in the American armed forces were raped showed remarkabl e self-restraint on the part of Uncle Sam's soldiers. His arrogance in not allo w ing BBC editors to see his script in advance worked, not for the first time, to his detriment. His defenders said he could not help living with the 1930s values he had acquired and somewhat dubiously went on to cite "gallantry" as chief a mo ng them. Cooke's raconteur style encouraged a whole generation of BBC men to th i nk of themselves as more important than the story. His treacly tones were the mo del for the regular World Service reports From Our Own Correspondent, known as F OOCs in the business. They may yet be his epitaph.

  24. At the beginning of the passage the writer sounds critical of ___.

  A. Cooke's obscure origins

  B. Cooke's broadcasting style

  C. Cooke's American citizenship

  D. Cooke's fondness of America

  25. The following adjectives can be suitably applied to Cooke EXCEPT ___.

  A. old-fashioned B. sincere C. arrogant D. popular 26. The writer comments on Cooke's life and career in a slightly ___ tone.

  A. ironic B. detached C. scathing D. indifferent

  TEXT D

   Mr Duffy raised his eyes from the paper and gazed out of his window on the cheerless evening landscape. The river lay quiet beside the empty distillery and from time to time a light appeared in some house on Lucan Road. What an end! Th e whole narrative of her death revolted him and it revolted him to think that he had ever spoken to her of what he held sacred. The cautious words of a reporter won over to conceal the details of a commonplace vulgar death attacked his stom ach. Not merely had she degraded herself, she had degraded him. His soul's comp a nion! He thought of the hobbling wretches whom he had seen carrying cans and bot tles to be filled by the barman. Just God, what an end! Evidently she had been u nfit to live, without any strength of purpose, an easy prey to habits, one of th e wrecks on which civilization has been reared. But that she could have sunk so low! Was it possible he had deceived himself so utterly about her? He remembered her outburst of that night and interpreted it in a harsher sense than he had ev er done. He had no difficulty now in approving of the course he had taken.

  As the light failed and his memory began to wander he thought her hand tou ched his. The shock which had first attacked his stomach was now attacking his n erves. He put on his overcoat and hat quickly and went out. The cold air met him on the threshold; it crept into the sleeves of his coat. When he came to the pu blic house at Chapel Bridge he went in and ordered a hot punch.

  The proprietor served him obsequiously but did not venture to talk. There were five or six working-men in the shop discussing the value of a gentleman's e state in County Kildare. They drank at intervals from their huge pint tumblers, and smoked, spitting often on the floor and sometimes dragging the sawdust over their heavy boots. Mr Duffy sat on his stool and gazed at them, without seeing o r hearing them. After a while they went out and he called for another punch. He sat a long time over it. The shop was very quiet. The proprietor sprawled on the counter reading the newspaper and yawning. Now and again a tram was heard swish ing along the lonely road outside.

  As he sat there, living over his life with her and evoking alternately the two images on which he now conceived her, he realized that she was dead, that s he had ceased to exist, that she had become a memory. He began to feel ill at ea se. He asked himself what else could he have done. He could not have lived with her openly. He had done what seemed to him best. How was he to blame? Now that s he was gone he understood how lonely her life must have been, sitting night afte r night alone in that room. His life would be lonely too until he, too, died, ce ased to exist, became a memory-if anyone remembered him.

  27. Mr Duffy's immediate reaction to the report of the woman's death wa s that of ___.

  A. disgust B. guilt C. grief D. compassion

  28. It can be inferred from the passage that the reporter wrote about the woman's death in a ___ manner.

  A. detailed B. provocative C. discreet D. sens ational

  29. We can infer from the last paragraph that Mr Duffy was in a(n) ___ mood.

  A. angry B. fretful C. irritable D. remorseful

  30. According to the passage , which of the following statements is NOT t rue?

  A. Mr Duffy once confided in the woman.

  B. Mr Duffy felt an intense sense of shame.

  C. The woman wanted to end the relationship.

  D. They became estranged probably after a quarrel.

  SECTION B SKIMMING AND SCANNING ( 10 min)

  In this section there are seven passages followed by ten multiple -choice q uestions. Skim or scan them as required and then mark your answers on the Colour ed Answer Sheet.

  TEXT E

  First read the following question.

  31. In the passage Bill Gates mainly discusses ___.

  A. a person's opportunity of a lifetime

  B. the success of the computer industry

  C. the importance of education

  D. high school education in the US

  Now go through TEXT E quickly and answer question 31.

  Hundreds of students send me e-mail each year asking for advice about educa tion. They want to know what to study, or whether it's OK to drop out of colleg e since that's what I did.

  My basic advice is simple and heartfelt." Get the best education you can. Take advantage of high school and college. Learn how to learn."

  It's true that I dropped out of college to start Microsoft, but I was at H a rvard for three years before dropping out-and I'd love to have the time to go b a ck. As I've said before, nobody should drop out of college unless they believe they face the opportunity of a lifetime. And even then they should reconsider.

  The computer industry has lots of people who didn't finish college, but I 'm not aware of any success stories that began with somebody dropping out of high school. I actually don't know any high school dropouts, let alone any successfu l ones.

  In my company's early years we had a bright part-time programmer who threa tened to drop out of high school to work full-time. We told him no.

  Quite a few of our people didn't finish college, but we discourage droppin g out.

  College isn't the only place where information exist. You can learn in a l i brary. But somebody handing you a book doesn't automatically foster learning. Y o u want to learn with other people, ask questions, try out ideas and have a way t o test your ability. It usually takes more than just a book.

  Education should be broad, although it's fine to have deep interests, too.

   In high school there were periods when I was highly focused on writing soft ware, but for most of my high school years I had wide-ranging academic interests . My parents encouraged this, and I'm grateful that they did.

  One parent wrote me that her 15-year old son "lost himself in the hole of t he computer. "He got an A in Web site design, but other grades were sinking, sh e said.

  This boy is making a mistake. High school and college offer you the best ch ance to learn broadly-math, history, various sciences-and to do projects with ot her kids that teach you firsthand about group dynamics. It's fine to take a dee p interest in computers, dance, language or any other discipline, but not if it j eopardizes breadth.

  In college it's appropriate to think about specialization. Getting real e x pertise in an area of interest can lead to success. Graduate school is one way t o get specialized knowledge. Choosing a specialty isn't something high school s t udents should worry about. They should worry about getting a strong academic sta rt.

  There's not a perfect correlation between attitudes in high school and su c cess in later life, of course. But it's a real mistake not to take the opportun i ty to learn a huge range of subjects, to learn to work with people in high schoo l, and to get the grades that will help you get into a good college.

  TEXT F

  First read the following question.

  32. The passage focuses on ___.

  A. the history and future of London

  B. London's manufacturing skills

  C. London's status as a financial centrer

  D. the past and present roles of London

  Now go through Text F quickly and answer question 32.

  What is London for? To put the question another way, why was London, by 190 0, incomparably the largest city in the world, which it remained until the bomba rdments of the Luftwaffe? There could be many answers to this question, but any history of London will rehearse three broad explanations. One is the importance of its life as a port. When the Thames turned to ice in February 1855,50,000 men were put out of work, and there were bread riots from those whose liveliboods h ad been frozen with the river. Today, the Thames could be frozen for a year with out endangering the livelihoods of any but a few pleasure-boatmen. 

  The second major cause of London's wealth and success was that it was easi l y the biggest manufacturing centre in Europe. At the beginning of the Industrial Revolution, Dutch looms and the stocking knitting frame were first pioneered in London. The vast range of London's manufacturing skills is another fact; almos t any item you can name was manufactured in London during the days of its prosper ity. In 1851, 13.75 percent of the manufacturing work-force of Great Britain was based in London. By 1961, this had dramatically reduced. By 1993, there were a mere 328,000 Londoners engaged in manufacturing. In other words, by our own time s, two of the chief reasons for London's very existence-its life as a pert and as a centre of manufacture-had dwindled out of existence.

  London's third great function, since the seventeenth century, has been tha t of national and international bourse: the exchange of stocks and shares, bankin g, commerce and, increasingly, insurance. Both In wood and Francis Sheppard, in London: A history, manage to make these potentially dry matters vivid to the gen eral reader, and both authors assure us that "The City" in the financial sense i s still as important as ever it was. Both, however, record the diminution of the City as an architectural and demographic entity, with the emptying of many city offices (since the advent of the computer much of the work can be done anywhere ) and the removal of many distinctive landmarks.

  TEXT G

  First read the following question.

  33. The primary purpose of the passage is to ___.

  A. discuss the impact of the internet

  B. forecast the future roles of the bookstore

  C. compare the publisher with the editor

  D. evaluate the limitations of the printed page

  Now go through TEXT G quickly and answer question 33.

  Since the advent of television people have been prophesying the death of the book. Now the rise of the World Wide Web seems to have revived this smolderi ng controversy from the ashes. The very existence of paper copy has been brought into question once more. It might be the bookstore, rather than the book itself, that is on the br ink of extinction. Many of you will have noted tom of bookseller websites poppin g up. They provide lists of books and let you read sample chapters, reviews from other customers and interviews with authors.

  What does all this mean? Browsing a virtual bookstore may not afford you the same dusty pleasure as browsing round a real shop, but as far as service, pr ice and convenience are concerned there is really no competition. This may chang e before long, as publishers' websites begin to offer direct access to new publ ications.

  Perhaps it is actually the publisher who is endangered by the relentless advance of the Internet. There are a remarkable number of sites republishing tex ts online——an extensive virtual library of materials that used to be handled pri marily by publishing companies.

  From the profusion of electronic-text sites available, it looks as if thi s virtual library is here to stay unless a proposed revision to copyright law ta kes many publications out of the public domain. However, can electronic texts st ill be considered books?

  Then again, it might be the editor at risk, in danger of being cut out of the publishing process. The Web not only makes it possible for just about anyon e to publish whatever they like whenever they like-there are virtually no costs involved. The editors would then be the millions of Internet users. And there is little censor ship, either.

  So possibly it is the printed page, with its many limitations, that is pe rishing as the implications of new technologies begin to be fully realized. Last year Stanford University published the equivalent of a 6,000 page Business Engl ish dictionary, online. There seem to be quite obvious benefits to housing these multi-volume reference sets on the Web. The perceived benefits for other books, such as the novel, are perhaps less obvious.

  TEXT H

  First read the following question.

  34. The reviewer's attitude towards the books is ___.

  A. ambiguous B. objective C. doubtful D. ho stile

  Now go through TEXT H quickly and answer question 34.

  The 1990s have witnessed a striking revival of the idea that liberal democr atic political system are the best basis for international peace. Western states men and scholars have witnessed worldwide process of democratization, and tend t o see it as a sounder basis for peace than anything we have had in the past.

  Central to the vision of a peaceful democratic world bas been the proposit ion that liberal democracies do not fight each other; that they may and frequent ly do get into fights with illiberal states, but not with other countries that a re basically similar in their political systems. The proposition appeals to poli tical leaders and scholars as well.

  Yet it is doubtful whether the proposition is strong enough to bear the va st weight of generalization that has been placed on it. Among the many difficult ies it poses, two stand out: first there are many possible exceptions to the rul e that democracies do not fight each other; and second, there is much uncertaint y about why democracies have, for the most part, not fought each other.

  Liberal Peace, Liberal War: American politics and international security b y John M. Owen is an attempt to explain the twin phenomena of liberal peace (why democracies do not fight each other) and liberal war (why they fight other sta tes, sometimes with the intent of making them liberal).

  Owen's analysis in the book strongly suggests that political leaders on a l l sides judged a given foreign country largely on the basis of its political sys tem; and this heavily influenced decisions on whether or not to wage war against it. However, be also shows that military factors, including calculations of the cost of going to war, were often influential in tipping the balance against war . In other words, democratic peace does not mean the end of power politics.

  Owen hints at, but never addresses directly, a sinister aspect of democrat ic peace theory: its assumption that there would be peace if only everybody else was like us. This can lead only too easily to attempts to impose the favoured s ystem on benighted foreigners by force-regardless of the circumstances and sensi bilities that make the undertaking hazardous, Owen's central argument is not st r engthened by the occasional repetition nor by the remorselessly academic tone of the more theoretical chapters. However, most of the writing is succinct; the hi storical accounts are clear and to the point; and the investigation of the causa l links between liberalism and war is admirably thorough.

  There are several grounds on which the book's thesis might be criticized. The most obvious is that some twentieth-century experience goes against the argu ment that liberal states ally with others, above all, because they perceive them as fellow liberals. In our own time, several liberal democracies have maintaine d long and close relations with autocracies. However, Owen's argument for a deg r ee of solidarity between liberal states provides at least part of the explanatio n for the continuation and even expansion of NATO in the post-Cold War era.

  TEXT I

  First read the following questions.

  35. In ___, the table of contents of the magazine was placed on its back cover.

  A. 1922 B. 1948 C. the 1930s D. the 1960s

  36. The magazine was criticized for failing to ___.

  A. appeal to the young B. attract old people C. interest readers aged 47 D. captivate rea ders in their 50s

  Now go through TEXT I quickly and answer questions 35 and 36.

  New York-Reader's Digest, the most widely read magazine in the world, will get a new look in a bid to attract younger readers, Reader' Digest Association Inc. announced on March 29. Beginning with the May issue, the world's largest- circulation magazine will move its table of contents off the front cover to mode rnize its look and make it easier for readers to navigate, editor in chief Chris top her Willcox said. "When you have the table of contents on the cover, it limits w hat you can say about what's in the magazine, "Willcox said. The magazine's f ami liar table of contents will be replaced with a photograph. The small size and fo cus of the editorial content will be unchanged, publisher Gregory Coleman said. "It will be a much more visual magazine, with more photography and less illustr ation," he said in an interview.

  Reader's digest was first published in 1922, with line drawings on the c o vers, and in the 1930s began listing the contents on the front. For a couple of years in the 1960s, Willcox said,the table of contents was shifted to the back c over. The May issue will feature a cover photo of a woman firefighter in San Fra ncisco for an except from a new book," Fighting Fire. "The names of a few arti cl es are listed on the cover, but the full table of contents will be on papes 2 an d 3. The issue began reaching subscribers on April 10 and will be on newsstands two weeks later. All 48 of the Digest's worldwide editions-27 million copies in 19 languages-are making the change. Publisher Gregory Coleman said he expe cted the redesign to boost advertising sales. "We've done a lot of research, a nd have tested the concept in the US, Sweden, and New Zealand," Coleman said.

  The move comes as Reader's Digest Association Inc. has struggled to boost profits. But industry analysts said its problems stretch beyond changes that wer e needed at the magazine. Publishing industry executives and Wall Street analyst s have criticized the magazine for failing to attract the next generation of rea ders. The company says its average reader is about 47,the same as the age for th e weekly new magazines, "They've been looking for ways to make the magazine a li ttle bit more the '90s than the '50s," said Doug Arthur at Morgan Stanley Dea n W itter & Co. "The company has to be addressing the response rate on its direct m a rketing campaign, "where its main problems lie. The company earned USD 133.5 mi l lion on sales of USD 2.8 billion in the year which ended last June. But it said, when it reported results, that profits would fall in the current year.

  In answer to a question, Coleman said the redesign was not done because of advertisers, although they were enthusiastic about the changes. "This is being done from a reader-driven standpoint, "he said. 

  TEXT J

  First read the following questions.

  37. Words in both the OWF and Longman Activator are ___.

  A. listed according to alphabetical order

  B. listed according to use frequency

  C. grouped according to similarities only

  D. grouped according to differences only

  38. To know the correct word for "boiling with a low heat", you will pr obably turn to first ___.

  A. page 10 B. page 99 C. page 100 D. page 448

  Now go through TEXT J quickly and answer questions 37 and 38.

  The Oxford Wordfinder (OWF)is a "production dictionary" designed for learn er s of English at Intermediate level and above, It is a useful tool with which to discover and encode (produce) meaning, rather than just to simply check the mean ing, grammar and pronunciation of words. The OWF encourages a reversal of the tr aditional role of the language learners' dictionary, which is normally to help decode and explain aspects of words that appear in a text.

  The OWF is based upon similar lines to the ground breaking Longman Activato r in that words in each dictionary are not simply listed in alphabetical order. Instead, they are grouped according to their similarities and differences in bot h meaning and use. Twenty-three main groups of 630 "keywords" (concepts) in al ph abetical order, assist the learner in exploring semantic areas such as: "People " , "Food and drink", and "Language and Communication". Each of these rather l arge areas contains cross-referencing in order to provide further helpful lexical in formation. Some of the keywords helpfully direct the learner to another keyword. Most keywords, however, have an index that shows how lexical items and their re lated terms are organized. Other keywords point to smaller sub-section headings whilst a few contain sections labeled "More", which deal with less frequently occurring vocabulary.

  The majority of words in the OWF are grouped together because they are clea rly related in meaning. Examples include: rucksack, "suitcase", trunk and hol d- all, on page 28, under the keyword "Bag". Other words are grouped together bec au se statistically they tend to "collocate", i.e. appear in English very near, i f not next to each other. The reader would, more often than not, find them in the same sentence or phrase. Examples include those for "butter", "spread" and "melt ", and those for Television on page 448: "watching", "turn on/off" and "pr ogramme".

  The OWF is an ideal supplementary resource for learners to engage in word-b uilding activities during topic based lessons. How is it best used? Let's say t h e learner wishes to know the correct word for "boiling with a low heat". The i nt ermediate learner, who will probably begin her search under "Cook" on page 99, l ocates the sub-section: "heating food in order to cook it" on page 100,then th e further sub-section "cooking food in water" and finally finds the definition f ol lowed by the word:-to boil slowly and gently: simmer. With the help of the OWF teachers could design a variety of such vocabulary exercises for a class, or eve n go on to designing a vocabulary-based syllabus.

  Definitions in the OWF are, as with all good dictionaries, concise but cle ar. They are obviously written according to a controlled defining vocabulary. Li nguistic varieties are also taken into consideration: formal/in formal labels ar e provided and, where it occurs, American English (AmE) is pointed out, e. g. fo r alcohol, liquor in AmE on page 10. The OWF also contains many drawings that ou tline meaning where words could not possibly do so or would require too much spa ce. Items chosen for inclusion in the OWF, along with example phrases outlining meaning are, it is assumed, based on evidence of frequency from a carefully cons tructed linguistic corpus, although this is not made clear.

  TEXT K

  First read the following questions.

  39. Students who wish to take courses in Dutch or French ___.

  A. should pass the TOEFL test first

  B. must speak Dutch or French fluently

  C. may receive language training

  D. must have a good command of English

  40. Belgian universities do NOT offer courses on ___.

  A. medical sciences

  B. computer science

  C. political and social sciences

  D. archaeology and art sciences

  Now go through TEXT K quickly and answer questions 39 and 40.

  To qualify to study in Belgium, it is essential to meet relevant requireme nts in (1) academic credentials, (2)linguistic skills, (3) academic objectives and (4) financial resources. Let us review these four points:

  1. Academic credentials

  Equivalence and admissibility of degrees will be assessed according to Belgian l aw and individual university regulations. Please submit a copy of your degree wi th a translation to the chosen university's admission board. 

  2. Language skills

  Chinese students who wish to follow courses in Dutch or French must realize that a superficial knowledge of the language will not do. The ability to speak Dutch or French is imperative in order to follow lectures and to pass examinations. A preparatory year of language instruction is available in some universities for already enrolled students. Please apply for information at the university of you r choice. Students who wish to attend lectures in English (post-academic trainin g international courses)must of course have a good command of that language. Uni versities will inform you about their individual TOEFL requirements.

  3. Programmes

  Belgian universities offer basic academic courses, advance academic training cou rses, doctoral programmes, post-academic training and various international stud y programmes (Master's) in the field of technology, law, economics and applied e conomics, political and social sciences, dentistry, pharmaceutical sciences, lan guage and literature/history, archaeology and art sciences, psychology and educa tional sciences, medical sciences, engineering and applied biological sciences.

  4. Financing

  Although precise determination of study and living expenses depends on individua l life style, one can assess that about 350,000 Belgian Francs (BEF)( about 88,0 00 RMB) is necessary for one year's study. This amount should include books, ho u sing, food, transport, and health insurance. It does not include registration fe es which can vary from about 25,000 BEF for a student under scholarship to 290,0 00 BEF for a self-financing student, according to the chosen study program.

  试卷二 (120 min)

  Part Ⅳ Translation (60 min)

  SECTION A CHINESE TO ENGLISH

  Translate the following underlined part of the text into English. Write your tra nslation on ANSWER SHEET THREE.

  中国科技馆的诞生来之不易。与国际著名科技馆和其他博物馆相比,它先天有些不足, 后天也常缺乏营养,但是它成长的步伐却是坚实而有力的。它在国际上已被公认为后起之秀 。

  世界上第一代博物馆属于自然博物馆,它是通过化石、标本等向人们介绍地球和各种生 物的演化历史。第二代属于工业技术博物馆,它所展示的是工业文明带来的各种阶段性结果 。这两代博物馆虽然起到了传播科学知识的作用,但是,它们把参观者当成了被动的旁观者 。

  世界上第三代博物馆是充满全新理念的博物馆。在这里,观众可以自己去动手操作,自 己细心体察。这样,他们可以更贴近先进的科学技术,去探索科学技术的奥妙。

  中国科技馆正是这样的博物馆!它汲取了国际上一些著名博物馆的长处,设计制作了力 学、光学、电学、热学、声学、生物学等展品,展示了科学的原理和先进的科技成果。

  SECTION B ENGLISH TO CHINESE

  Translate the following text into Chinese. Write your translation on ANSWER SH EET THREE.

  If people mean anything at all by the expression "untimely death", they m us t believe that some deaths nm on a better schedule than others. Death in old age is rarely called untimely-a long life is thought to be a full one. But with th e passing of a young person, one assumes that the best years lay ahead and the m easure of that life was still to be taken.

  History denies this, of course. Among prominent summer deaths, one recalls those of MariLarry Monroe and James Deans, whose lives seemed equally brief and co mplete. Writers cannot bear the fact that poet John Keats died at 26, and only h alf playfully judge their own lives as failures when they pass that year. The id ea that the life cut short is unfulfilled is illogical because lives are measure d by the impressions they leave on the world and by their intensity and virtue.

  Part Ⅴ Writing (60 min)

  Some people simply see education as going to schools or colleges, or as a m eans to secure good jobs; most people view education as a lifelong process. In y our opinion, how important is education to modem man?

  Write a composition of about 300 words on the following topic:

  EDUCATION AS A LIFELONG PROCESS

  In the first part of your writing you should present your thesis statement, and in the second part you should support the thesis statement with appropriate deta ils. In the last part you should bring what you have written to a natural conclu sion or a summary.

  Marks will be awarded for content, organization, grammar and appropriacy. Failur e to follow the above instructions may result in a loss of marks.

  Write your composition on ANSWER SHEET FOUR.

2000年英语专业四级试题及答案

Part Ⅱ DICTATION [15 MIN.]

  Listen to the following passage. Altogether the passage will be read to you four times. During the first reading, which will be read at normal speed, listen and try to understand the meaning. For the second and third readings, the passage w ill be read sentence by sentence, or phrase by phrase, with intervals of 15 seco nds. The last reading will be read at normal speed again and during this time yo u should check your work. You will then be given 2 minutes to check through your work once more.Please write the whole passage on ANSWER SHEET TWO

  Part Ⅲ LISTENING COMPREHENSION [20 MIN.]

  In Sections A, B, and C you will hear everything ONCE ONLY. Listen carefully an d then answer the questions that follow. Mark the best answer to each question o n your answer sheet.

  Section A STATEMENT

  In this section you will hear nine statements. At the end of the statement you w ill be given 10 seconds to answer each of the following nine questions.1.   What is said about Harry's brother?

  A.  He is happy with his job.

  B.  He is a very ambitious man.

  C.  He is too ambitious to be an engine driver.

  D.  He doesn't like to be an engine driver. 

  2.   What do you learn about Ms. Ellis?

  A.  She has been waiting.

  B.  She is examining her patient.

  C.  She is seeing her doctor.

  D. She wouldn't mind waiting. 

  3.   Joan is probably a___.

  A.  nurse        B. doctor       C.  lawyer     D. saleswoman 

  4.   The speaker sees Mary wear ___ different silk scarves in a wee k.

  A.  2      B.5      C.7     D. 6 

  5.   Where will the passengers change trains to go to Gilford?

  A.  East Croydon.    B.Victoria.    C.  Southeast.   D.Red Hill. 

  6.   What is the speaker probably doing?

  A.  Interviewing a clerk.

  B.  Writing a job ad.

  C.  Dismissing a clerk.

  D.  Making inquires 

  7.   What does the speaker mean?

  A.  Emily is neither honest nor trustworthy.

  B.  Emily used to be honest only.

  C.  Emily used to be trustworthy only.

  D.  Emily is more than honest and trustworthy. 

  8.   When does the next train leave?

  A.  6:56.     B. 7:00.    C.7:28.    D.8:38.

  9.   What was wrong with Malcolm?

  A.  He had trouble working hard.

  B.  He didn't know where to go.

  C.  He never went anywhere.

  D.  He worked hard but never succeeded. 

  SECTION B CONVERSATION

  In this section, you will hear eight short conversations between two speakers. A t the end of each conversation you will be given 10 seconds to answer each of th e following eight questions.

  10.  What's the probable relationship between the two speakers?

  A.  Teacher and student.

  B.  Doctor and patient.

  C.  Lawyer and client.

  D.  Boss and secretary. 

  11.  What is the weather usually like in November?

  A.  Hotter than the present weather.

  B.  More humid than the present weather.

  C.  Drier than the present weather.

  D.  Cooler than the present weather. 

  12.  What conclusion can we draw from this conversation?

  A.  Public buses are fast and cheap.

  B.  Parking is becoming a big problem.

  C.  Subway trains are even safer than taxis.

  D.  Taxis are more convenient than buses. 

  13.  What are the two speakers talking about?

  A.  Fixing the woman's computer.

  B.  Ordering some new parts by Friday.

  C.  Getting the new parts ready by Friday.

  D.  Sending the woman's computer for repair. 

  14.  What can we learn from the conversation?

  A.  Neither of them has a favourable opinion of the service.

  B.  The woman is having a terrible time serving in the restaurant.

  C.  Both agree it's time for the restaurant to fire some staff.

  D.  The man thinks the restaurant is all right, but the woman doesn't. 

  15.  Who will pay for the call?

  A.  The man. B.  The operator. C.  The man's sister. D.  The man and his sister. 

  16.  What does the man think of the woman's choice of clothing?

  A.  He thinks her choice is good.

  B.  He thinks her choice is terrible.

  C.  He doesn't like the colour.

  D.  He doesn't like the style. 

  17.  What happened to Mr. Runt's project?

  A.  It was fairly successful.

  B.  It was hard and futile.

  C.  It failed for lack of fund.

  D.  It stopped for lack of land. 

  SECTION C NEWS BROADCAST

  Question 18 is based on the following news. At the end of the news item, you wil l be given 1O seconds to answer the question.

  Now listen to the news.

  18.  According to the news, NATO and Russia___.

  A.  have finalized a charter on their new relationship

  B.  still have differences in military and political issues

  C.  will hold a fifth round of talks in Luxembourg

  D.  made no progress in this round of talks 

  Questions 19 and 20 are based on the following news. At the end of the news item , you will be given 20 seconds to answer the two questions.

  Now listen to the news.

  19.  ___ people were killed during the air crash.

  A. 61       B. 51      C. 41      D. 10

  20.  According to the news, the plane crashed___.

  A.  shortly before it landed

  B.  minutes after it took off

  C.  after it cleared the mountains

  D.  at the foot of the mountains 

  Questions 21 and 22 are based on the following news. At the end of the news item , you will be given 20 seconds to answer the two questions.

  Now listen to the news.

  21.  Which of the following is NOT listed as a terrorist group by the US ?

  A.  The pro-Iranian Hezbollah.  B.  The Palestinian group Hamas. C.  The Irish Republican Army.  D.  The Basque separatist group ETA. 

  22.  The affected groups will be prevented from___.

  A.  entering the United States legally

  B.  freezing US financial assets abroad

  C.  receiving support from other countries

  D.  giving weapons to other terrorist groups 

  Question 23 is based on the following news. At the end of the news item, you wil l be given 1O seconds to answer the question.

  Now listen to the news.

  23.  Israeli Prime Minister Benjamin Netanyahu___.

  A.  has been prosecuted by the Justice Ministry

  B.  may be prosecuted by the Justice Ministry

  C.  has been prosecuted by the police

  D.  will be prosecuted on Monday 

  Questions 24 and 25 are based on the following news. At the end of the news item , you will be given 20 seconds to answer the two questions.

  Now listen to the news.

  24.  The winners of the reported elections are___.

  A.  the left-wing Conservatives

  B.  the left-wing Socialists

  C.  the centre-right Conservatives

  D.  the centre-right Socialists 

  25.  If the left secures the parliamentary majority,___.

  A.  Chirac will share his presidential power with Jospin

  B.  Jospin will share his prime ministerial power with Chirac

  C.  Jospin will become prime minister, and Chirac will remain

  D. Jospin will become prime minister, and Chirac will resign 

  完型填空

  Part Ⅳ CLOZE [15 MIN.]

  Decide which of the choices given below would best complete the passage if inserted in the corresponding blanks. Mark the best choice for each blank on your answer sheet.

  The difference between a liquid and a gas is obvious ( 26 )    the conditions of temperature and pressure commonly found at the sur face of the Earth. A liquid can be kept in an open container and ( 27 ) it to the level of a free surface. A gas forms no free surface but ( 28 ) to diffuse throughout the space available; it must  ( 29 ) be kept in a closed container, as ( 30 )a planet's atmosphere. The distinction was a prominent feature of early theories ( 31 )the phases of matter. In the nineteenth century, for example, one theory maintained that a liquid could be "dissolved" in a vapor without losing its identity, and another theory held that the two phases are ( 32 ) different kinds o f molecules(分子). The theories now prevailing ( 33 ) a quit e different approach by emphasizing what liquids and gases have in common. They are both forms of matter that have no permanent structure, and they both flow ea sily. They are fluids.  

  The ( 34 ) similarly of liquids and gases becomes clear ly apparent when the temperature and pressure are raised somewhat.( 35 ) a closed container partially filled with a liquid is heated. The li quid expands or ( 36 ), becomes less dense; some of it evapor ates.( 37 ), the vapor above the liquid surface becomes dense r as the evaporated molecules are added to it. The combination of temperature an d pressure ( 38 ) the densities become equal is ( 3 9 )  the critical point. Above the critical point the liquid and the gas can no longer be ( 40 ); there is  a single, undifferentiated fluid phase of uniform density.

  26.  A. in     B. on C. under    D. beyond 

  27.  A. fills    B. be filled   C. filling   D. to fill 

  28.  A. intends  B. tends  C. inclines   D. contends 

  29.  A. however  B. nevertheless  C. so   D. therefore 

  30. A. in the event of        B. in the case of C. with a view to    D. with reference to 

  31.  A. having described     B. described C. describing    D. to have described 

  32.  A. made up of      B. consisted of C. constituted of      D. made from 

  33.  A. apply    B. adapt C. take    D. conduct 

  34.  A. elementary    B. crucial  C. rudimentary   D. fundamental 

  35.  A. Suppose        B. To suppose  C. Being supposed   D. Supposed 

  36.  A. in a word     B. in the meantime C. in other words     D. in that case 

  37.  A. Similarly   B. In contrast  C. Furthermore D. Instead 

  38.  A. on that   B. on which  C. at that    D. at which 

  39.  A. known  B. defined  C. called     D. referred to 

  40.  A. classified  B. recognized  C. categorized   D. distinguished

  选择

  Part Ⅴ GRAMMAR AND VOCABULARY [15 MIN.]

  There are twenty-five sentences in this section. Beneath each sentence there are four words or phrases marked A, B, C and D. Choose one word or phrase that best completes the sentence. Mark your answers on your answer sheet.

  41.  Acute hearing helps most animals sense the approach of thunderstorm s long before people___.

  A. do     B. hear       C. do them     D. hearing it 

  42. This is an illness that can result in total blindness ___ left u ntreated.

  A. after   B. if C. since D.unless 

  43.  The central provinces have floods in some years, and ___.

  A. drought in others   B. droughts are others C. while other droughts D. others in drought 

  44.  Do help yourself to some fruit,___ you?

  A. can't B. don't C. wouldn't  D. won't 

  45.  There___ nothing more for discussion, the meeting came to an e nd half an hour earlier.

  A. to be      B. to have been    C. being   D. be

  46.  My mother can't get ___ because she has rheumatism (风湿病). 

  A. about     B.on     C. through     D. in

  47. I was very much put ___ by Mark's rude behavior; it really annoy ed me.

  A.over    B.off C.up     D.by 

  48.  You ___ Jim anything about it. It was none of his business.

  A. needn't have told        B. needn't tell    C. mustn't have told        D. mustn't tell 

  49.  All of us would have enjoyed the party much more if there___ q uite such a crowd of people there.

  A. weren't  B. hasn't been   C. hadn't been    D. w ouldn't be 

  50. Firms that use computers have found that the number of staff ___ quality control can be substantially reduced.

  A.whose   B.as  C.what    D.that 

  51. ___ at in this way, the present economic situation doesn't seem so gloomy.

  A. Looking    B. Looked   C. Having looked   D. To look 

  52. Many people are ___ to insect bites, and some even have to go to hospital.

  A. insensitive    B. allergic     C. sensible    D. infected 

  53.  When you're driving on a motorway, you must obey the signs telling you to get into the right ___.

  A.way B.track   C.road    D.lane 

  54.  The motorist had to ___to avoid knocking the old woman down in the middle of the road.

  A. swerve  B. twist   C. depart  D. swing 

  55.  In winter drivers have trouble stopping their cars from ___ on icy roads.

  A. skating  B. skidding C. sliding  D. slipping 

  56.  This project would ___ a huge increase in defense spending.

  A. result  B. assure  C. entail  D. accomplish 

  57.  The chances of a repetition of these unfortunate events are ___ indeed.

  A. distant  B. slim   C. unlikely D. narrow 

  58.  We should make a clear ___ between 'competent' and 'proficient' for the purposes of our discussion.

  A. separation   B. division C. distinction   D. diffe rence 

  59.  In the present economic ___ we can make even greater progress than previously.

  A. air    B. mood   C. area   D. climate 

  60.  Rite of Passage is a good novel by any standards;___, it shoul d rank high on any list of science fiction.

  A. consistently  B. consequently  C. invariably   D. fortunately 

  61.  The diversity of tropical plants in the region represents a seeming ly___ source of raw materials, of which only a few have been utilized.

  A. exploited    B. controversial  C. inexhaustible  D. remarkable 

  62.  While he was in Beijing, he spent all his time ___ some import ant museums and buildings.

  A. visiting B. traveling C. watching D. touring 

  63.  You must let me have the annual report without ___ by ten o'cl ock tomorrow morning.

  A. failure   B. hesitation  C. trouble   D. fail

  64.  As the director can't come to the reception, I'm representing the c ompany

  A. on his account B. on his behalf  C. for his part  D. in his interest 

  65. Dreams are___ in themselves, but when combined with other data, they can tell us much about the dreamer.

  A. uninformative           B. startling C. harmless  D. uncontrollable

  阅读理解 A

  Part Ⅵ READING COMPREHENSION [30 MIN.]

  SECTION A READING COMPREHENSION[25 MIN.]

  In this section there are four passages followed by questions or unfinished statements, each with four suggested answers marked A, B, C and D. Choose the one that you think is the best answer.

  Mark your answers on your answer sheet.

  TEXT A

  Clearly if we are to participate in the society in which we live we must communicate with other people. A great deal of communicating is performed on a person-t o-person basis by the simple means of speech. If we travel in buses, buy things in shops, or eat in restaurants, we are likely to have conversations where we give information or opinions, receive news or comment, and very likely have our views challenged by other members of society.

  Face-to-face contact is by no means the only form of communication and during the last two hundred years the art of mass communication has become one of the dominating factors of contemporary society. Two things, above others, have caused t he enormous growth of the communication industry. Firstly, inventiveness has led to advances in printing, telecommunications, photography, radio and television. secondly, speed has revolutionised the transmission and reception of communications so that local news often takes a back seat to national news, which itself i s often almost eclipsed by international news.

  No longer is the possession of information confined to a privileged minority. In the last century the wealthy man with his own library was indeed fortunate, but today there are public libraries. Forty years ago people used to flock to the cinema, but now far more people sit at home and turn on the TV to watch a program me that is being channelled into millions of homes. Communication is no longer merely concerned with the transmission of information. The modem communication industry influences the way people live in society and broadens their horizons by allowing access to information, education and entertainment. The printing, broadcasting and advertising industries are all involved with informing, educating and entertaining.

  Although a great deal of the material communicated by the mass media is very valuable to the individual and to the society of which he is a part, the vast modem network of communications is open to abuse. However, the mass media are with us for better, for worse, and there is no turning back.

  66.  In the first paragraph the writer emphasizes the___ of face-t o-face contact in social settings.

  A. nature    B. limitation    C. usefulness  D. creativity 

  67.  It is implied in the passage that___.

  A.  local news used to be the only source of information.

  B.  local news still takes a significant place.

  C.  national news is becoming more popular.

  D.  international news is the fastest transmitted news. 

  68.  Which of the following statements is INCORRECT?

  A.  To possess information used to be a privilege.

  B.  Public libraries have replaced private libraries.

  C.  Communication means more than transmission.

  D.  Information influences ways of life and thinking. 

  69.  From the last paragraph we can infer that the writer is___.

  A.  indifferent to the harmful influence of the mass media

  B.  happy about the drastic changes in the mass media

  C.  pessimistic about the future of the mass media

  D.  concerned about the wrong use of the mass media 

  TEXT B

  The men and women of Anglo-Saxon England normally bore one name only. Distinguishing epithets were rarely added. These might be patronymic, descriptive or occupational. They were, however, hardly surnames. Heritable names gradually became general in the three centuries following the Norman Conquest in 1066. It was not until the 13th and 14th centuries that surnames became fixed, although for many years after that, the degree of stability in family names varied considerably in different parts of the country.

  British surnames fall mainly into four broad categories: patronymic, occupational, descriptive and local. A few names, it is true, will remain puzzling: foreign names, perhaps, crudely translated, adapted or abbreviated; or artificial names . In fact, over fifty per cent of genuine British surnames derive from place names of different kinds, and so they belong to the last of our four main categories. Even such a name as Simpson may belong to this last group, and not to the first , had the family once had its home in the ancient village of that name. Otherwise, Simpson means "the son of Simon", as might be expected.

  Hundreds of occupational surnames are at once familiar to us, or at least recognisable after a little thought: Archer, Carter, Fisher, Mason, Thatcher, Taylor, to name but a few. Hundreds of others are more obscure in their meanings an d testify to the amazing specialisation in medieval arts, crafts and functions. Such are "Day", (Old English for breadmaker) and "Walker" (a fuller whose job it was to clean and thicken newly made cloth).

  All these vocational names carry with them a certain gravity and dignity, which descriptive names often lack. Some, it is true, like "Long", "Short" or "Little", are simple. They may be taken quite literally. Others require more thinking: their meanings are slightly different from the modem ones. "Black" and "White " implied dark and fair respectively. "Sharp" meant genuinely discerning, alert, acute rather than quick-witted or clever. Place-names have a lasting interest since there is hardly a town or village in all England that has not at some time given its name to a family. They may be picturesque, even poetical; or they may be pedestrian, even trivial. Among the commoner names which survive with relatively little change from old-English times are "Milton"(middle enclosure) and "Hilton"(enclosure on a hill).

  70.  Surnames are said to be ___ in Anglo-Saxon England.

  A. common    B. vocational    C. unusual   D. descriptive 

  71.  We learn from the first paragraph ___ for many years after the 13th and 14th centuries.

  A. family names became descriptive and occupational

  B. people in some areas still had no surnames

  C. some people kept changing their surnames

  D. all family names became fixed in England 

  72.  "Patronymic" in the second paragraph is closest in meaning to "formed from ___.

  A. the name of one's father"   B. the family occupation" C. one's family home"   D. one's family history" 

  73.  Which of the following sentences is an opinion rather than a fact?

  A. hundreds of occupational names are at once familiar to us.

  B. "Black" and "White" implied "dark" and "fair" respectively.

  C. Vocational names carry with them a certain gravity and dignity.

  D. Every place in England has given its name to a family. 

  TEXT C

  Since the early 1930s, Swiss banks had prided themselves on their system of banking secrecy and numbered accounts. Over the years, they had successfully withstood every challenge to this system by their own government who, in turn, ha d been frequently urged by foreign governments to reveal information about the financial affairs to certain account holders. The result of this policy of secrecy was that a kind of mystique had grown up around Swiss banking. There was a widely-held belief that Switzerland was irresistible to wealthy foreigners, mainly because of its numbered accounts and bankers' reluctance to ask awkward questions of depositors. Contributing to the mystique was the view, carefully propagated by the banks themselves, that if this secrecy was ever given up, foreigners would fall over themselves in the rush to withdraw money, and the Swiss banking system would virtually collapse overnight.

  To many, therefore, it came like a bolt out of the blue, when, in 1977, the Swiss banks announced they had signed a pact with the Swiss National Bank (the Central Bank). The aim of the agreement was to prevent to improper use of the country's bank secrecy laws, and its effect was to curb severely the system of secrecy.

  The rules which the banks had agreed to observe made the opening of numbered accounts subject to much closer scrutiny than before. The banks would be required, if necessary, to identify the origin of foreign funds going into numbered and other accounts. The idea was to stop such accounts being used for dubious purposes. Also they agreed not to accept funds resulting from tax evasion or from crime.

  The pact represented essentially a tightening up of banking rules. Although the banks agreed to end relations with clients whose identities were unclear or who were performing improper acts, they were still not obliged to inform on a client to anyone, including the Swiss government. To some extent, therefore, the princ iple of secrecy had been maintained.

  74.  Swiss banks took pride in___.

  A.  the number of their accounts

  B.  withholding client information

  C.  being mysterious to the outsiders

  D.  attracting wealthy foreign clients 

  75.  According to the passage, the widely-held belief that Switzerland w as irresistible to wealthy foreigners was ___ by banks themselves.

  A.  denied    B.   criticized    C.   reviewed  D.  defended 

  76.  In the last paragraph, the writer thinks that___.

  A.  complete changes had been introduced into Swiss banks

  B.  Swiss banks could no longer keep client information

  C.  changes in the bank policies had been somewhat superficial

  D.  more changes need to be considered and made 

  TEXT D

  Coketown was a town of red brick, or of brick that would have been red if the smoke and the ashes had allowed it; but as matters stood it was a town of unnatural red and black like the painted face of a savage. It was a town of machinery an d tall chimneys, out of which smoke trailed themselves for ever and ever. It had a black canal in it, and a river that ran purple with ill-smelling dye, and vas t piles of buildings full of windows where there was a rattling and a trembling all day long, and where the piston of the steam-engine worked monotonously up an d down like the head of an elephant in a state of madness. The town contained several large streets all very like one another, and many small streets still more like one another, inhabited by people equally like one another.

  A sunny midsummer day. There was such a thing sometimes, even in Coketown. Seen from a distance in such weather, Coketown lay covered in a haze of its own. You only knew the town was there, because you knew there could have been no such blotch upon the view without a town.

  The streets were hot and dusty on the summer day, and the sun was so bright that it even shone through the haze over Coketown, and could not be looked at steadily. Workers emerged from low underground doorways into factory yards, and sat on posts and steps, wiping their faces and contemplating coals. The whole town see med to be frying in oil. There was a stifling smell of hot oil everywhere. The atmosphere of those places was like the breath of hell, and their inhabitants was ting with heat, toiled languidly in the desert. But no temperature made the mad elephants more mad or more sane. Their wearisome heads went up and down at the same rate, in hot weather and in cold, wet weather and dry fair weather and foul. The measured motion of their shadows on the walls, was the substitute Coketown had to show for the shadows of rustling woods; while for the summer hum of insects, it could offer all the year round, from the dawn of Monday to the night of S aturday, the whirr of shafts and wheels.

  77.  Which of the following adjectives is NOT appropriate to describe Co ketown?

  A. dull             B. dirty  C. noisy            D. savage 

  78.  From the passage we know that Coketown was mainly a(n) ___town .

  A. industrial     B. agricultural     C. residential  D. commercial 

  79. Only ___ were not affected by weather.

  A.  the workmen         B.   the inhabitants C.  the steam-engines        D.  the rustling woods 

  80.  Which is the author's opinion of Coketown?

  A.  Coketown should be replaced by woods.

  B.  The town was seriously polluted.

  C.  The town had too much oil in it.

  D. The town's atmosphere was traditional.

  Reading Comprehension B

  SECTION B SKIMMING AND SCANNING

  In this section there are seven passages with a total often multiple-choice questions. Skim or scan them as required and then mark your answers on your answer sheet.

  TEXT E

  First read the following question.

  81.  The writer is concerned about___.

  A.  budget housekeeping      B. the retail trade C.  computer skills         D. mental arithmetic 

  Now read Text E quickly and mark your answer on your answer sheet.

  A lot of attention is being given to children who leave school unable to read or write. I think there should be equal concern for those who are unable to cope with simple mental arithmetic -particularly girls. It is often stated that today's children are growing up in a computer world and they don't need the same skills that their grandparents did. But is it any wonder that many young girls trying to cope with budget housekeeping fail for the simple reason they cannot keep accurate checks on their purchases? Shopping in markets is no source of cheap purchasing unless one is able to keep pace with the apparent mental agility of the vendor. Must we face the thought that at some time in the distant future everyone will need to carry in their handbag or pocket one of the miniature calculators?

   TEXT F

  First read the following question.

  82.  This is a letter of___.

  A. reference           B. application  C. inquiry            D. complaint 

  Now read Text F quickly and mark your answer on your answer sheet.

  10 Garden Ave.

  Essex

  The Personnel Officer

  Belgian Medico Ltd.

  P0 Box 920

  Brussels

  Belgium

  5th May 200___ 

  Dear Sir,

  With reference to your advertisement in the "Daily Star", I'd like to apply for the position of translator with your firm. I hold a degree in German and French from the University of London. And I have worked as a translator for the past three years with Watson & Sons, Ltd., manufacturer of laboratory instruments, translating business correspondence from French and German into English. I am 25 years old and unmarried. I enjoy living and working in different countries and I should welcome the chance of moving to Belgium.

  Yours sincerely,

  (Miss) Janet Holbrooke

  TEXT G

  First read the following question.

  83.  The passage is mainly about___.

  A.  loneliness    B.  experience  C.memory     D.  isolation 

  Now read Text G quickly and mark your answer on your answer sheet.

  Loneliness is a curious thing. Most of us can remember feeling most lonely when we were not in fact alone at all, but when we were surrounded by people. Everyone has experienced, at some time, that strong sense of isolation that comes over you when you are at a party or in a room full of happy laughing people. It suddenly seems to you as if everybody knows everybody else, everybody knows what is going on; everybody, that is, except you.

  This feeling of loneliness which can overcome you when you are in a crowd is very difficult to get rid of. People living alone are advised to tackle their loneliness by joining a club or a society, by going out and meeting people.

  TEXT H

  First read the following question.

  84.  The author mainly discusses ___ of public transportation.

  A.  the price            B. the types   C.  the improvement         D. the advantage 

  Now read Text H quickly and mark your answer on your answer sheet.

  The price of public transportation in Beijing has doubled twice since 1989, but it is still a bargain. Using the subway and minibuses used to show class status; now people of all classes take them, while some wealthy prefer taxis or private cars. What a change in just a few years! But there are downfalls to having more cars on the roads. Fortunately, the government is aware of the problem. No-lead gasoline is the only one permitted in the city, and the rest of the country follows. Thousands of trees are planted in an d around the city every year. Children are taught why and how to protect the environment. At the same time, public transportation has marked real progress: buses are everywhere and run frequently. We no longer see those old buses with broke n windows. Instead, there are fast buses, double-decker buses, air-conditioned o r heated buses, all offering a good service.

  TEXT I

  First read the following questions.

  85. Each participating team should at least have ___

  A.  two     B.three    C.four    D.  five 

  86.  Participants can bring along their ___ to the competition.

  A.  Christmas trees B.  Christmas presents C.  festival costumes D.  decoration materials 

  Now read Text I quickly and mark your answers on your answer sheet.

  With Christmas Day around the corner, Hong Kong's Provisional Regional Council announced that a Christmas tree decoration competition will be held on Sunday in conjunction with the ongoing Regional Council Festival. Members of the public are welcome to take part in the competition as families or small groups. Each team should be formed by at least three persons. A total of 99 Christmas trees of 1.5 metres in height will be available for the participating teams to decorate. Participants can bring along their own decoration materials and to use their imagination and creativity to achieve the best results.

  Each participating team can take home the Christmas tree it has decorated as a souvenir. In addition, there will be cash awards for the winners.

  TEXT J

  First read the following questions.

  87.  If you only have time for a half-day trip, which day would you choo se?

  A.  Sunday 23 July. B.  Saturday 15 July. C.  Wednesday 9 August. D.  Saturday 5 August. 

  88.  Which of the following trips offers you the opportunity to see Geor gian architecture?

  A.  Trip One.     B. Trip Two.    C.  Trip Three.  D.  Trip Four. 

  Now read Text quickly and mark your answers on your answer sheet.

  Summer Outings

  Trip One

  Saturday 15 July Stratford-upon-Avon and "Julius Caesar"

  The coach will leave at 9 am, allowing a couple of hours to visit Stratford befo re the performance of "Julius Caesar" at the Royal Shakespeare Theatre. Back aro

  und 7:30 pm.

  Trip Two

  Sunday 23 July  Bath

  The spa town of Bath contains the country's finest Roman ruins, and much elegant Georgian architecture. The coach will depart at 9 am, returning at around 6:30 pm.

  Trip Three

  Saturday 5 August   Stratford-upon-Avon and "The Taming of the Shrew" Another chance to visit Stratford. "The Taming of the Shrew" stars Josie Lawrence in the title role. The coach will leave at 9 am, returning at around 7:30 pm.

  Trip Four

  Wednesday 9 August  Oxford and "A Midsummer Night's Dream" A half-day trip to Oxford. The coach will leave at 2:15 pm, allowing an afternoon to see the sights before one of Shakespeare's most popular plays at the Playhouse Theatre. Back after the show.

  TEXT K

  First read the following questions.

  89.  Which nation is thought to be business-minded?

  A.  The Dutch.       B.   The Italians.  C.  The British.       D. The Germans. 

  90.  The opinions seem to be most divided on___.

  A.  the Germans      B.   the Dutch   C.  the French   D.  the British 

  Now read Text K quickly and mark your answers on your answer sheet.

  Some of the data from a survey on national stereotypes in some European countries is summarized below:

  Germans Liked themselves best of all. Most Europeans agreed that the Germans had the highest proportion of good qualities. They considered themselves very tolerant, but nobody else did.

  French Not really admired by anyone except the Italians. Other Europeans found them conservative, withdrawn, brilliant, superficial. Also, not very friendl y.

  British Mixed reactions. Some found them calm, reserved, open- minded, others thought they were insular and superior. The British most admired the Dutch.

  Italians    Generally considered by everyone to be lazy and untrustworthy, an d the Italians agreed! Most also found them to be charming, hospitable and noisy

  . The Italians admired the French. Hardly anyone loved the Italians except the French.

  Dutch    Most admired people in Europe-except by their neighbours-the Belgians. Everyone agreed that the Dutch are hardworking, thrifty, good-natured, tolerant and business-minded.

  Part Ⅰ WRITING [45 MIN.]

  SECTION A COMPOSITION

  Write on ANSWER SHEET ONE a composition of about 150 words on the following topic: College life should be varied and colourful. And extracurricular activities are an important aspect of it. However, at present, there is much room for improvement in this regard. Write an article to the university radio entitled:

  The Importance of Extracurricular Activities 

  In the first part of your article you should clearly present your view, and in the second part you should support your opinion with appropriate details. In the last part you should bring what you have written to a natural conclusion or summary. Marks will be awarded for content, organization, grammar and appropriacy. Failure e to follow the above instructions may result in a loss of marks.

  SECTION B NOTE-WRITING[10 MIN.]

  Write on ANSWER SHEET ONE a note of about 50-60 words based on the following situation:

  You've read on the notice board that the university library is looking for a par t-time library assistant who can work at weekends. You think that your classmate , George, is a suitable person for this vacancy. Write him a note, telling him w hat you know about the vacancy and trying to persuade him to go for an interview Marks will be awarded for content organization, grammar and appropriacy.

2001年英语专业八级试题及答案

PART Ⅰ LISTENING COMPREHENSION

  SECTION A TALK

  The World Bank is one of the major channels through which development aid is passed from industrial west to the poor and developing nations of the world. Its scale of operations is vast, which is why its lending program exceeds 7 billion a year, and its work force numbers about 4500. In the last decade important changes have taken place in the size of the bank's operations and in the emphasis of its lending policies. What immediately strikes anyone looking at the lending figures over the last 10 years is the tremendous expansion in the bank's loan program. This has increased from 1 billion to nearly 7 billion. The figure includes hard loans, which are made at the current rate of interest, and soft loans, which are allocated to poor countries at concessionary rates, and usually channel led through the bank's affiliate-the International Development Association.

  In deciding the emphasis of its lending policy, the bank has had to take into account the population explosion which is occurring in many poor countries of the world. It is a fact that the fertility rate of the poor countries is often very high. This is one of the main reasons for these countries remaining poor. Unfortunately, wide-ranging country section programs do not usually reduce this r ate because this was a strong and deeply rooted tradition among people in these countries to have big families. What the bank discovered was that there was a link between economic and social development on the one hand, and reduction of fertility rate on the other. Thus by improving basic health services, by introducing better nutrition, by increasing literacy, and by promoting more even income distribution in a poor country, a lower and more acceptable fertility rate will be achieved. This advanced thinking persuaded the bank to change its overall lend ing strategy, where previously it concentrated on the big infrastructure project s, such as dams, roads and bridges. It begun to switch to projects which directly improve the basic services of the country. There was a shift, if you like, from building dams to digging water holes to provide clear water.

  A second reason for the change of approach was that the bank has learned a big lesson from projects financed in the 1960s. Many of its major capital investment had scarcely touched the lives of urban and rural poor, nor have they created much employment. The project did not have the trigger-down effect they have in industrialized countries. Instead the huge dams, steel-mills, and so on were left as monuments to themselves. This redirection of its lending has meant that the bank has tended to support labour intensive activities, rather than capita l intensive ones. Both rural and urban areas, there is a better chance in the first case, that its funds will benefit the bottom 40% of the country's population. 

  The bank is also looking for ways of stimulating the growth of the small businesses in many developing countries since this would create employment opportunities for people with lower incomes. Being such a big, obvious target, the bank has often come under fire. For example, its officials have been taken to task for u sing Concord supersonic aircraft so frequently, about 500 times in one year. Also, the large growth of the organization's personnel has not pleased some critic s . A more substantial criticism has concerned the bank's policy of setting annual target for lending to specified countries. This could lead to the deterioration in quality of loans, some say. One former bank official has said, rather than encourage growth for its own sake, the bank should begin to think of itself less as a foreign aid agency and more of a financial deal-maker, combining official wit h the private resources for specific purposes.

  Finally, some people maintain that the impact of the projects funded by the bank has been modest.  When one looks around the world at regions or countries that have successfully transformed to industrial status, it seems that one should be aware of over-estimating the bank's impact. Take Hong Kong for example. Its changes have come about as a result of trade offensive. The purpose has been to flood western market with low price goods made by capitalist methods of production. The example seems to indicate that some regions can prosper without the bank's aid as well.

  SECTION B CONVERSATION

  W: Well, it seems quite common actually. A lot of people in Australia no w are travelling and taking time off. And when I was actually travelling, I met so many people doing the same thing.

  M: Yeah, yeah, so where did you start off?

  W: Well, I went to New Zealand first. Eh, and got a job in a computer company as  a secretary. And I worked there for 4 months.

  M: Really? You can do that, can you? I mean it's possible for anyone to get a j ob in New Zealand, without being a New Zealander?

  W: No, not everybody, only Australians and New Zealanders can exchange either. Y ou know you can work in either country.'

  M: Right, yeah.

  W: So that was easy. So I worked there for 4 months and raised enough money for the rest of travels really. So from there I went to Indonesia, and travelled around the different islands around Indonesia, Malaysia, Singapore, Thailand, China , Nepal and India. 

  M: What about Indonesia? What did you do? Did you fly mostly between the Islands?

  W: Eh, I did a bit of that, and boats, mainly local boats between the Islands.

  M: What about Singapore? People said it's very very modern. But because it is s o modern, it's      rather boring. Did you find that?

  W: Well, it's difficult to say really. It has different attractions. You know t h e Chinese, Malay, and Hindu communities are there. Each has his own culture and custom, very different from the others. And it's a great big shopping center an d I really enjoy it from that point of view. And it was very clean.

  M: And after, you said you went what, Singapore, Malaysia, Thailand, and then China? That's a great country to travel in, isn't it?

  W: Eh, it was. Yeah, it was fabulous. It really was. You have been there then?

  M: No, I haven't. No, I mean it's very big. What did you do?

  W: Yeah. Well, I had only one month to travel in China, and that was too short f or such a vast country as China. I felt I didn't have enough time, so I sacrifced a lot of places and did the main tourist throughout really. I went to Beijing, the capital, Kaifeng, Yinchuan, and Tibet.

  M: Well, how exciting! You said after Beijing, you went to?

  W: That was Kaifeng in central China's Henan province. It's a charming city, and has got a lot to look around, like temples and pagodas, very traditional.

  M: Eh, eh.

  W: What fascinated me when I was there was that some Jews went to live in Kaifen g many years ago. As early as 16th century, there were Jewish families there. They have their synagogue and five books of Masses. Even today several hundred descendants of the original Jews still live in Kaifeng.

  M: Really, I've never heard of that. And where did you go after Kaifeng?

  W: I went to northwest to Yinchuan, the provincial city of Ningxia Hui autonomous region.

  M: Is this the place where there always is a shortage of water?

  W: No, no, on the contrary, it has got abundant supply of water, because it is n ear the Yellow River. In this sense, Yinchuan has a favorable geographical posit on in otherwise harsh surroundings.

  M: What did you see there then?

  W: Ningxia was once the capital of Western Xia during the 11th century. So outside Yinchuan,  you can still see the Western Xia mausoleum, where the Kings and t heir Kingdoms were buried. The tombs were scattered in a pretty big area at the foot of the Henan Mountain, and inside the city there are famous mosques in the architectural style of Middle East. It's really a place worth visiting. You got to know something about Chinese Moslems.

  M: And that sounds really interesting. Where did you travel after that?

  W: I was lucky enough to get into Tibet, and that was brilliant.

  M: Yeah. What was the most interesting place you visited, do you think?

  W: Well, I think actually Tibet is the most fascinating and exciting. I've never been anywhere so different. The people there are wonderful, the clothes .they w ear, the food they eat.

  M: And you said you went to Nepal as well?

  W: Yeah, eh, that's a sort of easier passion of Tibetans really and there are a  lot of Tibetans——there as well as other tribes and Nepalese, so that was good because I went trekking in Nepal, you should do that.

  M: How long did you trek for?

  W: Oh, I only did a short one, only for a week. I was lazy.

  M: Was it very tiring?

  W: No, it wasn't actually. I mean you just set your own pace, and don't pace y ourselves too hard, that was a stupid thing to do. Eh, you don't have to walk very far, so that was great.

  M: So how did you feel after all this travelling? How did you feel to stop travelling? I mean you were on the move alone for months and months ,and suddenly you're here ,and not traveling any more. How does it feel?

  W: I was ready to stop anyway. You get pretty sick, wearing the same clothes, and washing them in the different hotels. I never stay in the same place for longer than two days. And since I was ready to stop, I don't think I could keep doing it. I mean I've met people who've been travelling for 2 or 3 years. I couldn't do it.

  M: Yeah, yeah. Maybe it's something I should try after this.

  W: I think you really should.

  SECTION C    NEWS BROADCAST

  News Item 1(For Questions 11-12)

  Mike Tyson could sign a deal by Friday to face either Germany's Axo Shos or Denmark's Brian Nielsen here on August 21 in the heavy weight's first fight since his release from jail. The former World Heavy Weight champion was released on Monday after 4 months behind bars for an assault in the wake of a traffic accident last August. His deal with Showtime makes an August come back likely. "August 21st is certainly a day we're looking at," Showtime board director, James Lock en s said, "Hopefully in the next few days we'll have something concrete". Nevada boxing officials revoked Tyson's license for more than a year after he bit Evander Holyfield's ear off in June 1997. But they plan no action on Tyson because the license they granted him to fight doesn't expire until the end of the year. That came before the assault charge had been heard, and with the victim's support.

  News Item 2(For Question 13)

  The United States has begun to review Russians documents about the life and death of former president John F. Kennedy, and is expected to release them once the review is complete, the White House said on Monday. The documents which the Russians gave the United States on Sunday would be of particular interest because Kennedy's assassin Lee Harvey Oswald lived in the former Soviet Union for several years before he returned to the United States, and was arrested for killing t he former president on November 22, 1963. Russian President Boris Yeltsin surprised US president Bill Clinton on Sunday when he turned over what was described a s the result of exhaustive search of Russian government, military and private archives for papers about Kennedy and his assassination.

  News Item 3(For Questions 14-15)

  Hong Kong's unemployment rate has remained stable at 6.3% in the past 3 months, as business conditions have improved in the last month Figures indicate that from March to May this year, the size of the labor force was provisionally at 3,469,000, while the number of the unemployed people stood at 216,000. The number of cases of insolvency, sensational businesses and retrenchment, and numbers of workers affected these cases as recorded by the Labor Department have shown a declining trend in recent months. From March to May 1999, 78 such cases involving 3,882 workers were recorded as compared 93 cases affecting 5,220 workers for the 3 months from December 1998 to February 1999. Figures for the period from March to May 1999 when compared with those from February to April 1999 show an increase in the unemployment rate, mainly in renovation, maintenance, whole sale, and retail and transport sectors, which offset the decrease in construction, import and export and financing sectors 

  SECTION D NOTE-TAKING AND GAP-FILLING

  The Press Conference

  Press conferences are an all too familiar phenomena around us. However, when people start thinking about its advantages, it sometimes seems difficult to think of any that press conference provides for the competent news reporter. Use of the press conference by public officials and private entrepreneurs help give rise to the phrase "pseudo-event", and the event contrived to create news cove rage, where none has considered wanted. 

  However, having said that, one value of news conference by a public official is the symbolic nature of the event. And perhaps this is reasoned enough to continue the practice. At a press conference, a public official supposedly submits to examination by responding to unsolicited and perhaps hostile questions. A related advantage to the reporter is that press conference offers an opportunity to get the public official on the record, with regard to the government policies. Statements from the press conference can serve as criteria against subsequent statements and policies are measured. In that regard, press conference information may be used as reference point, more so than comments reported by single news medium. Also, when there is a single issue or topic to address, such as a new government program, an earthquake, or the nomination of a new government minister, the press conference offers benefits of efficiency in that officials can speak to a number of reporters at once on the issue of public concern and interest. The press conference may be at its best in this almost one-way format.

  Now let's move on to the disadvantages. And there are serious ones. Generally speaking, the press conference format, as it stands, makes it difficult for t he reporters to get worthwhile information. At press conference, particularly a large one, the well-prepared reporter may never get asked question and certainly is unlikely to have chance to ask follow-up questions. Another disadvantage in the press conference comes from the news source, that is, the news provider. The  news source generally decides who asks the questions, determines the length of the answers, and can avoid any follow-up question and rephrase tough questions t o his or her liking, and sets the time, place and duration of the interview. Press conferences televise live for the news audience, sometimes only highlights the disadvantages for the news reporters since there is little or no time to challenge, clarify or place context materials provided by the news source. Despite these disadvantages and others inherent how reporters get information, t he dews reporter can help assure the worthwhile information from a news source in share with the news audience. One way to reduce the likely heard of errors is to use multiple forms of interviewing by telephone, in person, and press conferences as well as multiple sources, common to the success of these approaches, how ever, are such interrelated ingredient as the preparation of the news reporter, the component of the news source and nature of the questions asked.

  Let's look at the preparation first.  The nature of much news coverage requires news reporters to be well-read on contemporary events. Like other professionals and craftsmen, the reporter must keep up to date on journalistic subject matter, that is the human condition. Although the subject is broad, it offers the advantages that there is a little a reporter can read or witness. That will not help in covering the news at one time or another. Next, being prepared for an interview includes giving some thought to the competence of the news source. The relationships between news reporters and news sources would benefit the news audience more, if reporters would frequently ask themselves: What is this news resource competent to talk about? What can this person tell the news audience that few others can? Towards the caution regarding the issue of competence, first, the reporter should not take for granted that, because of position or experience, the news source should know, does know, and can provide information. Second, the competence of t he news source needs to be linked with the news-gathering-methods. Let's just spend a few minutes on the first point. There are generally 4 conditions under which the reporter should not give prints to the news source information. One, the source may not know the information the reporter wants. Two, the source may have the information and want to share it, but may lack the verbal skills or concepts to do so. Three, the source may have the desired information but not to want to share it, or worse, may lie to avoid sharing information. Last, t he source may be willing to share this information but unable to recall it.

  Now in addition to preparation of the reporter and competence of the news source, there is one more important ingredient in successful news coverage, that is, he nature of questions asked. It is generally agreed that the nature of the question can shape the nature of the answer. General questions like "Are you for market economy" may lead to the respondent saying virtually anything, and s till being rather vague. On the other hand, a too narrow question may limit the respondent to one particular answer only. In order to gather information from news sources as accurately as possible, reporters can improve the question in the following 5 ways. Firstly, avoid words wit h double meanings. Secondly, avoid long questions. Thirdly, specify the time, pl ace and context you want the respondent to assume, and number 4, it is often helpful to ask questions in terms of the respondent's own immediate and recent experience rather than in generalities. Finally, either make explicit all the alternative the respondent should have in mind when answering the question or make non e of them explicit. Do not leave the news resource by suggesting a desired answer and not mentioning other alternatives.

  OK, to sum up, today's lecture has covered some of the advantages and disadvantages of the press conference, and three important factors and successful news coverage. In our next lecture, we will continue to discuss how to become a competent news reporter.

  答案与详解

  PAPER ONE

  PART Ⅰ LISTENING COMPREHENSION

  SECTION A TALK

  1.答案:A

  【问句译文】世界银行运行系统变大系统的变化指的是什么?

  【试题分析】本题为细节题。

  【详细解答】解答本题的关键句是"What immediately strikes anyone looking at the lending figures over the last 10 years is the tremendous expansion in the bank's loan program."由此可知,选项A为正确答案。

  2.答案:C

  【问句译文】是什么使得银行改变其借贷政策?

  【试题分析】本题为细节题。

  【详细解答】录音在提及银行借贷政策的改变时说"Thus by improving basic health services, by introducing better nutrition, by increasing literacy, and by promoting m ore even income distribution in a poor country, a lower and more acceptable fertility rate will be achieved. This advancing thinking persuaded the bank to change its overall lending strategy."这里提到了一些改变贫穷国家现状的措施,由"a lower and more acceptable"可知,从根本上促使政策改革的原因应为选项C。

  3.答案:D

  【问句译文】银行借贷政策重点的变化意味者银行将会怎样?

  【试题分析】本题为细节题。

  【详细解答】在提到投资对象时,谈话者说"Many of its major capital investment had scarcely touched the lives of urban and rural poor, nor have they created much employment."因此银行决定加大对劳动密集型活动的援助,故答案选D。

  4.答案:D

  【问句译文】下列哪一项不是对银行的批评?

  【试题分析】本题为细节题。

  【详细解答】谈话者说"…the bank should begin to think of itself less as a foreign aid agency and more of financial deal-maker,…"即世界银行应该作为财政的决策者 ,这一点应得到肯定,而不是批评,故选项D为正确答案。

  5.答案:C

  【问句译文】在整个谈话中,作者在介绍世界银行时是什么态度?

  【试题分析】本题为推理题。

  【详细解答】谈话者对世界银行的职能、现状、改进作了客观评论,并未表达出自己的个人感 情。因此选项C"客观的"为正确答案。

  SECTION B CONVERSATION

  6.答案:B

  【问句译文】男士对什么事实感到吃惊?

  【试题分析】本题为细节题。

  【详细解答】当女士说她在新西兰找了一份在计算机公司做秘书的工作时,男士说"Really? You can do that, can you? I mean it's possible for anyone to get a job in New Zealand, without being a New Zealander?"不是新西兰本土人也可以找到一份工作,因此 他感到惊奇,故选项B为正确答案。

  7.答案:D

  【问句译文】那位女土喜欢新加坡的主要原因是什么?

  【试题分析】本题为细节题。

  【详细解答】当男士提出由于新加坡过于的现代化,所以人们觉得它很乏味,接着他问女士的 感受。女士回答说:"And it's a great big shopping center and I really enjoy it from that point of view."由此可知吸引女士的是购物机遇,故答案选D。

  8.答案:B

  【问句译文】从对话中我们可得知开封和银川留给这位女士的怎样印象?

  【试题分析】本题为概括题。

  【详细解答】对话中提到了开封和银川的地理位置,犹太人的后代,古老的宝塔,归纳起来便 是历史情趣吸引了她,答案选B。

  9.答案:C

  【问句译文】下列哪一项最能形容女士对西藏的感受?

  【试题分析】本题为推理题。

  【详细解答】当男士问到她游历过的最有趣的地方时,女士回答说"I think actually Tibet is the most fascinating and exciting. I've never been anywhere so different." ,由此可知,选项C"狂喜"最能体现她的感受。

  10.答案:A

  【问句译文】根据谈话内容,是什么使得她暂停了旅行?

  【试题分析】本题为概括题。

  【详细解答】在谈到准备暂停旅行时,女士说"You get pretty sick, wearing the same clothes, and washing them in the different hotels. I never stay in the same place for longer than two days."穿着同样的衣服,在不同的旅馆里去洗它们使她感到厌烦, 在同一个地方呆的时间从没有超过两天,由此可推知:她想停下居无定所的生活,故答案选 A。

  SECTION C NEWS BROADCAST

  News Item 1

  11.答案:C

  【问句译文】泰森为何于去年八月被关入监狱?

  【试题分析】本题为主旨题。

  【详细解答】新闻中提到"…for an assault in the wake of traffic accident last August."由此可知,泰森是因为在一次交通事故中动手打人而被捕入狱。

  12.答案:A

  【问句译文】泰森在什么日期前得不到拳击比赛的许可证?

  【试题分析】本题为细节题。

  【详细解答】解答本题的关键句是"But they plan no action on Tyson because the license they granted him to fight doesn't expire until the end of the year."由此可知,答案选A。

  News Item 2

  13.答案:B

  【问句译文】为何俄罗斯的档案应受到重视?

  【试题分析】本题为细节题。

  【详细解答】新闻第二句说"The documents which the Russian gave the United States o n Sunday would be of particular interest because Kennedy's assassin Lee Harvey Oswald lived in the former Soviet Union for several years …",由于刺杀肯尼迪的 凶手逃至前苏联生活了好几年,因此俄罗斯的档案应受到重视。

  News Item 3

  14.答案:C

  【问句译文】在近三个月里,香港的失业率情况如何?

  【试题分析】本题为主旨题。

  【详细解答】新闻开头就说"Hong Kong's unemployment rate has remained stable at 6.3 % in the past 3 months,…"由此可知,在过去的三个月里,香港失业率稳定在6.3%,因 此答案选C。

  15.答案:C

  【问句译文】根据该新闻,下列哪一选项是正确的?

  【试题分析】本题为推理题。

  【详细解答】新闻中提到各部门失业率有不同幅度地上涨或下降,但新闻首句就告诉我们说失 业率保持在6.3%不变。由此可推断出一些部门失业率上升,另外一些部门失业率下降,从 而保持总的失业率稳定不变,故答案选C。

  SECTION D NOTE-TAKING AND GAP-FILLING

  1.答案:symbolic  

  【详细解答】在提到新闻发布会的第一个优点时说"…one value of news conference by a public official is the symbolic nature of the event."由此可直接填写"symbolic"。

  2.答案:reference  

  【详细解答】在提到新闻发布会的另一个优点时说"In that regard, press conference information may be used as reference point,…" 由此可直接填写"reference"。

  3.答案:reporters  

  【详细解答】由录音"…the press conference offers benefits of efficiency in that officials can speak to a number of reporters at once on the issue of public concern and interest."可知,这里说的是从"记者们"(reporters)的观点来看问题。

  4.答案:format  

  【详细解答】本段录音的最后一句说"The press conference may be at its best in this almost one-way format." 由此可直接填写"format"。

  5.答案:disadvantage

  【详细解答】前面讲的是新闻发布会的优点(advantage),下面接着就要转入谈它的缺点了 (disadvantage)。

  6.答案:component  

  【详细解答】由录音"…the component of the news source and nature of the questions asked."可直接填写"component"。

  7.答案:competence  

  【详细解答】由关键句"…the competence of the news source needs to be linked with the news-gathering-methods."可知,此处应填写"competence"。

  8.答案:verbal    

  【详细解答】由关键句"…but may lack the verbal skills or concepts to do so."可知 ,这里说的是"口头表达"(verbal)技能。

  9.答案:nature    

  【详细解答】由录音"…that is, the nature of questions asked." 可知,此处应填写" nature"。

  10.答案:asking

  【详细解答】这里提到了五条改进提问的方法,第四条是"it is often helpful to ask questions in terms of the respondent's own immediate",根据上下文,此处应该用它的 动名词形式,故填写"asking"。

  改错

  PART Ⅱ PROOFREADING AND ERROR CORRECTION

  1.答案: as→so 

  【详细解答】由于本句中的 much修饰的是名词 feeling,应改用so来修饰。

  2.答案:increasing→increasingly 

  【详细解答】 increasing可用来修饰名词,表示"正在增加",修饰名词topic不合题意。本 句中应该用副词作状语修饰形容词favorite,所以应该改用副词increasingly。

  3.答案:so→but 

  【详细解答】根据上下文判断,小麦价格在秋天不断下降,但是农民不能等着市场提升价格。 这里应该是转折关系,而不是因果关系。

  4.答案:soon或shortly 

  【详细解答】要表达"不久以后",要么用soon after,要么用shortly after,就是没有soon shortly after这种搭配。

  5.答案:rich→richer 

  【详细解答】根据上下文,这里是说由于价格上涨,投机商们变得更加富有,故应该改用形容 词的比较级形式。

  6.答案:asked∧→for 

  【详细解答】ask for为固定搭配,意为"要求,请求"。

  7.答案:involving→involved 

  【详细解答】be involved in为一固定搭配,意为"被卷入…",这里的be为连系动词,可用 become替换。

  8.答案:1ife→living 

  【详细解答】根据上下文,这里要表达的意思是"生活费用",应为living costs。life则意 为"生命",不能修饰costs。

  9.答案:handle→deal或with

  【详细解答】handle为及物动词,应直接带宾语。而固定搭配deal with与handle同义,都 表示 "处理(某事)"。

  10.答案:total→full

  【详细解答】total意为"总体的,全体的",指一种整体的概念。而这里要表达的是"全部 的权威",英语中常用full authority。

  阅读理解 A

  SECTION A 

  TEXT A  

  短文大意:本文介绍的是一个叫做Blackpool的地方。在二十年前是一片繁荣景象,如今 却是一片荒凉。当地政府正在采取措施。

  16.答案:D

  【参考译文】文章开头似乎在暗示Blackpool怎样?

  【试题分析】本题为推理题。

  【详细解答】文章的开头引用一位当地官员的话说"Twenty years ago, Blackpool turned its back on the sea and tried to make itself into an entertainment centre." 然后 又谈到了现在的情况"Now the thinking is that we should try, to refocus on the se a and make Blackpool a family destination again."通过前后对比可知,选项D "Blackpool打算恢复它以前的面貌"为正确答案。

  17.答案:B

  【参考译文】由该短文可知,Blackpool以前的状况怎样?

  【试题分析】本题为细节推理题。

  【详细解答】短文第三段最后一句说"For the first time since the industrial revolution Blackpool's waters are safe to swim in."由此可知,该处以前的水源同样是受到污染的,故答案选B。   

  18.答案:A

  【参考译文】Blackpool海滩给参观者印象最深的是什么?

  【试题分析】本题为细节题。

  【详细解答】 短文第五段首句说"The most striking thing about Blackpool these days compared with 20 years ago is how empty its beaches are."由此可知选项A为正确答 案。

  

  TEXT B  

  短文大意:本文主要论述的是1997年的亚洲金融危机的危害及其爆发的原因。

  19.答案:D

  【参考译文】根据该短文,下列哪一项不是作者的观点?

  【试题分析】本题为排除题。

  【详细解答】短文第二段最后几句说"In the face of the stampede, governments had no good options. If they let their currencies plunge inflation would soar and companies that had borrowed in dollars would go bankrupt; if they tried to support their currencies by pushing up interest rates, the same firms would probably go bust from the combination of debt burden and recession. In practice, countries' sp lit the difference- and paid a heavy price regardless."由此可知,亚洲经济危机时 期各国均陷入经济两难境地,既不能任由本国货币猛跌,也不能提高存款利率。选项D与该 说法相矛盾, 故不是作者的观点。

  20.答案:C

  【参考译文】作者认为那些亚洲国家如何?

  【试题分析】本题为细节推理题。

  【详细解答】短文第三段最后一句说"But the punishment was surely disproportionate t o the crime,…"由此可知,作者认为那些亚洲国家遭受的惩罚过度了,故答案选C。

  21.答案:A

  【参考译文】从该短文可以推知IMF的经济复苏政策如何?

  【试题分析】本题为推理题。

  【详细解答】虽然IMF对韩国的经济复苏起了一定作用,短文第四段却说"Never mind that other IMF clients have done far worse,…"接着以马来西亚为例,说明IMF的政策不是万能的,故答案选A。 

  22.答案:B

  【参考译文】在短文的末尾,作者认为亚洲国家经济全面复苏是怎样的?

  【试题分析】本题为推理题。

  【详细解答】短文最后一段的最后一句说"…they have a long way to go."即这些国家( 的经济全面复苏)有很长的路要走。故选项B"遥远的"为正确答案。

  TEXT C  

  短文大意:本文主要论述的是人类为何迁居以及迁居所带来的影响。

  23.答案:D

  【参考译文】下列哪一种说法是错误的?

  【试题分析】本题为细节题,可用排除法解答。

  【详细解答】短文第三段首句说"Migration is the dynamic undertow of population change: everyone's solution, everyone's conflict."由此可知,人口迁移对社会人口变化 有影响,故可排除选项A;短文第四段最后一句说"Migration helped create humans, dro ve us to conquer the planet, shaped our societies, and promises to reshape them again." 由此可知,人口迁移影响着社会政治经济的变化,同时也促进了社会的发展变化 ,故可排除选项B和C;而全文没有提及选项D的意思,故为正确答案。

  24.答案:B

  【参考译文】根据Kingsley Davis的观点,下列哪一项不是造成人类迁移的原因?

  【试题分析】本题为细节题,可用排除法解答。

  【详细解答】短文第八段分析了造成人类迁移的两个方面的原因。第一点是人类的适应性(ad apt to different conditions);第二点是人类文化的差异性及群体之间的不平等性(cultures began to differ, and inequalities developed between groups),故可排除选项 A、C、D。

  25.答案:A

  【参考译文】下列哪一项不是短文中提到的移居者?

  【试题分析】本题为细节题,可用排除法解答。

  【详细解答】 短文倒数第二段讲述了几起重大事件引起的人口迁移:宗教运动引起了早期移 民(religions spawned pilgrims or settlers);战争为殖民者提供了场所(wars drove refugees);经济技术革新吸引了大批工人与企业家(economic innovations drew workers and entrepreneurs)。因此可排除D(殖民者)、B(工人)和C(移居者)。

  26.答案:C

  【参考译文】重大事件与人类迁移之间是怎样的关系?

  【试题分析】本题为推理题。

  【详细解答】短文倒数第二段首句说"It is difficult to think of any great events that did not involve migration."该否定句强调说明了重大事件与人类迁移之间因果相承关 系,故答案选C。

  TEXT D  

  短文大意:本文论述的是科学家和艺术家们如何表达思想,进行社会交流的。

  27.答案:B

  【参考译文】富有创造力的艺术家与思想家是如何进行交流的?

  【试题分析】本题为细节题。

  【详细解答】 短文第一段第三句指出,富有创造力的艺术家与思想家是"… in one way tha t of creating a language, or creating a convention, or at least of developing th e language and conventions to the point where they are capable of bearing his pr ecise meaning."选项B与此意一致,故为正确答案。

  28.答案:D

  【参考译文】从事创造性工作的艺术家和科学家们共同的特征是什么?

  【试题分析】本题为细节题。

  【详细解答】 短文第一段最后几句说"Many artists and scientists share this fundamental unconcern about the ways in which their work will be received. They may be glad if it is understood and appreciated, hurt if it is not, but while the work is being done there can be no argument."由此可知,他们只看重作品本身,而不在意人 们对他们创作的反应,故答案选D。

  29.答案:C

  【参考译文】根据该短文,下列哪一种说法是不正确的?

  【试题分析】本题为排除题。

  【详细解答】从短文第二段中可找出选项A、B和D的意思。由"For it is not only a communication system outside him; it is also, however original he may be, a communication system which is in fact part of himself. "可知,无论任何人在创作过程中都必 将会受到社会的影响,故答案选C。

  30.答案:A

  【参考译文】文章结尾处暗示了那些具有创造力的人们感到孤独,其原因是他们怎样?

  【试题分析】本题为推理题。

  【详细解答】短文最后一句话"The society and the communication are there, but it is difficult to recognize them, difficult to be sure."这里是说,他们其实与当时的 社会以及同时代其他人之间都存在着许多共同点,但他们自己不承认。因此他们感到孤独的 真正原因是没有找到合适的表达方式。故答案选A。

  阅读理解 B

  TEXT E  

  短文大意:这是一篇关于几本新书的评论文章。

  31.答案:A

  【参考译文】本文的写作目的是什么?

  【试题分析】本题为主旨题。

  【详细解答】文章第一段第五句说"What you need is a good interior-design book."作 者于是分别介绍了几本有关室内设计的书刊。由此可见,作者是在建议人们看一些有关室内 设计的书。故答案选A。

  TEXT F  

  短文大意:本文主要介绍的是德国税收制度改革情况。

  32.答案:D

  【参考译文】本文作者的主要写作目的是什么?

  【试题分析】本题为主旨题。

  【详细解答】通过扫读文中的黑体字"One major reason","A reform","How do"," An approach"和"Comprehensive tax reform"可知本文作者主要是讲述税收制度改革措 施,故答案选D。

  TEXT G  

  短文大意:本文主要论述的是政府在保护大众健康方面应起的作用。

  33.答案:C

  【参考译文】本文主要强调了下列哪一项在健康话题中的角色?

  【试题分析】本题为主旨题。

  【详细解答】短文第一段最后一句提出问题"How can governments help people conserve t heir health and avoid premature death? "下文便开始讲述政府应在烟草、酒精、垃圾 广告各方面多作工作。由此可知,全文主要是谈论政府在健康问题上的角色。故答案选C。

 

  TEXT H  

  短文大意:本文主要阐述的是人类反叛的本性。

  34.答案:A

  【参考译文】随着文章论点的展开,文章的写作特点是什么?

  【试题分析】本题为推理题。

  【详细解答】分析文章段落内容从提出问题(must we conform?),然后对该问题进行分析, 最后回答解决问题的方法,由此可见,作者论述了"人类具有反叛的本性"这一论点,据有 一定的说服力。故答案选A"阐述性"。

  TEXT I  

  短文大意:本文是对美国未来的一些社会情况进行的预测。

  35.答案:B

  【参考译文】根据人口调查预计,到2050年美国男性平均寿命能达到多少岁?

  【试题分析】本题为细节题。

  【详细解答】短文第三段最后一句说"And by the year 2050, women will have a life exp ectancy of 83.6 years and men of at least 75.1."由此可直接得出正确答案为B。

  36.答案:D

  【参考译文】犯罪专家估计在不久的将来犯罪率将首先在哪些地区降低?

  【试题分析】本题为细节题。 【详细解答】短文最后一段最后一句指出"Crime expert Alfred Blumstein, who foresees a drop in crime over the coming decade, predicts that the Northeast and Midwest, …" 由此可直接得出正确答案为D。

  TEXT J 

  短文大意:本文是从1972到1998有关中美交往的记事表。

  37.答案:D

  【参考译文】中美何时建立正式的外交关系?

  【试题分析】本题为时间题。

  【详细解答】快速查看全文,在记事表第五段说"January 1, 1979: China and the US form ally established diplomatic ties."故答案选D。

  38.答案:B

  【参考译文】建立新型中美关系的五项基本原则是江泽民在何什么地方提出来的?

  【试题分析】本题为地点题。

  【详细解答】快速查看全文,在记事表第十二段说"November 14, 1994: Chinese President Jiang Zemin, on the sidelines of attending an informal APEC leadership meeting in Bogor, met US President Bill Clinton in Jakarta and put forward the Five Principles for the establishment of a new type of Sino-US relationship."这里记录的 是江泽民和克林顿在雅加达(Jakarta)会面签订了五项基本原则。故答案选B。

  TEXT K  

  短文大意:本文介绍的是五位作家及其主要作品。

  39.答案:A

  【参考译文】如果你对讽刺小说感兴趣,你很可能选择谁的作品阅读?

  【试题分析】本题为细节题。

  【详细解答】短文第四段第一句说"Alexander Pope, poet and satirist,…"由此可知,Al exander Pope是一位诗人和讽刺作家,故答案选A。

  40.答案:B

  【参考译文】下列哪一位作家是诺贝尔奖获得者?

  【试题分析】本题为细节题。

  【详细解答】文章第一段最后一句说"Galsworthy was awarded the Nobel Prize for Liter ature in l932。"由此可知,Galsworthy于1932年获得诺贝尔文学奖,故答案选B。

  翻译

  SECTION A CHINESE TO ENGLISH

  参考译文

  Qiao Yu took to fishing in his old age. He said: "Where there is fish and water, there is good environment, and good environment fills one's heart with joy (makes one feel delighted, delights everyone, gives delight to everyone). I think the best place for fishing is not a comfortable man-made fish-pond where hungry fish are ready (provided) for you, but an enchanting place in the wild where everything is natural. "Fishing (Angling) is a game that can help improve one's temperament (character). It is good for mental and bodily health. Qiao Yu said: "Fishing falls into three stages. The first stage is just for eating fish. The second stage is for eating fish and for enjoying the pleasure of fishing as well. (The second stage is for enjoying the pleasure of fishing as well as eating fish. ) The third stage is mainly for the pleasure of fishing facing a pool of green water, one casts (throws) aside all anxieties and worries and enjoys (takes) a good rest, both mental and physical."

  SECTION B ENGLISH TO CHINESE

  参考译文

  梭罗所理解的"低层次",即为了拥有而去拥有,或与所有的邻居明争暗斗而致拥有。他心目中的"高层次",则是这样一种积极的人生戒律,即要使自己对自然界永恒之物的感悟臻于完美。对于他从低层次上节省下来的时间和精力,他可将其致力于对高层次的追求。勿庸置疑,梭罗不赞成忍饥挨饿,但他在膳食方面所投入的精力仅果腹而已,只要可确保他能去从事更为重要的事务,他便别无所求。

  付出努力才是其本质所在,除非我们终生与困难为伴,否则就无幸福可谈。正如济慈所 言,除去不可能做的事,我们一生获得的满足有多大,取决于我们选择的困难有多强。罗伯 特·福剂罗斯特谈到"苦中求乐"时,也有异曲同工之理。就通常宣传的幸福而言,其致命弱 点在于声称不用付出努力即可获得幸福。

  我们甚至在竞赛中寻求困难。我们需要困难,因为没有困难也就无所谓竞赛,竞赛是制 造困难以求得乐趣之道。竞赛规则就是任意强加的困难。违犯竞赛规则就是破坏乐趣。下棋 时,随心所欲,肆意更改强制之规更易获胜。然而乐趣源于获胜而又遵守规则。没有困难就 没有乐趣。

  写作

  【参考范文】

  The Impact of the Internet on Education

  The Internet is making the world smaller and is changing every aspect of our life. There can be no doubt that the Internet will have greater impact on our society and economy. The impact, in my opinion, will be strongly felt in the aspect of education. 

  First, the Internet provides equal opportunities for all students, as long as they can get access to the Internet. In the traditional form of education, students who get higher marks in examination may enter a better school, and those who do not have to enter a relatively less good one. With the Internet, every student is provided with the same sources of education, so they can get equal opportunities to improve themselves. In that case, the general quality of education will be highly improved in our country.

  Second, the Internet poses a challenge, not a danger really to teacher. Traditionally, teachers are the dominator of education, and students are fixed to t he instruction of several specific teachers. This, in fact, may result in the laziness of teachers, since there is no competition. With the Internet, a student may choose any teacher's instruction as far as he likes it, which is an incentive to the faculty of teachers. Faced with this, a teacher is likely to try his best to improve his instruction and this will in turn improve the educational quality of our country.

  Last but not least, the internet makes education more convenient for the students. A student can learn at any time he likes and, at the same time, save the money spent on tuition and board. All of these may head to a revolution of education. 

  After ail, I am not exaggerating the influence of the Internet on education. It is true that the Internet may still remain out of the reach of some one and may be less amicable than human teachers. But the point is that, the Internet is sure to accelerate the development of education.

  With the Internet, let us make the education of our country into a better existence!

  本套真题测试的语言重点:

  重点单词:

  prevail   流行,盛行

  preposterous  荒谬的

  tumultuous  喧嚣的

  demographer  人口统计学家

  niche  【生】小生境

  bedraggled  衰败的,破旧的

  cirrhosis   【医】硬化

  annihilation  灭绝

  重点词组:

  bottom out  降至最低点

  average out  达到平均数

  sweep up  扫除,清除

  in a way   在某种程度上,稍微

  fall off  下降,跌落,减少,衰退

  strive for  为……奋斗,争取

2001年英语专业四级试题及答案
Part Ⅲ LISTENING COMPREHENSION[20 MIN.]

  In Sections A, B and C, you will hear everything once only. Listen carefully and then answer the questions that follow. Mark the correct answer to each question on your answer sheet.

  SECTION A STATEMENT

  In this section you will hear nine statements. At the end of each statement you will be given 10 seconds to answer the question.

  1. The speaker likes teaching because of ___.

  A. its interesting nature

  B. the good salaries

  C. contact with the young

  D. more summer holidays 

  2. What does the speaker mean?

  A. Bad living conditions are due to the poor city.

  B. Bad planning is responsible for poor living conditions.

  C. Living conditions are bad because the city is too big.

  D. Small cities have better living conditions than large ones. 

  3. What does the statement mean?

  A. Many people are concerned about their security.

  B. Social security bears no relation to population.

  C. Most social security problems are caused by a few people.

  D. Too many people may result in social security problems. 

  4. Passengers must check in to board Flight 998 by ___.

  A. 10:30 a.m B. 10:00 a.m C. 11:30 a.m D. 11:00 a.m 

  5. The speaker is probably a(n) ___.

  A. insurance agent B. fireman C. salesman D. policeman 

  6. The speaker thinks that___.

  A. Ian achieved a lot as an athlete

  B. Ian's blind eye prevented him from athletics

  C. Ian's success depended on his childhood experience

  D. Ian trained so hard in athletics as to lose one eye 

  7. Mrs. Clark is worried about her___.

  A. husband's health B. husband's work C. husband's illness D. own health 

  8. The relationship between Susan and Jenny is ___.

  A. neutral B. friendly C. unclear D strained 

  9. What do we learn about Jack?

  A. He is well-known for hard work.

  B. He is pretty busy working.

  C. He has overworked and hurt his sight.

  D. He doesn't like to have dinner with us. 

  SECTION B CONVERSATION

   In this section, you will hear nine short conversations between two speakers.At the end of each conversation you will be given 10 seconds to answer the question.

  10. What are they mainly talking about?

  A. Graduation date. B. Vacation plans. C. School courses. D. Job hunting. 

  11. The conversation probably takes place in___.

  A. a library B. a bookstore C. the classroom D. a department store 

  12. The relationship between the two speakers is probably___.

  A. man and wife B. lawyer and clientC. customer and waitress D. colleagues 

  13. We can infer from the conversation that the man is a(n) ___.

  A. plumber B. construction worker C. office boy D. porter 

  14. What will the man probably do next? 

  A. Turn off the tape recorder. 

  B. Turn up the tape recorder.

  C. Call the doctor. 

  D. Continue to play. 

  15. How does Lisa feel about her work? 

  A. Satisfied. B. Frustrated. C. Annoyed. D. Confident. 

  16. The woman is going to the___.

  A. library B. theatre C. research institute D. laboratory 

  17. Jackson changed his job because he ___.

  A. hurt himself during his work

  B. was not satisfied with his play

  C .wanted to work harder

  D. found the job too hard 

  18. What does the woman say about the film?

  A. It is hard to pronounce the name.

  B. It is not going to be well received.

  C. She has temporarily forgotten its name.

  D. She has never heard of the name.

  SECTION C NEWS BROADCAST 

  Questions 19 and 20 are based on the following news. At the end of the news item, you will be given 20 seconds to answer the question. 

  Now listen to the news.

  19. Nigeria returned to the Commonwealth after ___.

  A. she had sentenced minority rights activists to death

  B. the military had resumed control of the country

  C. power had been handed over to an elected president

  D. she had negotiated with Commonwealth leaders 

  20. The Commonwealth consists of ___countries which were former British colonies.

  A. 54 B. 29 C. 9 D. 95 

  Questions 21 and 22 are based on the following news. At the end of the news item, you will be given 20 seconds to answer the question.

  Now listen to the news.

  21. The space shuttle Discovery completed a ___mission upon to the Kennedy Space Centre.

  A. 11-day B. 94-day C. 10-day D. 49-day

  22. When the spacecraft was going to land, ___.

  A. it produced a lot of noise

  B. there were scattered showers

  C. people could see it high in the sky

  D. people could neither see nor hear it 

  Questions 23 and 24 are based on the following news. At the end of the news item, you will be given 20 second to answer the question.

  Now listen to the news.

  23. How many people died during the collision?

  A Two. B Eighteen. C. Three. D. Five. 

  24. Three Albanians were arrested for___.

  A. attacking the patrol boat

  B. smuggling in refugees

  C. causing the accident

  D. injuring refugees 

  Question 25 is based on the following news. At the end of the news item, you will be given 10 seconds to answer the question.

  Now listen to the news.

  25. The news item is mainly about___.

  A. efforts to salvage Sun Vista

  B. negotiation with the ship's owner

  C. threats Sun Vista poses to passing ships

  D. a newspaper's comment on Sun Vista 

  Part Ⅳ CLOZE [15 MIN.]

  Decide which of the choices given below would best complete the passage if inserted in the corresponding blanks. Mark the best choice for each blank on your answer sheet.

  The translator must have an excellent, up-to-date knowledge of his source languages, full facility in the handling of his target language, which will be his mother tongue or language of habitual ( 26 )and a knowledge and understanding of the latest subject-matter in his field of specialization.

  This is, as it were, his professional equipment. ( 27 ) this, it is desirable that he should have an inquiring mind,wide interests, a good memory and the ability to grasp quickly the basic principles of new developments. He should be willing to work ( 28 )is own, often at high speeds, but should be humble enough to consult others ( 29 )his own knowledge not always prove adequate to the task in hand. He should be able to type fairly quickly and accurately and, if he is working mainly for publication, should have more than a nodding ( 30 )with printing techniques and proof-reading. If he is working basically as an information translator, let us say, for an industrial firm, he should have the flexibility of mind to enable him to ( 31 )rapidly from one source language to another, as well as from one subject-matter to another,since this ability is frequently ( 32 )of him in such work. Bearing in mind the nature of the translator's work,i.e. the processing of the written word, it is, strictly speaking, ( 33 )that he should be able to speak thelanguage he is dealing with. If he does speak them, it is an advantage ( 34 )a hindrance, but this skill is in many ways a luxury that he can ( 35 )with. It is, ( 36 ),desirable that he should have an approximate idea about the pronunciation of his source languages even if this is restricted to ( 37 )how proper names and place names are pronounced. The same ( 38 ) to an ability to write his source languages. If he can, well and good; if he cannot, it does not ( 39 ). There are many other skills and ( 40 )that are desirable in a translator.

  26. A. application B. use C. utility D. usage 

  27. A. More than B. Except forC. Because of D. In addition to 

  28. A. of B. byC. for D. on 

  29. A. should B. whenC. because D. if 

  30. A. familiariy B. acquaintanceC. knowledge D. skill 

  31. A. change B. transformC. turn D. switch 

  32. A. lacked B. requiredC. faced D. confronted 

  33. A. essential B. unnecessaryC. advantageous D. useless 

  34. A. over B. despiteC. rather than D. instead 

  35. A. deal B. concernC. work D. do away 

  36. A. however B. accordinglyC. consequently D. thus 

  37. A. knowing B. having knownC. know D. have known 

  38. A. refers B. comesC. applies D. amounts 

  39. A. matter B. mindC. harm D. work 

  40. A. characteristics B. qualitiesC. distinctions D. features

  Part ⅤGRAMMAR AND VOCABULARY [15 MIN.]

  There are twenty-five sentences in this section. Beneath each sentence there are four words or phrases marked A, B, C and D. Choose one word or phrase that best completes the sentence.

  Mark your answers on your answer sheet.

  41. I can't go-for one thing, I have no money, and___I have too much work.

  A. what's more B. as well C. for another D. in addition 

  42. Even as a girl,___to be her life, and theater audiences were tobe her best teacher.

  A. performing by Melissa were

  B. it was known that Melissa's performances were

  C. knowing that Melissa's performances were

  D. Melissa knew that performing was 

  43. ___ him tomorrow?

  A. Why not to call on B. Why don't call on

  C. Why not calling on D. Why not call on 

  44. There is no doubt ___the company has made the right decision on the sales project.

  A. why B. that C. whether D. when 

  45. Intellect is to the mind ___sight is to the body.

  A. what B. as C. that D.like

  46. ___I sympathize, I can't really do very much to help them out of the difficulties.

  A. As long as B. As C. While D. Even

  47. The patient's progress was very encouraging as he could ___ get out of bed without help.

  A. nearly B. hardly C. merely D. barely

  48. He was___to tell the truth even to his closest friend.

  A. too much of a coward B. too much the coward C. a coward enough D. enough of a coward 

  49. Barry had an advantage over his mother ___he could speak French.

  A. since that B. in that C. at that D. so that

  50. You needn't worry ___ regards the cost of the operation.

  A. with B. which C. as D. about 

  51. ___ is not a serious disadvantage in life.

  A. To be not tall B. Not to be tall C. Being not tall D. Not being tall 

  52. During the famine, many people were ___ to going without food for days.

  A. sunk B. reduced C. forced D. declined

  53. The computer can be programmed to ___a whole variety of tasks.

  A. assign B. tackle C. realize D. solve

  54. The team's efforts to score were ___by the opposing goalkeeper.

  A. frustrated B. prevented C. discouraged D. accomplished 

  55. I only know the man by___ but I have never spoken to him.

  A. chance B. heart C. sight D. experience

  56. Being colour-blind, Sally can't make a ___between red and green.

  A. difference B. distinction C. comparison D. division 

  57. You must insist that students give a truthful answer ___ with the reality of their world.

  A. relevant B. simultaneous C. consistent D. practical 

  58. In order to raise money, Aunt Nicola had to ___with some of her most treasured possessions.

  A. divide B. separate C. part D. abandon.

  59. The car was in good working ___when I bought it a few months ago

  A. order B. form C. state D. circumstance 

  60. The customer expressed her ___for that broad hat.

  A. disapproval B. distaste C. dissatisfaction D. dismay 

  61. In order to repair barns, build fence, grow crops, and care for animals a farmer must indeed be___.

  A. restless B. skilled C. strong D. versatile 

  62. His expenditure on holidays and luxuries is rather high in___to his income.

  A. comparison B. proportion C. association D. calculation 

  63. Although he has become rich, he is still very ___ of his money.

  A. economic B. thrifty C. frugal D. careful 

  64. As the manager was away on a business trip, I was asked to ___the weekly staff meeting.

  A. preside B. introduce C. chair D. dominate 

  65. The ___ of the word is unknown, but it is certainly not from Greek.

  A. origin B. generation C. descent D. cause

  Part Ⅵ READING COMPREHENSION [30 MIN.]

  SECTION A READING COMPREHENSION [25 MIN.]

  In this section there are four passages followed by questions or unfinished statements, each with four suggested answers marked A, B, C and D. Choose the one that you think is the best answer. Mark your answers on your answer sheet.

  TEXT A

  The train clattered over points and passed through a station. Then it began suddenly to slow down, presumably in obedience to a signal. For some minutes it crawled along, then stopped; presently it began to move forward again. Another up-train passed them, though with less vehemence than the first one. The train gathered speed again. At that moment another train, also on a down-line, swerved inwards towards them, for a moment with almost alarming effect. For a time the two trains ran parallel, now, one gaining a little, now the other. Mrs. McGillicuddy looked from her window through the window of the parallel carriages. Most of the blinds were down, but occasionally the occupants of the carriages were visible. The other train was not very full and there were many empty carriages.

  At the moment when the two trains gave the illusion of being stationary, a blind in one of the carriages flew up with a snap. Mrs. McGillicuddy looked into the lighted first-class carriage that was only a few feet away.

  Then she drew her breath in with a gasp and half-rose to her feet.

  Standing with his back to the window and to her was a man. His hands were round the throat of a woman who faced him, and he was slowly, remorselessly, strangling her. Her eyes were starting from their sockets, her face was purple. As Mrs. McGillicuddy watched, fascinated, the end came; the body went limp and crumpled in the man's hands.

  At the same moment, Mrs. McGillicuddy's train slowed down again and the other began to gain speed. It passed forward and a moment or two later it had vanished from sight.

  Almost automatically Mrs. McGillicuddy's hand went up to the communication cord, then paused, irresolute. After all, what use would it be ringing the cord of the train in which she was traveling? The horror of what she had seen at such close quarters, and the unusual circumstances, made her feel paralysed. Some immediate action was necessary,-but what?

  The door of her compartment was drawn back and a ticket collector said, "Ticket, please."

  66. When Mrs. McGuillicuddy's train passed through a station, it___.

  A. gained speed suddenly B. kept its usual speed C. changed its speed D. stopped immediately 

  67. Mrs. McGuillicuddy seems to be a (an) ___ person.

  A. observant B. interested C. nosy D. nervous 

  68. What she saw in the parallel train made her feel___.

  A. excited B. anxious C. worried D. horrified 

  69. She didn't ring the communication cord immediately because___.

  A. she was very much afraid

  B. there was no point of doing so

  C. she was too shocked to move

  D. the ticket collector came in 

  TEXT B

   I am one of the many city people who are always saying that given the choice we would prefer to live in the country away from the dirt and noise of a large city. I have managed to convince myself that if it weren't for my job I would immediately head out for the open spaces and go back to nature in some sleepy village buried in the county. But how realistic is the dream?

  Cities can be frightening places. The majority of the population lives in massive tower blocks, noisy, dirty and impersonal. The sense of belonging to a community tends to disappear when you live fifteen floors up. All you can see from your window is sky, or other blocks of fiats. Children become aggressive and nervous - cooped up at home all day, with nowhere to play; their mothers feel isolated from the rest of the world. Strangely enough, whereas in the past the inhabitants of one street all knew each other, nowadays people on the same floor in tower blocks don't even say hello to each other.

  Country life, on the other hand, differs from this kind of isolated existence in that a sense of community generally binds the inhabitants of small villages together. People have the advantage of knowing that there is always someone to turn to when they need help. But country life has disadvantages too. While it is true that you may be among friends in a village, it is also true that you are cut off from the exciting and important events that take place in cities. There's little possibility of going to a new show or the latest movie. Shopping becomes a major problem, and for anything slightly out of the ordinary you have to goon an expedition to the nearest large town. The city-dweller who leaves for the country is often oppressed by a sense of unbearable stillness and quiet.

  What, then, is the answer? The country has the advantage of peace and quiet, but suffers from the disadvantage of being cut off: the city breeds a feeling of isolation, and constant noise batters the senses. But one of its main advantages is that you are at the centre of things, and that life doesn't come to an end at half-past nine at night. Some people have found (or rather bought) a compromise between the two: they have expressed their preference for the "quiet life" by leaving the suburbs and moving to villages within commuting distance of large cities. They generally have about as much sensitivity as the plastic flowers they leave behind-they are polluted with strange ideas about change and improvement which they force on to the unwilling original inhabitants of the villages.

   What then of my dreams of leaning on a cottage gate and murmuring "morning" to the locals as they pass by. I'm keen on the idea, but you see there's my cat, Toby. I'm not at all sure that he would take to all that fresh air and exercise in the long grass. I mean, can you see him mixing with all those hearty males down the farm? No, he would rather have the electric imitation-coal fire any evening.

  70. We get the impression from the first paragraph that the author___.

  A. used to live in the country 

  B. used to work in the city

  C. works in the city 

  D. lives in the country 

  71. In the author's opinion, the following may cause city people to be unhappy EXCEPT___.

  A. a strong sense of fear B. lack of communicationC. housing conditions D. a sense of isolation 

  72. The passage implies that it is easy to buy' the following things in the country EXCEPT___

  A. daily necessities B. fresh fruits C. designer clothes D. fresh vegetables 

  73. According to the passage, which of the following adjectives best describes those people who work in large cities and live in villages?

  A. Original. B. Quiet. C. Arrogant. D. Insensitive. 

  74. Do you think the author will move to the country?

  A. Yes, he will do so. B. No, he will not do so.C. It is difficult to tell. D. He is in two minds. 

  TEXT C

   Traditionally, the woman has held a low position in marriage partnerships. While her husband went his way, she had to wash, stitch and sew. Today the move is to liberate the woman, which may in the end strengthen the marriage union.

  Perhaps the greatest obstacle to friendship in marriage is the amount a couple usually see of each other. Friendship in its usual sense is not tested by the strain of daily, year-long cohabitation. Couples need to take up separate interests (and friendship) as well as mutually shared ones, if they are not to get used to the more attractive elements of each other's personalities.

  Married couples are likely to exert themselves for guests - being amusing, discussing with passion and point - and then to fall into dull exhausted silence when the guests have gone.

  As in all friendship, a husband and wife must try to interest each other, and to spend sufficient time sharing absorbing activities to give them continuing common interests. But at the same time they must spend enough time on separate interests with separate people to preserve and develop their separate personalities and keep their relationship fresh.

  For too many highly intelligent working women, home represents chore obligations, because the husband only tolerates her work and does not participate in household chores. For too many highly intelligent working men, home represents dullness and complaints - from an over-dependent wife who will not gather courage to make her own life.

  In such an atmosphere, the partners grow further and further apart, both love and liking disappearing. For too many couples with children, the children are allowed to command all time and attention, allowing the couple no time to develop liking and friendship, as well as love, allotting them exclusive parental roles.

  75. According to the passage, which of the following statements is CORRECT?

  A. Friendship in marriage means daily, year-long cohabitation.

  B. Friendship can be kept fresh by both separate and shared interests.

  C. Friendship in marriage is based on developing similar interests.

  D. Friendship in marriage is based on developing separate interest. 

  76. The passage suggests that married couples become___.

  A. unfriendly with guests

  B. uninterested in guests

  C. hostile when guests have left

  D. quiet when guest have left 

  77. The passage seems to indicate at the end that children___.

  A. help couples reinforce their friendship

  B. make no impact on the quality of friendship

  C. may pose obstacles in marital friendship

  D. command less time and care than expected 

  TEXT D

  Sending a child to school in England is a step which many parents do not find easy to take. In theory, at least, the problem is that there are very many choices to make. Let us try to list some of the alternatives between which parents are forced to decide. To begin with, they may ask themselves whether they would like their child to go to a single-sex school or a co-educational school. They may also consider whether he should go to a school which is connected to a particular church or religious group, or whether the school should have no such connections. Another decision is Whether the school should be one of the vast majority financed by the State or one of the very small but influential minority of private schools, though this choice is, of course, only available to the small number of those who can pay. Also connected with the question of money is whether the child should go to a boarding school or live at home. Then there is the question of what the child should do at school. Should it be a school whose curriculum lays emphasis, for instance, on necessary skills, such as reading, writing and mathematics, or one which pays more attention to developing the child's personality, morally, emotionally and socially. Finally, with dissatisfaction with conventional education as great as it is in some circles in England and certainly in the USA, the question might even arise in the parents' minds as to whether the child should be compelled to go to school at all. Although in practice, some parents may not think twice about any of these choices and send their child to the only school available in the immediate neighbourhood, any parent who is interested enough can insist that as many choices as possible be made open to him, and the system is theoretically supposed to provide them.

  78. Parents find choosing a school hard because___.

  A. there is a limited number of choices

  B. some schools are very expensive

  C. some schools are government schools

  D. they are faced with a variety of offers 

  79. According to the passage, some parents, if allowed, might let their children stay at home because they___.

  A. don't find conventional education satisfactory

  B. don't know how to choose among different schools

  C. intend to educate their children themselves

  D. find conventional education too expensive to pay for 

  80. What is implied at the very end of the passage ?

  A. Most parents are unconcerned about the choices available to them.

  B. Interested parents can request more school choices be open to them.

  C. The educational system may not provide as many choices as expected.

  D. Most parents usually send their children to the schools nearby.

  SECTION B SKIMMING AND SCANNING [5 MIN.]

  In this section there are seven passages with a total of ten multiple-choice questions. Skim or scan them as required and then mark your answers.

  TEXT E

  First read the following question.

  81. The main purpose of the passage is to___.

  A. warn people of pickpockets. B. tell people what to wear.C. describe how to catch thieves. D. explain how to contact the police. 

  Now, go through TEXT E quickly and answer question 81.

  Pickpockets operate in crowded places in the hope of getting easy pickings. Don't make it easy for them. Keep wallets, purses and other valuables out of sight. If wearing a jacket, an inside pocket is the best place to use. If not, your possessions are safest in a pocket with a button-down flap.

  Please co-operate with the police by reporting any crime or suspicious activity immediately, either by dialing 110 or calling at your nearest police station.

  TEXT F

  First read the following question.

  82. The main topic of the passage is ___.

  A. agricultural products B. irrigation methodsC. natural resources D. water shortages 

  Now, go through TEXT F quickly and answer question 82.

  It is widely accepted that China is a country faced with severe water shortages. Insufficient water resources have slowed agricultural development. And to

  make matters worse, some of the traditional Chinese irrigation methods have wasted an astonishing amount of water.

  In China today, the utilization efficiency of farming water is about 30-40 per cent. This figure stands in sharp contrast to developed countries' utilization average of 70-80 per cent. The low utilization efficiency has resulted from the adoption of some traditional Chinese irrigation methods.

  Only by using modern irrigation methods can we reduce water shortage in agriculture.

  One of the advantages of modern irrigation methods is that they alone can save 20-30 per cent of the present volume of wasted irrigation water.

  TEXT G

  First read the following question.

  83. The letter is about___.

  A. cities in South-east Asia B. holiday greetingsC. sightseeing D. travel plans 

  Now, go through TEXT G quickly and answer question 83.

  May 5th 2002

  Dear Mark,

  Hello again! Here are my holiday plans. I'll leave on a tour of South-east Asia in August and will arrive in Singapore in September. Hope we'll be able to

  meet there. These are my travel plans:

  August 28th London-Tokyo

  September 1st Tokyo-Bangkok

  September 4th Bangkok-Singapore

  September 7th Singapore-Manila

  September 9th Manila-London

  Looking forward to seeing you again.

  Best wishes

  Christopher

  TEXT H

  First read the following question.

  84. Who will read the following excerpt from a pamphlet?

  A. Travellers. B. Baby-sitters.C. Insurance agents. D. Trattic police. 

  Now, go through TEXT H quickly and answer question 84.

  DAY TRIPS

  Even if you are only going on a day trip to another country , accidents can happen. So please make sure you have adequate travel insurance.

  TAKE CARE IN WATER

  Bathing will cool you but remember that fatal accidents can happen very easily and in the most unexpected conditions. Adults should watch each other for signs of trouble when in water. Children should always be supervised by an adult who can swim well. Young children should never be left unattended near a stretch of water.

  TAKE CARE ON THE ROADS

  Traffic accidents are the major cause of death among travellers. Whether driver or pedestrian, always check on local traffic regulations.

  TEXT I

  First read the following questions.

  85. How many performances will the Irish dancing troupe give between June 23 and 25?

  A. One. B. Two. C. Three. D. Four. 

  86. Whose works will NOT be played at the concert?

  A. Chopin. B. Schumann. C. Beethoven. D. Liszt. 

  Now, go through TEXT I quickly and answer questions 85 and 86.

  Irish dance: The Irish International Dance Company, one of the most dynamic dance troupes in the world, will tour China with its classic production "Spirit of the Dance-the New Millennium."

  The dancers include such famous names as Patricia Murray, one of the Irish dancing champions, and first rate ballerina Claire Holding.

  Sponsored by China National Culture and Art Company Ltd., the dancing troupe will give three performances at the Century Theatre.

  Time:7:30 pm, June 23-25

  Place: Century Theatre, 40 Liangmaqiaolu, Chaoyang District

  Telephone: 6551 - 8888

  Piano solos: twenty Chinese and foreign piano music works will be playedby three young, promising pianists from the China Central Conservatory of Music.

  Programmes include: "Consolation No 3 in D-flat major" by Liszt, " For Elise" by Beethoven, "Turkish March" by Mozart, "Waltz in C-sharp minor" and "A Minute Waltz" by Chopin, and "Hungarian Dance" by Brahms.

  Time:7:30 pm, June 16

  Place: Beijing Concert Hall, 1 Beixinhuajie, Xicheng District

  Telephone: 6605- 5812

  TEXT J

  First read the following questions.

  87. When is the deadline for the competition?

  A. May 7. B. May 5. C. June 18. D. June 15. 

  88. The six lucky winners will ___.

  A. visit Guiyang City. B. contact the Press Office C. go to China Daily. D. take an overseas trip.

  Now, go through TEXT J quickly and answer questions 87 and 88.

  Guiyang Customs and Scenery Competition

  Notice

  Fifteen questions for the Guigyang Customs and Scenery Competition were published in China Daily on May 5 and 7, and on China Daily' s web edition on May 7. Participants, please answer the questions and mail the answer card to: Press Office, Guiyang Municipal People's Government 46 Zhongshan Xilu, Guiyang 550003, Guizhou, China or find the competition on www. chinadaily, com. cn. Then answer the questions, fill in all information needed and click the button below to send it back. The deadline for the competition is June 15(subject to postmark).

  The prize-drawing Ceremony will be held on June 18 in Guiyang City ,and six luck winners(three living in China, three from abroad)will be drawn from those who give correct answers to all questions. Their names will be published in China Daily and its web edition on June 19.

  The six lucky winners will be invited to visit Guiyang from August 8 to 18.

  TEXT K

  First read the following questions.

  89. If you want to travel to Shanghai on Air France on a Saturday, which flight would you take?

  A. AF129. B. AF128. C. AF111. D. AF112. 

  90. Does Lufthansa operate a flight between Beijing and Frankfurt everyday?

  A. No. B. Yes. C. Yes, except on Saturdays. D. No, only three days a week.

  Now, go through TEXT K quickly and answer questions 89 and 90.

  Flight Schedule

  Air France ……

  …… Tel: (010)6588 1388

  (020)6360 6688 

  Day From To Flight Departure Arrival

  1 - 7 Beijing Paris AFl29 09: 40 14:15

  1 - 7 Paris Beijing AFl28 15:55 07:40

  2,4,7 Shanghai Paris AFlll 10:55 17:05

  1,3,6 Paris Shanghai AFl12 15:55 09:05

  Lufthansa …… Tel: (010)6465 4488

  Tel: (010)6465 4488

  (021)6248 1100 

  Day From To Flight Departure Arrival

  1 - 7 Beijing Frankfurt LH721 10:30 14:25

  1 - 7 Frankfurt Beijing LH720 17:25 08:30

  1,2,3,4,5,7 Shanghai Frankfurt LH729 11:15 16:30

  1,2,3,4,5,7 Frankfurt Shanghai 111728 17:10 09:25

  Part ⅠWRITING [45 MIN.]

  SECTION A COMPOSITION

  Travel has become part of our life. And more and more of us have come to know the significance of travel through our own experience. Write on ANSWER SHEET ONE a composition of about 150 words on the following topic:

  TRAVEL BROADENS THE MIND 

   You are to write in three parts.

  In the first part, state what the topic actually means to you.

  In the second part, give one or two examples to illustrate your ideas.

  In the last part, bring what you have written to a natural conclusion or a summary.

  Marks will be awarded for content, organization, grammar and appropriacy. Failure to follow the instructions may result in a loss of marks.

  SECTION B NOTE-WRITING [10 MIN.]

   Write on ANSWER SHEET ONE a note of about 50-60 words based on the following situation:

  Yesterday you failed to turn up for the appointment with your teacher, Profess or Wang. Write him a note of apology and make a request for another meeting. You should also suggest the time for the requested meeting.Marks will be awarded for content, organization, grammar and appropriacy.

  PART Ⅱ DICTATION

  Characteristics of A Good Reader

  To improve your reading habits, you must understand the characteristics of a good reader. First, the good reader usually reads rapidly. Of course, he does not read every piece of material at the same rate. But whether he is reading a newspaper or a chapter in a physics text, his reading rate is relatively fast. He has learned to read for ideas rather than words one at a time. Next, the good reader can recognize and understand general ideas and specific details. Thus he is able to comprehend the material with a minimum of effort and a maximum of interest. Finally, the good reader has in his command several special skills, which he can apply to reading problems as they occur. For the college student, the most helpful of these skills include making use of the various aids to understanding that most text books provide and skim-reading for a general survey.

  PART Ⅲ LISTENING COMPREHENSION

  SECTION A  STATEMENT

  1. I have to teach the same course books several times in the summer holiday camp, which is sometimes boring and not well-paid, but by and large I'm quite delighted at being with young people.

  2. The poor living conditions in such a large city have resulted from the unplanned real estate development, which is rarely seen in small cities.

  3. At a recent seminar, many participants were worried about the fact that overpopulation may give rise to many social security problems.

  4. May I have your attention, please? Flight 998 is leaving at 11:30 a.m. Please check in half an hour prior to the departure.

  5. Having gone through your claims for fire damage, I don't think the policy you have provided protection against loss by fire.

  6. Ian lost one eye in a childhood accident, but he nonetheless had a very successful athletic career.

  7. Mr. and Mrs. Clark used to smoke. But now Mrs. Clark has stopped and she is afraid her husband will fall ill if he doesn't get rid of his bad habit of smoking both at home and at work.

  8. I heard from Mary that last semester, Susan found it difficult to stay on good terms with her roommate Jenny.

  9. Jack says that he is up to his eyes at work at present and really cannot afford the time to have dinner with us.

  SECTION B  CONVERSATION

  10. W: I want to find a part-time job during the summer vacation and earn some money. How about you?

  M: I'm going to take a few summer courses so that I can graduate early next year.

  11. W: Excuse me, I want some dictionaries. Where can I find them?

  M: The regular-priced ones are here and on that table in the corner of the room we have some on discount.

  W: Thank you.

  12. W: I wonder where I can take my girlfriend for dinner after work tonight.

  M: Have you been to the Chinese Restaurant near the school?

  13. M: Hello, the pipe in my bedroom is leaking. Can you come and get it repaired right away?

  W: Well, it depends on how soon I can finish the drains at the office building.

  14. W: Do you think you can play the music tape another time, dear? I've got a slight headache.

  M: Of course. Sorry. I didn't realize you could hear it. You want me to call the doctor?

  W: No, thanks. I'll be OK in a minute.

  15. M: Lisa, how are you getting along with our term paper?

  W: I've been writing and rewriting it. I simply don't know if I will ever get it finished.

  16. W: I must go to the library, the one near the laboratory, because I have to finish my research project by tomorrow. But if I could, I prefer to go with you to the theatre.

  M: I wish you could come along.

  17. M: Why did Jack suddenly decide to quit his job?

  W: He said he wouldn't break his back working for such low pay.

  M: I see.

  18. M: Are you sure you can remember the name of the film you saw last week?

  W: It's just on the tip of my tongue.

  SECTION C  NEWS BROADCAST

  News Item One (19-20)

  Commonwealth leaders agreed to lift Nigeria's 3-5 years' suspension on May 29, the day the military government hands over power to the elected president, the organization secretary general announced yesterday.

  Nigeria was suspended from the 54 nation group of mainly former British colonies in 1995 after it executed 9 minority rights activists including writer Ken Thawrawiwa. But now that the country has embarked on the return to democracy, Commonwealth heads of government have agreed to end this estrangement. Secretary general chief Ormiga Anyaco said in a statement: "I'm delighted an unfortunate episode in Nigeria Commonwealth relations will now come to an end and Nigeria is resuming its rightful place in the Commonwealth."

  News Item Two (21-22)

  The Space Shuttle Discovery made a real night landing at the Kennedy Space Center early on Thursday. The night landing, the 11th in the centre's 94 shuttle missions, ended a 10-day mission to outfit the orbiting international space station. Although the space craft created a solid boom that can be heard along much of Florida's eastern seaboard, witnesses on the ground could not see the orbiter until it was directly over the one-way lights. Scattered showers off the Florida coast had threatened to postpone the shuttle's return. But broadcasters gave a green light when they decided no rain will fall within 48 kilometres of the space centre.

  News Item Three (23-24)

  Five people died, two were missing and at least 18 were injured on Wednesday when an Italian petrol vessel collided with a dinghy filled with refugees crossing the Adriatic sea from Albania, authorities said. The victims were believed to be Albanians from either Albania or Kosovo, said authorities from Italy's Tax Police Division, which, along with the coast guard, patrols the nation's coast. The cause of the collision was not immediately known. Three Albanians,believed be smuggling the refugees were arrested a few hours after the accident.

  News Item Four (25)

  Malaysian authorities are discussing possible salvage efforts with Sun Cruisers, the Singapore owner of a large liner, that sunk off Malaysia last week, a news report said yesterday. Sun cruisers had received some advice from Malaysia on the matter. The Business Times newspaper quoted the company's spokeswoman Judy Shoo Asian. Judy and other Sun Cruiser's officials could not immediately be reached for further comment as they were away in Indonesia. The Sun Vista went down in international waters. The nearby Malaysia may have the right to order the wreck's removal, the newspaper said. Salvage experts said the wreck of the Sun Vista, which sank in 65 metres of water, poses no threat to ships passing over it. But Malaysia may still want it removed.

  Nigeria was suspended from the 54 nation group of mainly former British colonies in 1995 after it executed 9 minority rights activists including writer Ken Thawrawiwa. But now that the country has embarked on the return to democracy, Commonwealth heads of government have agreed to end this estrangement. Secretary general chief Ormiga Anyaco said in a statement: "I'm delighted an unfortunate episode in Nigeria Commonwealth relations will now come to an end and Nigeria is resuming its rightful place in the Commonwealth."

  SECTION A STATEMENT

  1. 答案:C

  【问句译文】讲话者喜欢教书是因为什么?

  【试题分析】本题为细节题。

  【详细解答】解答本题的关键是听清"I'm quite delighted in being with young people",也就是说他喜欢教书是因为喜欢和年轻人在一起,故选项C为正确答案。

  2. 答案:B

  【问句译文】讲话者是什么意思?

  【试题分析】本题为细节题。

  【详细解答】只要听清"…the poor living conditions…resulted from…unplanned realestate development…",就可知道很差的住居条件是由于没有好的规划,故选项B为正确答案。

  3. 答案:D

  【问句译文】这句话是什么意思?

  【试题分析】本题为细节题

  【详细解答】。解答本题的关键是听清"…overpopulation may give rise to many social security problems."就可得知人口过多会带来社会安全问题,故选项D 为正确答案。

  4. 答案:D

  【问句译文】乘坐998航班的乘客何时办理登机手续?

  【试题分析】本题为计算题。

  【详细解答】解答本题的关键是听清两个时间,"Flight 998 is leaving at ll:30…"和"…check in half an hour prior to departure",由此可以计算出办理登机手续的时间为11:00。

  5. 答案:A

  【问句译文】说话者是何职业?

  【试题分析】本题为推理题。

  【详细解答】解答本题的关键词有"policy"(保险单),"fire damage"(火灾损失),"loss"(损失)等,由此可以推测说话人的职业为insurance agent(保险经纪人)。

  6. 答案:A

  【问句译文】说话者认为怎样?

  【试题分析】本题为细节题。

  【详细解答】解答本题的关键是听清关键句"…but he nonetheless had a very successful athletic career",由此可知,Lan成为一名成功的运动员,故选项A 为正确答案。

  7. 答案:A

  【问句译文】Clark 夫人担忧什么?

  【试题分析】本题为细节题。

  【详细解答】只要听清"she's afraid her husband will fall ill…",就可得知她担心的是丈夫的健康。

  8. 答案:D

  【问句译文】Susan和Jenny关系如何?

  【试题分析】本题为细节题。

  【详细解答】解答本题的关键是听清"…Susan found it difficult to stay on good terms with her roommate Jenny",这里的"stay on good terms with"意为"与(某人)相处得好"由此可知,Susan与她关系紧张,故选项D 为正确答案。

  9. 答案:B

  【问句译文】我们知道Jack怎样?

  【试题分析】本题为细节题。

  【详细解答】解答本题的关键是听清并理解短语"up to one's eyes"的意思,该短语意为"深深陷在……,埋头于………"。由此可知, Jack正在埋头工作,故选项B 为正确答案。

  SECTION B CONVERSATION

  10. 答案:B

  【问句译文】他们主要谈论什么?

  【试题分析】本题为细节题。

  【详细解答】解答本题的关键是听清关键词 "summer vacation",对话者分别谈到几件不同的事情,但都是围绕"summer vacation"来谈的,故选项B 为正确答案。

  11. 答案:B

  【问句译文】对话很可能发生早哪儿?

  【试题分析】本题为推理题。

  【详细解答】解答本题的关键是听清关键词"dictionary","price","discount"等,显然是一人要买词典,询问价钱,还谈到折扣,由此可以推测两人对话的地点是在书店。

  12. 答案:D

  【问句译文】两位说话者是什么关系?

  【试题分析】本题为推理题。

  【详细解答】男士提出自己的问题是"I wonder where I can take my girlfriend for dinner after work tonight",女士给他以建议"Have you been to the Chinese restaurant near the school."两人之间最不可能是"夫妻"、"律师与当事人"或"顾客与侍者"的关系,而是"同事"关系。

  13. 答案:A

  【问句译文】从对话可以推理出那位男士是干什么的?

  【试题分析】本题为推理题。

  【详细解答】解答本题的关键是听清关键词"pipe","leaking","get it repaired",由此不难推测男士的职业是一名水暖工。

  14.答案:A

  【问句译文】那位男士接下来会做什么?

  【试题分析】本题为推理题。

  【详细解答】女士请求男士"play the music tape another time",男士回答说"of course",由此可以推测男土会"turn off the tape recorder",故选项A 为正确答案。

  15.答案:B

  【问句译文】Lisa觉得她的工作如何?

  【试题分析】本题为推理题。

  【详细解答】男士问女士的论文写得怎样了, 女士的回答有"writing and rewriting…",

  "…simply don't know if I'll ever get it finished",表现出一种厌倦、疲累的情绪,故选项B为正确答案。

  16.答案:A

  【问句译文】 女士要去哪儿?

  【试题分析】本题为推理题。

  【详细解答】女士说"But if I could,I'd prefer to go with you to the theater",这里用的是虚拟语气,表示与现实相反的情况,那么现在女士就不会去看戏,而是去图书馆故选项A为正确答案。

  17.答案:B

  【问句译文】Jackson为何换了工作?

  【试题分析】本题为细节题。

  【详细解答】解答本题的关键是听清关键句"…wouldn't break his back working for such low pay",由此可以推理出Jackson辞掉工作原因是"报酬太低",故选项B为正确答案。

  18.答案:C

  【问句译文】关于那场电影,女士说了什么?

  【试题分析】本题为细节题。

  【详细解答】解答本题的关键在于听懂短语"on the tip of one's tongue",该短语意为"即将想起(某事),(某事)就在嘴边",即她只是暂时忘了它的名字,故选项C为正确答案。

  SECTION C NEWS BROADCAST

  19.答案:C

  【问句译文】Nigeria何时回归英联邦的?

  【试题分析】本题为主旨题。

  【详细解答】解答本题的关键是听清新闻的第一句中,"Common wealth leaders agree to lift Nigeria's three and a half years' suspension…"接着讲了Nigeria回归英联邦的时间,即"the military government hands over power to an elected president",故选项C为正确答案。

  20.答案:A

  【问句译文】前英帝国共有多少个殖民国?

  【试题分析】本题为细节题

  【详细解答】。新闻第二句说"Nigeria was suspended from the fifty-four nation-groupof mainly former British colonies in l995…",由此可知,前英帝国共有54个殖民国。

  21. 答案:C

  【问句译文】"发现"号飞船执行了多少天的任务后回到肯尼迪航天中心的?

  【试题分析】本题为细节题。

  【详细解答】新闻第二句说"…ended a ten-day mission …",由此可知,"发现"号飞船用10天时间完成了任务。

  22. 答案:A

  【问句译文】飞船何时着陆?

  【试题分析】本题为细节题。

  【详细解答】新闻第三句说"…the spacecraft created a sonic boom…",由此可知,此时为飞船将要着陆时发出巨大的噪音。

  23.答案:D

  【问句译文】事故中有多少人死亡?

  【试题分析】本题为细节题。

  【详细解答】新闻第一句说"Five people died, two were missing…"即事故中有5人死亡。

  24.答案:B

  【问句译文】三位阿尔巴尼亚人为何被捕?

  【试题分析】本题为细节题。

  【详细解答】新闻最后一句说"Three Albanians,believed to have smuggled the refugees, were arrested a few hours after the accident."由此可知,那三位阿尔巴尼亚人被逮捕的原因是因为他们被怀疑在偷渡难民,故选项B为正确答案。

  25. 答案:A

  【问句译文】这篇新闻主要是关于什么的?

  【试题分析】本题为主旨题,要求概括本篇新闻的报道内容。

  【详细解答】新闻第一句说"…discussing possible salvage efforts with Sun Cruisers,…",由此可知,新闻报道的是有关方面正在协商营救一艘沉船的计划,故选项B为正确答案。

  完型填空

  PART IV CLOZE  

  短文大意:这篇文章讨论的是翻译所需的技能和素质。

  26. 答案:B

  【试题分析】本题为近义名词词义辨析题。

  【详细解答】这四个选项都有"应用,运用"的意思。application指"(理论知识的)应用";use表示"(工具等的)使用";utility指"实用",有效地使用某物使其发挥效用;usage侧重于"具体的用法"。此处是指语言的运用,故应该选 use。

  27. 答案:D

  【试题分析】本题为语法题,考查关联词的用法。

  【详细解答】 more than常用于比较级,意为"比…更加…";except for意为"除…外"的意思;because of意为"由于,因为";in addition to意为"加上;除…外,还有"。这里是说翻译人员除了应具备前面所说的基本知识外,还应有好学的品质,故选项D为正确答案。

  28. 答案:D

  【试题分析】本题考查对固定搭配的掌握。

  【详细解答】 on one's own指依靠某人自己的力量做某件事,"独自,独立"。of one'sown指某人自己拥有的东西;介词by和for都不与own搭配。根据句意,选项D为正确答案。

  29. 答案:A

  【试题分析】本题为语法题,考查对虚拟语气的掌握。

  【详细解答】本句是一个表示与将来事实相反的虚拟语气句。表示条件的从句中没有if,应用部分倒装。故选项A为正确答案。

  30. 答案:B

  【试题分析】本题考查对固定搭配的掌握。

  【详细解答】 a nodding acquaintance意为"点头之交";familiarity with意为"通晓;精通";acquaintance with指"(从经验所得的)知识;了解";knowledge指"知识",skill指"技能;熟练",都与of连用。

  31. 答案:D

  【试题分析】本题为近义动词词义辨析题。

  【详细解答】这四个选项都有"转变,变更"的意思。change意为"改变,变化",常用于change from…to…(由……转变成……);transform意为"使转化";turn意为"转变",常与介词from, into搭配;switch意为"转换",指完全或突然的转变。本句是指从一种语言转换到另一种语言,应用动词switch。 

  32. 答案:B

  【试题分析】本题考查对固定搭配的掌握。

  【详细解答】 lack 常用于短语"lack of"; require常用于"require sth. of sb.";face常用于"be faced with";confront常用于"be confronted with"。故选项B为正确答案。

  33.答案:B

  【试题分析】本题考查对上下文的理解。

  【详细解答】上文中的"the nature of the translator's work"就是"the processing ofthe written word",而下文"this skill"指的是"be able to speak the languages he is dealing with",故此处应选unnecessary,即他不必会讲他所处理的语言。

  34.答案:C

  【试题分析】本题考查连接词的用法。

  【详细解答】此处需要一个连词将"an advantage"和"a hindrance"连接起来。根据上下文,此处是肯定advantage否定hindrance,所以应选rather than表示"而不是"。

  35.答案:D

  【试题分析】本题考查对固定搭配的掌握。

  【详细解答】 concern常与about搭配,意为"关心",而不与with搭配;deal with意为"处理";work with意为"工作",都不符合句意。do away with意为"除掉,取消"。这里指"没有掌握说这些语言的技巧",故选项B为正确答案。

  36.答案:A.

  【试题分析】本题考查连词的用法。

  【详细解答】上文说的是"unnecessary",而下文说的是"desirable",所以此处应该填一个表示转折关系的连词。四个选项中,只有however是表转折的连词,其他选项都是表示结果关系的连词,故选项A为正确答案。

  37.答案:A

  【试题分析】本题为语法题,考查对非谓语动词的掌握。

  【详细解答】 be restricted to中to是介词,后面要求跟一个名词,故选项A为正确答案。

  38.答案:C

  【试题分析】本题考查对固定搭配的掌握。

  【详细解答】 refer to意为"提到,说起";come to意为"谈到,关于";apply to 意为"应用,适用";amount to意为"相当于,总共达"。这里是说用源语言写作的能力也是如此,故应选apply to。

  39.答案:A

  【试题分析】本题考查对固定搭配的掌握。

  【详细解答】 it does not matter意思是"不要紧,没关系"。这里是说如果译者没有这种能力,也不要紧。

  40.答案:B

  【试题分析】本题为形容词词义辨析题。

  【详细解答】 characteristic意为"特征;特色",比较注重外在的;quality指意为"特质;品质;素质",比较注重内在的;distinction意为"区别;分别";feature意为"特征,特色",侧重具体的某个特征。这里指内在品质,与skills并列,故应选quality。

  选择

  PART V GRAMMAR AND VOCABULARY

  41.答案:C

  【参考译文】我不能去,一则我没钱,再则我手头上的事情太多。

  【试题分析】本题考查对固定搭配的掌握。

  【详细解答】 what's  more意为"更甚的是";as well意为"也" ;for one thing…for another…意为"一则……再则……";in addition"此外"。根据句意,选项C为正确答案。

  42.答案:D 

  【参考译文】即使在她还是一个小姑娘的时候,梅丽莎就知道表演将成为她的生活,戏剧观众将成为她最好的老师。

  【试题分析】本题为语法题,考查对句式结构的理解。

  【详细解答】从句子结构看,本句缺少表示某个人的主语。在这四个选项中,只有 as可充当表示某个人的主语,故为正确答案。

  43.答案:D

  【参考译文】何不明天再去看他呢?

  【试题分析】本题考查对习惯用法的掌握。

  【详细解答】 why not是习惯用法,其后直接跟动词原形,常用作建议,意思是"为什么不……"。

  44.答案:B

  【参考译文】毫无疑问,该公司的销售计划是正确的。

  【试题分析】本题考查对习惯用法的掌握。

  【详细解答】 There is no doubt…后面应接由that引导的主语从句,故只能选B。

  45.答案:A

  【参考译文】智力对于大脑,就像视力对于躯体一样。

  【试题分析】本题为语法题,考查对句子结构的理解。

  【详细解答】 what引导的是一个类比性状语从句,说明主句行为的方式、方法,其结构为"A is to B what C is to D",意思是"A对于B来说,就像C对于D一样"。根据句意,选项A为正确答案。

  46.答案:C

  【参考译文】尽管我对此很同情,但我的确不能做太多的事情来帮他们摆脱困境。

  【试题分析】本题为语法题,考查对让步状语从句的理解。

  【详细解答】 while意为"虽然";as常引导原因状语从句;as long as意为"只要";even意为"即使"。根据句意,选项C 为正确答案。

  47.答案:A 

  【参考译文】病人的情况好转让人感到鼓舞,因为他不用人帮忙也能勉强下床。

  【试题分析】本题为语法题,考查副词的用法。

  【详细解答】 can hardly和can barely意为"几乎不能",是否定副词,不合题意。nearly意为"几乎";merely意为"仅仅"。根据句意,此处需要填表肯定性副词,故选项A 为正确答案。

  48.答案:A

  【参考译文】他胆小得不敢向最要好的朋友说真话。

  【试题分析】本题考查对习惯用法的掌握。

  【详细解答】 too…to…是习惯用法,意为"太……以至于不能……"。根据句意,选项A为正确答案。

  49.答案:B

  【参考译文】贝利比他母亲多具备一个优势,因为他会讲法语。

  【试题分析】本题为语法题,考查连词的用法。

  【详细解答】 in that意为"因为,既然",是一个表弱原因关系的连词,一般用于书面语,故为正确答案。

  50.答案:C 

  【参考译文】你不必担心手术费用

  【试题分析】本题考查对固定搭配的掌握。

  【详细解答】在四个选项中,只有as可与regards搭配。as regards意为"关于,在……方面,至于"等意思。

  51.答案:D

  【参考译文】在生活中,个子长不高并非一个十分严重的不利条件。

  【试题分析】本题为语法题,考查非谓语动词的用法。

  【详细解答】本题是不定式或分词的否定形式作主语,其正确形式应分别是not to be和not being。当分词作主语时,短语若需否定,否定词应放在动名词前。根据句意,该题应选分词作主语,故选项D为正确答案。

  52.答案:B

  【参考译文】在饥荒中,许多人沦落到数日无饭可吃的地步。

  【试题分析】本题考查动词的用法。

  【详细解答】 reduce与介词 to搭配,意为"使陷入不良境遇";sink意为"下沉;倒下"; force意为"迫使",常带有主观色彩,受某种主观力量的驱使;declined意为"衰落;降低"。根据句意和句中介词to,本句应选B。

  53.答案:B

  【参考译文】计算机可通过编程处理各种各样的任务。 

  【试题分析】本题为动词词义辨析题。

  【详细解答】 assign意为"分配";tackle意为"着手处理,对付"; realize意为"意识到" solve意为"解决,解答"。这里要表达的是"处理各种各样的任务",故选项B为正确答案。

  54.答案:A

  【参考译文】球队试图破门得分的努力因对方守门员而受挫。

  【试题分析】本题为动词词义辨析题。

  【详细解答】 frustrated意为"受挫";prevented意为"受阻止",后接介词from;discouraged意为"使沮丧",侧重于情绪; accomplished意为"完成"。根据句意,选项A为正确答案。

  55.答案:C

  【参考译文】我只跟那个人面熟,但从未说过话。

  【试题分析】本题考查对固定搭配的掌握。

  【详细解答】 by chance意为"偶然";by heart意为"用心";know sb by sight意为"跟某人面熟";by experience"通过经验"。根据句意,选项C为正确答案。

  56.答案:B 

  【参考译文】由于是色盲,Sally不能够区分红色和绿色。

  【试题分析】本题为名词词义辨析题。

  【详细解答】 difference意为"差别;差异",是指事物自身客观存在的差异; distinction意为"区别;分别",指主观意识能够辨别客观存在的差异;comparison意为"比较",指将两物放在一起进行对比的过程;division意为"分开;分配",并不强调事物的差异和关联,只是一个简单的外部行为。根据句意,选项B为正确答案。

  57.答案:C

  【参考译文】你必须要求学生给出真实的、与现实一致的答案。

  【试题分析】本题考查对固定搭配的掌握。Relevant与介词to搭配,意为"相关的";simultaneous意为"同时发生的";consistent 常构成短语be consistent with,意为"与……一致,与……共存的"; practical意为"实际的"。根据句意,选项C为正确答案。

  58.答案:C

  【参考译文】为了筹集资金, Nicola姑姑不得不卖掉她的一些最珍爱的财物。

  【试题分析】本题考查对固定搭配的掌握。

  【详细解答】 divide意为"分割;分开",侧重将一个整体分成几部分;separate意为"分开;分离";part with sth.意为"放弃",尤指并非出自自愿地放弃;abandon意为"放弃",为及物动词。根据句意,选项C为正确答案。

  59.答案:A

  【参考译文】几个月前刚买时我那辆车性能正常。

  【试题分析】本题考查对固定搭配的掌握。

  【详细解答】 in order表示"按顺序,井然有序,处于良好(或正常工作)状态,合宜的"。in(good或bad)form一般指(运动员、赛马等)"处于良好(不好)竞技状态"。in good (working) order意为"处于良好的工作状态"。in a state在口语中表示"不整洁,凌乱;兴奋;焦急"等。根据句意,选项A为正确答案。

  60.答案:B

  【参考译文】那个顾客不喜欢那顶宽边帽。

  【试题分析】本题考查对固定搭配的掌握。

  【详细解答】 disapproval 与介词of搭配,意为"不赞成,反对";distaste与介词for搭配,意为"不喜欢,厌恶";dissatisfaction与介词with搭配,意为"不满";dismay与介词at 搭配,意为"灰心,沮丧"。从句意和搭配上讲,选项B为正确答案。

  61.答案:D

  【参考译文】农民要修粮仓,建篱笆,种庄稼,养牲畜,必须是个多面手。

  【试题分析】本题为形容词词义辨析题。

  【详细解答】 restless意为"得不到休息的";skilled意为"有技能的,熟练的";strong意为"强壮的";versatile意为"多才多艺的,多面手"。句意可知,农民必须多才多艺,故选项D为正确答案。

  62. 答案:B

  【参考译文】与他的收入相比,他在度假和买豪华物品上的开支偏高了。

  【试题分析】本题考查对固定搭配的掌握。

  【详细解答】 in proportion to意为"与……成比例"。comparison意为"比较",常用于by/in comparison with结构中,意为"与……相比";association意为"联系;合伙",常用于in association with结构中,意为"与……联系;与……合伙";calculation意为"计算"。根据句意和搭配,选项B为正确答案。

  63.答案:C

  【参考译文】尽管富裕了,他还是很节俭。

  【试题分析】本题为形容词词义及用法辨析题。

  【详细解答】 economic意为"经济的",常用作定语,直接修饰中心词,不与介词搭配;thrifty意为"节俭的;节省的",常与with搭配;frugal意为"节约的;俭朴的",既可做前置定语,也可作表语,后与of搭配;careful意为"小心的"。故选项C为正确答案。

  64.答案:C

  【参考译文】由于经理出差,我受命主持每周的职工大会。

  【试题分析】本题为动词词义及用法辨析题。

  【详细解答】 preside和chair都可作"主持会议"讲,但preside需与介词at或over搭配使用,chair则可直接跟会议;introduce意为"介绍";dominate意为"支配,控制"。根据句意,选项C为正确答案。

  65.答案:A

  【参考译文】那个词的起源虽不为人所知,但肯定不是来自希腊语。

  【试题分析】本题为名词词义辨析题。

  【详细解答】 origin意为"起源,开端",往往与过去的活动、风俗、事件等有关;generation意为"产生,发生"; descent意为"祖籍,血统"; cause意为"原因,起因"。这里是说字母的起源,故选项A为正确答案。

  阅读 A

  PART VI READING COMPREHENSION  

  SECTION A READING COMPREHENSION

  TEXT A  

  短文大意:本文讲述Mrs.McGillicuddy在乘火车经过一个站台前后的一段奇特见闻。

  66.答案:B

  【参考译文】当McGillicuddy夫人乘坐的火车经过车站时,它如何?

  【试题分析】本题为细节题。

  【详细解答】短文开头就说 "The train clattered over points and passed through astation.""Then it began suddenly to slow down…",由此可知,火车既没有突然加速,也没保持原速,更没有立刻停下来,而是改变了它的速度。所以选项B为正确答案。

  67.答案:A

  【参考译文】McGillicuddy夫人是一个怎样的人?

  【试题分析】本题为推理题。

  【详细解答】这篇短文有很多细节描写,如有几列火车,各列车何时加速何时减速,怎么样平行行驶,在不同的时间有不同的情况发生,这些都是以McGillicuddy夫人的视角展开描写的。文中还写到,尽管大多数窗帘拉上了,但是她还是看见了另一列火车上的杀人案。由此可见,McGillicuddy夫人是一个善于观察的人。

  68.答案:D

  【参考译文】看见并行的火车上发生的事,她感觉如何?

  【试题分析】本题为细节题。

  【详细解答】短文第四段说"Then she drew her breath in with a gasp and half-rose to her feet",即可怕的景象吓得她发呆,故选项D为正确答案。

  69.答案:B

  【参考译文】她为什么没有马上按响她所乘的火车的警报?

  【试题分析】本题为细节题。

  【详细解答】短文倒数第二段说"…Mrs.McGillicuddy's hand went up to the communication cord,then paused,irresolute.After all,what use would it be ringing the cord of the train in which she was travelling?" McGillicuddy夫人看到的情景发生在另一列火车上,那么按响她所乘的火车的警报是不起任何作用的,也就是说她认为这样做没什么意义,故选项B为正确答案。

  TEXT B  

  短文大意:本文写的城乡生活的差异及作者的看法。

  70.答案:C

  【参考译文】第一段作者流给我们的印象是什么?

  【试题分析】本题为细节题。

  【详细解答】短文第一段第一句说"I am one of the many city people…if it weren't for my job I would…"说明作者是在城里工作,故选项C为正确答案。

  71.答案:A

  【参考译文】根据作者的意见,下列哪一项不会引起城里人不幸福?

  【试题分析】本题为细节题。

  【详细解答】短文第二段描写的是城市生活的弊端。由"The sense of belonging to a community tends to disappear…people on the same floor in tower blocks don't even say hello to each other."可知,城市生活"lack of communication";由"The majority of the population live in massive tower blocks,noisy,dirty and impersonal…All you can see from your window is sky,or other blocks of flats."可知,城市生活"housing conditions";由"Children…cooped up at home all day…their mothers feel isolated from the rest of the world."可知,城市生活是"a sense of isolation"。文中没有提及"a strong sense of fear",故A为正确答案。

  72.答案:C

  【参考译文】根据短文,在乡村不容易买到的东西是哪一项?

  【试题分析】本题为推理题。

  【详细解答】短文第三段倒数第二句说"Shopping becomes a major problem, and for anything slightly out of the ordinary you have to go on an expedition to the nearest large town.",即住在一个小村庄里,购物是一个大问题,即使只是稍微特别的物品,人们也必须到附近大城镇去买。由此可知,在乡村不容易买到的东西应是designer clothes。

  73.答案:C

  【参考译文】根据短文,用下列哪一个词来描述在大城市工作而在乡村生活的人最合适?

  【试题分析】本题为语义题。

  【详细解答】短文第四段最后一句说"They generally have about as much sensitivity…the unwilling original inhabitants of the villages."即他们一般还是比较敏感的,……他们也把某些改变强加给当地农村居民。故用"Arrogant" (骄横的)来描述在大城市工作而在乡村生活的人最合适。

  74.答案:B

  【参考译文】你认为作者会搬迁到农村去吗?

  【试题分析】本题为推理题。

  【详细解答】最后一段说"I'm keen on the idea, but you see there's my cat, Toby.I'm not at all sure that he would take to all that fresh air and exercise in the long grass. I mean, can you see him mixing with all those hearty males down the farm? No, he would rather have the electric imitation-coal fire any evening."作者以他的猫为例,说他没有把握它一定会喜欢新鲜空气及到草地练习,并在最后作了否定的答复。可见,作者是不会去农村生活的。

  TEXT C  

  短文大意:本文讨论的是如何维持健康的婚姻,保持婚姻活力。

  75.答案:B

  【参考译文】根据短文,下列哪一种说法是正确的?

  【试题分析】本题为概括题。

  【详细解答】短文第四段说"As in all friendship, a husband and wife must try to interest each other, and to spend sufficient time sharing absorbing activities to give them continuing common interests. But at the same time they must spend enough time on separate interests with separate people to preserve and develop theirseparate personalities and keep their relationship fresh."即正如所有友情一样,夫妇必须尽力保持对彼此的兴趣,给予充足的时间来分享有趣的活动,以不断提供共同兴趣;同时还必须花费足够的时间于个人兴趣方面,以保持独立的个性,并使他们的关系保持新鲜。选项B全面地概括了这短话的意思,故为正确答案。

  76.答案:D

  【参考译文】短文认为以婚夫妇变得怎样?

  【试题分析】本题为细节题。

  【详细解答】短文第三段说"Married couples are likely to exert themselves for guests…and then to fall into dull exhausted silence when the guests have gone", 即已婚夫妇很可能尽力使客人高兴,谈话时风趣、有感情且锐智;当客人走后,便陷入沉默。故选项D为正确答案。

  77.答案:C

  【参考译文】短文最后似乎暗示孩子怎样?

  【试题分析】本题为细节题。

  【详细解答】短文最后一段说"…the children are allowed to command all time and attention,allowing the couple no time to develop liking and friendship,as well as  love…",由此可知,夫妻间感情变淡甚至消失的原因可能是孩子占用了两人过多的时间,孩子成为了婚姻关系的障碍,故选项C为正确答案。

  TEXT D  

  短文大意:本文介绍的是英国父母是如何为孩子选择学校的。

  78.答案:D

  【参考译文】为何父母给孩子选择一所学校很困难?

  【试题分析】本题为细节题。

  【详细解答】短文的第二句说"In theory,at least,the problem is that there are very many choices to make."即在理论上,父母至少有很多选择,文章还列举了很多可供选择的学校模式。故父母给孩子选择一所学校很困难是因为"有太多的选择摆在他们面前"。

  79.答案:A

  【参考译文】根据短文,如果同意的话,为何有的父母可能让他们的孩子呆在家里?

  【试题分析】本题为细节题。

  【详细解答】短文倒数第二句说"Finally,with dissatisfaction with conventional education…the question might even arise in the parents' mind as to whether the child should be compelled to go to school at all."即,由于对传统教育不满意,父母们会考虑孩子是不是非得要去学校,故选项A为正确答案。

  80.答案:B

  【参考译文】短文最后暗示了什么?

  【试题分析】本题为细节题。

  【详细解答】短文最后一句说"Although in practice, some parents may not think twice about any of these choices and send their child to the only school available in the immediate neighbourhood, any parent who is interested enough can insist that as many choices as possible be made open to him, and the system is theoretically supposed to provide them." 这里是说,尽管实际上有些父母想都不想就把子女送到了附近唯一的一所学校,但是有足够兴趣的父母能够坚持要求开放尽可能多的选择,并且,这种选择学校的体制从理论上讲,应该提供这些选择。由以可知,教育制度并没有提供那么多可供选择的机会,故选项B为正确答案。

  阅读 B

  TEXT E  

  短文大意:这篇短文向我们介绍的是防备小偷的方法。

  81.答案:A

  【参考译文】该短文的写作意图是什么?

  【试题分析】本题为主旨题。

  【详细解答】从"Pickpockets operate in crowded places…. Don't make it easy for them"以及"co-operate with the police by reporting any crime…"可以看出这篇文章

  主要教人们如何警惕扒手,故选项A为正确答案。

  TEXT F  

  短文大意:这篇短文介绍的是中国由传统灌溉向现代灌溉方法的转变。

  82. 答案:B

  【参考译文】这篇短文的主题是什么?

  【试题分析】本题为主旨题。

  【详细解答】短文第一段谈到"some of the traditional Chinese methods have wasted an astonishing amount of water."第二段谈"The low utilization efficiency has resulted from the adoption of some traditional Chinese irrigation methods."其他两段也都谈到了"modern irrigation methods",可见这篇短文的中心论题就是irrigation methods。

  TEXT G  

  短文大意:这是写给朋友的一封有关个人假期旅行计划的信。

  83.答案:D

  【参考译文】这封信是关于什么的?

  【试题分析】本题为主旨题。

  【详细解答】本题问的是这封信的内容。信中提到"Here are my holiday plans.I'll leave on a tour of South-east Asia…"由此可知,这是一份假期到东南亚旅行的计划,信中详细列出了日期及旅行路线。

  TEXT H  

  短文大意:这篇短文介绍的是旅行中应注意的问题。

  84.答案:A

  【参考译文】谁会看这篇手册节选?

  【试题分析】本题为主旨题。

  【详细解答】从第一句"Even if you are only going on a day trip…",以及"Bathing will cool you but remember" 可知,旅游者会看这篇手册节选。

  TEXT I  

  短文大意:这是一篇有关最近的文化艺术表演活动的海报。

  85. 答案:C

  【参考译文】爱尔兰舞蹈团在6月23日至25日期间将演出几场呢?

  【试题分析】本题为细节题。

  【详细解答】海报第三段告诉我们"the dancing troupe will give three performances at the Century theatre."由此可知,正确答案为选项C。

  86. 答案:B

  【参考译文】钢琴独奏中将不会演出谁的作品呢?

  【试题分析】本题为细节题。

  【详细解答】在介绍钢琴独奏(Piano solos)时,海报中提及"… by Liszt, …by Mozart…,by Chopin,…by Brahms."只有Schumann没有提到,由此可知将不会演出Schumann的作品。

  TEXT J  

  短文大意:这是一则有关举办贵阳习俗及风景知识竞赛的通知。

  87. 答案:D

  【参考译文】竞赛的截止日期是哪一天?

  【试题分析】本题为细节题。

  【详细解答】文中有"The deadline for the competition is June 15." 由此可知,正确答案为选项D。

  88. 答案:A

  【参考译文】六名幸运的获胜者将会怎样?

  【试题分析】本题为细节题。

  【详细解答】文章最后一句说"The six lucky winners will be invited to visit Guiyang from August 8 to 18."由此可知,他们将游览贵阳。

  TEXT K  

  短文大意:这是一个航班时刻表,列出了两家航空公司的航班信息。

  89. 答案:D

  【参考译文】如果想在星期六乘坐Air France公司的航班去上海,应该乘座哪一班机呢?

  【试题分析】本题为细节题。

  【详细解答】在"Air France"下找到终点站Shanghai,对应的班机为AFll2,在星期六可以乘坐。

  90. 答案:B

  【参考译文】Lufthansa公司是否每天都有航班来往于北京和法兰克福之间?

  【试题分析】本题为细节题。

  【详细解答】。在"Lufthansa"下发现"1-7 Beijing Frankfurt LH721...; 1-7  Frankfurt Beijing LH720...",所以答案是肯定的,即从星期一到星期天每天都有班机在两地之间来往。


2002年英语专业八级试题及答案


  PART Ⅰ LISTENING COMPREHENSION [40 min]

  In Sections A, B and C you will hear everything ONCE ONLY. Listen carefully and then answer the questions that follow. Mark the correct response to each question on your COLOURED ANSWER SHEET.

  SECTION A TALK

  Questions 1 to 5 refer to the talk in this section. At the end of the talk you will be given 15 seconds to answer each of the following five questions.

  Now listen to the talk.

  1. According to the passage, during the 18th and 19th centuries cities were small in size mainly because ______.

  〔A〕 the urban population was stable

  〔B〕 few people lived in cities

  〔C〕 transport was backward

  〔D〕 it was originally planned

  2. Cities survived in those days largely as a result of ______.

  〔A〕 the trade activities they undertook

  〔B〕 the agricultural activities in the nearby areas

  〔C〕 their relatively small size

  〔D〕 the non-economic roles they played

  3. City dwellers were engaged in all the following economic activities EXCEPT ______.

  〔A〕 commerce

  〔B〕 distribution

  〔C〕 processing

  〔D〕 transportation

  4. Urban people left cities for the following reasons EXCEPT ______.

  〔A〕 more economic opportunities

  〔B〕 a freer social and political environment

  〔C〕 more educational opportunities

  〔D〕 a more relaxed religious environment

  5. Why did the early cities fail to grow as quickly as expected throughout the 18th century?

  〔A〕 Because the countryside attracted more people.

  〔B〕 Because cities did not increase in number.

  〔C〕 Because the functions of the cities changed.

  〔D〕 Because the number of city people was stable.

  SECTION B INTERVIEW

  Questions 6 to 10 are based on an interview. At the end of the interview you will be given 15 seconds to answer each of the following five questions.

  Now listen to the interview.

  6. According to Janet, the factor that would most affect negotiations is ______.

  〔A〕 English language proficiency

  〔B〕 different cultural practices

  〔C〕 different negotiation tasks

  〔D〕 the international Americanized style

  7. Janet's attitude towards the Americanized style, as a model for business negotiations is ______.

  〔A〕 supportive

  〔B〕 negative

  〔C〕 ambiguous

  〔D〕 cautious

  8. Which of the following can NOT be seen as a difference between Brazilian and American negotiators?

  〔A〕 Americans prepare more points before negotiations.

  〔B〕 Americans are more straightforward during negotiations.

  〔C〕 Brazilians prefer more eye contact during negotiations.

  〔D〕 Brazilians seek more background information.

  9. Which group of people seems to be the most straightforward?

  〔A〕 The British.

  〔B〕 Germans.

  〔C〕 Americans.

  〔D〕 Not mentioned.

  10. Which of the following is NOT characteristic of Japanese negotiators?

  〔A〕 Reserved.

  〔B〕 Prejudiced.

  〔C〕 Polite.

  〔D〕 Prudent.

  SECTION C NEWS BROADCAST

  Question 11 is based on the following news. At the end of the news item, you will be given 15 seconds to answer the question.

  Now listen to the news.

  11. The news item is mainly about ______.

  〔A〕 a call for research papers to be read at the conference

  〔B〕 an international conference on traditional Tibetan medicine

  〔C〕 the number of participants at the conference and their nationalities

  〔D〕 the preparations made by the sponsors for the international conference

  Questions 12 and 13 are based on the following news. At the end of the news item, you will be given 30 seconds to answer the questions.

  Now listen to the news.

  12. The news item mainly concerns ______ in Hong Kong.

  〔A〕 Internet centres

  〔B〕 an IBM seminar

  〔C〕 e-government

  〔D〕 broadcasting

  13. The aims of the three policy objectives include all the following EXCEPT ______.

  〔A〕 improvement of government efficiency

  〔B〕 promotion of e-commerce

  〔C〕 integration of service delivery

  〔D〕 formulation of Digital 21 Strategy

  Questions 14 and 15 are based on the following news. At the end of the news item, you will be given 30 seconds to answer the questions.

  Now listen to the news.

  14. Which of the following records was the second best time of the year by Donovan Bailey?

  〔A〕 9.98.

  〔B〕 9.80.

  〔C〕 9.91.

  〔D〕 9.95.

  15. The record shows that Bailey was ______.

  〔A〕 still suffering from an injury

  〔B〕 getting back in shape

  〔C〕 unable to compete with Greene

  〔D〕 less confident than before

  SECTION D NOTE-TAKING & GAP-FILLING

  Fill in each of the gaps with ONE word. You may refer to your notes. Make sure the word you fill in is both grammatically and semantically acceptable.

  Study Activities in University

  In order to help college and university students in the process of learning, four key study activities have been designed and used to encourage them to make knowledge their own.

  1. essay writing: central focus of university work esp. in the humanities, e.g. (1)______

  Benefits : 1)helping to select interesting content in books and to express understanding.

  2) enabling teachers to know progress and to offer(2) ______.

  3)familiarizing students with exam forms.

  2. seminars and classroom discussion: another form to internalize knowledge in specialized contexts.

  Benefits: 1) (3)______ enables you to know the effectiveness of and others response to your speech immediately.

  2) Within the same period of time, more topics can be dealt with than in(4)______.

  3) The use of a broader range of knowledge is encouraged.

  3. individual tutorials : a substitute for group discussion. Format: from teacher (5)______to flexible conversation.

  Benefit: encouraging ideas and interaction.

  4. lectures: a most (6) ______used study activity.

  Disadvantages: 1) less (7) ______ than discussions or tutorials.

  2) more demanding in note-taking.

  Advantages: 1) providing a general (8)______ of a subject under discussion.

  2) offering more easily understood versions of a theory.

  3) updating students on(9)______ developments.

  4) allowing students to follow different (10)______.

  PART Ⅱ PROOFREADING & ERROR CORRECTION [15 min]

  The passage contains TEN errors. Each indicated line contains a maximum of ONE error. In each case, only ONE word is involved. You should proofread the passage and correct it.

  There are great impediments to the general use

  of a standard in pronunciation comparable to that

  existing in spelling (orthography). One is the fact

  that pronunciation is learnt "naturally" and uncon-

  sciously, and orthography is learnt deliberately and (1)______

  consciously. Large numbers of us, in fact, remain

  throughout our lives quite unconscious with what (2)______

  our speech sounds like when we speak out, and (3)______

  it often comes as a shock when we firstly hear a (4)______

  recording of ourselves. It is not a voice we recog-

  nize at once, whereas our own handwriting is some-

  thing which we almost always know. We begin the (5)______

  "natural" learning of pronunciation long before we

  start learning to read or write, and in our early years

  we went on unconsciously imitating and practicing (6)______

  the pronunciation of those around us for many more

  hours per every day than we ever have to spend  (7)______

  learning even our difficult English spelling. This is  (8)______

  "natural", therefore, that our speech-sounds should

  be those of our immediate circle; after all, as we

  have seen, speech operates as a means of holding

  a community and to give a sense of "belonging". We  (9)______

  learn quite early to recognize a "stranger", someone

  who speaks with an accent of a different comm-

  unity - perhaps only a few miles far.   (10)______

  PART Ⅲ READING COMPREHENSION [40 min]

  SECTION A READING COMPREHENSION [30min]

  In this section there are four reading passages followed by a total of fifteen multiple-choice questions. Read the passages and then mark your answers on your COLOURED ANSWER SHEET.

  TEXT A

  Do you ever feel as though you spend all your time in meetings?

  Henry Mintzberg, in his book The Nature of Managerial Work, found that in large organizations managers spent 22 per cent of their time at their desk, 6 per cent on the telephone, 3 per cent on other activities, but a whopping 69 per cent in meetings. There is a widely-held but mistaken belief that meetings are for "solving problems" and "making decisions". For a start, the number of people attending a meeting tends to be inversely proportional to their collective ability to reach conclusions and make decisions. And these are the least important elements. Instead hours are devoted to side issues, playing elaborate games with one another. It seems, therefore, that meetings serve some purpose other than just making decisions. All meetings have one thing in common: role-playing. The most formal role is that of chairman. He sets the agenda, and a good chairman will keep the meeting running on time and to the point. Sadly, the other, informal, role-players are often able to gain the upper hand. Chief is the "constant talker", who just loves to hear his or her own voice. Then there are the "can t do" types who want to maintain the status quo. Since they have often been in the organization for a long time, they frequently quote historical experience as an excuse to block change: "It won t work, we tried that last year and it was a disaster." A more subtle version of the "can't do" type, the "yes, but ……", has emerged recently. They have learnt about the need to sound positive, but they still can t bear to have things change. Another whole sub-set of characters are people who love meetings and want them to continue until 5∶30 pm or beyond. Irrelevant issues are their speciality. They need to call or attend meetings, either to avoid work, or to justify their lack of performance, or simply because they do not have enough to do. Then there are the "counter-dependents", those who usually disagree with everything that is said, particularly if it comes from the chairman or through consensus from the group. These people need to fight authority in whatever form. Meetings can also provide attenders with a sense of identification of their status and power. In this case, managers arrange meetings as a means of communicating to others the boundaries of their exclusive club: who is "in", and who is not. Because so many meetings end in confusion and without a decision, another game is played at the end of meetings, called reaching a false consensus. Since it is important for the chairman to appear successful in problem solving and making a decision, the group reaches a false consensus. Everyone is happy, having spent their time productively. The reality is that the decision is so ambiguous that it is never acted upon, or, if it is, there is continuing conflict, for which another meeting is necessary. In the end, meetings provide the opportunity for social intercourse, to engage in battle in front of our bosses, to avoid unpleasant or unsatisfying work to highlight our social status and identity. They are, in fact, a necessary though not necessarily productive psychological sideshow. Perhaps it is our civilized way to moderating, if not preventing, change.

  16. On role-playing, the passage seems to indicate that chairman ______.

  〔A〕 talks as much as participants

  〔B〕 is usually a "constant talker"

  〔C〕 prefers to take the role of an observer

  〔D〕 is frequently outshone by participants

  17. Which of the following is NOT a distinct characteristic of the three types of participants?

  〔A〕 Submissiveness.

  〔B〕 Stubbornness.

  〔C〕 Disobedience.

  〔D〕 Lack of focus.

  18. The passage suggests that a false consensus was reached at the end of a meeting in order to ______.

  〔A〕 make room for another meeting

  〔B〕 bring an illusory sense of achievement

  〔C〕 highlight the importance of a meeting

  〔D〕 go ahead with the agreed programme

  TEXT B

  Cooperative competition. Competitive cooperation. Confused? Airline alliances have travellers scratching their heads over what s going on in the skies. Some folks view alliances as a blessing to travellers, offering seamless travel, reduced fares and enhanced frequent-flyer benefits. Others see a conspiracy of big businesses, causing decreased competition, increased fares and fewer choices. Whatever your opinion, there's no escaping airline alliances: the marketing hype is unrelenting, with each of the two mega-groupings, Oneworld and Star Alliance, promoting itself as the best choice for all travellers. And, even if you turn away from their ads, chances are they will figure in any of your travel plans. By the end of the year, Oneworld and Star Alliance will between them control more than 40% of the traffic in the sky. Some pundits predict that figure will be more like 75% in 10 years.

  But why, after years of often ferocious competition, have airlines decided to band together? Let's just say the timing is mutually convenient. North American airlines, having exhausted all means of earning customer loyalty at home, have been looking for ways to reach out to foreign flyers. Asian carriers are still hurting from the region-wide economic downturn that began two years ago-just when some of the airlines were taking delivery of new aircraft. Alliances also allow carriers to cut costs and increase profits by pooling manpower resources on the ground (rather than each airline maintaining its own ground crew)and code-sharing-the practice of two partners selling tickets and operating only one aircraft.

  So alliances are terrific for airlines-but are they good for the passenger? Absolutely, say the airlines: think of the lounges, the joint FFP(frequent flyer programme)benefits, the round-the-world fares, and the global service networks. Then there's the promise of "seamless" travel: the ability to, say, travel from Singapore to Rome to New York to Rio de Janeiro, all on one ticket, without having to wait hours for connections or worry about your bags. Sounds utopian? Peter Buecking, Cathay Pacific's director of sales and marketing, thinks that seamless travel is still evolving. "It's fair to say that these links are only in their infancy. The key to seamlessness rests in infrastructure and information sharing. We're working on this." Henry Ma, spokesperson for Star Alliance in Hong Kong, lists some of the other benefits for consumers: "Global travellers have an easier time making connections and planning their itineraries." Ma claims alliances also assure passengers consistent service standards.

  Critics of alliances say the much-touted benefits to the consumer are mostly pie in the sky, that alliances are all about reducing costs for the airlines, rationalizing services and running joint marketing programmes. Jeff Blyskal, associate editor of Consumer Reports magazine, says the promotional ballyhoo over alliances is much ado about nothing. "I don't see much of a gain for consumers: alliances are just a marketing gimmick. And as far as seamless travel goes, I'll believe it when I see it. Most airlines can't even get their own connections under control, let alone coordinate with another airline."

  Blyskal believes alliances will ultimately result in decreased flight choices and increased costs for consumers. Instead of two airlines competing and each operating a flight on the same route at 70% capacity, the allied pair will share the route and run one full flight. Since fewer seats will be available, passengers will be obliged to pay more for tickets.

  The truth about alliances and their merits probably lies somewhere between the travel utopia presented by the players and the evil empires portrayed by their critics. And how much they affect you depends on what kind of traveller you are.

  Those who've already made the elite grade in the FFP of a major airline stand to benefit the most when it joins an alliance: then they enjoy the FFP perks and advantages on any and all of the member carriers. For example, if you re a Marco Polo Club "gold" member of Cathay Pacific s Asia Miles FFP, you will automatically be treated as a valuable customer by all members of Oneworld, of which Cathay Pacific is a member-even if you've never flown with them before.

  For those who haven t made the top grade in any FFP, alliances might be a way of simplifying the earning of frequent flyer miles. For example, I belong to United Airline s Mileage Plus and generally fly less than 25, 000 miles a year. But I earn miles with every flight I take on Star Alliance member-All Nippon Airways and Thai Airways.

  If you fly less than I do, you might be smarter to stay out of the FFP game altogether. Hunt for bargains when booking flights and you might be able to save enough to take that extra trip anyway. The only real benefit infrequent flyers can draw from an alliance is an inexpensive round-the-world fare.

  The bottom line: for all the marketing hype, alliances aren't all things to all people-but everybody can get some benefit out of them.

  19. Which is the best word to describe air travellers reaction to airline alliances?

  〔A〕 Delight.

  〔B〕 Indifference.

  〔C〕 Objection.

  〔D〕 Puzzlement.

  20. According to the passage, setting up airline alliances will chiefly benefit ______.

  〔A〕 North American airlines and their domestic travellers

  〔B〕 North American airlines and their foreign counterparts

  〔C〕 Asian airlines and their foreign travellers

  〔D〕 Asian airlines and their domestic travellers

  21. Which of the following is NOT a perceived advantage of alliances?

  〔A〕 Baggage allowance.

  〔B〕 Passenger comfort.

  〔C〕 Convenience.

  〔D〕 Quality.

  22. One disadvantage of alliances foreseen by the critics is that air travel may be more expensive as a result of ______.

  〔A〕 less convenience

  〔B〕 higher operation costs

  〔C〕 less competition

  〔D〕 more joint marketing

  23. According to the passage, which of the following categories of travellers will gain most from airline alliances?

  〔A〕 Travellers who fly frequently economy class.

  〔B〕 Travellers who fly frequently business class.

  〔C〕 Travellers who fly occasionally during holidays.

  〔D〕 Travellers who fly economy class once in a while.

  TEXT C

  It is nothing new that English use is on the rise around the world, especially in business circles. This also happens in France, the headquarters of the global battle against American cultural hegemony. If French guys are giving in to English, something really big must be going on. And something big is going on.

  Partly, it s that American hegemony. Didier Benchimol, CEO of a French e-commerce software company, feels compelled to speak English perfectly because the Internet software business is dominated by Americans. He and other French businessmen also have to speak English because they want to get their message out to American investors, possessors of the world s deepest pockets.

  The triumph of English in France and elsewhere in Europe, however, may rest on something more enduring. As they become entwined with each other politically and economically, Europeans need a way to talk to one another and to the rest of the world. And for a number of reasons, they've decided upon English as their common tongue.

  So when German chemical and pharmaceutical company Hoechst merged with French competitor Rhone-Poulenc last year, the companies chose the vaguely Latinate Aventis as the new company name- and settled on English as the company's common language. When monetary policymakers from around Europe began meeting at the European Central Bank in Frankfurt last year to set interest rates for the new Euroland, they held their deliberations in English. Even the European Commission, with 11 official languages and traditionally French-speaking bureaucracy, effectively switched over to English as its working language last year.

  How did this happen? One school attributes English s great success to the sheer weight of its merit. It s a Germanic language, brought to Britain around the fifth century A.D. During the four centuries of French-speaking rule that followed Norman Conquest of 1066, the language morphed into something else entirely. French words were added wholesale, and most of the complications of Germanic grammar were shed while few of the complications of French were added. The result is a language with a huge vocabulary and a simple grammar that can express most things more efficiently than either of its parents. What's more, English has remained ungoverned and open to change-foreign words, coinages, and grammatical shifts-in a way that French, ruled by the purist Academic Francaise, had not.

  So it's a swell language, especially for business. But the rise of English over the past few centuries clearly owes at least as much to history and economics as to the language's ability to economically express the concept win-win. What happened is that the competition-first Latin, then French, then, briefly, German-faded with the waning of the political, economic, and military fortunes of, respectively, the Catholic Church, France, and Germany. All along, English was increasing in importance: Britain was the birthplace of the Industrial Revolution, and London the world's most important financial centre, which made English a key language for business. England s colonies around the world also made it the language with the most global reach. And as that former colony the U.S. rose to the status of the world's preeminent political economic, military, and cultural power, English became the obvious second language to learn.

  In the 1990s more and more Europeans found themselves forced to use English. The last generation of business and government leaders who hadn't studied English in school was leaving the stage. The European Community was adding new members and evolving from a paper-shuffling club into a serious regional government that would need a single common language if it were ever to get anything done. Meanwhile, economic barriers between European nations have been disappearing, meaning that more and more companies are beginning to look at the whole continent as their domestic market. And then the Internet came along.

  The Net had two big impacts. One was that it was an exciting, potentially lucrative new industry that had its roots in the U.S., so if you wanted to get in on it, you had to speak some English. The other was that by surfing the Web, Europeans who had previously encountered English only in school and in pop songs were now coming into contact with it daily.

  None of this means English has taken over European life. According to the European Union, 47% of Western Europeans (including the British and Irish)speak English well enough to carry on a conversation. That's a lot more than those who can speak German (32%) or French (28%), but it still means more Europeans don't speak the language. If you want to sell shampoo or cell phones, you have to do it in French or German or Spanish or Greek. Even the U.S. and British media companies that stand to benefit most from the spread of English have been hedging their bets-CNN broadcasts in Spanish; the Financial Times has recently launched a daily German-language edition.

  But just look at who speaks English: 77% of Western European college students, 69% of managers, and 65% of those aged 15 to 24. In the secondary schools of the European Union's non-English-speaking countries, 91% of students study English, all of which means that the transition to English as the language of European business hasn't been all that traumatic, and it s only going to get easier in the future.

  24. In the author s opinion, what really underlies the rising status of English in France and Europe is ______.

  〔A〕 American dominance in the Internet software business

  〔B〕 a practical need for effective communication among Europeans

  〔C〕 Europeans eagerness to do business with American businessmen

  〔D〕 the recent trend for foreign companies to merge with each other

  25. Europeans began to favour English for all the following reasons EXCEPT its ______.

  〔A〕 inherent linguistic properties

  〔B〕 association with the business world

  〔C〕 links with the United States

  〔D〕 disassociation from political changes

  26. Which of the following statements forecasts the continuous rise of English in the future?

  〔A〕 About half of Western Europeans are now proficient in English.

  〔B〕 U.S. and British media companies are operating in Western Europe.

  〔C〕 Most secondary school students in Europe study English.

  〔D〕 Most Europeans continue to use their own language.

  27. The passage mainly examines the factors related to ______.

  〔A〕 the rising status of English in Europe

  〔B〕 English learning in non-English-speaking E.U. nations

  〔C〕 the preference for English by European businessmen

  〔D〕 the switch from French to English in the European Commission

  TEXT D

  As humankind moves into the third millennium, it can rightfully claim to have broken new ground in its age-old quest to master the environment. The fantastic achievements of modern technology and the speed at which scientific discoveries are translated into technological applications attest to the triumph of human endeavour.

  At the same time, however, some of these applications threaten to unleash forces over which we have no control. In other words, the new technology Man now believes allows him to dominate this wider cosmos could well be a Frankenstein monster waiting to turn on its master.

  This is an entirely new situation that promises to change many of the perceptions governing life on the planet. The most acute challenges facing the future are likely to be not only those pitting man against his fellow man, but those involving humankind's struggle to preserve the environment and ensure the sustainability of life on earth.

  A conflict waged to ensure the survival of the human species is bound to bring humans closer together. Technological progress has thus proved to be a double-edged sword, giving rise to a new form of conflict: a clash between Man and Nature.

  The new conflict is more dangerous than the traditional one between man and his fellow man, where the protagonists at least shared a common language. But when it comes to the reactions of the ecosystems to the onslaught of modern technology, there is no common language.

  Nature reacts with weather disturbances, with storms and earthquakes, with mutant viruses and bacteria-that is, with phenomena having no apparent cause and effect relationship with the modern technology that supposedly triggers them.

  As technology becomes ever more potent and Nature reacts ever more violently, there is an urgent need to rethink how best to deal with the growing contradictions between Man and Nature.

  For a start, the planet, and hence all its inhabitants, must be perceived as an integral whole, not as a dichotomous mass divided geographically into the rich and developed and the poor and underdeveloped.

  Today, globalization encompasses the whole world and deals with it as an integral unit. It is no longer possible to say that conflict has shifted from its traditional east-west axis to a north-south axis. The real divide today is between summit and base, between the higher echelons of the international political structure and its grassroots level, between government and NGOs, between state and civil society, between public and private enterprise.

  The mesh structure is particularly obvious on the Internet. While it is true that to date the Internet seems to be favouring the most developed sectors of the international community over the less developed, this need not always be the case. Indeed, it could eventually overcome the disparities between the privileged and the underdeveloped.

  On the other hand, the macro-world in which we live is exposed to distortions because of the unpredictable side-effects of a micro-world we do not and cannot totally control.

  This raises the need for a global system of checks and balances, for mandatory rules and constraints in our dealings with Nature, in short, for a new type of veto designed to manage what is increasingly becoming a main contradiction of our time: the one between technology and ecology.

  A new type of international machinery must be set in place to cope with the new challenges. We need a new look at the harnessing of scientific discoveries, to maximize their positive effects for the promotion of humanity as a whole and to minimize their negative effects. We need an authority with veto powers to forbid practices conducive to decreasing the ozone hole, the propagation of AIDS, global warming, desertification-an authority that will tackle such global problems.

  There should be no discontinuity in the global machinery responsible for world order. The UN in its present form may fall far short of what is required of it, and it may be undemocratic and detrimental to most citizens in the world, but its absence would be worse. And so we have to hold on to the international organization even as we push forward for its complete restructuring.

  Our best hope would be that the functions of the present United Nations are gradually taken over by the new machinery of veto power representing genuine democratic globalization.

  28. The mention of Man s victory over Nature at the beginning of the passage is to highlight ______.

  〔A〕 a new form of conflict

  〔B〕 Man s creative powers

  〔C〕 the role of modern technology

  〔D〕 Man s ground-breaking work

  29. According to the passage, which is NOT a responsibility of the proposed international authority?

  〔A〕 Monitoring effects of scientific discoveries.

  〔B〕 Dealing with worldwide environmental issues.

  〔C〕 Vetoing human attempts to conquer Nature.

  〔D〕 Authorizing efforts to improve human health.

  30. When commenting on the present role of the UN, the author expresses his ______.

  〔A〕 dissatisfaction

  〔B〕 disillusionment

  〔C〕 objection

  〔D〕 doubt

  SECTION B SKIMMING & SCANNING [10 min]

  In this section there are seven passages with ten multiple-choice questions. Skim or scan them as required and then mark your answers on your COLOURED ANSWER SHEET.

  TEXT E

  First read the following question.

  31. What is the most appropriate topic of the following passage?

  〔A〕 Strikes.

  〔B〕 Pensions.

  〔C〕 Retirement Ages.

  〔D〕 Government decisions.

  Now go through TEXT E quickly and answer the question.

  In addition to the national social security system, 17 special pension schemes are among the social advantages that government employees are not prepared to give up.

  Under the national scheme, retirement is at the age of 65, whereas the special schemes offer retirement at 55 or even 50.

  Most of the pension schemes are in the red and have to be topped up by the state. The total state contribution in 1994 was F125 billion ( $ 25 billion).

  The prime minister says he wants to keep the special schemes. There are three solutions for keeping them afloat: lengthening the contribution period, increasing contributions, or reducing the pensions paid out. The government chose the first solution in the plan that it announced on November 15. Private sector employees were required in 1993 to contribute for 40 instead of 37.5 years, in order to qualify for a full pension. State employees could still retire after 37.5 years service provided they had reached the age limit.

  The prime minister's announcement touched off strikes on the railways, Paris's transport services and government departments. Facing increasing opposition to this proposal, the prime minister said on December 5 that working more years would no longer be a condition for reforming the special pension schemes.

  A government commission that will examine pensions will, however, be free to propose changes in the retirement age in certain professions. But it will take into consideration the hardships involved in the work and the constraints of working hours.

  At the moment, the minimum retirement age is 60-as in the private sector before 1983-for 65 percent of public service employees. It is 55, or even 50, for 35 per cent of employees considered to be doing work "involving special risks or exceptional fatigue".

  Primary school teachers can retire at 55, but the limit for new, better qualified recruits is 60. Postal workers at sorting offices can retire at 55. The retiring age for police officers is 50, prison officers 50, nurses 55, and railwaymen 50 and 55 for others. The 30, 000 employees of the Paris Metro have an average retirement age of 53.

  Two-thirds of the "active" employees and those working in conditions that can damage health in the public gas and electric utility retire at 55. Retirement age for notary s clerks is 55 for women, and 60 for men. For miners, retirement is at 55.

  Comparing the national pension scheme and the special schemes is not easy, because state employees receive bonuses-some of them substantial-which are not included in calculating their contributions or their pensions.

  TEXT F

  First read the following question.

  32. In the following passage the author intends to ______.

  〔A〕 explain how the Gulf Stream is formed

  〔B〕 compare global warming with global cooling

  〔C〕 explain the composition of the sea currents

  〔D〕 deliver a warning of a coming ice age

  Now go through TEXT F quickly and answer the question.

  It seems obvious that trapping more of the sun's heat will make the planet hotter. But what seems obvious isn't always true. According to some respected scientists, there is a chance that global warming could plunge us into, of all things, an ice age.

  The argument hinges on the Gulf Stream, the ocean current that brings warm surface water north and east and heats Europe. As it travels, some of the water evaporates; what's left is saltier and thus denser. Eventually the dense surface water sinks to the sea bottom, where it flows back southward. And then, near the equator, warm, fresh water from tropical rivers and rain dilutes the salt once again, allowing the water to rise to the surface, warm up and begin flowing north again.

  But with global warming, melting ice from Greenland and the Arctic Ocean could pump fresh water into the North Atlantic; so could the increased rainfall be predicted for northern latitudes in a warmer world. Result: the Gulf Stream's water wouldn't get saltier after all and wouldn't sink so easily. Without adequate re-supply, the southerly underwater current would stop, and the Gulf Stream would in turn be shut off.

  If that happens, Europe will get very cold. Rome is, after all, at the same latitude as Chicago, and Paris is about as far north as North Dakota. More snow will fall, and the bright snow cover will reflect more of the sun's energy back into space, making life even chillier. Beyond that, the Gulf Stream is tied into other ocean currents, and shutting it down could rearrange things in a way that would cause less overall evaporation.

  Worst of all, the experts believe, such changes could come on with astonishing speed-perhaps within a decade or less. And while we might have a great deal of trouble adjusting to a climate that gets 2℃ warmer over the next century, an ice age by mid-century would be unimaginably devastating. The lingering uncertainty about whether our relentless production of greenhouse gases will keep heating our planet or ultimately cool it suggests that we should make a better effort to leave the earth's thermostat alone.

  TEXT G

  First read the following question.

  33. What is the main theme of the following passage?

  〔A〕 Strengths of paper books over E-books.

  〔B〕 Projected extinction of paper books.

  〔C〕 Market prospects of E-books.

  〔D〕 The history of paper books.

  Now go through TEXT G quickly and answer the question.

  Experts predict that the printed paper and glue book will be rendered obsolete by electronic text delivery systems, of which one, the Microsoft Reader, is already on the market, offering "book" on a pocket PC manufactured by Hewlett-Packard. This is not impossible; already much of the written communication that used to be handled by letters, newspapers and magazines has shifted to computer screens and to the vast digital library available over the Internet. If the worst comes true and the paper book joins the papyrus scroll and parchment codex in extinction, we will miss, I predict, a number of things about it.

  The book as furniture. Shelved rows of books warm and brighten the starkest room. By bedside and easy chair, books promise a cozy, swift and silent release from this world into another. For ease of access and speed of storage, books are tough to beat.

  The book as sensual pleasure. Smaller than a breadbox, bigger than a TV remote, the average book fits into the human hand with a seductive nestling, a kiss of texture, whether of cover cloth, glazed jacket or flexible paperback. The weight can rest on the little finger of the right hand for hours without strain, while the thumb holds the pages open and the fingers of the other hand turn them.

  The rectangular block of type, a product of five and a half centuries of printers lore, yields to decipherment so gently that one is scarcely aware of the difference between immersing oneself in an imaginary world and scanning the furniture of one's own room.

  The book as souvenir. One's collection comes to symbolize the contents of one s mind. Books read in childhood, in yearning adolescence, at college and in the first self-conscious years of adulthood travel along, often, with readers as they move from house to house. My mother's college texts sat untouched in a corner of our country bookcase.

  The bulk of my own college books are still with me, rarely consulted but always there, reminders of moments, of stages, in a pilgrimage. The decades since add their own drifts and strata of volumes read or half read or intended to be read. Books preserve, daintily, the redolence of their first reading-this beach, that apartment, that summer afternoon, this flight to Indonesia.

  Books as ballast. As movers and the moved both know, books are heavy freight, the weight of refrigerators and sofas broken up into cardboard boxes. They make us think twice about changing addresses. How many aging couples have decided to stay put because they can t imagine what to do with the books? How many divorces have been forestalled by love of the jointly acquired library?

  Books hold our beams down. They act as counterweight to our fickle and flighty natures. In comparison, any electronic text delivery device would lack substance. Further, speaking of obsolescence, it would be outdated in a year and within 15 years as inoperable as my formerly treasured Wang word processor from the mid-80's. Electronic equals immaterial. Without books, we might melt into the airwaves, and be just another set of blips.

  TEXT H

  First read the following question.

  34. The passage intends primarily to ______ in some Asian cities.

  〔A〕 explain how porters work

  〔B〕 introduce top-end eateries

  〔C〕 provide advice on tipping

  〔D〕 describe how taxis are metered

  Now go through TEXT H quickly and answer the question.

  It's difficult to determine what constitutes an appropriate tip in any country. In Japan, if you leave a couple of coins on the table, the waiter is liable to chase after you to return your forgotten change. In New York, on the other hand, if you leave less than 15%, your reservation might not hold up next time. Asia, with its multiplicity of cultures and customs, is a particularly difficult terrain. To make your next trip a little easier, here s a guide to tipping across the region:

  HONG KONG

  Tipping is de rigreur in this money-mad metropolis at all but the lowest establishments. Even bathrooms in posh hotels have little dishes for loose change.

  Restaurants: Most places automatically add a 10% service charge to the bill, but the surcharge often ends up in the pocket of the owner, not the staff kitty. If the service is good, add another 10% to the bill, up to HK $ 100 if you've in an especially nice restaurant.

  Porters: HK $ 10 should do it at all but the nicest hotels where a crisp HK $ 20 bill may be more acceptable.

  Taxis: Round up to the nearest dollar, although many drivers will do this on their own when making change.

  MANILA

  Tipping is common in Manila, and anything above 10% will gain you undying loyalty.

  Restaurants: Even if a service charge is included, custom dictates adding another 5%~10% to the bill.

  Porters: Service in top hotels is good and should be rewarded with 20 pesos per bag.

  Taxis: Most cabs are metered, and rounding up to the next five pesos is a good rule of thumb.

  SEOUL

  Tipping is not part of Korean culture, although it has become a matter of course in international hotels where a 10% service charge is often added.

  Restaurants: If you re at a Korean barbecue joint, there s no need to add anything extra. But a sleek Italian restaurant may require a 10% contribution.

  Porters: If you re at a top-end hotel, international standards apply, so expect to give 500~1, 000 won per bag.

  Taxis: Drivers don t expect a tip, so unless you re feeling remarkably generous, keep the change for yourself.

  SINGAPORE

  According to government mandate in the Lion City, tipping is a no-no. It's basically outlawed at Changi Airport and officials encourage tourists not to add to the 10% service charge that many high-end hotels add on to the bill.

  Porters: Hotel staff are the one exception to the no-tipping rule. As a general guide, S $ 1 should be adequate for baggage-lugging service.

  Taxis: Drivers don t expect tipping, but they won t refuse if you want to round up the fare to the next Singaporean dollar.

  TEXT I

  First read the following questions.

  35. If you want to see a performance by the Beijing Peking Opera Theatre, which phone number would you ring?

  〔A〕 6841-9283.

  〔B〕 6848-5462.

  〔C〕 6301-6688.

  〔D〕 6523-3320.

  36. Supposing you have some free time after 7 pm on July 1st, which performance or exhibition can you go to?

  〔A〕 Traditional Chinese music.

  〔B〕 Chinese modern operas.

  〔C〕 Peking Opera.

  〔D〕 Lao Dao s recent paintings.

  Now go through TEXT I quickly and answer the questions.

  WHAT'S ON

  CONCERTS

  New concert hall: The movie theatre of the National Library of China has been turned into a concert hall after months of renovation.

  The Guotu Concert Hall will open to the public for the first time on June 30. After the opening ceremony, the China National Song and Dance Theatre will present highlights of Chinese modern operas from the past 50 years.

  Programme: excerpts from Chinese modern operas including "The White-haired Girl", "Red Rocks" and more.

  Time: 7∶30 pm, June 30

  Place: Guotu Concert Hall at the National Library of China

  Tel: 6841-9283

  Chinese music: The Traditional Band of China National Song and Dance Theatre will perform traditional Chinese music, under Liu Wenjin, composer and director of the theatre.

  Programme: "Butterfly Lovers", "Moonlight Reflected on Number Two Spring", "The Night is Deep" and other traditional pieces.

  Time: 7∶30 pm, July 1~2

  Place: Guotu Concert Hall at the National Library of China

  Tel: 6848-5462

  EXHIBITIONS

  One-man show: Lao Dao is presenting his most recent paintings at the Wanfung Gallery.

  Titled "Spanning the Space", the exhibition features about 30 works created from synthetic materials. The paintings are composed of mottled ancient doors with faded couplets pasted on them, leading the viewers into ancient stories hidden behind the door.

  Time: 9 am~5 pm until July 1st

  Place: 136 Nanchizi Dajie, Dongcheng District

  Tel: 6523-3320

  Charm of ink: The Huangshicheng Gallery is hosting a solo show of ink-and-colour paintings by veteran calligrapher and painter Qin Tang. More than just visually appealing, Qin's work impresses the viewer with its vividness and simplicity.

  Time: 9 am~5 pm until July 5th

  Place: Nanchizi Dajie, Dongcheng District

  Tel: 6528-9103

  STAGE

  Peking Opera: The Liyuan Theatre presents traditional Peking Opera excerpts in short programmes for foreign audiences and in original styles. With an explanation in English, the performances are from the Beijing Opera Theatre.

  Time: 7∶30 pm July 3~5

  Place: Liyuan Theatre, Qianmen Jianguo Hotel, Xuanwu District

  Tel: 6301-6688

  TEXT J

  First read the following questions.

  37. Who's the author of Culture/Metaculture ?

  〔A〕 Linda Anderson.

  〔B〕 Peter Childs.

  〔C〕 Adam Roberts.

  〔D〕 Francis Mulhern.

  38. Which of the following books draws on case studies?

  〔A〕 Modernism .

  〔B〕 Science Fiction .

  〔C〕 Autobiography .

  〔D〕 Culture/Metaculture .

  Now go through TEXT J quickly and answer the questions.

  Autobiography

  Linda Anderson, University of Newcastle, UK

  This wide-ranging introduction to the study of autobiography offers a historical overview of autobiographical writing from St Augustine to the present day. Linda Anderson follows the important developments in autobiographical criticism in the last thirty years, paying particular attention to psychoanalytic, post-structuralist and feminist approaches. This volume:

  ● outlines the main theoretical issues and concepts of this difficult area

  ● looks at the different forms from confessions to narratives to memoirs to diaries

  ● considers the major writers of this historical tradition.

  Culture/Metaculture

  Francis Mulhern, Middlesex University, UK

  Culture/Metaculture is a stimulating introduction to the meanings of "culture" in contemporary Western society. This essential survey examines:

  ● culture as an antidote to "mass" modernity, in the work of Thomas Mann, Julien Benda, Karl Mannheim and F. R. Leavis

  ● post-war theories of "popular" culture and the rise of Cultural Studies, paying particular attention to the key figures of Raymond Williams and Stuart Hall

  ● theories of "metaculture", or the ways in which culture, however defined, speaks of

death among travelers. Whether driver or

  pedestrian, always check on local traffic regulations.

  TEXT I

  First read the following questions.

  85. How many performances will the Irish dancing troupe give between June 23 and 25?

  A. One. B. Two. C. Three D. Four

  Whose works will NOT be played at the concert?

  A. Chopin. B. Schumann. C. Beethoven. D. Liszt.

  Now, go through TEXT I quickly and answer Questions 85 and 86.

  Irish dance: The Irish International Dance Company, one of the most dynamic dance

  troupes in the world, will tour China with its classic production" Spirit of the Dance——the

  New Millennium."

  The dancers include such famous names as Patricia Murray, one of the Irish dancing

  champions, and first-rate ballerina Claire Holding.

  The dancers include such famous names as Patricia Murray,

  Irish dancing

  Sponsored by China National Culture and Art Company Ltd. , the dancing troupe will

  give three performances at the Century Theatre.

  Time: 7:30 pm. June 23——25

  Place: Century Theatre, 40 Liangmaqiaolu,

  Telephone: 6551-8888

  Piano solos: twenty Chinese and foreign piano music works will be played by three

  young, promising pianists from the China Central Conservatory of Music.

  Programmes include: "Consolation No 3 in D-flat major" by Liszt, "For Elise' by

  Beethoven, "Turkish March" by Mozart, "Waltz in C-sharp minor" and "A Minute Waltz" by

  Chopin, and "Hungarian Dance" by Brahms.

  Time: 7:30 pm, June 16

  Place: Beijing Concert Hall,

  Telephone: 6605-5812

  TEXT J First read the following questions.

  87. When is the deadline for the competition?

  A. May 7. B. May 5. C. June 18. D. June 15

  88. The six lucky winners will _________

  A. visit Guiyang City. B. contact the Press Office.

  C. go to China Daily. D. take an overseas trip.

  Now, go through TEXT J quickly and answer Questions 87 and 88.

  Guiyang Customs and Scenery Competition Notice

  Fifteen questions for the Guiyang Customs and Scenery Competition were published in

  China Daily on May 5 and 7, and on China Daily's web edition on May 7. Participants,

  please answer the questions and mail the answer card to:

  Press Office, Guiyang Municipal People's Government

  46 Zhongshan Xilu, Guiyang 550003, Guizhou, China

  or find the competition on www. chinadaily, com. on. Then answer the questions, fill in

  all information needed and click the button below to send it back.

  The deadline for the competition is June 15 (subject to postmark)

  The prize-drawing Ceremony will be held on June 18 in Guiyang City, and six lucky

  winners ( three living in China, three from abroad) will be drawn from those who give correct

  answers to all questions. Their names will be published in China Daily and its web edition on

  June 19

  The six lucky winners will be invited to visit Guiyang from August 8 to 18.

  TEXT K

  First read the following questions

  89. If you want to travel to Shanghai on Air France on a Saturday, which flight would you

  take?

  A. AFl29. B. AFl28. C. AFlll. D. AFll2.

  90. Does Lufthansa operate a flight between Beijing and Frankfurt every day?

  A. No. B. Yes.

  C. Yes, except on Saturdays.

  D. No, only three days a week.

  Now, go through TEXT K quickly and answer Questions 89 and 90.

  Flight Schedule

  Air France Tel: (010) 6588 1388

  (020) 6360 6688

  Day From To Flight Departure Arrival

  1 —— 7 Beijing Paris AF129 09:40 14:15

  1 —— 7 Paris Beijing AF128 15:55 07:40

  2,4,7 Shanghai Paris AF111 10:55 17:05

  1,3,6 Paris Shanghai AF112 15:55 09:05

  Lufthansa

  Day From To Flight Departure Arrival

  1 —— 7 Beijing Frankfurt LH721 10:30 14:25

  1 —— 7 Frankfurt Beijing LH720 17:25 08:30

  1,2,3,4,5,7 Shanghai Frankfurt LH729 11:15 16:30

  1,2,3,4,5,7 Frankfurt Shanghai LH728 17:10 09:25

  答案:

  Part One Writing Section A Composition

  The Best Way to Stay Healthy

  If you have recently started or restarted an exercise program, you are not alone, and it proves to be the best way to stay healthy.

  In the first place, exercise makes your body, most importantly, your heart stronger-by helping it pump more blood with each heartbeat. The blood then deliver more oxygen to your body. Exercise can also lower blood pressure, reduce your risk of heart disease. Besides, exercise is the best way to lose weight. Burning calories and working off the fat will help you look and feel better. In the second place, more and more people realize that mental health is nevertheless important than physical health. Focusing your energy on taking care of yourself with proper exercise is the best way to not just transform your body but to lift your spirits.

  If you are blessed with a healthy body and healthy mind by getting into exercises like soccer, ice skating, jogging, running, swimming, bicycling or anything that involves lots of activities, you can be confident that you are the “ wealthiest”, thus the happiest man on the planet earth.

  Section B Note-writing

  May 6, 2002

  Dear Jack,

  I have heard that you wish to sell your walkman. I am very interested in it. I am just wondering about its condition. Give me a call when you have a moment. My telephone number is 555-2436. I am willing to offer 30 dollars for it if everything is OK.

  Looking forward to hearing from you soon.

  Yours,

  Sam.

  Part Two Dictation Disappearing Forests

  The world‘s forests are disappearing. As much as a third of the total tree cover has been lost since agriculture began some 10,000 years ago. The remaining forests are home to half of the world’s species thus becoming the chief resource for their survival. Tropical rain forests once covered twelve percent of the land of the planet. As well as supporting at least half of the world‘s species of plants and animals, these rain forests are home to millions of people. But there are other demands on them. For example, much has been cut for timber. An increasing amount of forestland(林地) has been used for industrial purposes or for agricultural development, such as crop growing. By the 1990s less that half of the earth’s original rain forests remained, and they continues to disappear at an alarming rate every year. As a result, the world‘s forests are now facing gradual extinction.

  Part Three Listening Comprehension Section A Statement 1. D 2. A 3. C 4.B 5. C

  6. C 7. A 8. D

  Section B Conversation 9. A 10. B 11. C 12. B 13. D

  14. C 15. D 16.A 17. C

  Section C News Broadcast

  18. D 19.B 20 C 21.A 22. C

  23.A 24.D 25.C

  Part Four Cloze 26. B 27. A 28. C 29. C 30. A

  31. B 32. D 33. A 34. B 35. C

  36. D 37. A 38. D 39. B 40. A

  Part Five Grammar & Vocabulary41. A 42. C 43. B 44. B 45.A

  46. C 47. B 48. B 49. B 50. A

  51. B 52. D 53. C 54. A 55. D

  56. B 57. A 58.B 59. D 60. B

  61. C 62. D 63. B 64. A 65. B

  Part Six Reading Comprehension Section A Reading Comprehension 66. B 67. A 68. D 69. C 70. A

  71. D 72. C 73. A 74. B 75. C

  76. A 77. C 78. B 79. D 80. B

  Section B Skimming and Scanning 81. A 82. D 83. D 84. A 85. C

  86. B 87. D 88. A 89. D 90. B

2003年英语专业八级试题及答案

试卷一  (95 min)

  Part ⅠListening Comprehension(40min)

  In Sections A, B and C you will hear everything ONCE ONLY. Listen carefully and then answer the questions that follow. Mark the correct answer to each question on your COLORED ANSWER SHEET. 

  SECTION A TALK

  Questions 1 to 5 refer to the talk in this section. At the end of the talk you will be given 15 seconds to answer each of the following five questions. Now listen to the talk. 

  1. Which of the following statements about offices is NOT true according to the talk?

  A. Offices throughout the world are basically alike. 

  B. There are primarily two kinds of office layout. 

  C. Office surroundings used to depend on company size. 

  D. Office atmosphere influences workers' performance. 

  2. We can infer from the talk that harmonious work relations may have a direct impact on your ____. 

  A. promotion  B. colleagues  C. management     D. union 

  3. Supposing you were working in a small firm, which of the following would you do when you had some grievances?

  A. Request a formal special meeting with the boss. 

  B. Draft a formal agenda for a special meeting. 

  C. Contact a consultative committee first. 

  D. Ask to see the boss for a talk immediately. 

  4. According to the talk, the union plays the following roles EXCPET ____. 

  A. mediation B. arbitration  C. negotiation  D. representation 

  5. Which topic is NOT covered in the talk?

  A. Role of the union.   B. Work relations. 

  C. Company structure.   D. Office layout. 

  SECTION B INTERVIEW

  Questions 6 to 10 are based on an interview. At the end of the interview you will be given 15 seconds to answer each of the following five questions. Now listen to the interview. 

  6. Which of the following statements is INCORRECT about David's personal background?

  A. He had excellent academic records at school and university. 

  B. He was once on a PhD programme at Yale University. 

  C. He received professional training in acting. 

  D. He came from a single-parent family. 

  7. David is inclined to believe in ____. 

  A. aliens      B. UFOs

  C. the TV character D. government conspiracies 

  8. David thinks he is fit for the TV role because of his ____. 

  A. professional training  B. personality

  C. life experience     D. appearance 

  9. From the interview, we know that at present David feels ____. 

  A. a sense of frustration  B. haunted by the unknown things

  C. confident but moody   D. successful yet unsatisfied 

  10. How does David feel about the divorce of his parents?

  A. He feels a sense of anger.   B. He has a sense of sadness. 

  C. It helped him grow up.     D. It left no effect on him. 

  SECTION C NEWS BROADCAST

  Question 11 is based on the following news. At the end of the news item, you will be given 15 seconds to answer the question. Now listen to the news. 

  11. What is the main idea of the news item?

  A. US concern over the forthcoming peace talks. 

  B. Peace efforts by the Palestinian Authority. 

  C. Recommendations by the Mitchell Commission. 

  D. Bomb attacks aimed at Israeli civilians. 

  Question 12 is based on the following news. At the end of the news item, you will be given 15 seconds to answer the question. Now listen to the news. 

  12. Some voters will waste their ballots because ____. 

  A. they like neither candidate

  B. they are all ill-informed

  C. the candidates do not differ much

  D. they do not want to vote twice 

  Questions 13 to 15 are based on the following news. At the end of the news item, you will be given 15 seconds to answer each of the questions. Now listen to the news. 

  13. According to the UN Human Development Report, which is the best place for women in the world?

  A. Canada.  B. The US.  C. Australia.  D. Scandinavia. 

  14. ____ is in the 12th place in overall ranking. 

  A. Britain B. France C. Finland D. Switzerland 

  15. According to the UN report, the least developed country is ____. 

  A. Ethiopia B. Mali C. Sierra Leon D. Central African Republic 

  SECTION D NOTE-TAKING AND GAP-FILLING

  In this section you will hear a mini-lecture. You will hear the lecture ONCE ONLY. While listening, take notes on the important points. 

  Fill in each of the gaps with ONE word. You may refer to your notes. Make sure the word you fill in is both grammatically and semantically acceptable. 

  Maslow's Hierarchy of Needs 

  Abraham Maslow has developed a famous theory of human needs, which can be arranged in order of importance. 

  Physiological needs: the most (1)____ ones for survival. They include such needs as food, water, etc. And there is usually one way to satisfy these needs. 

  (2)____ needs: needs for 〖ZK(〗a)physical security;

  b) (3)____ security. 〖ZK)〗

  The former means no illness or injury, while the latter is concerned with freedom from (4)____, misfortunes, etc. These needs can be met through a variety of means, e. g. job security, (5)____ plans, and safe working conditions. 

  Social needs: human requirements for 〖ZK(〗a) love and affection;

  b) a sense of belonging. 〖ZK)〗

  There are two ways to satisfy these needs: 〖ZK(〗a) 〖ZK(〗formation of relationships at workplace;〖ZK)〗

  b) 〖ZK(〗formation of relationships outside workplace. 〖ZK)〗〖ZK)〗

  Esteem needs: 〖ZK(〗a) self-esteem, i. e. one's sense of achievement;

  b) 〖ZK(〗esteem of others, i. e. others' respect as a result of one's (6)____. 〖ZK)〗〖ZK)〗

  There needs can be fulfilled by achievement, promotion, honours, etc. 

  Self-realization needs: need to realize one's potential. Ways to realize these needs are individually (7)____. 

  Features of the hierarchy of needs: 〖ZK(〗a) 〖ZK(〗Social, esteem and self-realization needs are exclusively 

  (8)____ nees. 〖ZK)〗

  b) 〖ZK(#〗Nesds are satisfied in a fixed order from the bottom up. 

  c) (9)____ for needs comes from the lowest un-met level. 

  d) Different levels of needs may (10)____ when they comes into play. 〖ZK)〗

  []

  (1)____

  (2)____

  (3)____

  (4)____

  (5)____

  (6)____

  (7)____

  (8)____

  (9)____

  (10)____〖DZ〗〗

  Proofreading and Error Correction  (15 min)

  The passage contains TEN errors. Each indicated line contains a maximum of ONE error. In each case, only ONE word is involved. You should proofread the passage and correct it in the following way:

  For a wrong word, underline the wrong word and write the correct one in the blank provided at the end of the line. 

  For a missing word, mark the position of the missing word with a "∧" sign and write the word you believe to be missing in the blank provided at the end of the line. 

  For an unnecessary word, cross the unnecessary word with a slash "/"and put the word in the blank provided at the end of the line. 

  Example

  When ∧ art museum wants a new exhibit, [JY](1)[ZZ(Z]an[ZZ)]

  it never buys things in finished form and hangs [JY](2)[ZZ(Z]never[ZZ)]

  them on the wall. When a natural history museum

  wants an [ZZ(Z]exhibition[ZZ)], it must often build it. [JY](3)[ZZ(Z]exhibit[ZZ)]〖FK)〗〖CSD〗〖CSX〗

  Demographic indicators show that Americans in the postwar period were more eager than over to establish families. They quickly

  brought down the age at marriage for both men and women and

  brought the birth rate to a twentieth century height after more than [JY](1)____

  a hundred years of a steady decline, producing the "baby boom". [JY](2)____

  There young adults established a trend of early marriage and relatively

  large families that went for more than two decades and caused a major but [JY](3)____

  temporary reversal of long-term demographic patterns. From the 1940s

  through the early 1960s, Americans married at a high rate and at a [JY](4)____

  younger age than their Europe counterparts. [JY](5)____

  Less noted but equally more significant, the man and women [JY](6)____

  who formed families between 1940 and 1960 nevertheless reduced [JY](7)____

  the divorce rate after a postwar peak; their marriages remained intact

  to a greater extent than did that of couples who married in earlier [JY](8)____

  as well as later decades. Since the United States maintained its [JY](9)____

  dubious distinction of having the highest divorce rate in the world,

  the temporary decline in divorce did not occur in the same extent in [JY](10)____

  Europe. Contrary to fears of the experts, the role of breadwinner

  and homemaker was not abandoned. 

  Part Ⅲ Reading Comprehension  (40 min)

  SECTION A READING COMPREHENSION    (30 min)

  In this section there are four reading passages followed by a total of fifteen multiple-choice questions. Read the passages and then mark your answers on your COLORED ANSWER SHEET. 

  TEXT A

  Hostility to Gypsies has existed almost from the time they first appeared in Europe in the 14th century. The origins of the Gypsies, with little written history, were shrouded in mystery. What is known now from clues in the various dialects of their language, Romany, is that they came from northern India to the Middle East a thousand years ago, working as minstrels and mercenaries, metal smiths and servants. Europeans misnamed them Egyptians, soon shortened to Gypsies. A clan system, based mostly on their traditional crafts and geography, has made them a deeply fragmented and fractious people, only really unifying in the face of enmity from non-Gypsies, whom they call gadje. Today many Gypsy activists prefer to be called Roma, which comes from the Romany word for "man". But on my travels among them most still referred to themselves as Gypsies. 

  In Europe their persecution by the gadje began quickly, with the church seeing heresy in their fortune-telling and the state seeing anti-social behaviour in their nomadism. At various times they have been forbidden to wear their distinctive bright clothes, to speak their own language, to travel, to marry one another, or to ply their traditional crafts. In some countries they were reduced to slavery-it wasn't until the mid-1800s that Gypsy slaves were freed in Romania. In more recent times the Gypsies were caught up in Nazi ethnic hysteria, and perhaps half a million perished in the Holocaust. Their horses have been shot and the wheels removed from their wagons, their names have been changed, their women have been sterilized, and their children have been forcibly given for adoption to non-Gypsy families. 

  But the Gypsies have confounded predictions of their disappearance as a distinct ethnic group, and their numbers have burgeoned. Today there are an estimated 8 to 12 million Gypsies scattered across Europe, making them the continent's largest minority. The exact number is hard to pin down. Gypsies have regularly been undercounted, both by regimes anxious to downplay their profile and by Gypsies themselves, seeking to avoid bureaucracies. Attempting to remedy past inequities, activist groups may overcount. Hundreds of thousands more have emigrated to the Americans and elsewhere. With very few exceptions Gypsies have expressed no great desire for a country to call their own-unlike the Jews, to whom the Gypsy experience is often compared. "Romanestan," said Ronald Lee, the Canadian Gypsy writer, "is where my two feet stand. "

  16. Gypsies are united only when they ____. 

  A. are engaged in traditional crafts

  B. call themselves Roma

  C. live under a clan system

  D. face external threats 

  17. In history hostility to Gypsies in Europe resulted in their persecution by all the following EXCEPT ____. 

  A. the Egyptians  B. the state

  C. the church   D. the Nazis 

  18. According to the passage, the main difference between the Gypsies and the Jews lies in their concepts of ____. 

  A. language B. culture C. identity D. custom 

  Text B

  I was just a boy when my father brought me to Harlem for the first time, almost 50 years ago. We stayed at the Hotel Theresa, a grand brick structure at 125th Street and Seventh Avenus. Once, in the hotel restaurant, my father pointed out Joe Louis. He even got Mr. Brown, the hotel manager, to introduce me to him, a bit paunchy but still the champ as far as I was concerned. 

  Much has changed since then. Business and real estate are booming. Some say a new renaissance is under way. Others decry what they see as outside forces running roughshod over the old Harlem. 

  New York meant Harlem to me, and as a young man I visited it whenever I could. But many of my old haunts are gone. The Theresa shut down in 1966. National chains that once ignored Harlem now anticipate yuppie money and want pieces of this prime Manhattan real estate. So here I am on a hot August afternoon, sitting in a Starbucks that two years ago opened a block away from the Theresa, snatching at memories between sips of high-priced coffee. I am about to open up a piece of the old Harlem-the New York Amsterdam News-when a tourist asking directions to Sylvia's, a prominent Harlem restaurant, penetrates my daydreaming. He's carrying a book: Touring Historic Harlem. 

  History. I miss Mr. Michaux's bookstore, his House of Common Sense, which was across from the Theresa. He had a big billboard out front with brown and black faces painted on it that said in large letters:"World History Book Outlet on 2,000,000,000 Africans and Nonwhite Peoples. "An ugly state office building has swallowed that space. 

  I miss speaker like Carlos Cooks, who was always on the southwest corner of 125th and Seventh, urging listeners to support Africa. Harlem's powerful political electricity seems unplugged-although the streets are still energized, especially by West African immigrants. 

  Hardworking southern newcomers formed the bulk of the community back in the 1920s and '30s, when Harlem renaissance artists, writers, and intellectuals gave it a glitter and renown that made it the capital of black America. From Harlem, W. E. B. Dubois, Langston Hughes, Paul Robeson, Zora Hurston, and others helped power America's cultural influence around the world. 

  By the 1970s and '80s drugs and crime had ravaged parts of the community. And the life expectancy for men in Harlem was less than that of men in Bangladesh. Harlem had become a symbol of the dangers of inner-city life. 

  Now, you want to shout "Lookin' good!"at this place that has been neglected for so long. Crowds push into Harlem USA, a new shopping centre on 125th, where a Disney store shares space with HMV Records, the New York Sports Club, and a nine-screen Magic Johnson theatre complex. Nearb, a Rite Aid drugstore also opened. Maybe part of the reason Harlem seems to be undergoing a rebirth is that it is finally getting what most people take for granted. 

  Harlem is also part of an "empowerment zone"-a federal designation aimed at fostering economic growth that will bring over half a billion in federal, state, and local dollars. Just the shells of once elegant old brownstones now can cost several hundred thousand dollars. Rents are skyrocketing. An improved economy, tougher law enforcement, and community efforts against drugs have contributed to a 60 percent drop in crime since 1993. 

  19. At the beginning the author seems to indicate that Harlem ____. 

  A. has remained unchanged all these years

  B. has undergone drastic changes

  C. has become the capital of Black America

  D. has remained a symbol of dangers of inner-city life 

  20. When the author recalls Harlem in the old days, he has a feeling of ____. 

  A. indifference B. discomfort C. delight D. nostalgia 

  21. Harlem was called the capital of Black America in the 1920s and '30s mainly because of its ____. 

  A. art and culture    B. immigrant population

  C. political enthusiasm  D. distinctive architecture 

  22. From the passage we can infer that, generally speaking, the author ____. 

  A. has strong reservations about the changes

  B. has slight reservations about the changes

  C. welcomes the changes in Harlem

  D. is completely opposed to the changes 

  TEXT C

  The senior partner, Oliver Lambert, studied the resume for the hundredth time and again found nothing he disliked about Mitchell Y. McDeere, at least not on paper. He had the brains, the ambition, the good looks. And he was hungry; with his background, he had to be. He was married, and that was mandatory. The firm had never hired an unmarried lawyer, and it frowned heavily on divorce, as well as womanizing and drinking. Drug testing was in the contract. He had a degree in accounting, passed the CPA exam the first time he took it and wanted to be a tax lawyer, which of course was a requirement with a tax firm. He was white, and the firm had never hired a black. They managed this by being secretive and clubbish and never soliciting job applications. Other firms solicited, and hired blacks. This firm recruited, and remained lily white. Plus, the firm was in Memphis, and the top blacks wanted New York or Washington or Chicago. McDeere was a male, and there were no women in the firm. That mistake had been made in the mid-seventies when they recruited the number one grad from Harvard, who happened to be a she and a wizard at taxation. She lasted four turbulent years and was killed in a car wreck. 

  He looked good, on paper. He was their top choice. In fact, for this year there were no other prospects. The list was very short. It was McDeere, or no one. 

  The managing partner, Royce McKnight, studied a dossier labeled "Mitchell Y. McDeere-Harvard. "An inch thick with small print and a few photographs; it had been prepared by some exCIA agents in a private intelligence outfit in Bethesda. They were clients of the firm and each year did the investigating for no fee. It was easy work, they said, checking out unsuspecting law students. They learned, for instance, that he preferred to leave the Northeast, that he was holding three job offers, two in New York and one in Chicago, and that the highest offer was $ 76 000 and the lowest was $ 68 000. He was in demand. He had been given the opportunity to cheat on a securities exam during his second year. He declined, and made the highest grade in the class. Two months ago he had been offered cocaine at a law school party. He said no and left when everyone began snorting. He drank an occasional beer, but drinking was expensive and he had no money. He owed close to $ 23 000 in student loans. He was hungry. 

  Royce McKnight flipped through the dossier and smiled. McDeere was their man. 

  Lamar Quin was thirty-two and not yet a partner. He had been brought along to look young and act young and project a youthful image for Bendini, Lambert & Locke, which in fact was a young firm, since most of the partners retired in their late forties or early fifties with money to burn. He would make partner in this firm. With a six-figure income guaranteed for the rest of his life, Lamar could enjoy the twelve-hundred-dollar tailored suits that hung so comfortably from his tall, athletic frame. He strolled nonchalantly across the thousanddollaraday suite and poured another cup of decaf. He checked his watch. He glanced at the two partners sitting at the small conference table near the windows. 

  Precisely at two-thirty someone knocked on the door. Lamar looked at the partners, who slid the resume and dossier into an open briefcase. All three reached for their jackets. Lamar buttoned his top button and opened the door. 

  23. Which of the following is NOT the firm's recruitment requirement?

  A. Marriage.  B. Background.  C. Relevant degree.  D. Male. 

  24. The details of the private investigation show that the firm ____. 

  A. was interested in his family background

  B. intended to check out his other job offers

  C. wanted to know something about his preference

  D. was interested in any personal detail of the man 

  25. According to the passage, the main reason Lama Quin was there at the interview was that ____. 

  A. his image could help impress McDeere

  B. he would soon become a partner himself

  C. he was good at interviewing applicants

  D. his background was similar to McDeere's 

  26. We get the impression from the passage that in job recruitment the firm was NOT ____. 

  A. selective B. secretive C. perfunctory D. racially biased 

  TEXT D

  Harry Truman didn't think his successor had the right training to be president. "Poor Ike-it won't be a bit like the Army,"he said. "He'll sit there all day saying 'do this, do that,'and nothing will happen. "Truman was wrong about Ike. Dwight Eisenhower had led a fractious alliance-you didn't tell Winston Churchill what to do-in a massive, chaotic war. He was used to politics. But Truman's insight could well be applied to another, even more venerated Washington figure: the CEO-turned cabinet secretary. 

  A 20-year bull market has convinced us all that CEOs are geniuses, so watch with astonishment the troubles of Donald Rumsfeld and Paul O' Neill. Here are two highly regarded businessmen, obviously intelligent and well-informed, foundering in their jobs. 

  Actually, we shouldn't be surprised. Rumsfeld and O' Neill are not doing badly despite having been successful CEOs but because of it. The record of senior businessmen in government is one of almost unrelieved disappointment. In fact, with the exception of Robert Rubin, it is difficult to think of a CEO who had a successful career in government. 

  Why is this? Well, first the CEO has to recognize that he is no longer the CEO. He is at best an adviser to the CEO, the president. But even the president is not really the CEO. No one is. Power in a corporation is concentrated and vertically structured. Power in Washington is diffuse and horizontally spread out. The secretary might think he's in charge of his agency. But the chairman of the congressional committee funding that agency feels the same. In his famous study "Presidential Power and the Modern Presidents,"Richard Neustadt explains how little power the president actually has and concludes that the only lasting presidential power is "the power to persuade. "

  Take Rumseld's attempt to transform the cold-war military into one geared for the future. It's innovative but deeply threatening to almost everyone in Washington. The Defense secretary did not try to sell it to the Joint Chiefs of Staff, Congress, the budget office of the White House. As a result, the idea is collapsing. 

  Second, what power you have, you must use carefully. For example, O' Neill's position as Treasury secretary is one with little formal authority. Unlike Finance ministers around the world, Treasury does not control the budget. But it has symbolic power. The secretary is seen as the chief economic spokesman for the administration and, if he plays it right, the chief economic adviser for the president. 

  O' Neill has been publicly critical of the IMF's bailout packages for developing countries while at the same time approving such packages for Turkey, Argentina and Brazil. As a result, he has gotten the worst of both worlds. The bailouts continue, but their effect in holstering investor confidence is limited because the markets are rattled by his skepticism. 

  Perhaps the government doesn't do bailouts well. But that leads to a third rule: you can't just quit. Jack Welch's famous law for re-engineering General Electric was to be first or second in any given product category, or else get out of that business. But if the government isn't doing a particular job at peak level, it doesn't always have the option of relieving itself of that function. The Pentagon probably wastes a lot of money. But it can't get out of the national-security business. 

  The key to former Treasury secretary Rubin's success may have been that he fully understood that business and government are, in his words, "necessarily and properly very different. "In a recent speech he explained, "Business functions around one predominate organizing principle, profitability …… Government, on the other hand, deals with a vast number of equally legitimate and often potentially competing objectives-for example, energy production versus environmental protection, or safety regulations versus productivity. "

  Rubin's example shows that talented people can do well in government if they are willing to treat it as its own separate, serious endeavour. But having been bathed in a culture of adoration and flattery, it's difficult for a CEO to believe he needs to listen and learn, particularly from those despised and poorly paid specimens, politicians, bureaucrats and the media. And even if he knows it intellectually, he just can't live with it. 

  27. For a CEO to be successful in government, he has to ____. 

  A. regard the president as the CEO

  B. take absolute control of his department

  C. exercise more power than the congressional committee

  D. become acquainted with its power structure 

  28. In commenting on O' Neill's record as Treasury Secretary, the passage seems to indicate that ____. 

  A. O' Neill has failed to use his power well

  B. O' Neill policies were well received

  C. O' Neill has been consistent in his policies

  D. O' Neill uncertain about the package he's approved 

  29. According to the passage, the differences between government and business lie in the following areas EXCEPT ____. 

  A. nature of activity B. optin of withdrawal

  C. legitimacy of activity D. power distribution 

  30. The author seems to suggest that CEO-turned government officials ____. 

  A. are able to fit into their new roles

  B. are unlikely to adapt to their new roles

  C. can respond to new situations intelligently

  D. may feel uncertain in their new posts 

  SECTION B SKIMMING AND SCANNING    (10 min)

  In this section there are seven passages with ten multiple-choice questions. Skim or scan them as required and then mark your answers on COLORED ANSWER SHEET. 

  TEXT E

  First read the question. 

  31. The passage is mainly concerned with ____ in the U. S. A. 

  A. traveling  B. big cities C. cybercafes  D . inventions 

  Now go through TEXT E quickly to answer question 31. 

  Planning to answer your e-mail while on holiday in New York? That may not be easy. The Internet may have been invented in the United States, but America is one of the least likely places where a traveller might find an Internet cafe. "Every major city in the world has more cybercafes than New York,"says Joie Kelly, who runs CyberCafeGuide. com. The numbers seem to bear her out: according to various directories, London has more than 30, Paris 19, Istanbul 17, but New York has only 8. Other U. S. cities fare just as poorly: Los Angeles has about 11, Chicago has 4. "Here it's quite hard work to find a cafe. I was surprised,"says Michael Robson, a sportswriter from York, England, who was visibly relieved to be checking his e-mail at CyberCafe near New York's Times Square. 

  Why the lack of places to plug in? Americans enjoy one of the highest rates of Internet access from work and home in the world, and they've never really taken to cafes. About 80 percent of CyberCafe's clients, for instance, are tourists from overseas. Greek tycoon Stelios HajiIoannou also thinks high prices drive away locals. Last November he oppened a branch of his Internet-cafe chain easyEverything in Times Square. With 800 terminals, it's the largest Net cafe in the world. While the typical American cafe charges $ 8 to $ 12 an hour, easyEverything charges $ 1 to 4. Marketing manager Stephaine Engelsen says half the cafe's customers are locals. "We get policemen, firemen, nurses who don't work at desks with computers, actors between auditions. "easyEverything is now planning to open new locations in Harlem, and possibly SoHo. Unless there's some cultural shift afoot, however, New York will continue to lag behind metropolises from Mexico City to Moscow. 

  TEXT F

  First read the question. 

  32. In the passage below the author primarily attempts to ____. 

  A. criticize yogis in the West  B. define what yoag is

  C. teach yoga postures      D. experiment with yoga 

  Now go through TEXT F quickly to answer question 32. 

  Most of the so-called yogis in the West seem to focus on figure correction, not true awareness. They make statements about yoga being for the body, mind and soul. But this is just semantics. Asanas (postures), which get such huge play in the West, are the smallest aspect of yoga. Either you practice yoga as a whole or you don't. If one is practicing just for health, better to take up walking. Need to cure a disease? See a doctor. Yoga is not about fancy asanas or breath control. Nor is it a therapy or a philosophy. Yoga is about inside awareness. It is the process of union of the self with the whole. Yoga is becoming the Buddha. 

  Yogis are experimentalists. In the West, scientists research mainly external phenomena. Yogis focus on the inside. They know that the external world is maya (illusionary) and everything inside is sathya (truth). In maya everything goes, but if you know yourself nothing goes. The West tends to practice only what we call cultural asanas that focus on the external. We don't practice asanas just to become fit. Indian yogis have discovered 8. 4 million such postures. It is essential to train our bodies to find the most comfortable pose that we can sit in for hours. Beyond that there is no role for physical yoga. 

  Basically yoga is made up of two parts: bahirang (external yoga) and antarang (internal yoga). The West practices only the former. It needs to enter into antarang yoga. After that begins the trip to the unknown where the master makes the student gradually aware at every stage, where you know that you are not the body or the mind and not even the soul. That is when you get the first taste of moksha, or enlightenment. It is the sense of the opening of the silence, the sense where you lose yourself and are happy doing it, where for the first time your ego has merged with the superconsciousness. You feel you no longer exist, for you have walked into the valley of death. And if you start walking more and more in this valley, you become freer. 

  TEXT G

  First read the question. 

  33. The reviewer's comments on Henry Kissinger's new book are basically ____. 

  A. negative  B. noncommittal  C. unfounded  D. positive 

  Now go through TEXT G quickly to answer question 33. 

  Whatever you think of Henry Kissinger, you have to admit: the man has staying power. With a new book-Does America Need a Foreign Policy?-on the shelves, Kissinger is once again helping to shape American thinking on foreign relations. This is the sixth decade in which that statement can be said to be true. 

  Kissinger's new book is terrific. Plainly intended as an extended tutorial on policy for the new American Administration, it is full of good sense and studded with occasional insights that will have readers nodding their heads in silent agreement. A particularly good chapter on Asia rebukes anyone who unthinkingly assigns China the role once played by the Soviet Union as the natural antagonist of the U. S. 

  Kissinger's book can also be read in another, and more illuminating, light. It is, in essence, an extended meditation on the end of a particular way of looking at the world: one where the principal actors in international relations are nation-states, pursuing their conception of their own national interest, and in which the basic rule of foreign policy is that one nation does not intervene in the internal affairs of another. 

  Students of international relations call this the "Westphalian system,"after the 1648 Peace of Westphalia that ended Europe's Thirty Years War, a time of indescribable carnage waged in the name of competing religions. The treaties that ended the war put domestic arrangements-like religion-off limits to other states. In the war's aftermath a rough-and-ready commitment to a balance of power among neighbours took shape. Kissinger is a noted school of the balance of power. And he is suspicious of attempts to meddle in the internal business of others. 

  Yet Kissinger is far too sophisticated to attempt to recreate a world that is lost. "Today,"he writes,"the Westphalian order is in systematic crisis. "In particular, nation-states are no longer the sole drivers of the international system. In some cases, groups of states-like the European Union or Mercosur-have developed their own identities and agendas. Economic globalization has both blurred the boundaries between nations and given a substantial international role to those giant companies for whom such boundaries make little sense. In today's world, individuals can be as influential as nations; future historians may consider the support for public health of the Bill and Melinda Gates Foundation to be more noteworthy than last week's United Nations conference on AIDS. And a large number of institutions are premised on the assumption that intervention in the internal affairs of others is often desirable. Were that not the case, Slobodan Milosevic would not have been surrendered last week to the jurisdiction of the war crimes tribunal in the Hague. 

  The consequences of these changes are profound. Kissinger is right to note that globalization has undermined the role of the nation-state less in the case of the U. S. (Why? Because it's more powerful than anyone else. ) Elsewhere, the old ways of thinking about the "national interest"-that guiding light of the Westphalian system-have fewer adherents than they once did. 

  TEXT H

  First read the question. 

  34. In the passage the author expresses his concern about ____. 

  A. the survival of small languages

  B. globalization in the post-Cold War era

  C. present-day technological progress

  D. ecological imblance 

  Now go through TEXT H quickly to answer question 34. 

  During the past century, due to a variety of factors, more than 1 000 of the world's languages have disappeared, and it is possible to foresee a time, perhaps 100 years from now, when about half of today's 6 000 languages will either be dead or dying. 

  This startling rate of linguistic extinction is possible because 96 per cent of the world's languages are now spoken only by 4 per cent of the world's population. 

  Globalization in the post-Cold War era has witnessed the coming of the information age, which has played an important role in promoting economic co-operation but which has, at the same time, helped facilitate the assimilation of smaller cultural systems into a larger, mostly English-speaking whole. 

  Internet and other forms of mass media have succeeded in making English the worldwide standard. 

  In 1998, the Seminar on Technological Progress & Development of the Present-day World was held in China. At the seminar, many participants expressed concern over the potential risks associated with excessive dependency on information technology. These critics claimed a move from "information monopoly" to "information hegemony" could possibly become just another way for the strong to dominate the weak, culturally as well as economically. 

  In other words, life in a technology-and information-based global society may lead to a new social stratification, in which linguistic assimilation will lead to cultural assimilation and social injustice will abound. 

  In the 20th century, human society's over-development caused the deterioration of the environment and ecological imbalance. The extinction of myriad biological species aroused deep concern which led people to an understanding of the special importance of protecting rare animals and plants on the brink of extinction. 

  Now we face the question, is the maintenance of cultural and linguistic diversity as important as the preservation of pandas and Chinese white-flag dolphins?

  Given the open society in which we live, or wish to live, this question becomes complicated. A balance must be struck between promoting international exchanges on the one hand, and taking measures to protect "small" languages on the other hand. 

  Most widely used languages, such as the six working languages-including English and Chiese-used in the United Nations, have little to fear and need no special protection. 

  But for other, more marginal languages some measures should be taken. Professionals should be trained to study and use them in order to keep them alive. Effective measures such as bilingual or multilingual education should also be implemented to protect them from extinction. 

  To some, 6 000 may seem like an inexhaustible number of languages. To those same people, it may seem irrelevant if one or two of those languages cease to be used. 

  But what many fail to realize is that language and culture are linked. Without one, the other dies, and so with the death of different languages we have the death of different cultures. The extinction of languages is equal to animal extinction in this respect. The fading away of a language, no matter how small, causes real damage to the "ecological balance" in the field of culture. 

  TEXT I

  First read the questions. 

  35. The work of Project Manager is chiefly concerned with ____. 

  A. emergency relief programmes    B. agricultural rehabilitation

  C. helicopter assisted surveys    D. strategic planning 

  36. The working contract is offered on a ____ basis. 

  A. two-month  B. twenty-monty  C. ten-month  D. twelve-month 

  Now go through TEXT I quickly to answer questions 35 and 36. 

  Project Manager

  AGRICULTURAL REHABILITATION PROJECT, NORTHERN ETHIOPIA

  SCF started work in Ethiopia in 1973 with an emergency relief programme in response to the famine of that year. Since then SCF has been involved in a range of longer-term relief and development programmes to secure lasting benefits for children. 

  As a result of a helicopter assisted survey undertaken in the northem highlands of Ethiopia in 2000, SCF has been involved in a number of interventions aimed at engaging with the agricultural sector in order to promote food security in the most vulnerable areas of North Wollo. 

  As Project Manager your key task will be to manage, promote and develop all SCF's activities in the agriculture / livestock and natural resources sectors in Wollo. You will also play a major role in developing policy at national level. 

  To meet the challenge of this exciting new post you will need a relevant post graduate qualification; substantial experience in managing agricultural development projects in Africa with an emphasis on providing institutional support to the capacity of extension services while prompting farmer participation; ability to think and plan strategically; proven team management skills; report writing and financial skills; willingness to travel extensively and live and work in an isolated location. 

  This post is offered on a twelve-month contract with a salary of £ 19 294(normally tax-free). You can also expect a generous benefits package including all flights and reasonable living and accommodation expenses. 

  For further details and an application form please apply with CV to Jenny Thomas, Overseas Personnel Administrator, SCF, 17 Grove Lane, London SE5 8RD

  Closing date: 30th November 2001.

  TEXT J

  First read the questions. 

  37. Who have found a protein called M2?

  A. Scientists from a Belgium University.  B. Drug-makers in Belgium. 

  C. Doctors in a Belgium hospital.      D. It is not mentioned. 

  38. How many causes of bad breath does the passage cite?

  A. One.  B. Two.  C. Three.  D. Four. 

  Now go through TEXT J quickly to answer questions 37 and 38. 

  The Common Cold?

  The conventional wisdom says no, but by mid-century that assessment-along with the sniffles-may well be ancient history. Colds are considered incurable today because it would take months to come up with a vaccine for every new strain. That's fine for the flu, which breeds in animals and only jumps over to humans every year or two. But colds mutate even while they're infecting you, and new strains pop up so often that by the time drug-makers create a vaccine against one variation, the serum is already out of date. 

  The flu may yet point the way toward a cold cure though. Scientists at the University of Ghent, in Belgium, have found a protein called M2 that seems to be present in virtually every flu strain known to man. Using that knowledge, they have made a vaccine that they think could protect against all flus-old, new and those not yet in existence. 

  If a similar protein is found in cold viruses-a protein that's present no matter what strain is involved-then it is possible that by 2025 or so, children could be getting a universal cold vaccine. And then they will have to listen to us old geezers reminsice about the days when we used to carry a small white cloth called a handerchief. 

  Bad Breath?

  Afraid not. Bad breath isn't an illness; it's merely a symptom of something else. In some cases, the something else really is an illness-some kidney disorder or an infection. Infections can usually be cured, and if you're suffering from an incurable one or from another serious condition, bad breath is the least of your problems. 

  Another cause is foods like onions or garlic, in which case you're out of luck: essential oils from such foods get into the blood, then into the lungs, then out with each exhaled breath. Even in the 21st century, if you want the flavour, you risk disflavour. 

  The most common reason for bad breath, though, is, to put it delicately, food molecules rotting in the mouth. Mouthwash masks te smell, but ultimately you have to get rid of the stuff. Brushing removes larger particles, but dentists suggest brushing the back of the tongue as well, where food residues and bacteria congregate. The microscopic bits that remain must be flushed down by drink or saliva. But if you're waiting for a true cure, it won't happen until we eat all our food in pill form. In other words, don't hold you breath. 

  TEXT K

  First read the questions. 

  39. When did Moore receive his first commission?

  A. In 1948.   B. In 1946.   C. In 1931.    D. In 1928. 

  40. Where did Moore win his first international prize?

  A. In London.  B. In Venice.  C. In New York.  D. In Hamburg. 

  Now go through TEXT K quickly to answer questions 39 and 40. 

  Henry Moore, the seventh of eight children of Raymond Spencer Moore and his wife Mary, was born in Yorkshire on 30 July 1898. After graduating from secondary school, Moore taught for a short while. Then the First World War began and he enlisted in the army at the age of eighteen. After the war he applied for and received an ex-serviceman's grant to attend Leeds School of Art. At the end of his second year he won a scholarship to the Royal College of Art in London. 

  In 1928 Moore met Irina Radesky, a painting student at the college, whom he married a year later. The couple then moved into a house which consisted of a small ground-floor studio with an equally small flat above. This remained their London home for ten years. 

  Throughout the 1920's Moore was involved in the art life of London. His first commission, received in 1928, was to produce a sculpture relief for the newly opened headquarters of London Transport. His first one-man exhibition opened at the Warren Gallery in 1928; it was followed by a show at the Leicester Galleries in 1931 and his first sale to a gallery abroad-the Museum fur Kunst und Gewerbe in Hamburg. His success continued. 

  In 1946 Moore had his first foreign retrospective exhibition at the Museum of Modern Art, New York. In 1948 he won the International Sculpture Prize at the 24th Venice Biennale, the first of countless international accolades acquired in succeeding years. At the same time sales of Moore's work around the world increased, as did the demand for his exhibitions. By the end of 1970's the number of exhibitions had grown to an average of forty a year, ranging from the very small to major international retrospectives taking years years of detailed planning and preparation. 

  The main themes in Moore's work included the mother and child, the earliest work created in 1922, and the reclining figure dating from 1926. At the end of the 1960's came stringed figures based on mathematical models observed in the Science Museum, and the first helmet head, a subject that later developed into the internal-external theme-variously interpreted as a hard form covering a soft, like a mother protecting her child or a foetus inside a womb. 

  A few years before his death in 1986 Moore gave the estate at Perry Green with its studios, houses and cottages to the Trustees of the Henry Moore Foundation to promote sculpture and the fine arts within the cultural life of the country and in particular the works of Henry Moore. 

  试卷二 (120 min)

  Part ⅣTranslation  (60 min)

  SECTION A CHINESE TO ENGLISH

  Translate the following text into English. Write your translation on ANSWER SHEET THREE. 

  在得病以前,我受父母宠爱,在家中横行霸道,一旦隔离,拘禁在花园山坡上一幢小房子里,顿感被打入冷宫,十分郁郁不得志起来。一个春天的傍晚,园中百花怒放,父母在园中设宴,霎时宾客云集,笑语四溢。我在山坡的小屋里,悄悄掀起窗帘,窥见园中大千世界,一片喧闹。自己的哥姐,堂表弟兄,也穿插其间,个个喜气洋洋。一霎时,一阵被人摈弃,为世所遗忘的悲愤兜上心头,禁不住痛哭起来。

  SECTION B ENGLISH TO CHINESE

  Translate the following text into Chinese. Write your translation on ANSWER SHEET THERR. 

  In his classic novel, "The Pioneers", James Fenimore Cooper has his hero, a land developer, with his cousin on a tour of the city he is building. He decribes the broad streets, rows of houses, a bustling metropolis. But his cousin looks around bewildered. All she sees is a forest. "Where are the beauties and improvements which you were to show me?" she asks. He's astonished she can't see them. "Where! Why everywhere," he replies. For though they are not yet built on earth, he has built them in his mind, and they are as concrete to him as if they were already constructed and finished. 

  Cooper was illustrating a distinctly American trait, future-mindeness: the ability to see the present from the vantage point of the future; the freedom to feel unencumbered by the past and more emotionally attached to things to come. As Albert Einstein once said, "Life for the American is always becoming, never being. "

  Part Ⅴ Writing  (60 min)

  An English newspaper is currently running a discussion on whether young people in China today are (not) more self-centred and unsympathetic than were previous generations. And the paper is inviting contributions from university students. You have been asked to write a short article for the newspaper to air your views. 

  Your article should be about 300 words in length. In the first part of your article you should state clearly your main argument, and in the second part you should support your argument with appropriate details. In the last part you should bring what you have written to a natural conclusion or a summary. 

  You should supply a title for your article. 

  Marks will be awarded for content, organization, grammar and appropriacy. Failure to follow the above instructions may result in a loss of marks. 

  Write your composition on ANSWER SHEET FOUR.

  专业八级 (2003)  答案部分

  听力原文

  PART Ⅰ LISTENING COMPREHENSION

  SECTION A TALK

  When we talk about a modern company, we usually have managers, employees, products, research and development or marketing in mind. However, in reality, a company is not just made up of these elements. There are other things that make a company what it is. This morning, we are going to look at some other aspects of a company. Let's first take a look at the offices. The physical surroundings of most modern companies, especially offices are becoming more and more similar. Although there are some differences from country to country, one office looks much like another. Office furniture and equipment tends to be similar, desks, chairs, filing cabinets, computers, etc. "What is important about offices?" you may ask, "What the atmosphere of the work place can often influence the effectiveness of a company's employees?" Modern offices are more spacious and better laid, heated, ventilated and air-conditioned than in the past. But of course, this is the feature that varies from firm to firm, and may be dependant on the size of the company and its cooperate philosophy. In some companies, the employees work in large, open-plan offices without walls between the departments; in others, the staff members work more privately in individual offices. No matter what the office's law is like, modern companies pay special attention to the physical surroundings in order to create an atmosphere conducive to higher working efficiency. Another related point when talking about offices is the work relations with other people at the place of work. They include relationships with fellow employees, workers or colleagues. A great part of work or job satisfaction, some people say the major portion, comes from getting on with others at work. Work relations were also included those between management and employees. These relations are not always straightforward, particularly as the management's assessment of how your performing can be crucial to your future career. 

  Now I'd like to say a bit more about the relations between management and employees. There will also be matters about which employees will want to talk to the management. In small businesses, the boss will probably work alongside his or her workers. Anything that needs to be sorted out will be done face to face as soon as the problem arises. There will be no formal meetings for procedures. But the larger the business, the less direct contact there will be between employees and management. Special meetings have to be held and procedures set up to say when, where, how and what circumstances the employees can talk to the management. Some companies have specially organized consultive committees for this purpose. In many countries of the world today, particularly in large firms, employees join a trade union and ask the union to represent them to the management. Through the union all categories of employees can pass on the complaints they have and try to get things changed. The process, through which unions negotiate with management on behalf of their members is called, collective bargaining. Instead of each employee trying to bargain alone with the company, the employees join together and collectively put forward their views. Occasionally a firm will refuse to recognize the right of a union to negotiate for its members, and its dispute over union recognition will arise. Whether there is an agreement, bargaining or negotiation will take place. A compromise agreement may be reached. When this is not possible, the sides can go to arbitration and bring in a third party from outside to say what they think should happen. 

  However, sometimes one of the sides decides to take industrial action. The management can lock out the employees and prevent them from coming to work. This used to be quite common, but it's rarely used today. The main courses of action open to a trade union are strike, a ban on working overtime, "working to rule", that is when employees work according to the company rule book, "go slows", which means that employees may spend more time doing the same job, and "picketing", which means the employees stand outside the entrance to the business location, hoarding outside to show that they are in conflict with the management. Every country has its own tradition of industrial relations, so it's difficult to generalize. In some businesses, unions are not welcomed by the management, but it others, the unions play an important role both in the everyday working relations of individual companies, and also in the social and political life of the country. 

  SECTION B INTERVIEW

  If you are going to create a TV show that plays week after week, it needs an actor who can play a believer, you know, a person who tends to believe everything. Tonight in our show we have David Duchovney, who has starred in the popular TV series, "The X·Files". Thanks to his brilliant performance in the TV series, David has become one of best-known figures in the country. 

  Interviewer: Good evening, David, I'm so glad to have you here. 

  David: It's my pleasure. Thank you for inviting me on the show. 

  Interviewer: David, have you often been on the radio shows? 

  David: Oh, yes, quite often. To be frank, I love to be on the show. 

  Interviewer: Why? 

  David: You know, I want to know what people think about the TV series and about me, my acting, etc. 

  Interviewer: OK, David, let's first talk about the character you played in 'The X·Files'. The character, whose name is Mulder is supposed to be a believer. He deals with those unbelievable, wild and often disastrous events. He must be, I mean, Mulder, someone who really believes in the things he meets in order to keep on probing into those mysteries. 

  David: That's true. Remember those words said by Mulder: What is so hard to believe? Whose intensity makes even a most skeptical viewer believe the paranormal and our rigorous government consipiracies, without every reason to believe that life in the persistent survey is driving us out of our territorial sphere, etc. , etc. ? 

  Interviewer: I believe, I guess, David, your contribution to the hot series is quite aparent.  Now let's talk about your personal experience. From what I have read, I know that starting from your childhood, you were always a smart boy, went to the best private school, and were accepted at most of the Ivy League colleges. Not bad for a low middle class kid from a broken family on New York's Lower Eastside. It's even more surprising when you, who were on your way to a doctorate at Yale to took a few acting classes and got beaten by the book. 

  David: You bet. My mother was really surprised when I decided to give up all that in order to become an actor. 

  Interviewer: Sure. But talking about Mulder, the believer in 'The X·Files', what about you, David? Do you believe at all in real life, the aliens, people from outer space, you know, UFOs, government conspiracies, all the things that the TV series deal with? 

  David: Well, government conspiracies, I think, are a little far fetched. Because I mean, it's very hard for me to keep a secret with a friend of mine. And you can tell me that the entire government is going to come together and hide the aliens from us? I find that hard to believe. In terms of aliens, I think that they are real. They must be. 

  Interviewer: So you could believe in aliens? 

  David: Oh, yeah. 

  Interviewer: The character you played in 'The X·Files', Fox Mulder, is so dark and moody. Are you dark and moody in life? 

  David: I think so. I think what they wanted was somebody who could be this hearted, driven person, but not behave that way and therefore be hearted and driven but also appear to be normal and not crazy at the same time. And I think that I could, I can, I can afford that. 

  Interviewer: What haunts you now? What drives you now? 

  David: What drives me is failure and success and all those things, so …… 

  Interviewer: Where are you now? Are you haunted and driven, failed or successful, which? 

  David: Yeah, both. 

  Interviewer: All of the above? 

  David: I always feel like a failure. 

  Interviewer: Do you mean now you feel like a failure? 

  David: Yeah, I mean, sometimes you know, like I come back to New York, so its like, everything is different. So I lie on bed and think, two years ago, three years ago, very different. Maybe I'm doing well, but then I think, you know there are just so many other things that I want to do and …… 

  Interviewer: Your father and mother divorced when you were eleven. Does that have effect on your life today that you recognize? 

  David: Well, yeah, I think that the only way to think of it is that, you know, people are saying 'your wound is your goal', you know, 'wherever you're hurt, that's where you'll become stronger. ' So, that's what, that's what it's really about ……

  Interviewer: OK. It's time for short break. We'll be back in a minute. David Duchovney in 'The X·Files', don't go away. 

  SECTION C NEWS BROADCAST

  News Item 1 (For Question 11)

  The Bush administration is warning that continuing mid-east violence threatens to overwhelm US efforts to revise Israeli-Palestinian Peace talks, using the recommendations of the Mitchell commission to bring the two sides together. The administration officials are openly worried the violence and particularly the car bomb attack injured Isreali civilians could undermine what they see as a positive opening towards renewed peace talks presented by the Mitchell report. The US appeal came in the week of the bomb blast Wednesday in Israeli coastal town of Netanya that injured several Israelies. Responsibility for the bombing was claimed by the Palestinian group, Islamic Jihad. At the state department, sopkesman, Phillip Reeker said there can be no justification for terrorism and targeting its civilians, and he urged the Palestinian authority to do all they can to put an end to such incidents which is said to threaten to overtake the latest peace efforts. 

  News Item 2

  Voters in Peru head to the post today to cast their ballots in a run-off presidential election that many hope will mark the end of the nation's political crisis. Opinion polls last week show the modern candidate Arhumdred Toledo with a narrow lead over a left-leaning former President Ellen Gaceya. Both candidates have campaigned on similar populous platforms. Meanwhile pre-election Service indicates that up to 25% of voters in Peru plan to spoil or leave their ballots blank to show their dissatisfaction with both candidates. 

  News Item 3 (For Questions 13-15)

  Canada for the seventh consecutive year ranks the best place to live in the world. But if you are a woman, you are better off in Scandinavia since the UN Human Development Report (2000) released yesterday. Norway is in second place you know for ranking followed by the United States, Australia, Iceland, Sweden, Belgium, the Netherlands Japan and Britain. Finland is in eleventh place followed by France, Switzerland, Germany, Denmark, Austria, Luxembourg, Ireland, Italy and New Zealand. At the other end of the scale, the ten least developed countries that provide the fewest service to their people, from the bottom up, a war-devastated Sierra Leone, Niger, Burkina Faso, Ethiopia, Brandi, Guinean Bissau, Mozambique, Chad, Central African Republic and Mali. 

  SECTION D NOTE-TAKING AND GAP-FILLING

  Good morning, everybody. Today's lecture is about Abraham Maslov's hierarchy of needs. This seems like a physiological topic. Actually it is something psychological. Abraham Maslov is a psychologist, and he is especially known for his theory of human needs. 

  OK, first of all, what is the need? Here, we can simply define it as a personal requirement. Maslov believes that humans are wanting beings, who seek to fulfill a variety of needs. According to his theory, these needs can be arranged in an order according to their importance. It is this order that has become known as Maslov's hierarchy of needs. In this hierarchy of needs, at the most basic level are physiological needs. Fundamentally, humans are just one species of animal. We need to keep ourselves alive. Physiological needs are what we require for survival. These needs include food and water, shelter and sleep. At this level for us humans, Maslov also includes the need for clothing. How are these needs usually satisfied? It is mainly through adequate wages. 

  Then what is the next level of needs? At the next level are safety needs, the things we require for physical and emotional security. Physical security is easy to understand. Everybody needs to keep his body safe from injury, illness, etc. Then what is emotional security? Well, that may be the point in this hierarchy of needs, where humans begin to differ from other animals. We are thinking animals. We have worries, what we fear may be losing a job, or being struck down by a severe disease. Besides physical Security, we need to think we are safe from misfortunes both now and in a forseeable future. How can these needs be met then? According to Maslov, safety needs may be satisfied through job security, health insurance, pension plans and safe working conditions. 

  After this stage come the levels of needs that are particular to human beings. The immediate following level are the social needs. Under this category, Maslov puts our requirements for love and affection and the sense of belonging. We need to be loved, we need to belong to a group not just the family in which we can share with others in common interest. In Maslov's view, this need can be satisfied through the work environment and some informal organizations. Certainly, we also need social relationships beyond the work place, for example, with family and friends. Next, the level of esteem needs. What are esteem needs then? They include both the needs of self-esteem and the need of esteem of others. Self-esteem is a sense of our own achievements and worth. We need to believe that we are successful, we are no worse if no better than others. The esteem of people is the respect and recognition we gain from other people, by or through our work or our activities in other social groups. The ways to satisfy esteem needs include personal achievements, promotion to more responsible jobs, various honors and awards and other forms of recognition. 

  What follows is the top level of this hierarchy of needs. These are the self-realization needs. In other words, they are the needs to grow and develop as people, the needs to become all that we are capable of being. These are the most difficult needs to satisfy. Whether one can achieve this level or not, perhaps determines whether one can be a great man or just an ordinary man. Of course, it depends on different people. The means of satisfying them tend to vary greatly with the individual. For some people, learning a new skill, starting a new career after retirement could quite well satisfy their self-realization needs. While for other people, it could be becoming the best in certain areas. It could be becoming the president of IBM, anyway, being great or ordinary is what others think, while self-realization is largely individual. Maslov suggested that people work to satisfy their physiological needs first, then their safety needs and so on up the needs ladder. In general, they are motivated by the needs at the lowest level that remain unsatisfied. However, needs at one level do not have to be completely satisfied before needs at the next higher level come into play. If the majority of a person's physiological and safety needs are satisfied, that person will be motivated primarily by social needs. But any physiological and safety needs that remain Unsatisfied will keep playing an important role. 

  OK, that's the general picture of Maslov's hierarchy of needs. Just to sum up, I briefly introduce to you Maslov's theory. Maslov thinks there are five kinds of human needs with each one being more important than the preceding one. I hope that you find his ideas interesting and in our next lecture, we will mainly discuss the practical implications of his theory. 

  Now, you have 2 minutes to check your notes, then please complete the 15-minute gap-filling task on Answer Sheet One. This is the end of Part One. 

  答案与详解

  PAPER ONE

  PART Ⅰ LISTENING COMPREHENSION

  SECTION A TALK

  1. 答案: B

  【问句译文】根据该谈话内容,关于办公室的下列哪一种说法是不正确的?

  【试题分析】本题为细节题,可用排除法解答。

  【详细解答】由谈话中提到的"Let's first take a look of the offices,the physical surroundings of most modern companies,especially offices are becoming more and more similar."可知"全球的办公室基本上是一样的"故可排除选项A;根据听到的"this is the feature that...,may be dependent on the size of the company"和"...modem companies pay special attention to the physical surrounding,in order to create an atmosphere conducive to higher working efficiency."可知,办公环境设置与公司规模有一定联系并影响着工作人员的办公效率,可排除选项C和D。只有选项B不合题意,故为正确答案。 

  2. 答案: A

  【问句译文】 由谈话可以推知,和谐的工作关系对你的什么产生直接的影响?

  【试题分析】 本题为细节题。

  【详细解答】 由谈话中提到的"...particularly as the management's assessment of how are you performing can be crucial to your future career."可知,工作表现会直接影响到未来的事业,故选项A promotion(提升,晋级)为正确答案。 

  3. 答案: D

  【问句译文】 假设你在一家小公司工作,有什么不满时会怎么做?

  【试题分析】 本题为细节题。

  【详细解答】 由谈话中提到的"In small businesses,the boss will probably work along side his/her workers.Anything that needs to be sorted out will be done face to face as soon as the problem arises."可知,在小的公司里,有问题应尽快与老板直接面谈。故选项D为正确答案。 

  4.  答案: B

  问句译文:根据该谈话内容,工会在下列哪一方面不起作用?

  【试题分析】 本题为细节题。

  【详细解答】 由谈话中提到的 "When this is not possible,the sides can go to arbitration and bring in a third party from outside to say what they think should happen."可知,当工会与公司自身不能调节问题时,就会请第三者进行仲裁,所以工会不具备仲裁的职能,故选项B为正确答案。 

  5.  答案: C

  问句译文:谈话不包含下列那一项内容?

  【试题分析】 本题为细节题,可用排除法解答。

  【详细解答】 由谈话可知,其内容先后涉及工作关系(work relations)、工会角色(role of the union)和办公室设置(office layout)。故可分别排除选项A、B、D,正确答案为选项C。 

  SECTION B INTERVIEW

  6. 答案: C 

  【问句译文】 关于David的个人背景,下列那一种说法是错误的?

  【试题分析】 本题为细节题。

  【详细解答】 由对话中主持人提到的"...took a few acting classes...",可知David只是参加了几次表演班,并没有接受过专门的职业培训,故选项C的说法是错误的。 

  7. 答案: D

  【问句译文】 David倾向于相信什么?

  【试题分析】 本题为细节题。

  【详细解答】 在对话中,David提到"Well,government conspiracies,I think,are a little far fetched..."由此可知,他倾向于相信政府阴谋,故选项D为正确答案。 

  8. 答案: C 

  【问句译文】 David为何认为他适合电视角色?

  【试题分析】 本题为细节推理题。

  【详细解答】 在谈到David对电视的贡献时,主持人说"I believe,I guess,David,your contribution to the heat series is quite ability.Now let's talk about your personal experience...",由此可知David的个人经验帮助了他的演艺事业的成功。故选项C为正确答案。 

  9. 答案: A

  【问句译文】 由谈话可知,David目前的感觉怎样?

  【试题分析】 本题为细节题。

  【详细解答】 当主持人问到David的现状时,他回答"I always feel like a failure."由此可见,他总是有一种"挫败感",故选项A 为正确答案。 

  10. 答案: C 

  【问句译文】 对于父母离婚一事,David的感想是什么?

  【试题分析】 本题为细节题。

  【详细解答】 在提到父母离婚一事时,David说 "...whenever you are hurt,that's where you'll become stronger."由此可见,他认为父母的离异促进了他的成长。故选项C为正确答案。 

  

  SECTION C NEWS BROADCAST 

  News Item 1 

  11. 答案: A 

  【问句译文】 该新闻的主题是什么?

  【试题分析】 本题为综合题。

  【详细解答】 新闻的第二句话提到"The administration officials are openly worried the violence ……,could under mine what they see as a positive opening……",新闻中还多次提到相关人员担心巴以和平进程,故选项A为正确答案。 

  News Item 2 

  12. 答案: A

  【问句译文】 为何有些选民会浪费他们的选票?

  【试题分析】 本题为细节题。

  【详细解答】 由新闻中提到的"……plan to spoil or leave their ballots blank to show their dissatisfaction with both candidates."可知,有些选民对两个候选人都不满意,故选项A为正确答案。 

  News Item 3 

  13. 答案: D 

  【问句译文】 根据联合国人类发展报告,世上哪儿的妇女的地位最高?

  【试题分析】 本题为细节题。

  【详细解答】 新闻的第二句话说"But if you are a woman,you are better off in Scandinavia since the UN Human Development(2000) released yesterday."由此可知,选项D为正确答案。 

  14. 答案: B

  【问句译文】 哪个国家位居第十二位?

  【试题分析】 本题为细节题。

  【详细解答】 由新闻中提及的"Finland is the eleventh place followed by France……"可知,法国紧随其后,位居第十二位。答案选B。 

  15. 答案: C 

  【问句译文】 根据联合国的报告,最不发达的国家是哪一国?

  【试题分析】 本题为细节题。

  【详细解答】 新闻的最后一句话提到"...from the bottom up war-deviated,Sierra Leone...,"由此可知,选项C为正确答案。 

  SECTION D NOTE-TAKING AND GAP-FILLING 

  1. 答案:basic (或 fundamental)  

  【详细解答】 在谈到Physiological needs时,录音中说"In this hierarchy of needs, at the most basic level physiological needs",由此可知,此处应填写basic 或 fundamental。 

  2.  答案:safety

  【详细解答】 当录音中说到"Then what is the next level of needs?"我们就应集中注意力听下文," At the next level are safety needs,..."由此可知,此处应填写safety。 

  3.  答案:emotional

  【详细解答】 紧接上题,录音解释了safety needs所包含的内容"...the things we require for physical and emotional security. " 

  4.  答案:worries

  【详细解答】 根据录音中提到的"We have worries, what we find may lost my job, what we find ...",此处应填写worries。 

  5.  答案:pension 

  【详细解答】 在谈到解决safety needs的方法时,录音中说到"...safety needs may be satisfied through job security, health insurance, pension plan and safe working conditions. " 故此处应填写pension。 

  6.  答案:work

  【详细解答】 在谈到esteem needs时,录音中说"The esteem of others is the respect and recognition we gain from other people, by or through our work or our achievements and worth. " 故此处应填写work。 

  7.  答案:variable

  【详细解答】 在谈到self-realization needs时,录音中说"The means of satisfying them tend to vary greatly with the individual. " 故此处应填写variable。 

  8.  答案:human

  详细解答:由上下文可知,social,esteem and self-realization needs 应该是 human needs。 

  9.  答案:motivation

  【详细解答】 根据录音中提到的"In general, they are motivated by the needs at the lowest level that remain unsatisfied ...",此处应填写motivation。 

  10.  答案:coexist

  【详细解答】 根据录音中提到的"But any physiological and safety needs that remain unsatisfied will keep playing an important role.",此处应填写coexist。 

  PART Ⅱ PROOFREADING AND ERROR CORRECTION

  1.  答案: height→high

  【详细解答】 height为不可数名词,意为"高度,身高,海拔,顶点"等,故此处应改为可数名词high。 

  2. 答案:a

  【详细解答】 此处steady decline指稳定下降的行为、过程而不是其结果,为不可数名词,故应去掉定冠词a。 

  3. 答案:went∧→on

  【详细解答】 go on为固定搭配,意为"持续"。 

  4. 答案:high→higher

  【详细解答】 根据上下文,此处应为比较级。 

  5. 答案:Europe→European 

  【详细解答】 根据上下文,此处应用形容词作定语修饰名词counterparts。 

  6. 答案: more

  【详细解答】 由上下文可知,more与equally矛盾,故应去掉。 

  7. 答案:nevertheless→also

  【详细解答】 由上下文可知,此处讲的内容与前部分内容之间为递进关系,而非转折关系。 

  8. 答案: that→those

  【详细解答】 由上下文可知,此处所指代的应为前面复数形式的marriages,故指示代词也应该用复数形式。 

  9. 答案: Since→Although(或While)

  【详细解答】 从逻辑上讲,此处应表达让步关系,而非因果关系。 

  10. 答案:in→to

  【详细解答】 to...extent为固定搭配,意为"到……程度"。 

  PART Ⅲ READING COMPREHENSION

  SECTION A 

  TEXT A

  短文大意 :这篇短文介绍的是吉卜赛人在欧洲受敌视的状况。

  16. 答案:D 

  【参考译文】 只有在什么时候吉卜赛人才会联合起来?

  【试题分析】 本题为细节理解题。

  【详细解答】 短文第一段中有这么一句话:"……,only really unifying in the face of enmity from non-Gypsies,……"即"只有面对非吉卜赛人的威胁时才联合起来",由此可知选项D为正确答案。 

  17. 答案:A

  【参考译文】 历史上,除了下列哪类人,在欧洲对吉卜赛人的仇视导致了各种人对他们的迫害?

  【试题分析】 本题为综合理解题。

  【详细解答】 短文第二段中有这么两句话"In Europe their persecution by gadje began quickly,with the church seeing heresy in their fortune-telling and the state seeing anti-social behaviour in their nomadism."即"在欧洲gadje人对他们的迫害迅速开始,教堂认为他们的算命是异教行为,政府认为他们的游牧生活是反社会行为"。 "In more recent times the Gypsies were caught up in Nazi ethnic hysteria,and perhaps half a million perished in the holocaust."即"在更近期的历史中,吉卜赛人被卷入了纳粹党的歇斯底里的种族迫害中,大约有50万人死于这次大屠杀"。由此可知选项B、C、D都对吉卜赛人造成了迫害。所以答案应选A。 

  18.  答案:C

  【参考译文】 根据文章内容,吉卜赛人和犹太人的主要区别在于他们对于什么的观念不同?

  【试题分析】 本题为细节理解题。

  【详细解答】 短文最后一句说"with very few exceptions Gypsies have expressed no great desire for a country to call their own——unlike the Jews,……"即"绝大部分吉卜赛人对建立一个自己民族的国家没有多大欲望,不像犹太人那样",由此可知,他们的主要区别在于他们对他们的身份所持的不同观点上,所以答案应为C。文中未将吉卜赛人与犹太人的语言、文化或习俗进行比较,故A、B、D项都不符合题意。 

  TEXT B 

  短文大意 :在这篇短文中,作者以自身经历讲述了哈莱姆市(Harlem)的变化,表达了他的怀念之情。

  19.  答案:B 

  【参考译文】 在文章开头,作者似乎在暗示哈莱姆市怎么样?

  【试题分析】 本题为推断题。

  【详细解答】 文章第一段讲述了作者还是个小男孩时去哈莱姆市的情景。当时住在Theresa旅店,它是一幢雄伟的砖头建筑。在旅店餐馆里,父亲指出乔·路易斯(一位美国职业拳击运动员)。在第二段开头,作者指出Much has changed since then.接着说,Business and real estate are booming.(商业和房地产繁荣起来。)由此可见,作者在开头想指出哈莱姆市经历了剧烈的变化,故选项B为正确答案。 

  20.  答案:D 

  【参考译文】 当作者回忆旧时的哈莱姆市时,他有怎样的感情?

  【试题分析】 本题为细节理解题。

  【详细解答】 由短文的第四、五段"I miss Mr.Michaux's bookstore...,I miss speaker like Carlos Cooks...",可以推断出作者是很怀念过去的时光的,故选项D为正确答案。 

  21.  答案:A

  【参考译文】 在20世纪20、30年代,哈莱姆市被称作为美国黑人的首都主要是因为它的什么?

  【试题分析】 本题为细节理解题。

  【详细解答】 短文第六段开头说"...in the l920s and '30s,when Harlem renaissance artists,writers,and intellectuals gave it a glitter and renown that made it capital of black America."即"在20世纪20、30年代Harlem新兴的艺术家、作家和知识分子给Harlem带来了光明和名望,使得其成为美国黑人的首都",由此可知,在20世纪20、30年代Harlem被称为首都主要是因为它的艺术和文化,所以答案应为A。 

  22.  答案:C

  【参考译文】 从这篇文章我们可以推断出,从总体上来说,作者的态度怎么样?

  【试题分析】 本题为综合理解题。

  【详细解答】 文章说作者首次来到Harlem大约在20世纪40、50年代,而那时由于20、30年代艺术和文化的发展,Harlem成为美国黑人的首都,但随着经济的发展和商业的繁荣,美国正经历另一次的rebirth,尽管随之而来的也有一些社会问题,如毒品和犯罪等。第八段还另外提到:"Now,you want to shout'Lookin'good!'at this place that has been neglected for so long,"所以作者对Harlem的变化大体上是持肯定态度的。 

  TEXT C 

  短文大意 :这篇文章详细描绘的是某律师事务所的几位工作人员在对应聘者进行面试前的准备工作以及他们的心理状况。

  23. 答案:B 

  【参考译文】 下列哪一项不是公司的招聘要求?

  【试题分析】 本题为细节题,可用排除法解答。

  【详细解答】 短文第一段提到了公司招聘的要求。由"He was married,and that was mandatory."可知A项marriage在要求之内。由"He had a degree in accounting,passed the CPA exam the first time he took it and wanted to be a tax lawyer,which of course was a requirement with a tax firm."可知C项relevant degree也在要求之内。由"McDeere was a male,and there were no women in the firm."可知D项male也是正确选项,所以只有B不符合题意。 

  24.  答案:D

  【参考译文】 秘密调查的详细内容表明公司怎么样?

  【试题分析】 本题为推断题。

  【详细解答】 根据文章第三段的描述,该公司派人调查了McDeere在校的表现甚至是一些琐碎的私人生活细节,可以推断出该公司对McDeere的任何个人细节都感兴趣。 

  25.  答案:A

  【参考译文】 根据文章内容,Lama Quin在面试现场的主要原因是什么?

  【试题分析】 本题为细节理解题。

  【详细解答】 根据文中第五段中的"Lamar Quin was thirty-two and not yet a partner.He had been brought along to look young and act young and project a youthful image for Bendini,Lambert & Locke which in fact was a young firm..."可知Lamar Quin出现在此次面试中的主要原因应为A项,即"他的形象可以给McDeere留下印象"。 

  26.  答案:C

  【参考译文】 读这篇文章,我们会有这种印象,即该公司在招募人员时不怎么样?

  【试题分析】 本题为综合理解题。

  【详细解答】 根据文中第一段及第二段中的内容可知该公司在招聘人员时是精挑细选的(selective)。该公司还派私人侦探去调查McDeere的个人情况,可知他们的招聘是秘密的(secretive)。该公司在工作招聘中对应聘人员的要求是有种族偏见的(racially biased),这由第一段中"He was white,and the firm had never hired a black."可得出结论。所以只有C项perfunctory(敷衍了事,马马虎虎)不合题意。 

  TEXT D 

  短文大意 :这篇短文论述的是当过CEO的政府官员政绩不佳的原因。

  27. 答案:D

  【参考译文】 一位CEO要想在政府部门成功,他必须怎样做?

  【试题分析】 本题为细节题,可用排除法解答。

  【详细解答】 A项是视总统为CEO,这与原文第四段中"But even the president is not really the CEO."是不符的;B项也不合题意,因为"Power in Washington is diffuse and horizontally spread out."(政府权力是分散的,是水平分布的。)所以他对其负责部门不必绝对控制,而是要扮演好总统顾问(adviser to President)的角色;C项在文中并未提及。故正确答案为D。 

  28. 答案:A

  【参考译文】 文章在评论O'Neill当财政部长的经历时,似乎指明什么?

  【试题分析】 本题为推断题。

  【详细解答】 短文第六段以O'Neill当财政部长为例,论述了必须小心行使权力。接着在第七段指出O'Neill公开批评国际货币基金组织给予发展中国家紧急援助的建议,与此同时却赞同针对土尔其、阿根廷和巴西的这类建议,结果两边都不讨好。可见,O'Neill没有行使好自己的权力。故选项A为正确答案。 

  29. 答案:C

  【参考译文】 根据文章内容,政府和企业的不同之处不包括下列哪一项?

  【试题分析】 本题为综合理解题。

  【详细解答】 短文倒数第二段第二句说"Business functions around one predominate organizing principle,profitability...Government,on the other hand,deals with a vast number of equally legitimate and often potentially competing objectives..."可以看出两者的行为性质是不同的,所以A项是区别之一。短文倒数第三段第二句说"...you can't just quit.Jack Welch's famous law for re-engineering General Electric was to be first or second in any given product category,or else get out of that business.But if the government isn't doing a particular job at peak level,it doesn't always have the option of relieving itself of that function."可以看出商业部门有其自由撤出的权力而政府部门一般不可,所以B项也是区别之一。此外两者权力分布也是不同的,正如文中第四段所述"Power in a corporation is concentrated and vertically structured.Power in Washington is diffuse and horizontally spread out."由此看来只有C项不是两者的区别,故为正确答案。 

  30.  答案:B

  【参考译文】 作者似乎在暗指由CEO转变过来的政府官员怎样?

  【试题分析】 本题为推断题。

  【详细解答】 在短文的最后一段,作者指出,CEO长期习染崇拜、奉承的文化,因而他们很难相信自己有必要去听取别人的意见或学习他人的优点,特别是那些为人们所不齿的政客、官僚和媒体,而他们即使清楚地认识有这个必要,也不会那样做,故选B。 

  SECTION B 

  TEXT E 

  短文大意 :本文介绍的是有关美国网吧的状况。

  31.  答案:C

  【参考译文】 文章主要是关于美国的什么?

  【试题分析】 本题为综合判断题。

  【详细解答】 从文中大量有关美国网吧的数字,就可判断本文主要是关于美国网吧的。 

  TEXT F 

  短文大意 :这篇短文介绍了有关瑜珈的情况。

  32.  答案:A

  【参考译文】 在这篇短文中,作者试图做什么?

  【试题分析】 本题为综合判断题。

  【详细解答】 快速浏览全文可知,第一段第一句话直接道出西方练瑜珈的人只注重形体修塑,没注意意识修炼。第二段的开头介绍了瑜珈功的正确、本来的原理,指出瑜珈功重在内心修炼。第三段指出瑜珈由外部瑜珈与内部瑜珈组成,并详细介绍了内部瑜珈的境界感受,这是为了体现内心修炼的重要性,从而批评了西方练瑜珈的人的不当之处,故选项A为正确答案。 

  TEXT G 

  短文大意 :这是一篇有关基辛格的新书Does American Need a Foreign Policy的书评。

  33.  答案:D

  【参考译文】 评论者对亨利·基辛格的新书的评价基本上怎么样?

  【试题分析】 本题为细节题。

  【详细解答】 根据短文第2段和第3段的第1句、第6段的第2句可知,作者对基辛格的新书还是持赞扬态度,故正确答案为D。 

  TEXT H 

  短文大意 :本文论述的是有关小语种逐渐消失的问题。

  34.  答案:A

  【参考译文】 在这篇文章中,作者表达了对什么的关心?

  【试题分析】 本题为细节题。

  【详细解答】 由文章第一段可知,小语种在逐渐消失。接着浏览各段开头或文中词语可知,文章主要围绕language,linguistic extinction进行论述。而且从文中倒数第六段和倒数第三段可知,文中还谈及了如何挽救小语种。故本题选A项。 

  TEXT I 

  短文大意 :这是一份招聘工程管理员的广告。

  35.  答案:B

  【参考译文】 工程管理员的工作主要与什么有关?

  【试题分析】 本题为细节题。

  【详细解答】 从招聘启事的副标题AGRICULTURAL REHABILITATION PROJECT可知正确答案为选项B。 

  36.  答案:D

  【参考译文】 工作合同多久签一次?

  【试题分析】 本题为细节题。

  【详细解答】 由短文倒数第三段中的"a twelve-month contract with a salary"可知正确选项为D。 

  TEXT J 

  短文大意 :这是一篇有关感冒的医学文章。

  37.  答案:A

  【参考译文】 谁发现了名为M2的蛋白质?

  【试题分析】 本题为细节题。

  【详细解答】 在文中迅速找寻M2一词。从第二段第二句可得出  是比利时的一所大学里的科学家发现了M2,故正确答案为选项A。 

  38.  答案:C

  【参考译文】 文章引证了导致口气不清新的几条原因?

  【试题分析】 本题为细节题。

  【详细解答】 在标题为BAD BREATH的三段文字中,分别给出了导致口气不清新的三个原因:an illnes,foods like onions or garlic,food molecules,故正确答案为选项C。 

  TEXT K 

  短文大意 :这是一篇介绍Moore生平的文章。

  39.  答案:D

  【参考译文】 Moore第一次受委托办事是在何时?

  【试题分析】 本题为细节题。

  【详细解答】 在文中寻找first commission。从第三段第二句可知His first commission,received in l928……,故正确答案为选项D。 

  40.  答案:B

  【参考译文】 Moore在哪儿第一次获得国际性奖项?

  【试题分析】 本题为细节题。

  【详细解答】 在文中寻找international prize。从第四段第二句可知"……won the International Sculpture Prize at the 24th Venice Biennale,the first of countless international accolades……,由此可知,正确答案为选项B。 

  PAPER TWO 

  PART Ⅳ TRANSLATION

  SECTION A CHINESE TO ENGLISH

  【参考译文】 

  Before I fell ill, my parents doted on me a lot. I could have my way at home. Once I was isolated and confined in a chamber on the hillside of the garden, I suddenly felt I was neglected and became very depressed. One spring evening, my parents held a Banquet in the garden, where all sorts of flowers were in full bloom. In no time, a crowd of their guests collected and laughter was heard all over there. I, without being noticed, lifted the curtain in my small room, only to spy the bustle of a kaleidoscopic world down in the garden, and my elder sisters, brothers and my cousins, each full of the joys of spring, were shuttling among the guests. Quickly enough, I was thrown into a fist of sorrowful anger at being forgotten and discarded by the rest and could not help crying my heart out. 

  SECTION B ENGLISH TO CHINESE

  【参考译文】〖HT5"SS〗

  在其经典小说《开拓者》中,詹姆士·菲尼摩尔·库珀让主人公,一个土地开发商,带他的表妹参观正在由他承建的一座城市。他描述了宽阔的街道,林立的房屋,热闹的都市。他的表妹环顾四周,大惑不解。她所看见的只是一片树林。"你想让我看的那些美景和改造了的地方在哪儿啊?"她问道。他见表妹看不到那些东西,感到很惊讶。"哪儿?到处都是啊!"他答道。虽然那些东西还未建成在大地上,但他已在心中将它们建好了。对他来说,它们都是实实在在的,宛如已建成竣工一样。

  库珀这里阐明的是一种典型的美国人特性:着眼于未来,即能够从未来的角度看待现在;可以自由地不为过去所羁绊,而在情感上更多地依附于未来的事物。正如阿尔伯特·爱因斯坦曾经说过的那样:"对美国人来说,生活总是在发展变化中,从来不会静止不变。"

  PART Ⅴ WRITING

  【参考范文】 

  Love, and Then Be Loved

  In recent years, more and more teachers complain that their students are indifferent to others. Some even worry that the young generation might ruin the future of China.  To be sure, many of our young people cannot see eye to eye with this view.  However, as a university student, I myself would like to content that we young people today are in general more self-centered and unsympathetic than our previous generations. 

  To start with, most, if not all, young people choose to attend exclusively to their own needs. In their eyes, It is all too natural to seek satisfaction from what they do, even if it may mean inconvenience to others.  Take my dormitory for example.  It is a common scene here that a roommate cheerfully talks to his girlfriend on the phone at midnight when others are struggling for a sound sleep. One may complain now and then, but to no avail. In fact, the others, to the exclusion of me, live their dormitory life much in the same way. When I take a nap at noon, they often play cards.  They have no regard for others.  Life is a joy to them, yet they often enjoy it to the neglect of others' feelings. In sharp contrast, our caring parents always pay heed to our needs and those of others. Whenever my father comes back home late in the night, he tiptoes In for fear that he might awake me. 

  Moreover, our young people tend to be insensitive to others' difficulty.  When a classmate falls ill, few people offer to help, but regard it as none of their business.  Some students in my class come from poor families. Yet, they are active mobile phone users, who may spend twice as much as what their parents earn from arduous labor.  When asked why they behave so, they answer that their parents have the obligation to accommodate their expenses. Personally, I detest their answer, for I know my parents never thought that way when they were young.  Being aware of their parents' financial difficulty, they managed to save every penny they could. 

  For the above reasons and those not mentioned here, I subscribe to the view that young people in today's China are more self-centered and unsympathetic than were our previous generations. It is high time that we learned from older generations so that a harmonious and splendid future can be anticipated. 

  

  本套真题测试的语言重点:

  重点单词:

  confound 挫败,使落空

  mercenary 外国雇佣兵

  nomadism游牧生活,流浪生活

  burgeon迅速增长,发展繁荣

  venerated受尊敬的

  founder失败,崩溃

  holster  维持

  重点词组:

  fragmented and fractious分裂而且难以驾驭的

  downplay their profile贬低他们的形象或影响

  decry...as 谴责...

  run/ride roughshod over  残暴地(或盛气凌人地)对待,对……横行霸道

  at peak level处于最好的水平


2003年英语专业四级试题及答案

听力原文

  PART Ⅱ DICTATION

  Salmon

  Every year, millions of salmon swim from the ocean into the mouths of rivers and then steadily up the rivers. Passing through waters, around rocks and waterfalls, the fish finally reach their original streams or lakes. They dig out nests in the riverbed and lay their eggs. Then, exhausted by their journey, the parent salmon die. They have finished the task that nature has given them. Months, or years later, the young fish start their trip to the ocean. They live in the salt water from 2-7 years, until they, too are ready to swim back to reproduce. Their life cycle helps man provide himself with a basic food-fish. When the adult salmon gather at the river mouths for the annual trip up the rivers, they are in the best possible condition, and nearly every harbor has its salmon fishing fleet ready to catch thousands for markets.

  Now, you have two minutes to check through your work.

  PART Ⅲ LISTENING COMPREHENSION

  In sections A, B and C, you will hear everything once only. Listen carefully and then answer the questions that follow. Mark the correct answer to each question on your answer sheet.

  SECTION A  STATEMENT

  In this section, you will hear seven statements. At the end of each statement you will be given 10 seconds to answer the question.

  1. You must relax. Don't work too hard. And do watch your drinking and smoking.

  2.We hadn't quite expected the committee to agree to rebuild the hospital, so we were taken aback when we got to know that it had finally agreed.

  3.The coach leaves the station every 20 minutes. It's 9:15 now, and you have to wait for five minutes for the next one.

  4.Perhaps Jane shouldn't have got married in the first place. No one knows what she might have been doing now, but not washing up. That's for sure!

  5.I happen to be working on a similar project at the moment. I am only too pleased to help you.

  6.The man arrived for the ceremony with patched jackets and faded jeans that the average person would save for mowing the lawn in his garden at the weekend.

  7.Mark! Here you are! This is the last place in the world I would have expected to find you.

  SECTION B  CONVERSATION

  In this section you will hear 10 short conversations between two speakers. At the end of each conversation, you will be given 10 seconds to answer the question.

  8.  W: I couldn't stand this morning. My right leg went stiff.

  M: I'm afraid it's probably a side-effect from the drugs I put you on.

  9.  W: How did your writing go this morning? Is the book coming along alright?

  M: I'm not sure. I think the rest of it will be difficult to write.

  10. W: Is there anything you can do to make the cold go away more quickly?

  M: No, there isn't. And a cold isn't really serious enough for a visit to a doctor.

  11. W: Look! What have I got here!

  M: Oh. So you did go to that bookstore!

  12. M: Excuse me. Has there been an emergency?

  W: Oh, no sir. There's just a storm, so the plane will leave a little later this afternoon.

  13. W: I wish I hadn't hurt Linda's feeling like that yesterday. You know I never meant to.

  M: The great thing about Linda is that she doesn't hold any grudges. By tomorrow she'll have forgotten all about it.

  14. M: My grades are not bad, but not good enough. I know I didn't study at all this semester. Now I have to work very hard next semester to keep my scholarship.

  W: I'll see you in the library, then.

  15. W: I'll wear this blue jacket for the evening. I like the color on me, don't you think?

  M:I think it looks terrific on you-really!

  16. W: Do you know that Sam turned down that job offer by a travel agency?

  M: Yeah. The hours were convenient, but had he accepted it, he wouldn't have been able to make ends meet.

  17. W: At the rate it is being used, the printer is not going to make it through the rest of the year.

  M: The year? It is supposed to be good for four!

  SECTION C  NEWS BROADCAST

  Questions 18 and 19 are based on the following news. At the end of the news item you will be given 20 seconds to answer the questions. Now listen to the news. 

  The U.N resolution calls for greater international intelligence and law enforcement cooperation. And it requires states to change their banking laws in order to police the global network of terrorisms financiers. It makes providing funds for terror activities a criminal offence and would freeze bank accounts of those who sponsor terrorism.

  Questions 20 and 21 are based on the following news. At the end of the news item you will be given 20 seconds to answer the questions. Now listen to the news. 

  A police spokesman said the devices were made safe by explosive experts in the Ardorn district, where a woman was shot in the leg and 13 police officers were injured during a second successive night of violence. Northern Ireland's police chief had earlier called on community leaders to work together to end the violence. The violence has erupted sporadically throughout a summer of Sectarian tension in northern Belfast.

  Question 22 is based on the following news. At the end of the news item you will be given 10 seconds to answer the question. Now listen to the news. 

  Airlines are being hit with huge increases to ensure their planes after the terrorist attacks in the United States. Goshork Insurance Holdings, which ensures aircraft around the world, said rates had soared as much as 10 fold since the September 11th terror attacks. Airlines around the world have cut services and dismissed staff as their business has plunged in the wake of the crisis. They are also struggling with increased security costs.

  Questions 23 and 24 are based on the following news. At the end of the news item you will be given 20 seconds to answer the questions. Now listen to the news. 

  A Pakistani lawyer said the resumption of the trial of eight foreign aid workers accused of preaching Christianity in Afghanistan has been put off until Sunday. He had met earlier Saturday with the aid workers, 2 Americans, 2 Australians, and 4 Germans. They insist they were in Afghanistan to help the poor, not to convert them. The penalty for these captured aid workers could range from expulsion to a jail term and death sentence.

  Question 25 is based on the following news. At the end of the news item you will be given 10 seconds to answer the question. Now listen to the news. 

  On the 20th anniversary of the first official report on AIDS, the head of the United Nations AIDS program warns that the deadly disease may only be at its early stages in many parts of the world. Dr. Piu said the disease has already reached staggering proportions since first being identified in 1981. 58 million people worldwide have contracted the HIV virus, which causes AIDS, while 22 million have died from related illnesses. The UN estimates the world's HIV positive population at 36 million, including 25 million in sub-Saharan Africa.

  International officials warn the disease will have disastrous political, social, and economic consequences in many developing countries.

  This is the end of listening comprehension. 

  答案与详解

  PART Ⅰ WRITING

  SECTION A  COMPOSITION

  THE IMPORTANCE OF KEEPING A GOOD MOOD

  People in modern society live under a lot of pressure. I see three kinds of pressure working on people today: pressure from education, family and career. It is easy to blame the school for charging too much money, the family members for the heavy burden, the society for the fierce competition. I think people should relax. It is important for them to keep a good mood under whatever circumstances.

  Long gone are the days when people lived their life with a certain relaxation, sampling a wide variety of activities-film, music, art, poetry. But now, a lot of people suffer from a lot of pressure. They can't communicate well with co-workers and family members, and have unbalanced, one-dimensional lives. Some people complain of symptoms of stress, for instance, loss of appetite, a complete sense of exhaustion, insomnia and low morale. Thus have destructive effect on their health. People tend to lose temper easily, and this may interfere personal relationship. What's more, a high rate of suicide may warn people of their emotional well-being.

  To be a healthy person physically and psychologically, one should keep a good mood, according to some psychology experts. Those little things may seem relatively insignificant-reading mystery novels, playing volleyball, spending time with family and friends, maintaining your emotional well-being, getting plenty of rest, going fishing. They can help you keep a good mood.

  SECTION B  NOTE-WRITING

  Oct. 18, 2003

  Dear Clare, 

  Thank you for inviting me to your house-warming party this weekend. But I'm afraid I couldn't go there, because I have an important business appointment on that day. I saw some pictures of your new house, which is so cozy and comfortable. I sincerely hope that you have a good day.

  Yours,

  Gou Ming

  PART III LISTENING COMPREHENSION

  SECTION A STATEMENT

  1. 答案:D

  【问句译文】 关于听者哪一句是不对的?

  【试题分析】 本题为细节题。

  【详细解答】 只要听懂了这句话"Don't work too hard, and do watch your drinking and smoking", 由此可以判断听者工作辛苦,既抽烟又喝酒。故选项D为正确答案。

  2. 答案:C

  【问句译文】 当讲话者听到这个消息,他感觉如何?

  【试题分析】 本题为细节题。

  【详细解答】 只要听懂了关键词 "We were taken aback……", 意为"吃惊",这与选项C的意思一致。

  3. 答案:C

  【问句译文】 下一班长途汽车什么时候离站?

  【试题分析】 本题为计算题。

  【详细解答】 本题的关键词是 "It's 9:15.", "……wait for 5 minutes……" 意为现在9:15,下一班车等5分钟。由此可以判断下一班长途汽车9:20离站。

  4. 答案:B

  【问句译文】 讲话者认为简可能会?

  【试题分析】 本题为细节题。

  【详细解答】 这句话的关键词是"but not washed-up...", 其中"washed up" 意为"不再有希望的,没有发展前途的",前面还有一句话 "No one knows what she might be doing now……"由此可以判断讲话者谈论简的事业还是有希望的。故B为正确答案。

  5. 答案:C

  【问句译文】 这句话是什么意思?

  【试题分析】 本题为细节题。

  【详细解答】 只要听清 "I'm only too pleased to help you," 就得出选项C为正确答案。

  6. 答案:B

  【问句译文】 这句话隐含了什么意思?

  【试题分析】 本题为推理题。

  【详细解答】 本题的关键句 "with patched jackets and faded jeans that average people would save for mowing the lawn in his garden……". 字里行间隐藏的信息说明他的穿着不合适,故B为正确答案。

  7. 答案:B

  【问句译文】 讲话者是什么意思?

  【试题分析】 本题为细节题。

  【详细解答】 本题的关键句 "This is the last place in the world I would expect to find you." 由此可以判断讲话者没想到会在这儿找到他。

  

  SECTION B CONVERSATION

  8. 答案:C

  【问句译文】 两位说话者之间是什么关系?

  【试题分析】 本题为细节题。

  【详细解答】 录音中女士抱怨腿发硬,男士说 "……side effect from the drugs I put you on." 意为可能是我给你开的药的副作用。可见两人是医生-病人关系。故选项C为正确答案。

  9. 答案:A

  【问句译文】 男士认为他的写作如何?

  【试题分析】 本题为细节题。

  【详细解答】 本题的关键词是 "difficult",男士说 "rest of it would be difficult to write" 意为余下的部分不好写,故选项A为正确答案。

  10. 答案:D

  【问句译文】 我们可以从对话中知道什么?

  【试题分析】 本题为细节题。

  【详细解答】 本题的关键句是 "Cold is not serious enough for a visit a doctor." 意为感冒不至于如此严重,而要去看医生。故选项D为正确答案。

  11. 答案:A

  【问句译文】 男士以前假定什么?

  【试题分析】 本题为细节题。

  【详细解答】 本题的关键句是 "so you did go to that bookstore." 意为你确实去了书店。

  故推测男士在此以前认为女士会去书店。故选项A为正确答案。

  12. 答案:A

  【问句译文】 关于这次航班,我们知道什么?

  【试题分析】 本题为细节题。

  【详细解答】 本题的关键词是 "leave a little later" 由此断定,航班将有短期延误。故选

  项A为正确答案。

  13. 答案:C

  【问句译文】 男士对琳达的评价如何?

  【试题分析】 本题为推理题。

  【详细解答】 本题的关键句是"She doesn't hold any grudges","forget"。 意为"她没有怀恨在心";"忘记了"。由此可见,选项C"她是宽容的"为正确答案。"forgetful"意为健忘的;"considerate"意为体贴的,关心的;"careless"意为粗心的。

  14. 答案:A

  【问句译文】 女士是什么意思?

  【试题分析】 本题为语气判断题。

  【详细解答】 男士说下学期他要用心读书,女士回答"I'll see you in the library then",从女士的语气判断,她不相信男士能够做到。

  15. 答案:A

  【问句译文】 男士认为女士的服饰选择如何?

  【试题分析】 本题为细节题

  【详细解答】 本句关键句是"looks terrific on you", 意为女士的穿着很漂亮。

  16. 答案:B

  【问句译文】 为什么山姆拒绝了这份工作?

  【试题分析】 本题为细节题。

  【详细解答】 本题的关键句是"to make ends meet"表明收支相抵,由此推断山姆拒绝这份工作是因为薪水不够高。

  17. 答案:C

  【问句译文】 男士的语气如何?

  【试题分析】 本题为推理题。

  【详细解答】 录音中女士说印刷商的速度太慢,今年不可能完成任务。男士说恐怕四年才行。本题的关键句是 "it is supposed to be good for four." 由此断定男士是持幽默的语气。

  SECTION C NEWS BROADCAST

  18. 答案:D

  【问句译文】 这项联合国决议是有关什么的严格控制?

  【试题分析】 本题为细节题。

  【详细解答】 这篇新闻讲的是联合国采取措施,严格控制恐怖活动。新闻中讲到"……providing funds for terror activities…… "即严格控制提供恐怖活动的基金,由此可知,故选项D为正确答案。

  19.  答案:B

  【问句译文】 这项联合国决议明确地要求州政府做到什么?

  【试题分析】 本题为细节题。

  【详细解答】 新闻中说"And it requires states to change its banking laws.",即要求州政府改变它的银行法规。revise意为"修改,改变",与这句话意思一致,故为正确答案。

  20.  答案:D

  【问句译文】 一共有多少人在这次暴力中受伤?

  【试题分析】 本题为细节题。

  【详细解答】 新闻中说"……a woman was shot in the leg, 13 police officers were injured during the violence."一共有14人在这次暴力中受伤。只有D为正确内容。

  21.  答案:C

  【问句译文】 这次暴力持续了多长时间?

  【试题分析】 本题为细节题。

  【详细解答】 解答本题的关键是听清"……throughout a summer of……."就可以选出正确答案C。

  22.  答案:D

  【问句译文】 在这次美国的恐怖袭击以后,保险费用上涨幅度为多大?

  【试题分析】 本题为细节题。

  【详细解答】 这篇新闻讲的是美国 "9.11" 恐怖事件以后,保险费用幅度上涨,航空业所面临的危机。解答本题的关键是听清"……rates had soared as much as ten fold ……",就可知道保险费用幅度上涨了10倍。故选项D为正确答案。

  23.  答案:C

  【问句译文】 在阿富汗,为什么八名外援工人被捕?

  【试题分析】 本题为细节题。

  【详细解答】 这篇新闻讲的是在阿富汗,八名外援工人被指控传播宗教思想,以及他们将可能面临的刑罚。只要听清"……accused of preaching Christianity……",即被指控传播宗教思想,所以C项是正确答案。

  24.  答案:A

  【问句译文】 下列哪一项不是他们将可能面临的刑罚?

  【试题分析】 本题为细节题。

  【详细解答】 新闻最后一句说"The penalty for these captured aid workers could range from expulsion to a jail term, and death sentence.",即"被抓的八名外援工人能面临的刑罚可能是驱逐出境、入狱或死刑。" 故选项A为正确答案。

  25.  答案:B

  【问句译文】 根据这篇报道,有多少爱滋病HIV阳性患者?

  【试题分析】 本题为计算题。

  【详细解答】 这篇新闻讲的是全世界爱滋病状况的严峻性。新闻中说"58 million people worldwide have contracted HIV virus which causes aids, while 22 million have died from related illness." 即全世界58,000,000 人患爱滋病,其中22,000,000死于爱滋病引起的相关疾病。可通过计算,得到这题的答案。如果听清 "The UN estimated HIV positive population at 36 million,"联合国统计爱滋病HIV阳性患者达到36,000,000人。也能得到这题的答案。故选项B为正确答案。

  PART IV CLOZE

  短文大意 :这篇短文介绍麦当劳薯条的发展及其成功之处。

  26. 答案:C

  【试题分析】 本题为动词词义辨析题。

  【详细解答】 这句话是说"在厨房,土豆去皮切成条状煎炸。"scaled意为"刮(鱼)鳞"; stripped意为"脱衣;剥夺"; peeled意为"除掉(水果等)皮";sliced意为"将某物切成薄片"。故选项C为正确答案。

  27. 答案:A

  【试题分析】 本题为语法题,考查连词的用法。

  【详细解答】 As后面可接从句,选项B、C、D后面只能接名词词组。此处需要连词,故选项A为正确答案。

  28. 答案:B

  【试题分析】 本题为动词词义辨析题。

  【详细解答】 ensure意为"确保、保证",后面可接从句;而ensue意为"因而发生,接着";enrich意为"使……富有";enable意为"使……能够", 选项A,C,D不能接从句,故选B。

  29. 答案:A

  【试题分析】 本题为动词词义辨析题。

  【详细解答】 这句话是说:麦当劳在1966年转向冷冻薯条的制作。switching to意为"改变";diverting from/to意为"转移";modifying 意为"修改";altering 意为"改动(强调事物本身大小、好坏的变化)"。故本题A最恰当。

  30. 答案:D

  【试题分析】 本题考查副词的用法

  【详细解答】  本句译为:然而,这个变化在国家的农业和饮食上产生了深远的影响。still 意为"仍然";anyway 意为"无论如何";besides 意为"除……之外";nevertheless意为"然而"。根据上下文,此处需要转折关系的副词,故选项D为正确答案。

  31.  答案:C

  【试题分析】 本题考查名词的用法。

  【详细解答】  brand意为"商标,商品的牌子";stuff意为"东西";commodity意为"商品,产品",故选项C为正确答案。

  32.  答案:B

  【试题分析】 本题考查关系代词的用法。

  【详细解答】 这是考查定语从句的用法,"that"关系代词,指代前面的plants,而A、C、D均无此功能。

  33.  答案:D

  【试题分析】 本题考查介词的用法。

  【详细解答】  介词"of"表示所属关系,本句译为:麦当劳的扩张和低成本、高产量薯条的普及改变了美国人的饮食方式。

  34.  答案:A

  【试题分析】 本题为副词用法辨析题。

  【详细解答】  本句译为:薯条长期受到消费者、竞争者,甚至食品评论家的赞扬。根据上下文,正确答案为选项A 。

  35.  答案:A

  【试题分析】 本题为形容词词义辨析题。

  【详细解答】 distinctive意为"有特色的,与从不同的";distinct意为"清晰的,明白的";distinguished意为"卓越的,著名的";distinguishable意为"可区别的"。根据上下文可知选项A为正确答案。

  36.  答案:B

  【试题分析】 本题为动词词义辨析题。

  【详细解答】 possesses意为"拥有,占有";buys意为"购买";acquires意为"获得,取得";grows意为"种植"。本句意为:他们与众不同的口味,并不是源于麦当劳所购买的土豆,以及他们所拥有的技术。故选项B为正确答案。

  37.  答案:C

  【试题分析】 本题为形容词词义辨析题。

  【详细解答】 exact意为"确切的";identical意为"完全相同的,一模一样的",如:identical twins(同卵双胞胎);same意为"同样的";alike意为"相同的",但alike一般在句尾,不修饰名词。故C为正确答案。

  38. 答案:D

  【试题分析】 本题为名词词义辨析题。

  【详细解答】 woks意为"锅";pots意为"罐,壶";boilers意为"锅炉";fryers意为"油煎锅"。故选项D为正确答案。

  39. 答案:D

  【试题分析】 本题为副词词义辨析题。

  【详细解答】 这句话是说"薯条的口味在很大程度上取决于烹饪用的油。"adequately意为"足够地"; massively意为"巨大地";plentifully意为"丰富地";largely意为"在很大程度上",故选项D为正确答案。

  40. 答案:A

  【试题分析】 本题为名词词义辨析题。

  【详细解答】 本句是说"这种混合油使得薯条有着与众不同的口味。"flavour意为"口味";fragrance意为"芳香,香气";smell意为"气味";perfume意为"香水",故A为正确答案。

  

  PART V GRAMMAR AND VOCABULARY

  41. 答案:D

  参考译文:农业是这个国家财富主要的来源,其中小麦是最主要的谷类作物。

  【试题分析】 本题考查对非谓语动词的掌握。

  【详细解答】 这是一个独立主格结构。wheat前用逗号,且没有连词,表示这一句起补充说明的作用,该空不能用动词的形式,则being为唯一之选。

  42. 答案:A

  参考译文:到目前为止,杰克已经失踪两天了,我开始担心他的安全。

  【试题分析】 本题考查时态的用法。

  【详细解答】 由句中时间状语"for some time now"可知,这里要用现在完成时;选项A为现在完成进行时,表示动作从过去某一时间一直持续到现在,故为正确答案。

  43. 答案:C

  参考译文:树的后面有小山,它的壮丽景色在河面上真实地反射出来。

  【试题分析】 本题为语法题,考查关系代词的用法。

  【详细解答】 这是一个定语从句,此处缺少一个关系代词。whose 指代前面的hills, whose magnificence表示它的壮丽景色。

  44. 答案:B

  参考译文:你说今天下午谁会到办公室找我?

  【试题分析】 本题考查插入语的用法。

  【详细解答】 did you say是一个插入语。比如说 Who do you think is the best teacher i

  n our school? 即你认为谁是我们学校最好的老师?

  45. 答案:D

  参考译文:——艾伦喜欢汉堡包吗?——喜欢,以至于他几乎每天吃。

  【试题分析】 本题考查连词的用法。

  【详细解答】 这句话实际上是 (He likes Hamburgers) so much so that he eats them almost every day. 连词"so that"如此……以致。

  46. 答案:B

  参考译文:你的想法,如同她的想法一样,对我来说不同寻常。

  【试题分析】 本题考查对代词的掌握。

  【详细解答】 只有hers可指代她的想法。而her, herself表示她自己。故B为正确答案。

  47. 答案:A

  参考译文:开幕式是一件大事,我们必须准备。

  【试题分析】 本题为语法题,考查对虚拟语气的掌握。

  【详细解答】 在类似essential, necessary之类的形容词后面,句子的动词形式为 (should)+动词原形。故选项B不对。 而essential 后面一般跟to/for, 故A为正确答案。

  48.  答案:B

  参考译文:如果时间允许,庆祝会如期举行。

  【试题分析】 本题考查独立主格结构的用法。

  【详细解答】 当从句的主语和主句的主语不一样时,且没有连词,从句的主语用独立主格结构。Time permitting相当于If time is permitted. 故B为正确答案。

  49. 答案:D

  参考译文:尽管我喜欢经济学,但我更喜欢社会学。

  【试题分析】 本题考查连词的用法。

  【详细解答】 在这四个选项中, as表示"尽管",引导的条件状语从句时,从句需倒装,即将形容词、副词提至句首,故选项D为正确答案。

  50.  答案:C

  参考译文:进一步讨论这个事情是无益的,因为今天你我都在任何事情上不愿达成一致

  【试题分析】 本题为语法题,考查对主谓一致原则的掌握。

  【详细解答】 若两个或两个以上的主语由 "or", "nor", "either……or", "neither……nor", "not only……but also",连接,遵守就近原则。故选项C为正确答案。

  51.  答案:A

  参考译文:他们克服所有的困难,提前两个月完成这个项目,这是我们没有预料到的。

  【试题分析】 本题考查关系代词的用法。

  【详细解答】 这是一个非限制性定语从句,此处缺少一个关系代词,指代前面的整个句子。在这四个选项中,只有选项A有此功能,因此A项是正确答案。

  52.  答案:B

  参考译文:由于多年的劳作,他身体状况很差。他已不是二十年前的他了。

  【试题分析】 本题为考查关系词的用法。

  【详细解答】 这是一个定语从句,此处缺少一个关系代词。选项B为正确答案。

  53.  答案:C

  参考译文:如果她稍微改变一点会更可爱,不是吗?

  【试题分析】 本题为考查反意疑问句的用法。

  【详细解答】 该题句子的主干 "She would have been more agreeable", 故附加疑问句为 "wouldn't she".

  54.  答案:D

  参考译文:三千英尺处, 没有了宽广的平原, 遥远的山脉随处可见。

  【试题分析】 本题为词语辨析题。

  【详细解答】 on view意为"展览着";at a glance意为"一看就"; on the scene意为"在场";in sight 意为"(指某物)看得见"。根据句意,选项D为正确答案。

  55.  答案:A

  参考译文:文明发展的最初两个阶段是发明了武器和发现了火,尽管没有人确切地知道后者使用的时间。

  【试题分析】 本题为形容词词义辨析题。

  【详细解答】 latter意为"(刚提到的两者之间的)后者"; latest意为"最近的,最新的";later意为"后来"; last意为"最后的"。根据句意,只有选项A表示"后者",选项A为正确答案。

  56. 答案:C

  参考译文:把交通阻塞的时间顾及在内,我们将花二十分钟到达火车站。

  【试题分析】 本题为词语辨析题。

  【详细解答】  acknowledging意为"承认,供认"; affording意为"提供,给予",allowing for意为"顾及"; accounting for意为"对……作出满意的解释"。 根据句意,选项C为正确答案。

  57. 答案:B

  参考译文:他将为他的无礼行为承担后果。

  【试题分析】 本题为动词短语词义辨析题。

  【详细解答】  answer to意为"对……有反应,顺从"; answer for意为"对…负责,承担后果"; answer back意为"无礼回嘴,顶嘴"。根据句意,选项B为正确答案。

  58. 答案:A

  参考译文:除了极个别情况外,前总统不公开露面。

  【试题分析】 本题为词语辨析题。

  【详细解答】  rare意为"稀少的,罕有的,";unusual意为"不平常的"; extraordinary意为"不平常的,特别的"; unique意为"唯一的,独一无二的"。故选项A为正确答案。

  59. 答案:B

  参考译文:我们一直听到赞扬你工作做得好的话。

  【试题分析】 本题为形容词词义辨析题。

  【详细解答】  favoured意为"有利的,称心的"; favourable意为"令人满意的,表示赞许的";favourite意为"最受喜欢的"。根据句意,选项B为正确答案。

  60.  答案:D

  参考译文:在夏季度假期间,这个海边旅馆没有空房间。

  【试题分析】 本题考查固定搭配的掌握。

  【详细解答】  empty意为"空的",强调房间里无物体; blank意为"空白的",如 "a blank page空白的一页"; deserted意为"荒废的,(因被弃)无人住的"; vacant意为"(地方等)空的,未被占用的",如 "vacant house / room空屋子(房间)"。根据句意,选项D为正确答案。

  61.  答案:A

  参考译文:笔直往前开,然后你将看到一个上海-南京高速公路的指示牌。

  【试题分析】 本题为名词词义辨析题。

  【详细解答】 sign意为"牌子,指示牌";mark意为"(书写的,印刷的)符号";signal意为"信号,手势,暗号";board意为"木板"。根据句意,选项A为正确答案。

  62.  答案:C

  参考译文:只要有可能,艾就炫耀他的日语讲得好。

  【试题分析】 本题为动词短语词义辨析题。

  【详细解答】 show up指"出现";show around指"为某人作向导";show off指"炫耀,卖弄"; 没有show out词组。根据句意,选项C为正确答案。

  63.  答案:B

  参考译文:除了一些纸片和布以外,这个房客什么也没留下。

  【试题分析】 本题考查固定搭配的掌握。

  【详细解答】 sheets指"片,块,张",可以说 "a sheet of paper",但一般不能和 "cloth"搭配;scraps意为"小片,碎片";pages指"页", 不能和 "paper, cloth"搭配;slices指"(食物上切下的)薄片"。根据句意,选项B为正确答案。

  64.  答案:D

  参考译文:由于世界经济的衰退,股票市场上的股票有所波动。

  【试题分析】 本题为动词词义辨析题。

  【详细解答】 turned意为"旋转,转动";changed意为"改变";floated意为"漂浮";fluctuated意为"波动,起伏"。根据句意,选项D为正确答案。

  65.  答案:A

  参考译文:我认为你可以选一个中级语言课程提高你的英语。

  【试题分析】 本题为形容词词义辨析题。

  【详细解答】 intermediate意为"中级的",如 "intermediate course /level /book 中级课程/水平/书本"; middle意为"中间的",表示方位; medium意为"传播媒介";mid意为"在……中间",诗歌中用的较多。故选项A为正确答案。

  

  PART Ⅵ READING COMPREHENSION

  SECTION A READING COMPREHENSION

  TEXT A

  短文大意 :这篇文章讲的是人们用社会空间反映人与人之间的社会关系。回顾了早期到美洲的欧洲移民生活方式的改变。比较了墨西哥人和美国人家庭空间感的不同。

  66. 答案:B

  参考译文:什么原因造成了早期移民生活方式的改变?

  【试题分析】 本题为细节题。

  【详细解答】 本题可从文中第五句找到答案"With the development of ideas about individualism, people soon began to shift to the use of individual cups and plates;"即随着个人主义思想的发展,人们开始转向使用个人的茶杯和碟子。故选项B为正确答案。

  67. 答案:A

  参考译文:下列哪一项文中没有提到?

  【试题分析】 本题为细节题。

  【详细解答】 短文第三句话说"……in which one room was used for eating, entertaining guests, and sleeping."即一个房间作为吃饭、款待客人和睡觉的房间。此句说明房间的功能;第一段最后一句话说 "They began to build their houses with separate rooms to entertain guests…… kitchen, laundry room, and separate bathrooms."即他们开始用单独的房间款待客人……有厨房,洗衣房和洗澡间。此句说明房间的布局。选项B"他们之间的社会关系"是文章的主旨。由此可知,选项B,C,D文中已提。而选项A即家庭空间的概念,文中没有提到,故为正确答案。此题采取排除法。

  TEXT B

  短文大意 :这篇文章首先介绍了人们对数字的迷信,以及对人们生活的影响。

  68.  答案:C

  参考译文:根据短文,下面哪一组数字给人们带来好运?

  【试题分析】 本题为细节题。

  【详细解答】 文中在提到幸运数字时讲到"Seven was another significant number, usually regarded as a bringer of good luck." "Nine is usually thought of as lucky number……"即七是另外一个有特殊意义的数字,能带来好运。九也通常被认为是一个幸运数字。故选项C为正确答案。

  69.  答案:B

  参考译文:认为数字十三带来坏运起源于什么?

  【试题分析】 本题为细节题。

  【详细解答】 文中在提到数字十三时说"The common belief is that this derives from the fact that there were 13 people at Christ's Last Supper."即普遍认为是起源于耶稣的最后的晚餐上十三个人这个事实。故选项B为正确答案。

  70.  答案:A

  参考译文:对于人们对数字的迷信,作者持什么观点?

  【试题分析】 本题为态度题。

  【详细解答】 文中最后一段提到"Yet oddly enough, to be born on the 13th of the month is not regarded with any fear at all, which just shows how irrational we are in our superstitious beliefs."然而非常奇怪的是,在十三日出生一点也不令人可怕,这恰恰显示出我们的迷信多么没有理性。故选项A即温和地讽刺,为正确答案。

  TEXT C

  短文大意 :这篇有关大脑结构的文章,介绍男性、女性大脑结构及工作方式的不同,以及科学家的研究成果。

  71.  答案:C

  参考译文:下列哪一项是正确的?

  【试题分析】 本题为细节题。

  【详细解答】 选项A即心理学家已经放弃,而生物学家正在展开研究工作。而文中第三句话说 "Psychologists view the subject either as a matter of frustration or a joke."即心理学家认为这个研究不是一件沮丧的事,就是一个笑话。此句和选项A含义不一样;选项B即大脑区别显示出一个性别优于另外一个性别。而文中第一段最后一句话说 "But being different, they point out hurriedly, is not the same as being better or worse." 即他们指出尽管有区别,但并不能说明谁优谁劣,此句和选项B含义不一样;选项D两性大脑结构的区别早已为人所知。而文中第三段第三句话说 "This is the first time that a structural difference has been found between the brains of women and men……" 即这是第一次人们发现了两性大脑结构的不同。此句和选项D含义不一样。由此可知,选项C为正确答案。此题采取排除法。

  72.  答案:D

  参考译文:根据这篇文章,普遍认为大脑的区别由什么因素引起的?

  【试题分析】 本题为细节题。

  【详细解答】 短文第三段倒数第二句话说 "We tend to think that is the influence of society that produces these differences."即我们认为社会的影响造成了这些区别。故选项D为正确答案。

  73.  答案:A

  参考译文:第五段"这些区别"指的是什么?

  【试题分析】 本题为细节题。

  【详细解答】 短文第五段主题句说"But it isn't all that easy to explain the actual differences between skills of men and women on this basis."即在此基础上解释两性技能的区别并不是很容易的事。由此可见,"这些区别"指的是两性技能。

  74.  答案:D

  参考译文:文章的结尾,作者提议在哪些方面进行更多的研究工作?

  【试题分析】 本题为细节题。

  【详细解答】 文章的结尾,作者说我们不知道学校学生的能力和大脑两半球功能之间的确切关系。我们不懂大脑两半球是如何通过corpus callosum相互作用的。间接地提出对于corpus callosum的影响,多作研究工作

  75.  答案:A

  参考译文:文章的主要目的是什么?

  【试题分析】 本题为主旨题。

  【详细解答】 短文介绍科学家在有关大脑结构方面的研究成果。故选项A为正确答案。

  TEXT D

  短文大意 :这篇文章预测信息技术对我们生活方式及职业的影响。

  76. 答案:D

  参考译文:信息技术对以下哪一项没有影响?

  【试题分析】 本题为细节题。

  【详细解答】 短文第二段说"By 2005, 83%of American management personnel will be knowledge workers. Europe and Japan are not far behind." "In the United States, 5 of the 10 faster-growing careers between now and 2005 will be computer related." 由此可见信息技术对美国及欧洲人事管理的影响,及对美国人的职业取向的影响。故选项D为正确答案。此题采取排除法。

  77.  答案:A

  参考译文:第四段"数字分化"指的是什么?

  【试题分析】 本题为细节题。

  【详细解答】 运用第四段的具体数据进行分析。文中说2000年早期,一项调查发现,50%的白人家庭拥有计算机,43%的美国黑人家庭有计算机,他们的数字迅速扩大。由此可见,"数字分化"指的是拥有计算机的人的数字差距。故选项A为正确答案。

  78.  答案:A

  参考译文:根据这篇文章,下列哪一选项是错误的?

  【试题分析】 本题为细节题。

  【详细解答】 短文中说"By 2005, nearly all college texts and many high school and junior high books will be tied to Internet sites that provide source material, study exercises, and relevant news articles to aid in learning."这里提到了"几乎所有的大学课本,许多高中、初中课本将和因特网址连接,在学习中提供资源、学习材料及相关的文章。"由此可见,电子资料是一种辅助教学。而选项A说所有的大学和学校学习资料将转成电子教学是错误的。

  79.  答案:B

  参考译文:文中哪一点没有谈到?

  【试题分析】 本题为细节题。

  【详细解答】 文中谈及未来职业的取向、种族差异、学校及图书馆。此题采取排除法。

  80.  答案:C

  参考译文:文章的结尾,作者强调在高科技时代,我们注意什么?

  【试题分析】 本题为细节题。

  【详细解答】 短文最后一段最后一句有"……the opportunity for training is becoming one of the most desirable benefits any job can offer."据此知人们对于教育的需求。故选项C为正确答案。

  SECTION B SKIMMING AND SCANNING

  TEXT E

  短文大意 :这篇短文向我们介绍的是阿思匹林的药效。

  81. 答案:B

  参考译文:该短文主要讨论什么的效果?

  【试题分析】 本题为主旨题。

  【详细解答】 从文章第一段和第二段的主题句 "Aspirin may be the most familiar drug in the world - but its power to heal goes far beyond the usual aches and pain." "Some of the major illnesses and conditions that aspirin or aspirin-like drugs might help prevent are:"可见这篇短文的中心论题就是阿思匹林的药效,故选项B为正确答案。

  TEXT F

  短文大意 :这篇短文介绍的是保护环境的几种方式。

  82.  答案:D

  参考译文:文中提出了几个方案?

  【试题分析】 本题为细节题。

  【详细解答】 短文谈到教育、媒体、法规和未来的计划四个保护环境的方案。

  TEXT G

  短文大意 :这篇短文预测了世界七大工业国家的人均寿命。

  83.答案:A

  【参考译文】 这篇短文的主旨是关于七大工业国的什么问题?

  【试题分析】 本题为主旨题。

  【详细解答】 短文中说"…… people in the G-7 nations may be living anywhere from 1.3 years to 8 years longer than official estimates now predict."即在七大发达工业国家,人们的寿命比现在官方预测的长1.3-8年。由此可知,正确答案为选项A。

  TEXT H

  短文大意 :这篇短文给计算机使用者提了几条建议。

  84. 答案:C

  参考译文:这篇文章的最佳题目是什么?

  【试题分析】 本题为主旨题。

  【详细解答】 短文谈及为了避免用眼过度和头痛,给计算机使用者提了几条建议。由此可知,正确答案为选项C。

  TEXT I

  短文大意 :这是一个中国主要城市和国外城市天气预报的图表。

  85.  答案:C

  参考译文:中国哪几个城市的天气预报是晴天呢?

  【试题分析】 本题为细节题。

  【详细解答】 抓住关键词 "clear",中国主要城市的天气预报图表告诉我们"Beijing、Changchun、Guangzhou、Wuhan" 是晴天。由此可知,正确答案为选项C。

  86.  答案:B

  参考译文:哪一个国外城市的最高气温是最高呢?

  【试题分析】 本题为细节题。

  【详细解答】 从国外城市天气预报的图表中得知,Bangkok气温为33,最高。做对此题,学生必须知道Max 表示最高的,而Min表示最低的

  TEXT J

  短文大意 :这是一则信息。有关旅游、住宿、警察局、医院、图书馆、邮局的地址及电话号码。

  87.  答案:B

  参考译文:如果你需要旅游信息,应在何处寻找?

  【试题分析】 本题为细节题。

  【详细解答】 抓住关键词 "travel shop information", 可在 "Tourist Information Center"处查询。正确答案为选项B。

  88.  答案:D

  参考译文:在哪儿能找到警察局?

  【试题分析】 本题为细节题。

  【详细解答】 抓住关键词 "Police Station", 查询地址。

  TEXT K

  短文大意 :这是一则学校课程简介。

  89.  答案:C

  参考译文:商业英语课程时间有多长?

  【试题分析】 本题为细节题。

  【详细解答】 在"English Language Programs"下找到 "4-week Business English",正确答案为选项C。

  90.  答案:A

  参考译文:哪一个课程没有提及?

  【试题分析】本题为细节题。

  【详细解答】在"Post-graduate certificates in 12 months"下发现"Global Operations management; Communications/Networking Engineering; Advanced Software Technologies...",所以只有选项A没有提及。故选项A为正确答案。

  本套真题测试的语言重点

  重点单词

  intermediate  中级的;

  ensure  确保、保证;

  proposal建议;

  favourable  令人满意的;

  distinctive  有特色的;

  vacant  空的

  重点词组

  show off  炫耀;

  answer for  对负责;

  allow for顾及;

  stem from    源于;

  tend to      倾向于;

  shift to    转向

2004年英语专业八级试题及答案

Part Ⅰ Listening Comprehension (40 min)

  In Sections A, B and C you will hear everything ONCE ONLY. Listen carefully and then answer the questions that follow. Mark the correct answer to each question on your coloured answer sheet.

  SECTION A TALK

  Questions 1 to 5 refer to the talk in this section. At the end of the talk you will be given 75 seconds to answer the questions.

  Now listen to the talk.

  1.

  A) the coordination based on individual actions      B) the number of individual participants      C) the necessity of individual actions      D) the requirements for participants

  2.

  A) individual      B) combined      C) distinct      D) social

  3.

  A) the manner of language use

  B) the topic and content of speech

  C) the interactions between speaker and audience

  D) the relationship between speaker and audience

  4.

  A) hide their real intentions

  B) voice others' intentions

  C) play double roles on and off stage

  D) only imitate other people in life

  5.

  A) the absence of spontaneity

  B) the presence of individual actions

  C) the lack of real intentions

  D) the absence of audience

  SECTION B INTERVIEW

  Questions 6 to 10 are based on an interview. At the end of the interview you will be given 75 seconds to answer the questions.

  Now listen to the interview.

  6.

  A) Students worked very hard.

  B) Students felt they needed a second degree.

  C) Education was not career oriented.

  D) There were many specialized subjects.

  7.

  A) To turn out an adequate number of elite for the society.

  B) To prepare students for their future career.

  C) To offer practical and utilitarian courses in each programme.

  D) To set up as many technical institutions as possible.

  8.

  A) require good education

  B) are secondary to education

  C) don't call for good education

  D) don't conflict with education

  9.

  A) Shifting from one programme to another.

  B) Working out ways to reduce student number.

  C) Emphasizing better quality of education.

  D) Setting up stricter examination standards.

  10.

  A) those who can adapt to different professions

  B) those who have a high flexibility of mind

  C) those who are thinkers, historians and philosophers

  D) those who possess only highly specialized skills

  SECTION C NEWS BROADCAST

  Questions 11 to 13 are based on the following news. At the end of the news item, you will be given 45 seconds to answer the questions.

  Now listen to the news.

  11. Which of the following regions in the world will witness the sharpest

  drop in life expectancy?

  A) Latin America.

  B) Sub Saharan Africa.

  C) Asia.

  D) The Caribbean.

  12. According to the news, which country will experience small life expectancy drop?

  A) Burma.

  B) Botswana.

  C) Cambodia.

  D) Thailand.

  13. The countries that are predicted to experience negative population growth are mainly in ____

  A) Asia.

  B) Africa.

  C) Latin America.

  D) The Caribbean.

  14. The trade dispute between the European Union and the US was caused by ____.

  A) US refusal to accept arbitration by WTO

  B) US imposing tariffs on European steel

  C) US refusal to pay compensation to EU

  D) US refusal to lower import duties on EU products

  15. Who will be consulted first before the EU list is submitted to WTO?

  A) EU member states.

  B) The United States.

  C) WTO.

  D) The steel corporations.

  SECTION D NOTE-TAKING AND GAP-FILLING

  In this section you will hear a mini lecture. You will hear the lecture ONCE ONLY. While listening to the lecture, take notes on the important points. Your notes will not be marked, but you will need them to complete a 15 minute gap filling task on ANSWER SHEET ONE after the mini lecture. Use the blank sheet for note taking.

  Part Ⅱ Proofreading and Error Correction (15 min)

  The passage contains TEN errors. Each indicated line contains a maximum of ONE error. In each case, only ONE word is involved. You should proofread the passage and correct it in the following way:

  For a wrong word, underline the wrong word and write the correct one in the blank provided at the end of the line.

  For a missing word, mark the position of the missing word with a "∧" sign and write the word you believe to be missing in the blank provided at the end of the line.

  For an unnecessary word, cross the unnecessary word with a slash "/"and put the word in the blank provided at the end of the line.

  Example

  When ∧ art museum wants a new exhibit, (1) an

  it never buys things in finished form and hangs (2) never

  them on the wall. When a natural history museum

  wants an [ZZ(Z]exhibition[ZZ)], it must often build it. (3)exhibit

  Proofread the given passage on ANSWER SHEET TWO as instructed.

  One of the most important non-legislative functions of the U.S Congress

  is the power to investigate. This power is usually delegated to committees - either

  standing committees, special committees set for a specific (1)____

  purpose, or joint committees consisted of members of both houses. (2)____

  Investigations are held to gather information on the need for

  future legislation, to test the effectiveness of laws already passed, 

  to inquire into the qualifications and performance of members and

  officials of the other branches, and in rare occasions, to lay the (3)____

  groundwork for impeachment proceedings. Frequently, committees

  rely outside experts to assist in conducting investigative hearings (4)____

  and to make out detailed studies of issues. (5)____

  There are important corollaries to the investigative power. One

  is the power to publicize investigations and its results. Most (6)____

  committee hearings are open to public and are reported (7)____

  widely in the mass media. Congressional investigations

  nevertheless represent one important tool available to lawmakers (8)____

  to inform the citizenry and to arouse public interests in national issues.

  (9)____

  Congressional committees also have the power to compel

  testimony from unwilling witnesses, and to cite for contempt

  of Congress witnesses who refuse to testify and for perjury

  these who give false testimony. (10)____

  Part Ⅲ Reading Comprehension (30 min)  (开始Part Ⅲ Reading Comprehension (30 min)计时)

  In this section there are four reading passages followed by a total of fifteen multiple choice questions. Read the passages and then mark your answers on your coloured answer sheet.

  TEXT A

  Farmers in the developing world hate price fluctuations. It makes it hard to plan ahead. But most of them have little choice: they sell at the price the market sets. Farmers in Europe, the U.S. and Japan are luckier: they receive massive government subsidies in the form of guaranteed prices or direct handouts. Last month U.S. President Bush signed a new farm bill that gives American farmers $190 billion over the next 10 years, or $83 billion more than they had been scheduled to get, and pushes U.S. agricultural support close to crazy European levels. Bush said the step was necessary to "promote farmer independence and preserve the farm way of life for generations". It is also designed to help the Republican Party win control of the Senate in November's mid-term elections.

  Agricultural production in most poor countries accounts for up to 50% of GDP, compared to only 3% in rich countries. But most farmers in poor countries grow just enough for themselves and their families. Those who try exporting to the West find their goods whacked with huge tariffs or competing against cheaper subsidized goods. In 1999 the United Nations Conference on Trade and Development concluded that for each dollar developing countries receive in aid they lose up to $14 just because of trade barriers imposed on the export of their manufactured goods. It's not as if the developing world wants any favours, says Gerald Ssendwula, Uganda's Minister of Finance. "What we want is for the rich countries to let us compete."

  Agriculture is one of the few areas in which the Third World can compete. Land and labour are cheap, and as farming methods develop, new technologies should improve output. This is no-pie-in-the-sky speculation. The biggest success in Kenya's economy over the past decade has been the boom in exports of cut flowers and vegetables to Europe. But that may all change in 2008, when Kenya will be slightly too rich to qualify for the "least developed country" status that allows African producers to avoid paying stiff European import duties on selected agricultural products. With trade barriers in place, the horticulture industry in Kenya will shrivel as quickly as a discarded rose. And while agriculture exports remain the great hope for poor countries, reducing trade barriers in other sectors also works: Americas African Growth and Opportunity Act, which cuts duties on exports of everything from handicrafts to shoes, has proved a boon to Africa's manufacturers. The lesson: the Third World can prosper if the rich world gives it a fair go.

  This is what makes Bush's decision to increase farm subsidies last month all themore depressing. Poor countries have long suspected that the rich world urges rade liberalization only so it can wangle its way into new markets. Such suspicions caused the Seattle trade talks to break down three years ago. But last November members of the World Trade Organization, meeting in Doha, Qatar, finally agreed to a new round of talks designed to open up global trade in agriculture andtextiles. Rich countries assured poor countries, that their concerns were finally being addressed. Bush's handout last month makes a lie of America's commitment to those talks and his personal devotion to free trade.

  16. By comparison, farmers ____ receive more government subsidies than others.

  A) in the developing world

  B) in Japan

  C) in Europe

  D) in America

  17. In addition to the economic considerations, there is a ____ motive behind Bush's signing of the new farm bill.

  A) partisan

  B) social

  C) financial

  D) cultural

  18. The message the writer attempts to convey throughout the passage is that ____.

  A) poor countries should be given equal opportunities in trade

  B) "the least developed country" status benefits agricultural countries

  C) poor countries should remove their suspicions about trade liberalization

  D) farmers in poor countries should also receive the benefit of subsidies

  19. The writer's attitude towards new farm subsidies in the U.S. is ____.

  A) favourable

  B) ambiguous

  C) critical

  D) reserved

  TEXT B

  Oscar Wilde said that work is the refuge of people who have nothing better to do. If so, Americans are now among the world's saddest refugees. Factory workers in the United States are working longer hours than at any time in the past half century. America once led the rich world in cutting the average working week-from 70 hours in 1850 to less than 40 hours by the 1950s. It seemed natural that as people grew richer they would trade extra earnings for more leisure. Since the 1970s, however, the hours clocked up by American workers have risen, to an average of 42 this year in manufacturing.

  Several studies suggest that something similar is happening outside manufacturing: Americans are spending more time at work than they did 20 years ago. Executives and lawyers boast of 80 hour weeks. On holiday, they seek out fax machines and phones as eagerly as Germans bag the best sun loungers. Yet working time in Europe and Japan continues to fall. In Germany's engineering industry the working week is to be trimmed from 36 to 35 hours next year. Most Germans get six weeks' paid annual holiday; even the Japanese now take three weeks. Americans still make do with just two.

  Germany responds to this contrast with its usual concern about whether people's aversion to work is damaging its competitiveness. Yet German workers, like the Japanese, seem to be acting sensibly: as their incomes rise, they can achieve a better standard of living with fewer hours of work. The puzzle is why America, the world's richest country, sees things differently. It is a puzzle with sinistersocial implications. Parents spend less time with their children, who may be left alone at home for longer. Is it just a coincidence that juvenile crime is on the rise?

  Some explanations for America's time at work fail to stand up to scrutiny. One blames weak trade unions that leave workers open to exploitation. Are workers being forced by cost cutting firms to toil harder just to keep their jobs? A recent study by two American economists, Richard Freeman and Linda Bell, suggests not: when asked, Americans actually want to work longer hours. Most German workers, in contrast, would rather work less.

  Then, why do Americans want to work harder? One reason may be that the real earnings of many Americans have been stagnant or falling during the past two decades. People work longer merely to maintain their living standards. Yet many higher skilled workers, who have enjoyed big increases in their real pay, have been working harder too. Also, one reason for the slow growth of wages has been the rapid growth in employment-which is more or less where the argument began.Taxes may have something to do with it. People who work an extra hour in America are allowed to keep more of their money than those who do the same in Germany. Falls in marginal tax rates in America since the 1970s have made it all the more profitable to work longer.

  None of these answers really explains why the century long decline in working hours has gone into reverse in America but not elsewhere (though Britain shows signs of following America's lead). Perhaps cultural differences-the last refuge of the defeated economist-are at play. Economists used to believe that once workers earned enough to provide for their basic needs and allow for a few luxuries, their incentive to work would be eroded, like lions relaxing after a kill. But humans are more susceptible to advertising than lions. Perhaps clever marketing has ensured that "basic needs"-for a shower with built in TV, for a rocket propelled car-expand continuously. Shopping is already one of America's most popular pastimes. But it requires money-hence more work and less leisure.

  Or try this: the television is not very good, and baseball and hockey keep being wiped out by strikes. Perhaps Wilde was right. Maybe Americans have nothing better to do.

  20. In the United States, working longer hours is ____.

  A) confined to the manufacturing industry

  B) a traditional practice in some sectors

  C) prevalent in all sectors of society

  D) favoured by the economists

  21. According to the third paragraph, which might be one of the consequences of working longer hours?

  A) Rise in employees' working efficiency.

  B) Rise in the number of young offenders.

  C) Rise in people's living standards.

  D) Rise in competitiveness.

  22. Which of the following is the cause of working longer hours stated by

  the writer?

  A) Expansion of basic needs.

  B) Cultural differences.

  C) Increase in real earnings.

  D) Advertising.

  TEXT C

  The fox really exasperated them both. As soon as they had let the fowls out, inthe early summer mornings, they had to take their guns and keep guard; and thenagain as soon as evening began to mellow, they must go once more. And he was so sly. He slid along in the deep grass; he was difficult as a serpent to see. And he seemed to circumvent the girls deliberately. Once or twice March had caught sight of the white tip of his brush, or the ruddy shadow of him in the deep grass, and she had let fire at him. But he made no account of this.

  The trees on the wood edge were a darkish, brownish green in the full light-for it was the end of August. Beyond, the naked, copper like shafts and limbs of the pine trees shone in the air. Nearer the rough grass, with its long, brownish stalks all agleam, was full of light. The fowls were round about-the ducks were still swimming on the pond under the pine trees. March looked at it all, saw it all, and did not see it. She heard Banford speaking to the fowls in the distance-and she did not hear. What was she thinking about? Heaven knows. Her consciousness was, as it were, held back.

  She lowered her eyes, and suddenly saw the fox. He was looking up at her. His chin was pressed down, and his eyes were looking up. They met her eyes. And he knew her. She was spellbound-she knew he knew her. So he looked into her eyes, and her soul failed her. He knew her, he has not daunted.

  She struggled, confusedly she came to herself, and saw him making off, with slow leaps over some fallen boughs, slow, impudent jumps. Then he glanced over his shoulder, and ran smoothly away. She saw his brush held smooth like a feather, she saw his white buttocks twinkle. And he was gone, softly, soft as the wind.

  She put her gun to her shoulder, but even then pursed her mouth, knowing it was nonsense to pretend to fire. So she began to walk slowly after him, in the direction he had gone, slowly, pertinaciously. She expected to find him. In her heart she was determined to find him. What she would do when she saw him again she did not consider. But she was determined to find him. So she walked abstractedly about on the edge of the wood, with wide, vivid dark eyes, and a faint flush in her cheeks. She did not think. In strange mindlessness she walked hither and thither……

  As soon as supper was over, she rose again to go out, without saying why.

  She took her gun again and went to look for the fox. For he had lifted his eyesupon her, and his knowing look seemed to have entered her brain. She did not somuch think of him: she was possessed by him. She saw his dark, shrewd, unabashedeye looking into her, knowing her. She felt him invisibly master her spirit. She knew the way he lowered his chin as he looked up, she knew his muzzle, the golden brown, and the greyish white. And again she saw him glance over his shoulder at her, half inviting, half contemptuous and cunning. So she went, with her great startled eyes glowing, her gun under her arm, along the wood edge. Meanwhilethe night fell, and a great moon rose above the pine trees.

  23. At the beginning of the story, the fox seems to the all EXCEPT ____.

  A) cunning

  B) fierce

  C) defiant

  D) annoying

  24. As the story proceeds, March begins to feel under the spell of ____.

  A) the light

  B) the trees

  C) the night

  D) the fox

  25. Gradually March seems to be in a state of ____.

  A) blankness

  B) imagination

  C) sadness

  D) excitement

  26. At the end of the story, there seems to be a sense of ____ between March and the fox.

  A) detachment

  B) anger

  C) intimacy

  D) conflict

  27. The passage creates an overall impression of ____.

  A) mystery

  B) horror

  C) liveliness

  D) contempt

  TEXT D

  The banners are packed, the tickets booked. The glitter and white overalls have been bought, the gas masks just fit and the mobile phones are ready. All that remains is to get to the parties.

  This week will see a feast of pan European protests. It started on Bastille Day, last Saturday, with the French unions and immigrants on the streets and the first demonstrations in Britain and Germany about climate change. It will continue tomorrow and Thursday with environmental and peace rallies against President Bush. But the big one is in Genoa, on Friday and Saturday, where the G8 leaders will meet behind the lines of 18, 000 heavily armed police.

  Unlike Prague, Gothenburg, Cologne or Nice, Genoa is expected to be Europe's Seattle, the coming together of the disparate strands of resistance to corporate globalisation.

  Neither the protesters nor the authorities know what will happen, but some things are predictable. Yes, there will be violence and yes, the mass media will focus on it. What should seriously concern the G8 is not so much the violence, the numbers in the streets or even that they themselves look like idiots hiding behind the barricades, but that the deep roots of a genuine new version of internationalism are growing.For the first time in a generation, the international political and economic condition is in the dock. Moreover, the protesters are unlikely to go away, their confidence is growing rather than waning, their agendas are merging, the protests are spreading and drawing in all ages and concerns.

  No single analysis has drawn all the strands of the debate together. In the meantime, the global protest "movement" is developing its own language, texts, agendas, myths, heroes and villains. Just as the G8 leaders, world bodies and businesses talk increasingly from the same script, so the protesters' once disparatepolitical and social analyses are converging. The long term project of governments and world bodies to globalise capital and development is being mirrored by the globalisation of protest.

  But what happens next? Governments and world bodies are unsure which way to turn. However well they are policed, major protests reinforce the impression of indifferent elites, repression of debate, overreaction to dissent, injustice and unaccountable power.

  Their options-apart from actually embracing the broad agenda being put to them-are to retreat behind even higher barricades, repress dissent further, abandon global meetings altogether or, more likely, meet only in places able to physically resist the masses.

  Brussels is considering building a super fortress for international meetings. Genoa may be the last of the European super protests.

  28. According to the context, the word "parties" at the end of the first paragraph refers to ____.

  A) the meeting of the G8 leaders

  B) the protests on Bastille Day

  C) the coming pan European protests

  D) the big protest to be held in Genoa

  29. According to the passage, economic globalisation is paralleled by ____.

  A) the emerging differences in the global protest movement

  B) the disappearing differences in the global protest movement

  C) the growing European concern about globalisation

  D) the increase in the number of protesters

  30. According to the last paragraph, what is Brussels considering doing?

  A) Meeting in places difficult to reach.

  B) Further repressing dissent.

  C) Accepting the protesters' agenda.

  D) Abandoning global meetings.

  (结束Part Ⅲ Reading Comprehension (30 min)计时)

  SECTION B SKIMMING AND SCANNING (10 MIN)  (开始SECTION B SKIMMING AND SCANNING (10 MIN)计时)

  In this section there are seven passages with ten multiple choice questions. Skim or scan them as required and then mark your answers on your coloured answer sheet.

  TEXT E

  First read the question. 31.

  The main purpose of the passage is to ____. A.demonstrate how to prevent crime B.show the seriousness of crime

  C.look into the causes of crime D.call for more government efforts. Now go through TEXT E quickly to answer question 31.

   For three weeks, every night at 11 p.m., correspondents, officers and judges from justice courts, police departments and prisons, psychiatrists, criminologists, victims and even criminals in prisons made their appearance on TV to debate on a topic "Crime in the United States".

   Indeed, crime has been disturbing the American people and has become a serious social problem just next to the unemployment problem. Some figures are terrifying : 1 of 4 Americans has been a victim of some kind of crimes; nearly 22 million crime cases occurred last year throughout the country. A simple arithmetic calculation indicates that on average, a crime is being committed in every 2 seconds. Now the Americans are living in a horrible environment. Their safety and property are threatened by various crimes: robbery, theft, rape, kidnapping, murder, arson, vandalism and violence.

   The most worrisome problem comes from the fact that about one third of crime cases were committed by the juvenile and 53% of criminals in jails are youngsters below 25. A poll indicates that about 73% of citizens said they avoided teenagers in streets, especially at night.

   To protect themselves from crime, according to a released figure, 52% of Americans keep guns at home. But some gun owners turn out to be potential criminals. Some people demand that strict law for gun control be enforced; but others oppose the ban of gun. No decision is in sight.

   Some experts said poverty, unemployment and racial discrimination are the cause of crime. They cited figures to show that 47% of crime cases were committed by t he black, though they account for only about 12% of the population of the nation . Others argued that about 54% of convicted criminals came from families associated with these evils.

   The American state government and federal government spend billions of dollars each year in maintaining the police departments and jails. But police authorities complain that they have not sufficient well trained hands and advanced equipment to detect and stop crimes. Several cases of criminal insurgence were reported as a result of resentment at overcrowded prisons. Taxpayers complain that they pay more and more tax but receive less and less protection from crime for their lives and property.

   Though the host of the live TV programme made great efforts to search for a solution, so far no participant could put forward a measure that was approved by most of the attendants.

  31. The main purpose of the passage is to ____.

  A) demonstrate how to prevent crime

  B) show the seriousness of crime

  C) look into the causes of crime

  D) call for more government efforts

  TEXT F

  First read the question.

  32.What is the main topic of the following passage?

  A. Differences between modes of learning. B.Deficiencies of formal learning.

  C.Advantages of informal learning. D.Social context and learning systems.

  Now go through TEXT F quickly to answer question 32.

  The term "formal learning" is used in this paper to refer to all learning that takes place in the classroom, irrespective of whether such learning is informed by conservative or progressive ideologies. "Informal learning", on the other hand, is used to refer to learning which takes place outside the classroom.

  These definitions provide the essential, though by no means sole, difference between formal and informal learning. Formal learning is decontextualised from daily life and, indeed, as Scribner and Cole (1973:553) have observed, may actually "promote ways of learning and thinking which often run counter to those nurtured in practical daily life". A characteristic feature of formal learning is the centrality of activities that are not closely paralleled by activities outside the classroom. The classroom can prepare for, draw on, and imitate the challenges of adult life outside the classroom, but it cannot, by its nature, consist of these challenges.

   In doing this, language plays a critical role as the major channel for information exchange. "Success" in the classroom requires a student to master this abstract code. As Bernstein (1969:152) noted, the language of the classroom is more similar to the language used by middle class families than that used by working class families. Middle class children thus find it easier to acquire the language of the classroom than their working class peers.

   Informal learning, in contrast, occurs in the setting to which it relates, making learning immediately relevant. In this context, language does not occupy such an important role: the child's experience of learning is more holistic, involving sight, touch, taste, and smell-senses that are under utilised in the classroom.

  While formal learning is transmitted by teachers selected to perform this role, informal learning is acquired as a natural part of a child's development. Adults or older children who are proficient in the skill or activity provide - sometime s unintentionally - target models of behaviour in the course of everyday activity. Informal learning, therefore, can take place at any time and is not subject to the limitations imposed by institutional timetabling. The motivation of the learner provides another critical difference between the two modes of learning. The formal learner is generally motivated by some kind of external goal such as parental approval, social status, and potential financial reward. The informal learner, however, tends to be motivated by successful completion of the task itself and the partial acquisition of adult status.

  32. What is the main topic of the following passage?

  A) Differences between modes of learning.

  B) Deficiencies of formal learning.

  C) Advantages of informal learning.

  D) Social context and learning systems.

  TEXT G First read the question.

  33.The three approaches mentioned in the passage aim at ____.

  A. restructuring economy B.improving the tax system

  C.improving the living conditions D.reducing poverty

  Now go through TEXT G quickly to answer question 33.

  As a rule, it is essential that the poor's productive capabilities be mobilized and the conditions for developing these human resources be improved. In this connection, German development policy has developed the following three approaches: 

  - Structural reform: Structural reform is the preferred approach for reducing poverty because it eliminates the causes of poverty rather than just its symptoms. It is vital that economic, political and social conditions which can alleviate poverty be established at national and international levels. Efforts at international level focus on fair conditions for international trade and competition. At national level, the poor must be helped through structural reform such as the introduction of democratic government, options for independent private enterprise, decentralization and agricultural reform. Development policy tools for realizing such reforms include political dialogue, political advisory services, structural adjustment measures and personnel and material support for reform efforts in the government, business and administrative sectors.

  - Direct measures: Projects of this category are aimed at directly helping the poor and improving their living conditions or increasing their job options and earning potential. Of special importance are those projects which provide help for self help in reducing poverty. The material support and advisory services offered by these projects reinforce the poor's will to help themselves and help eable them to lead self sufficient lives. Typical direct aid projects include the construction of simple housing by self help groups, the creation of a savings and loan system for the poorer segments of society and support for women's self help organizations.

   -Indirect measures: A project's beneficiaries - its target group - are not only often difficult to identify clearly, they are also not necessarily all poor people. In these cases, the project in question must be integrated into one of the partner nation's overall or sector specific policies that aim at reducing poverty. A good illustration of this type of project is the use of advisory services to improve the tax system. Advising and upgrading the qualifications of personnel working in the fiscal system can lead to increased tax revenues which could be allocated for anti poverty measures. In keeping with this focus, German development assistance concentrates on the poorest nations and on projects to reduce poverty. In 1993, some 10 percent of the commitments Germany made for bilateral financial and technical assistance went to self help projects aimed at reducing poverty. Basic needs projects comprised 48 percent of all projects and almost 30 percent of the commitments made for financial and technical assistance were allocated for the world's least developed countries (LDCs).

  33. The three approaches mentioned in the passage aim at ____.

  A) restructuring economy

  B) improving the tax system

  C) improving the living conditions

  D) reducing poverty

  TEXT H First read the question.

  34.What is the following passage mainly concerned with? A.Educational facilities in Africa. B.Founding a university for women.

  C.Agricultural production in Zimbabwe. D.Women's role in agricultural production.

  Now go through TEXT H quickly to answer question 34. Access to education facilities is inadequate in sub Saharan Africa. And women and girls there face greater disadvantages. They are often denied education as customs dictate they marry early and have children.

   Two Zimbabwean academics plan to open a university to help African women whose education was interrupted by either family commitments or financial constraints. The university will initially be in Harare, but will be relocated to Marondera, 80 kilometres east. The academics, Hope Sadza, former deputy commissioner of Zimbabwe's Public Service Commission and Fay Chung, former Minister of Education, are to open the university this month. It will initially have 400 students. Students will be split into groups of 100 and placed in one of four faculties: social science, agriculture, environmental studies or science and technology. The university is for women aged 25 or older.

  The need for a university for women is more acute in Africa, where women are the poorest and most disadvantaged. When they do have access to education they often must endure sexual harassment. Most women drop out because they lack educational materials or the schools are inaccessible.

   "In Africa, women till the land and produce the bulk of the food, yet they have no understanding about marketing, " Sadza siad. "Agriculture is another area w here we can empower women."

   The university will have a 285 hectare farm and courses will include agricultural production and marketing. Women account for 80 per cent of Africa's agricultural production, but have no control over either the resources or policies.

   The university since August has raised about Z$32.5 million (US$591, 000) in donations and pledges. The university will be open to students from across Africa. It will be the second women's university - after Sudan's Ahfad University - in Africa.

  34. What is the following passage mainly concerned with?

  A) Educational facilities in Africa.

  B) Founding a university for women.

  C) Agricultural production in Zimbabwe.

  D) Women's role in agricultural production.

  TEXT I First read the questions. 35.Which president advocated the lifting of the ban on women teachers? A.Xu Yangqiu. B.Wu Yifang. C.Tao Xingzhi. D.Chen Heqin. 36.What is Guo Juefu? A.A painter. B.A poet. C.A biologist. D.A psychologist. Now go through TEXT I quickly to answer questions 35 and 36. Many presidents of the century old Nanjing Normal University (NJNU) have put forward insightful and inspiring education theories and practices, which have had a far reaching impact on China's education history. Jiang Qian and Guo Bingwen proposed a school running principle that advocated the balance between versatility and specialization, liberal arts and sciences. Tao Xingzhi, a well known educator, carried out many important reforms in the university. For the first time in China, he advocated the lifting of the ban on women teachers and opened adult training classes in summer vacations. Wu Yifang, China's first woman university president, emphasized normal education, regarding it as the parent engine and heavy industry of education. Chen Heqin established a Chinese style and scientific theory for modern educati on for children. There have also been many noted scholars and artists. Educator Xu Yangqiu was one of China's earliest scholars to study American education theory. Professor Luo Bingwen devoted himself to normal education theory and Chinese and foreign education history, advocating that teachers should be models of virtue for the students so that their behaviour guides the students. Psychologist Guo Juefu is an important figure in China's psychological history. China Psychological History〖WTBZ〗, a book he authored, has made its mark in international psychological circles.Zhang Daqian, a well known master of traditional Chinese painting, advised his students to read books systematically and selectively to rid themselves of worldliness, fickleness and pedantry. Zhang also pointed out that success comes largely from one's own endeavours, but partly from circumstance. Sun Wang, a poet versed in the poems popular in the Tang Dynasty (AD 618 907), told students to map out a long term schedule for their studies and to work to wards fulfillment of their goal phase by phase. Biologist Chen Bangjie overcame formidable difficulties to collect plant specimen and became China's father of bryology. Generations of talented educators have given Nanjing Normal University a fine reputation.

  35. Which president advocated the lifting of the ban on women teachers?

  A) Xu Yangqiu.

  B) Wu Yifang.

  C) Tao Xingzhi.

  D) Chen Heqin.

  36. What is Guo Juefu?

  A) A painter.

  B) A poet.

  C) A biologist.

  D) A psychologist.

  TEXT J First read the questions.

  37.The Chicago GSB M.B.A. Programme for Executives is scheduled to be completed within ____.

  A.22 months     B.20 months C.16 weeks     D.14 weeks

  38.If you are in Malaysia, when is your attendance date?

  A. January 17th. B.January 15th. C.January 29th. D.February 27th.

  Now go through TEXT J quickly to answer questions 37 and 38.

  CHICAGOWorldwide campuses.World renowned faculty.World class M.B.A. degree. A world of opportunity. Limitless, lifelong opportunity awaits you when you attend the University of Chicago Graduate School of Business, and now you can do so from anywhere in the world. Experience international business firsthand at the only top ranked graduate school with campuses worldwide. The Chicago GSB M.B.A. Programme for Executives spreads 16 weeks of class sessions over 20 months so you can earn this renowned degree without leaving your job or relocating. Base your studies in Singapore; then collaborate with executives at our Chicago and Barcelona campuses. Learn not just the business theories of today but the business framework of tomorrow from the most acclaimed faculty in the world. Establish a global network of accomplished peers. And benefit for the rest of your life from the leadership training, the thinking, the relationships that become yours at Chicago GSB.

  If you are a top level manager seeking an unparalleled general management education, apply to the Chicago GSB M.B.A. Programme for Executives.And be among those who shape the future. The University of Chicago Graduate School of Business Where world class leaders emerge. Chicago GSB / Asia Campus 101 Penang Road, Singapore 238466 telephone 65 238 2196〓fax 65 835 6483email singapore.inquiries@gsb.uchicago.edu www.gsb.uchicago.edu/execMBASia Please reserve your attendance by fax or email. Jakarta 15/Jan, Tuesday The Grand Hyatt Hotel 19:00-21:30 Manila 24/Jan, Tuesday Taipei The Shangri La Edsa Plaza 19:00-21:30 The Grand Formosa Regent Hotel Hotel 17/Jan, Tuesday Kuala Lumpur 19/Feb, Tuesday 19:00-21:30 The Regent Hotel 19:00-21:30 29/Jan, Tuesday Bangkok 19:00-21:30 Singapore The Grand Hyatt Erawan Hotel GSB Asia Campus 22/Jan, Tuesday Hong Kong 27/Feb, Tuesday 19:00-21:30 The Mandarin Oriental Hotel 19:00-21:30 05/Feb, Tuesday Tokyo 19:00-21:30 The Imperial Hotel

  37. The Chicago GSB M.B.A. Programme for Executives is scheduled to be completed within ____.

  A) 22 months

  B) 20 months

  C) 16 weeks

  D) 14 weeks

  38. If you are in Malaysia, when is your attendance date?

  A) January 17th.

  B) January 15th.

  C) January 29th.

  D) February 27th.

  TEXT K First read the questions.

  39.Who has written Cultural Amnesia: America's Future and the Crisis of Memory?

  A. Michael G.Zey. B.Stephen Bertman. C.Don Tapscott, et al. D.Marvin Cetron et al.

  40.Which book is a collection of papers?

  A. Digital Capital: Harnessing the Power of Business Webs.

  B. Cheating Death: The Promise and the Future Impact of Trying to Live Forever.

  C. The Future Factor: The Five Forces Transforming Our Lives and Shaping Human De stiny. D.The University in Transformation: Global Perspectives on the Future of the Uni versity.

  Now go through TEXT K quickly to answer questions 39 and 40.

  Digital Capital: Harnessing the Power of Business Webs by Don Tapscott, David Ticoll, and Alex Lowy. Harvard Business School Press. 2000. 272 pages. Electronic business webs have demolished the rules of competition. Innovative partnerships of digitally linked producers, suppliers, service providers, and customers are accelerating productivity and generating wealth in entirely new ways. This book offers a behind the scenes look at success stories such as Linux, eBay, and Cisco, and provides a step by step process for implementing an effective business web strategy. Regular Price:$27.50 The University in Transformation: Global Perspectives on the Future of the University edited by Sohail Inayatullah and Jennifer Gidley. Bergin & Garvey/Greenwood Publishing Group. 2000. 270 pages. This anthology of essays from scholars around the world describes how the forces of technology and economic globalization may alter what we think of as higher education. Topics include the virtual university, paying for college, feminist alternative universities, the role of corporations in higher education, and the rise of "multiversities". Regular Price:$65.00 The Future Factor: The Five Force Transforming Our Lives and Shaping Human Destiny by Michael G.Zey. McGraw Hill. 2000. 289 pages. This optimistic vision of the human future argues that unprecedented opportuniti es for growth are emerging from breathtaking innovations in biotechnology, comput ing, robotics, medicine, energy development, and space technology. Powerful new forces altering society and the global economy include cybergenesis, the merging of humans and smart machines, and biogenesis, the harnessing of genetic technol ogies to improve ourselves. Regular Price: $24.95 Cheating Death: The Promise and the Future Impact of Trying to Live Forever by Marvin Cetron and Owen Davies. St. Martin's Press. 1998. 224 pages. With advances in medicine and new gene research, the human life span could exte nd hundreds of years. But a future of billions of people "cheating death" could have devastating impacts on societies, the economy, the environment, and family life. Regular Price: $21.95 Cultural Amnesia: America's Future and the Crisis of Memory by Stephen Bertman. Praeger. 2000. 176 pages. American society is losing its memory: 60% of American adults cannot name the pr esident who ordered the dropping of the first atomic bomb, and 42% of college se niors cannot place the Civil War in the correct half of the nineteenth century. This loss of culture memory, as insidious as Alzheimer's disease, eats away at t he soul of the nation, says Bertman, author of Hyperculture. He argues that, t o build a culture worthy of the future, Americans need to move away from their materialistic, present oriented lives and get more in touch with other dimension s of time. Regular Price: $35.00

  39. Who has written Cultural Amnesia: America's Future and the Crisis of Memory?

  A) Michael G.Zey.

  B) Stephen Bertman.

  C) Don Tapscott, etal.

  D) Marvin Cetron etal.

  40. Which book is a collection of papers?

  A) Digital Capital: Harnessing the Power of Business Webs.

  B) Cheating Death: The Promise and the Future Impact of Trying to Live Forever.

  C) The Future Factor: The Five Forces Transforming Our Lives and Shaping Human Destiny.      D) The University in Transformation: Global Perspectives on the Future of the University.

  Part Ⅳ Translation (60 min)

  SECTION A CHINESE TO ENGLISH

  Translate the underlined part of the following text into English. Write your translation on ANSWER SHEET THREE.

  在人际关系问题上我们不要太浪漫主义。人是很有趣的,往往在接触一个人时首先看到的都是他或她的优点。这一点颇像是在餐馆里用餐的经验。开始吃头盘或冷碟的时候,印象很好。吃头两个主菜时,也是赞不绝口。愈吃愈趋于冷静,吃完了这顿宴席,缺点就都找出来了。于是转喜为怒,转赞美为责备挑剔,转首肯为摇头。这是因为,第一,开始吃的时候你正处于饥饿状态,而饿了吃糠甜如蜜,饱了吃蜜也不甜。第二,你初到一个餐馆,开始举筷时有新鲜感,新盖的茅房三天香,这也可以叫做"陌生化效应"吧。

  SECTION B ENGLISH TO CHINESE

  Translate the underlined part of the following text into Chinese. Write your translation on ANSWER SHEET THREE.

  For me the most interesting thing about a solitary life, and mine has been that for the last twenty years, is that it becomes increasingly rewarding. When I can wake up and watch the sun rise over the ocean, as I do most days, and know that I have an entire day ahead, uninterrupted, in which to write a few pages, take a walk with my dog, read and listen to music, I am flooded with happiness. I'm lonely only when I am overtired, when I have worked too long without a brea k, when fro the time being I feel empty ad need filling up. And I am lonely somet imes when I come back home after a lecture trip, when I have seen a lot of peopl e and talked a lot, and am full to the brim with experience that needs to be sor ted out. Then for a little while the house feels huge and empty, and I wonder where my se lf is hiding. It has to be recaptured slowly by watering the plants and perhaps, by looking again at each one as though it were a person. It takes a while, as I watch the surf blowing up in fountains, but the moment co mes when the worlds falls away, and the self emerges again from the deep unconsc ious, bringing back all I have recently experienced to be explored and slowly un derstood.

  Part Ⅴ Writing (60min)

  It was reported in the press some time ago that a few second-and third-year students in a provincial university decided to try their hands at business in order to get prepared for the future. They opened six small shops near their university. Their teachers and classmates had different opinions about this phenomenon. Some thought that the students' business experience would help them adapt better to society after graduation, while others held a negative view, saying that running shops might occupy too much of the students' time and energy which should otherwise be devoted to their academic study. What do you think? Write a composition of about 300 words on the following topic:

  Should University Students Go in for Business?

  In the first part of your writing you should state clearly your main argument, and in the second part you should support your argument with appropriate details. In the last part you should brig what you have written to a natural conclusion or a summary.

  Marks will be awarded for content, organization, grammar and appropriateness. Failure to follow the above instructions may result in a loss of marks.

2004年英语专业四级试题及答案

TIME LIMIT: 140 MIN.

  Part Ⅰ

  WRITING  [45 MIN.]

  SECTION A COMPOSITION[35 MIN.]

  Nowadays young people tend to phone more often than write to each other. So, some say that phones will kill letter writing. What is your opinion?

  Write on ANSWER SHEET ONE a composition of about 150 words on the following topic:

  WILL PHONES KILL LETTER WRITING?

  You are to write in three parts.

  In the first part, state specifically what your view is.

  In the second part, support your view with one or two reasons.

  In the last part, bring what you have written to a natural conclusion or a summary.

  Marks will be awarded for content, organization, grammar and appropriateness. Failure to follow the instructions may result in a loss of marks.

  SECTION B NOTE-WRITING [10 MIN.]

  Write on ANSWER SHEET ONE a note of about 50-60 words based on the following situation:

  Your friend, Jane, has failed in the final exam, and is feeling very unhappy about it. Write a note to comfort her and give her some encouragement.

  Marks will be awarded for content, organization, grammar and appropriateness.

  Part Ⅱ DICTATION [15 MIN.]

  Listen to the following passage. Altogether the passage will be read to you four times: During the first reading, which will be read at normal speed, listen and try to understand the meaning. For the second and third readings, the passage will be read sentence by sentence, or phrase by phrase, with intervals of 15 seconds. The last reading will be read at normal speed again and during this time you should check your work. You will then be given 2 minutes to check through your work once more.

  Please write the whole passage on ANSWER SHEET TWO.

  Part Ⅲ  LISTENING COMPREHENSION  [20 MIN.]

    In Sections A, B and C you will hear everything ONCE ONLY. Listen carefully and then answer the questions that follow. Mark the correct answer to each question on your answer sheet.

  SECTION A STATEMENT

    In this section you will hear seven statements. At the end of each statement you will be given

  10 seconds to answer the question.

  1.Where is Lily working now?

  A. In the police department.

  B. In a drama society.

  C. In a university.

  D. In a primary school.

  2.Passengers must check in to board Flight 5125 by ____.

  A. 11:00  B. 11:20  C. 11:30  D.11:50

  3.Which of the following statements is true?

  A. There is a strike across the country.

  B. Many trains have been cancelled.

  C. A few trains have been cancelled.

  D. There is a strike in the North Region.

  4.The death and missing numbers in the floods are respectively ____.

  A. 60/9.  B. 16/9.  C.9/60.  D. 9/16.

  5.What is John supposed to do on Sunday?

  A. Call the office.  B. Revise his paper.

  C. Solve the problem.  D. Hand in the paper.

  6.What do we know about Mary Jackson?

  A. She is the speaker's friend.  B. She likes stories.

  C. She is an author.  D. She gave a gift.

  7.What do we know about the speaker?

  A. The speaker can get good tips.

  B. The speaker pays for the meals.

  C. The speaker can get good wages.

  D. The speaker lives comfortably.

  8.What will the speaker probably do next?

  A. To buy some medicine.

  B. To buy a new cupboard.

  C. To ignore the matter.

  D. To investigate the matter.

  SECTION B CONVERSATION

    In this section, you will hear nine short conversations between two speakers. At the end of each conversation you will be given 10 seconds to answer the question.

  9.When will they discuss the agenda?

  A. Before dinner.  B. During dinner.

  C. After dinner.  D. Tomorrow.

  10.What can be inferred about the woman?

  A. She'll be travelling during the vacation.

  B. She'll be working during the vacation.

  C. She's looking forward to going home.

  D. She will offer her help to Jane.

  11.What is the cause of their complaint?

  A. The place.  B. The heat.

  C. The workload.  D. The facilities.

  12.What can be concluded about Janet?

  A. She has come to the party.  B. She is hosting the party.

  C. She hasn't turned up.  D. She is planning a party.

  13.Where does the conversation probably take place?

  A. In a hotel.  B. At a bus station.

  C. In a restaurant.  D. At an airport.

  14.What does the woman intend to do?

  A. Get a job on campus.  B. Get her resume ready.

  C. Visit the company.  D. Apply for a job with PICC.

  15.What are the man and woman doing?

  A. Listening to the radio.  B. Looking at the photos.

  C. Watching television.  D. Reading a newspaper.

  16.What does the man mean?

  A. He hopes the party will be successful.

  B. He will see the woman around five.

  C. He is eager to help the woman.

  D. He is unenthusiastic about the party.

  17.What is NOT a change to the literature class?

  A. Class location.  B. Class times.

  C. Class length.  D. Class size.

  SECTION C NEWS BROADCAST

    Questions 18 and 19 are based on the following news. At the end of the news item, you will be given 20 seconds to answer the questions.

  Now listen to the news.

  18.The journalist was brought to court because ____.

  A. he was working for a British newspaper.

  B. he published an untrue story.

  C. the story was published in Britain.

  D. he was working with other foreign journalists.

  19.How did the lawyer defend for the journalist?

  A. He was an American journalist.

  B. He worked for a British newspaper.

  C. His story was published elsewhere.

  D. Foreigners are not subject to local laws.

  Questions 20 and 21 are based on the following news. At the end of the news item, you will be given 20 seconds to answer the questions.

  Now listen to the news.

  20.Afghanistan's first match will be against ____.

  A. Mongolia.  B. South Korea.  C. Iran.  D. Qatar.

  21.Which of the following statements is NOT true?

  A. The announcement was made by AFA.

  B. Afghanistan was a founding member of AFC.

  C. Afghanistan had been in chaos  for long.

  D. The football player were under 23.

  Question 22 and 23 are based on the following news. At the end of the news item, you will be given 10 seconds to answer the questions.

  Now listen to the news.

  22.The expected life-span of Beijing residents has gone up by ____ compared with that a decade earlier.

  A. 1.5 years    B. 1.4 years  C. 1.2 years  D. 1.1 years

  23.The ____ mortality rate had gone up greatly during the past 10 years.

  A. infant  B. maternal  C. male  D. middle-aged

  Questions 24 and 25 are based on the following news. At the end of the news item, you will be given 20 seconds to answer the questions.

  Now listen to the news.

  24.According to Pakistan's President, the chances of the two countries going to war were ____.

  A. great  B. small  C. growing  D. greater than before

  25.Recent tensions between the two countries were a direct result of ____.

  A. their border conflicts  B. their military build-up

  C. killings in the two countries  D. their mutual distrust

  

  Part Ⅳ CLOZE  [15 MIN.]

  Decide which of the choices given below would best complete the passage if inserted in the corresponding blanks. Mark the best choice for each blank on your answer sheet.

  The normal human daily cycle of activity is of some 7-8 hours' sleep alternation with some 16-17 hours' wakefulness and that the sleep normally coincides(26) ____ the hours of darkness. Our present concern is with how easily and to what extent this(27)____ can be modified.

  The question is no mere academic one. The ease with which people can change from working in the day to working at night is a(28)____ of growing importance in industry where automation(29)____ round-the-clock working of machines. It normally(30)____ from five days to one week for a person to adapt to a(31)____ routine of sleep and wakefulness, sleeping during the day and working at night. (32)____, it is often the case in industry that shifts are changed every week. This means that no sooner has he got used to one routine(33)____ he has to change to another, (34)____ much of his time is spent neither working nor sleeping very(35) ____.

  One answer would seem to be(36)____ periods on each shift, a month, or even three months. (37)____, recent research has shown that people on such systems will revert to go back to their(38)____ habits of sleep and wakefulness during the week-end and that this is quite enough to destroy any(39)____ to night work built up during the week. The only real solution appears to be to hand over the night shift to those permanent night workers whose(40)____ may persist through all week-ends and holidays.

  26.A.in          B.with          C.of          D.over

  27.A.cycle      B.period        C.circle      D.round

  28.A.problem    B.difficulty    C.trouble      D.matter

  29.A.asks        B.invites        C.calls for    D.reacts to

  30.A.takes      B.spends        C.demands      D.asks

  31.A.former      B.returned      C.reversed    D.regular

  32.A.Therefore  B.UnfortunatelyC.In a word    D.In comparison

  33.A.as          B.when          C.then        D.than

  34.A.though      B.so that        C.while        D.as

  35.A.efficiently B.good          C.easily      D.happily

  36.A.shorter    B.better        C.longer      D.nicer

  37.A.So          B.In short      C.Similarly    D.However

  38.A.new        B.normal        C.temporary    D.favourite

  39.A.change      B.return        C.adaptation  D.tendency

  40.A.wakefulness B.sleep          C.preference  D.habit

  Part Ⅴ GRAMMAR AND VOCABULARY  [15 MIN.]

    There are twenty-five items in this section. Beneath each sentence there are four words or phrases marked A, B, C and D. Choose one that best completes the sentence.

  Mark your answers on your answer sheet.

  41.That trumpet player was certainly loud. But I wasn't bothered by his loudness ____ by his lack of talent.

  A. so much as      B. rather than    C. as  D. than

  42.____, I'll marry him all the same.

  A. Was he rich or poor  B. Whethere rich or poor

  C. Were he rich or poor  D. Be he rich or poor

  43.The government has promised to do ____ lies in its power to ease the hardships of the victims in the flood-stricken area.

  A. however  B. whichever  C. whatever  D. wherever

  44.____ if I had arrived yesterday without letting you know beforehand?

  A. Would you be surprised  B. Were you surprised

  C. Had you been surprised  D. Would you have been surprised

  45.If not ____ with the respect he feels due to him, Jack gets very ill-tempered and grumbles all the time.

  A. being treated  B. treated

  C. be treated      D. having been treated

  46.It is imperative that students ____ their term papers on time.

  A. hand in          B. would hand in 

  C. have to hand in  D. handed in

  47.The less the surface of the ground yields to the weight of a fully-loaded truck, ____ to the truck.

  A. the greater stress is  B. greater is the stress

  C. the stress is greater  D. the greater the stress

  

  48.The Minister of Finance is believed ____ of imposing new taxes to raise extra revenue.

  A. that he is thinking    B. to be thinking

  C. that he is to think    D. to think

  49.Issues of price, place, promotion, and product are ____ conventional concerns in planning marketing strategies.

  A. these of the most  B. most of those

  C. among the most      D. among the many of

  50.____ both sides accept the agreement ____ a lasting peace be established in this region.

  A. Only if, will  B. If only, would

  C. Should, will    D. Unless, would

  51.Mr Wells, together with all the members of his family, ____ for Europe this afternoon.

  A. are to leave  B. are leaving    C. is leaving  D. leave

  52.It was suggested that all government ministers should ____ information on their financial interests.

  A. discover  B. uncover  C. tell  D. disclose

  53.As my exams are coming next week, I'll take advantage of the weekend to ____ on some reading.

  A. catch up  B. clear up  C. make up  D. pick up

  54.I'm surprised they are no longer on speaking terms. It's not like either of them to bear a ____.

  A. disgust  B. curse  C. grudge  D. hatred

  55.Mary hopes to be ____ from hospital next week.

  A. dismissed    B. discharged  C.expelled  D. resigned

  56.Once a picture is proved to be a forgery, it becomes quite ____.

  A. invaluable  B. priceless  C. unworthy  D.worthless

  57.Jimmy earns his living by ____ works of art in the museum.

  A. recovering  B. restoring  C. renewing  D. reviving

  58.I couldn't sleep last night because the tap in the bathroom was ____.

  A. draining    B. dropping    C. spilling  D. dripping

  59.The book gives a brief ____ of the course of his research up till now.

  A. outline  B. reference  C. frame  D. outlook

  60.She was sanding outside in the snow, ____ with cold.

  A. spinning  B. shivering  C. shaking  D. staggering

  61.All the rooms on the second floor have nicely ____ carpets, which are included in the price of the house.

  A. adapted  B. equipped  C. suited  D. fitted

  62.He plays tinnis to the ____ of all other sports.

  A. eradication  B. exclusion  C. extension  D. inclusion

  63.She answered with an ____ "No" to the request that she attend the public hearing.

  A. eloquent  B. effective  C. emotional  D. emphatic

  64.Everyone who has visited the city agrees that it is ____ with life.

  A. vibrant    B. violent  C. energetic  D. full

  65.We met Mary and her husband at a party two months ago. ____ we've had no further communication.

  A. Thereof  B. Thereby  C. Thereafter  D. Thereabouts

  Part Ⅵ  READING COMPREHENSION  [30 MIN.]

  SECTION A READING COMPREHENSION  [25 MIN.]

  In this section there are four passages followed by questions or unfinished statements, each with four suggested answers marked A, B, C and D. Choose the one that you think is the best answer.

  Mark your answers on your answer sheet.

  TEXT A

  It often happens that a number of applicants with almost identical qualifications and experience all apply for the same position. In their educational background, special skills and work experience, there is little, if anything, to choose between half a dozen candidates. How then does the employer make a choice? Usually on the basis of an interview.

  There are many arguments for and against the interview as a selection procedure. The main argument against it is that it results in a wholly subjective decision. As often as not, emplyers do not choose the best candidate, they choose the candidate who makes a good first impression on them. Some employers, of course, reply to this argument by saying that they have become so experienced in interviewing staff that they are able to make a sound assessment of each candidate's likely performance. The main argument in favour of the interview - and it is, perhaps, a good argument - is that an employer is concerned not only with a candidate's ability, but with the suitability of his or her personality for the particular work situation. Many employers, for example, will overlook occasional inefficiencies from their secretary provided she has a pleasant personality.

  It is perhas true to say, therefore, that the real purpose of an interview is not to assess the assssable aspects of each candidate but to make a guess at the more intangible things, such as personality, character and social ability. Unfortunately, both for the employers and applicants for jobs, there are many people of great ability who simply do not interview well. There are also, of course, people who interview extremely well, but are later found to be very unsatisfactory employees. Candidates who interview well tend to be quietly confident, but never boastful; direct and straightforward in their questions and answers; cheerful and friendly, but never over-familiar; and sincerely enthusiastic and optimistic. Candidates who interview badly tend to be at either end of the spectrum of human behaviour. They are either very shy or over-confident. They show either a lack of enthusiasm or an excess of it. They either talk too little or never stop talking. They are either over-polite or rudely abrupt.

  66.We can infer from the passage that an employer might tolerate his secretary's occasional mistakes, if the latter is ____

  A. direct.  B. cheerful.  C. shy.  D. capable.

  67.What is the author's attitude towards the interview as a selection procedure?

  A. Unclear.    B. Negative.  C. Objective.  D. Indifferent.

  68.According to the passage, people argue over the interview as a selection procedure mainly because they have ____.

  A. different selection procedures

  B. different puposes in the interview

  C. different standards for competence

  D. different experiences in interviews

  69.The purpose of the last paragraph is to indicate ____.

  A. a link between success in interview and personality

  B. connections between work abilities and personality

  C. differences in interview experience

  D. differences in personal behaviour

  TEXT B

  Every year thousands of people are arrested and taken to court for shop-lifting. In Britain alone, about HK$3,000,000's worth of goods are stolen from shops every week. This amounts to something like HK$150 million a year, and represents about 4 per cent of the shops' total stock. As a result of this "shrinkage" as the shops call it, the honest public has to pay higher prices.

  Shop-lifters can be divided into three main categories: the professionals, the deliberate amateur, and the people who just can't help themselves. The professionals do not pose much of a problem for the store detectives, who, assisted by closed circuit television, two-way mirrors and various other technological devices, can usually cope with them. The professionals tend to go for high value goods in parts of the shops where security measures are tightest. And, in any case, they account for only a small percentage of the total losses due to shop-lifting.

  The same applies to the deliberate amateur who is, so to speak, a professional in training. Most of them get caught sooner or later, and they are dealt with severely by the courts.

  The real problem is the person who gives way to a sudden temptation and is in all other respects an honest and law-abiding citizen. Contrary to what one would expect, this kind of shop-lifter is rarely poor. He does not steal because he needs the goods and cannot afford to pay for them. He steals because he simply cannot stop himself. And there are countless others who, because of age, sickness or plain absent-mindedness, simply forget to pay for what they take from the shops. When caught, all are liable to prosecution, and the decision whether to send for the police or not is in the hands of the store manager.

  In order to prevent the quite incredible growth in ship-lifting offences, some stores, in fact, are doing their best to separate the thieves from the confused by prohibiting customers from taking bags into the store. However, what is most worrying about the whole problem is, perhaps, that it is yet another instance of the innocent majority being penalized and inconvenienced because of the actions of a small minority. It is the aircraft hijack situation in another form. Because of the possibility of one passenger in a million boarding an aircraft with a weapon, the other 999,999 passengers must subject themselves to searches and delays. Unless the situation in the shops improves, in ten years' time we may all have to subject ourselves to a body-search every time we go into a store to buy a tin of beans!

  70.Why does the honest public have to pay higher prices when they go to the shops?

  A. There is a "shrinkage" in market values.

  B. Many goods are not available.

  C. Goods in many shops lack variety.

  D. There are many cases of shop-lifting.

  

  71.The third group of people steal things because they ____

  A. are mentally ill.  B. are quite absent-minded.

  C.can not resist the temptation.  D. can not afford to pay for goods.

  72.According to the passage, law-abiding citizens ____.

  A. can possibly steal things because of their poverty

  B. can possibly take away goods without paying

  C. have never stolen goods from the supermarkets

  D. are difficult to be caught when they steal things

  73.Which of the following statements is NOT true about the main types of shop-lifting?

  A. A big percentage of the total losses are caused by the professionals.

  B. The deliberate amateurs will be punished severely if they get caught.

  C. People would expect that those who can't help themselves are poor.

  D. The professionals don't cause a lot of trouble to the store detectives.

  74.The aircraft hijack situation is used in order to show that ____.

  A. "the professionals do not pose much of a problem for the stores"

  B. some people "somply forget to pay for what they take from the shops"

  C. "the honest public has to pay higher prices"

  D. the third type of shop-lifters are dangerous people

  TEXT C

  My bones have been aching again, as they often do in humid weather. They ache like history: things long done with, that still remain as pain. When the ache is bad enough it keeps me from sleeping. Every night I yearn for sleep, I strive for it; yet it flutters on ahead of me like a curtain. There are sleeping pills, of course, but the doctor has warned me against them.

  Last night, after what seemed hours of damp turmoil, I got up and crept slipperless down the staris, feeling my way in the faint street light that came through the window. Once safely arrived at the bottom, I walked into the kitchen and looked around in the refrigerator. There was nothing much I wanted to eat: the remains of a bunch of celery, a blue-tinged heel of bread, a lemon going soft. I've fallen into the habits of the solitary; my meals are snatched and random. Furtive snacks, furtive treats and picnics. I made do with some peanut butter, scooped directly from the jar with a forefinger: why dirty a spoon?

  Standing there with the jar in one hand and my finger in my mouth, I had the feeling that someone was about to walk into the room - some other woman, the unseen, valid owner - and ask me what in hell I was doing in her kitchen. I've had it before, the sense that even in the course of my most legitimate and daily actions - peeling a banana, brushing my teeth - I am trespassing.

  At night the house was more than ever like a stranger's. I wandered through the front room, the dining room, the parlour, hand on the wall for balance. My various possessions were floating in their own pools of shadow, denying my ownership of them. I looked them over with a burglar's eye, deciding what might be worth the risk of stealing, what on the other hand I would leave behind. Robbers would take the obvious things - the silver teapot that was my grandmother's, perhaps the hand-painted china. The television set. Nothing I really want.

  75.The author could not fall asleep because ____.

  A. it was too damp in the bedroom

  B. she had run out of sleeping pills

  C. she was in very poor health

  D. she felt very hungry

  76.The author did not like the food in the refrigerator because it was NOT ____.

  A. fresh  B. sufficientC. nutritious  D. delicious

  77.By "At night the house was more than ever like a stranger's"(Line 1, Para. 4), the author probably means that ____.

  A. the house was too dark at night

  B. ther were unfamiliar rooms in the house

  C. she felt much more lonely at night

  D. the furniture there didn't belong to her

  TEXT D

  The chief problem in coping with foreign motorists is not so much remembering that they are different from yourself, but that they are enormously variable. Cross a frontier without adjusting and you can be in deep trouble.

  One of the greatest gulfs separating the driving nations is the Atlantic Ocean. More precisely, it is the mental distance between the European and the American motorist, particularly the South American motorist. Compare, for example, an English driver at a set of traffic lights with a Brazilian.

  Very rarely will an Englishman try to anticipate the green light by moving off prematurely. You will find the occasional sharpie who watches for the amber to come up on the adjacent set of lights. However, he will not go until he receives the lawful signal. Brazilians view the thing quite differently. If, in fact, they see traffic    lights at all, they regard them as a kind of roadside decoration.

  The natives of North America are much more disciplined. They demonstrate this in their addiction to driving in one lane and sticking to it - even if it means settling behind some great truck for many miles.

  To prevent other drivers from falling into reckless ways, American motorists try always to stay close behind the vehicle in front which can make it impossible, when all the vehicles are moving at about 55 mph, to make a real lane change. European visitors are constantly falling into this trap. They return to the Old World still flapping their arms in frustration because while driving in the State in their car they kept failing to get off the highway when they wanted to and were swept along to the next city.

  However, one nation above all others lives scrupulously by its traffic regulations - the Swiss. In Switzerland, if you were simply to anticipate a traffic light, the chances are that the motorist behind you would take your number and report you to the police. What is more, the police would visit you; and you would be convicted. The Swiss take their rules of the road so seriously that a diver can be ordered to appear in court and charged for speeding on hearsay alone, and very likely found guilty. There are slight regional variations among the French, German and Italian speaking areas, but it is generally safe to assume that any car bearing a CH sticker will be driven with a high degree of discipline.

  78.The fact that the Brazilians regard traffic lights as a kind of roadside decoration suggests that ____.

  A. traffic lights are part of street scenery

  B. they simply ignore traffic lights

  C. they want to put them at roadsides

  D. there are very few traffic lights

  

  79. The second and third paragraphs focus on the difference between ____.

  A. the Atlantic Ocean and other oceans

  B. English drivers and American drivers

  C. European drivers and American drivers

  D. European drivers and South American drivers

  80.The phrase "anticipate the green light"(Line 1, Para. 3) is closest in meaning to ____.

  A. wait for the green light to be on

  B. forbid others to move before the green light

  C. move off before the green light is on

  D. follow others when the green light is on

  SECTION B SKIMMING AND SCANNING  [5 MIN.]

  In this section there are seven passages with a total of ten multiple-choice questions. Skim or scan them as required and then mark your answers on your answer sheet.

  TEXT E

  First read the following question.

  81.This paper will mainly discuss ____

  A. China's economic policies in general.

  B. China's special economic zones.

  C. significance of investment in China.

  D. China's recent development.

  Now, go through TEXT E quickly and answer question 81.

  Over the past decade, there have been a lot of changes in China's economic policies. Like other developing countries which are attempting to become more export-orientated, China has started to set up free trade zones. These zones are called "Special Economic Zones"(SEZ's) and feature various incentives designed to encourage foreign investment. What is the significance of these zones? Have they really played an important role in the development of significance of these zones? Have they really played an important role in the development of the economy of China? In this paper I first describe the background to the establishment of these zones. Then I describe some of the aims and characteristics of the SEZ's. Lastly, I attempt to assess the significance of the SEZ's in the development of the wider Chinese economy.

  TEXT F

  First read the following question.

  82.This is a letter of ____.

  A. introduction  B. apology  C. complaint  D. recommendation

  Now, go through TEXT F quickly and answer question 82.

  June 15,200

  Dear Sir,

  Your shipment of twelve thousand "Smart" watches was received by our company this morning. However, we wish to make a number of complaints concerning the serious delay in delivery and your failure to carry out our instructions with regard to this order.

  Late delivery of the goods has caused us to disappoint several of our most valued customers.

  The second complaint concerns the mismatch in colour between the watches we ordered and those delivered.

  As a result of the above problems, therefore, we feel that the most suitable course of action is to return to you unpaid any of the goods considered unsatisfactory. We look forward to your prompt reply.

  Yours sincerely, 

  Marks Swift

  Managing Director, 

  Johnson & Sons Ltd.

  TEXT G

  First read the following question.

  83.The purpose of the pamphlet is to show ____.

  A. how much money the card holder can take at a cash machine

  B. how many more benefits the card holder can now enjoy

  C. how card holders can use cash machines of other banks

  D. how travelers canuse cash machines when abroad

  Now, go through TEXT G quickly and answer question 83.

  NEW DESIGN, MORE BENEFITS

  Here is your new Cashpoint Card. You can use it in exactly the same way as your present card, and the Plus sign means you can take money from your account at even more cash machines.

  At any of the 2,400 Lloyds Bank Cashpoint machines in the UK you can take out up to £200 a day so long as there is enough money in your account and check how much money is in your account, and order a new statement.

  You can also use the cash machines of the Bank of Scotland, Barclays Bank and the Royal Bank of Scotland.

  When you are abroad, you can take out up to £200 a day in local currency from most machines with a VISA or Plus sign-so long as there is enough money in your account.

  TEXT H

  First read the following question.

  84.From its contents' page, we know that the book mainly discusses ____.

  A. German development policy  B. German rural development

  C. German development assistance  D. German development agencies

  Now, go through TEXT H quickly and answer question 84.

  Contents

  Chapter One Basic elements: Principles and general framework of German development policy  1 

  Domestic conditions and development:

  Basic criteria for German development policy  6

  Chapter Two Priority concerns of German development policy:

  Poverty, education, environmental protection  9

  Poverty  11

  Strategies for reducing poverty  13

  Education  18

  Environmental protection and resource conservation  33

  Chapter Three  Implementation of German development policy:

  Organization, instruments and procedures  50

  Bilateral German development assistance  58

  Financial cooperation  63

  Technical cooperation  68

  Manpower cooperation    71

  Development assistance at EU level  76

  Multilateral development assistance    81

  TEXT I

  First read the following questions.

  85.Where is the museum's main entrance?

  A. On the third floor.B. On the fourth floor.

  C. On the fifth floor.D. On the sixth floor.

  86.If you want to see stuffed fish and birds, which floor should you go to?

  A. The third floor.  B. The fourth floor.

  C. The fifth floor.  D. The sixth floor.

  

  Now, go through TEXT I quickly and answer questions 85 and 86.

  The Museum of Natural History is one of the most interesting museums at the University of Kansas. The museum opened in 1903, and its first exhibit was L. L. Dyche's collection of stuffed animals. Today, the museum has over 130 exhibits on four floors.

  The first thing visitors see from the museum's main entrance on the fourth floor is a very large display called a panorama. This exhibit of North American plants and animals was L. L. Dyche's collection. Down one floor is a large collection of fossils found in the Kansas area. On the fifth floor, visitors can learn about North American Indians. Going up one more floor, visitors can see a working beehive, live snakes, stuffed fish and birds, and many other displays of Kansas plants and animals.

  TEXT J

  First read the following questions.

  87.Wher is Cambridge?

  A. In the North End in Boston.  B. In the suburbs of Boston.

  C. Near Beacon Hill in Boston.  D. Near Faneuil Hall in Boston.

  88.How do most people get around in Boston?

  A. By the subway.  B. By car.C. By bus.  D. On foot.

  Now, go through TEXT J quickly and answer questions 87 and 88.

  Boston is a beautiful big city with historical landmarks, museums and cultural sites. There are a number of fine arts venues and more than 50 colleges and universities in the area, including Harvard in Cambridge, one of the bigger Botson suburbs.

  To see 372-year-old Boston, put on your tennis shoes and tour the streets on foot. Most of the city's sights can be seen within a five-square-mile area in the North End, the historic center of the city. Most people use the city's subway to get around. From Faneuil Hall to Beacon Hill to Harvard, Paul Revere's house or the site of the Boston Massacre, visitors can find a huge chunk of the nation's heritage in one afternoon.

  TEXT K

  First read the following questions.

  89.How many exhibits does Old Shoes Museum have?

  A. About 780.  B. About 501.C. About 1000.  D. About 930.

  90.Which of the following can NOT be seen inside the aquarium?

  A. The Oriental TV Tower.  B. The underwater viewing tunnel.

  C. Large themed exhibition areas.  D. More than 10,000 precious fish.

  Now, go through TEXT K quickly and answer questions 89 and 90.

  CITY TOURS

  Old Shoes Museum

  Bai Lu Tang, the only comprehensive museum of old shoes in China, is the best place to appreciate the history of Chinese footwear and its place in national culture. Among more than its place in national culture. Among more than 1,000 pieces, the most representative are the three-inch embroidered shoes, accessories and old photos. These rare treasure are very artistic and enjoyable. Yang Shaorong, the curator, has exhibited his collection in countries like Canada and Singapore.

  Place:    Room 501, No 8, Lane 780, Hongzhou Lu

  TEL:        64460977,64450432

  Time:      9:30 am-5:30 pm

  Ocean Aquarium

  The Ocean Aquarium, located near the Oriental TV Tower, is one of the largest in Asia, and features the longest underwater viewing tunnel at 155 metres. The aquarium is divided into eight zones with 28 large themed exhibition areas, displaying more than 300 species and a total of more than 10,000 precious fish around the world.

  Place:    158 Yincheng Beilu, Pudong

  TEL:        5879988

  Time:        9:00 am-9:00 pm

2005年英语专业八级试题及答案

TEST FOR ENGLISH MAJORS (2005)

  -GRADE EIGHT-

  PART I LISTENING COMPREHENSION (30 MIN)

  SECTION A MINI-LECTURE

  In this section you will hear a mini-lecture. You will hear the lecture ONCE ONLY. While listening, take notes on the important points. Your notes will not be marked, but you will need them to complete a gap-filling task after the mini-lecture. When the lecture is over, you will be given two minutes to check your notes, and another ten minutes to complete the gap-filling task on ANSWER SHEET ONE. Use the blank sheet for note-taking.

  Writing a Research Paper

  I. Research Papers and Ordinary Essay

  A. Similarity in (1) __________:

  e.g. -choosing a topic

  -asking questions

  -identifying the audience

  B. Difference mainly in terms of (2) ___________

  1. research papers: printed sources

  2. ordinary essay: ideas in one's (3) ___________

  II. Types and Characteristics of Research Papers

  A. Number of basic types: two

  B. Characteristics:

  1. survey-type paper:

  -to gather (4) ___________

  -to quote

  -to (5) _____________

  The writer should be (6) ___________.

  2. argumentative (research) paper:

  a. The writer should do more, e.g.

  -to interpret

  -to question, etc.

  b. (7) _________varies with the topic, e.g.

  -to recommend an action, etc.

  III. How to Choose a Topic for a Research Paper

  In choosing a topic, it is important to (8) __________.

  Question No. 1: your familiarity with the topic

  Question No. 2: Availability of relevant information on the chosen topic

  Question No. 3: Narrowing the topic down to (9) _________

  Question No. 4: Asking questions about (10) ___________

  The questions help us to work out way into the topic and discover its possibilities.

  SECTION B INTERVIEW

  In this section you will hear everything ONCE ONLY. Listen carefully and then answer the questions that follow. Mark the correct answer to each question on your coloured answer sheet.

  Questions 1 to 5 are based on an interview. At the end of the interview you will be given 10 seconds to answer each of the following five questions.Now listen to the interview.

  1. What is the purpose of Professor McKay's report?

  A. To look into the mental health of old people.

  B. To explain why people have negative views on old age.

  C. To help correct some false beliefs about old age.

  D. To identify the various problems of old age

  2. Which of the following is NOT Professor McKay's view?

  A. People change in old age a lot more than at the age of 21.

  B. There are as many sick people in old age as in middle age.

  C. We should not expect more physical illness among old people.

  D. We should not expect to find old people unattractive as a group.

  3. According to Professor McKay's report,

  A. family love is gradually disappearing.

  B. it is hard to comment on family feeling.

  C. more children are indifferent to their parents.

  D. family love remains as strong as ever.

  4. Professor McKay is ________ towards the tendency of more parents living apart from their children.

  A. negative

  B. positive

  C. ambiguous

  D. neutral

  5. The only popular belief that Professor McKay is unable to provide evidence against is

  A. old-age sickness.

  B. loose family ties.

  C. poor mental abilities.

  D. difficulities in maths.

  SECTION C NEWS BROADCAST

  In this section you will hear everything ONCE ONLY. Listen carefully and then answer the questions that follow. Mark the correct answer to each question on your coloured answer sheet.

  Question 6 is based on the following news. At the end of the news item, you will be given 10 seconds to answer the question. Now listen to the news.

  6. Scientists in Brazil have used frog skin to

  A. eliminate bacteria.

  B. treat burns.

  C. Speed up recovery.

  D. reduce treatment cost.

  Question 7 is based on the following news. At the end of the news item, you will be given 10 seconds to answer the question. Now listen to the news.

  7. What is NOT a feature of the new karaoke machine?

  A. It is featured by high technology.

  B. It allows you to imitate famous singers.

  C. It can automatically alter the tempo and tone of a song.

  D. It can be placed in specially designed theme rooms.

  Question 8 is based on the following news. At the end of the news item, you will be given 10 seconds to answer the question. Now listen to the news.

  8. China's Internet users had reached _________ by the end of June.

  A. 68 million

  B. 8.9 million

  C. 10 million

  D. 1.5 million

  Question 9 and 10 are based on the following news. At the end of the news item, you will be given 20 seconds to answer the question. Now listen to the news.

  9. According to the WTO, Chinese exports rose _________ last year.

  A. 21%

  B. 10%

  C. 22%

  D. 4.73

  10. According to the news, which trading nation in the top 10 has reported a 5 per cent fall in exports?

  A. The UK.

  B. The US.

  C. Japan.

  D. Germany.

  PART II READING COMPREHENSION (30 MIN)

  TEXT A

  I remember meeting him one evening with his pushcart. I had managed to sell all my papers and was coming home in the snow. It was that strange hour in downtown New York when the workers were pouring homeward in the twilight. I marched among thousands of tired men and women whom the factory whistles had unyoked. They flowed in rivers through the clothing factory districts, then down along the avenues to the East Side.

  I met my father near Cooper Union. I recognized him, a hunched, frozen figure in an old overcoat standing by a banana cart. He looked so lonely, the tears came to my eyes. Then he saw me, and his face lit with his sad, beautiful smile -Charlie Chaplin's smile.

  "Arch, it's Mikey," he said. "So you have sold your papers! Come and eat a banana."

  He offered me one. I refused it. I felt it crucial that my father sell his bananas, not give them away. He thought I was shy, and coaxed and joked with me, and made me eat the banana. It smelled of wet straw and snow.

  "You haven't sold many bananas today, pop," I said anxiously.

  He shrugged his shoulders.

  "What can I do? No one seems to want them."

  It was true. The work crowds pushed home morosely over the pavements. The rusty sky darkened over New York building, the tall street lamps were lit, innumerable trucks, street cars and elevated trains clattered by. Nobody and nothing in the great city stopped for my father's bananas.

  "I ought to yell," said my father dolefully. "I ought to make a big noise like other peddlers, but it makes my throat sore. Anyway, I'm ashamed of yelling, it makes me feel like a fool. "

  I had eaten one of his bananas. My sick conscience told me that I ought to pay for it somehow. I must remain here and help my father.

  "I'll yell for you, pop," I volunteered.

  "Arch, no," he said, "go home; you have worked enough today. Just tell momma I'll be late."

  But I yelled and yelled. My father, standing by, spoke occasional words of praise, and said I was a wonderful yeller. Nobody else paid attention. The workers drifted past us wearily, endlessly; a defeated army wrapped in dreams of home. Elevated trains crashed; the Cooper Union clock burned above us; the sky grew black, the wind poured, the slush burned through our shoes. There were thousands of strange, silent figures pouring over the sidewalks in snow. None of them stopped to buy bananas. I yelled and yelled, nobody listened.

  My father tried to stop me at last. "Nu," he said smiling to console me, "that was wonderful yelling. Mikey. But it's plain we are unlucky today! Let's go home."

  I was frantic, and almost in tears. I insisted on keeping up my desperate yells. But at last my father persuaded me to leave with him.

  11. "unyoked" in the first paragraph is closest in meaning to

  A. sent out

  B. released

  C. dispatched

  D. removed

  12. Which of the following in the first paragraph does NOT indicated crowds of people?

  A.Thousands of

  B. Flowed

  C. Pouring

  D. Unyoked

  13. Which of the following is intended to be a pair of contrast in the passage?

  A. Huge crowds and lonely individuals.

  B. Weather conditions and street lamps.

  C. Clattering trains and peddlers' yells.

  D. Moving crowds and street traffic.

  14. Which of the following words is NOT suitable to describe the character of the son?

  A. Compassionate

  B. Responsible

  C. Shy

  D. Determined

  15. What is the theme of the story?

  A. The misery of the factory workers.

  B. How to survive in a harsh environment.

  C. Generation gap between the father and the son.

  D. Love between the father and the son.

  16. What is the author's attitude towards the father and the son?

  A. Indifferent

  B. Sympathetic

  C. Appreciative

  D. Difficult to tell

  TEXT B

  提示:原文出自美国时代杂志(TIME) 日期Jan. 29, 2001

  文章标题No Fall Insurance 作者AN K. SMITH, M.D.

  When former President Ronald Reagan fell and broke his hip two weeks ago, he joined a group of more than 350,000 elderly Americans who fracture their hips each year. At 89 and suffering from advanced Alzheimer's disease, Reagan is in one of the highest-risk groups for this type of accident. The incidence of hip fractures not only increases after age 50 but doubles every five to six years as the risk of falling increases. Slipping and tumbling are not the only causes of hip fractures; weakened bones sometimes break spontaneously. But falling is the major cause, representing 90% of all hip fractures. These…… ……

  17. The following are all specific measures to guard against injuries with the EXCEPTION of

  A. removal of throw rugs.

  B. easy access to devices

  C. installation of grab bars

  D. re-arrangement of furniture

  18. In which paragraph does the author state his purpose of writing?

  A. The third paragraph

  B. The first paragraph

  C. The last paragraph

  D. The last but one paragraph

  19. The main purpose of the passage is to

  A. offer advice on how to prevent hip fractures

  B. emphasize the importance of health precautions

  C. discuss the seriousness of hip fractures.

  D. identify the causes of hip fractures.

  TEXT C

  提示:原文同2003年专八英译汉翻译试题相同

  In his classic novel, "The Pioneers", James Fenimore Cooper has his hero, a land developer, take his cousin on a tour of the city he is building. He describes the broad streets, rows of houses, a teeming metropolis. But his cousin looks around bewildered. All she sees is a forest. "Where are the beauties and improvements which you were to show me?" she asks. He's astonished she can't see them. "Where! Everywhere," he replies. For though they are not yet built on earth, he has built them in his mind, and they as concrete to him as if they were already constructed and finished.

  Cooper was illustrating a distinctly American trait, future-mindedness: the ability to see the present from the vantage point of the future; the freedom to feel unencumbered by the past and more emotionally attached to things to come. As Albert Einstein once said, "Life for the American is always becoming, never being."…… ……

  20. The third paragraph examines America's future-mindedness from the _________ perspective.

  A. future

  B. realistic

  C. historical

  D. present

  21. According to the passage, which of the following is NOT brought about by future-mindedness?

  A. Economic stagnation

  B. Environmental destruction

  C. High divorce rates

  D. Neglect of history

  22. The word "pooh-pooh" in the sixth paragraph means

  A. appreciate

  B. praise

  C. shun

  D. ridicule

  23. According to the passage, people at present can forecast ________ of a new round of future-mindedness.

  A. the nature

  B. the location

  C. the variety

  D. the features

  24. The author predicts in the last paragraph that the study of future-mindedness will focus on

  A. how it comes into being

  B. how it functions

  C. what it brings about

  D. what it is related to.

  TEXT D

  25. The phrase "men's sureness of their sex role" in the first paragraph suggests that they

  A. are confident in their ability to charm women.

  B. take the initiative in courtship.

  C. have a clear idea of what is considered "manly".

  D. tend to be more immoral than women are.

  26. The third paragraph does NOT claim that men

  A. prevent women from taking up certain professions.

  B. secretly admire women's intellect and resolution.

  C. doubt whether women really mean to succeed in business.

  D. forbid women to join certain clubs and societies.

  27. The third paragraph

  A. generally agrees with the first paragraph

  B. has no connection with the first paragraph

  C. repeats the argument of the second paragraph

  D. contradicts the last paragraph

  28. At the end of the last paragraph the author uses humorous exaggeration in order to

  A. show that men are stronger than women

  B. carry further the ideas of the earliest paragraphs

  C. support the first sentence of the same paragraph

  D. disown the ideas he is expressing

  29. The usual idea of the cave man in the last paragraph

  A. is based on the study of archaeology

  B. illustrates how people expect men to behave

  C. is dismissed by the author as an irrelevant joke

  D. proves that the man, not woman, should be the wooer

  30. The opening quotation from Margaret Mead sums up a relationship between man and woman which the author

  A. approves of

  B. argues is natural

  C. completely rejects

  D. expects to go on changing

  PART III GENERAL KNOWLEDGE (10 MIN)

  31. ______ is the capital city of Canada.

  A. Vancouver

  B. Ottawa

  C. Montreal

  D. York

  32. U.S. presidents normally serves a (an) _________term.

  A. two-year

  B. four-year

  C. six-year

  D. eight-year

  33. Which of the following cities is NOT located in the Northeast, U.S.?

  A. Huston.

  B. Boston.

  C. Baltimore.

  D. Philadelphia.

  34. ________ is the state church in England.

  A. The Roman Catholic Church.

  B. The Baptist Church

  C. The Protestant Church

  D. The Church of England

  注:The Church of England is the officially established Christian church in England and acts as the mother and senior branch of the worldwide Anglican Communion as well as a founding member of the Porvoo Communion.

  35. The novel Emma is written by

  A. Mary Shelley.

  B. Charlotte Bront?.

  C. Elizabeth C. Gaskell.

  D. Jane Austen.

  36. Which of following is NOT a romantic poet?

  A. William Wordsworth.

  B. George Elliot.

  C. George G. Byron.

  D. Percy B. Shelley.

  37. William Sidney Porter, known as O. Henry, is most famous for

  A. his poems.

  B. his plays.

  C. his short stories.

  D. his novels

  注:O. Henry was the pen name of William Sydney Porter (September 11, 1862 - June 5, 1910), He was famous for his short stories and a master of the surprise ending, O. Henry is remembered best for such enduring favorites as "The Gift of the Magi" and "The Ransom of Red Chief." The combination of humor and sentiment found in his stories is the basis of their universal appeal.

  38. Syntax is the study of

  A. language functions.

  B. sentence structures.

  C. textual organization.

  D. word formation.

  注:Definition of Syntax:

  a. The study of the rules whereby words or other elements of sentence structure are combined to form grammatical sentences.

  b. A publication, such as a book, that presents such rules.

  c. The pattern of formation of sentences or phrases in a language.

  d. Such a pattern in a particular sentence or discourse.

  39. Which of the following is NOT a distinctive feature of human language?

  A. Arbitrariness. 任意性

  B. Productivity. 丰富性

  C. Cultural transmission. 文化传播性

  D. Finiteness. 局限性 ?

  注:design feature: features that define our human languages,such as arbitrariness,duality,creativity,displacement,cultural transmission,etc.

  相关内容请点击查看:胡壮麟《语言学教程》课后答案

  40. The speech act theory was first put forward by

  A. John Searle.

  B. John Austin. √

  C. Noam Chomsky.

  D. M.A.K. Halliday.

  注:John Langshaw Austin (March 28, 1911 - February 8, 1960) was a philosopher of language, who developed much of the current theory of speech acts. He was born in Lancaster and educated at Balliol College, Oxford. After serving in MI6 during World War II, Austin became White's Professor of Moral Philosophy at Oxford. He occupies a place in the British philosophy of language alongside Wittgenstein in staunchly advocating the examination of the way words are used in order to elucidate meaning.

  PART V TRANSLATION (60 MIN)

  提示:今年专八翻译部分的选材均出自《散文佳作108篇(汉英·英汉对照)》

  作  者: 乔萍 翟淑蓉 宋洪玮,建议大家熟读此书。点击查看该书简介及文章目录

  SECTION A CHINESE TO ENGLISH

  Translate the following text into English. Write your translation on ANSWER SHEET THREE.

  提示:本文节选自文章《生命的三分之一》作者:马南邨 该文原始出处为《燕山夜话》(北京出版社1980年版)。

  初中语文自读课本七年级上册第8课 《短文两篇》中也有此文。

  一个人的生命究竟有多大意义,这有什么标准可以衡量吗?提出一个绝对的标准当然很困难;但是,大体上看一个人对待生命的态度是否严肃认真,看他对待劳动、工作等等的态度如何,也就不难对这个人的存在意义做出适当的估计了。

  古来一切有成就的人,都很严肃地对待自己的生命,当他活着一天,总要尽量多劳动、多工作、多学习,不肯虚度年华,不让时间白白地浪费掉。我国历代的劳动人民及大政治家、大思想家等等都莫不如此。

  部分译文:Accomplished men of all ages treat their lives very seriously. As long as they are living, they always labor, work, and study as hard as possible, unwilling to spend time in vain, let alone waste even a single moment of their lives.

  SECTION B ENGLISH TO CHINESE

  Translate the following underlined part of the text into Chinese. Write your translation on ANSWER SHEET THREE.

  提示:本文原文标题About Reading Books 作者:Virginia Woolf

  2004年6月大学四级考前预测模拟试卷阅读理解中有此文

  It is simple enough to say that since books have classes fiction, biography,poetry-we should separate them and take from each what it is right that each should give us. Yet few people ask from books what books can give us. Most commonly we come to books with blurred and divided minds, asking of fiction that it shall be true, of poetry that it shall be false, of biography that it shall be flattering, of history that it shall enforce our own prejudices. If we could banish all such preconception when we read, that would be an admirable beginning. Do not dictate to your author; try to become him. Be his fellowworker and accomplice.If you hang back, and reserve and criticize at first, you are preventing yourself from getting the fullest possible value from what you read. But if you open your mind as widely as possible, then signs and hints of almost imperceptible finess, from the twist and turn of the first sentences, will bring you into the presence of a human being unlike any other. Steep yourself in this, acquaint yourself with this, and soon you will find that your author is giving you, or attempting to give you, something far more definite.

  谈 读 书

  维吉尼亚·伍夫

  既然书籍有不同的门类,如小说、传记、诗歌等,我们就应该把它们区分开来,并从每种中汲取它应当给我们提供的正确的东西;这话说起来固然容易,然而,很少有人要求从书籍中得到它们所能提供的东西,通常我们总是三心二意地带着模糊的观念去看书:要求小说情节真实,要求诗歌内容虚构,要求传记阿谀奉承,要求历史能加深我们自己的偏见。如果我们读书时能抛弃所有这些成见,那将是一个极可贵的开端。我们对作者不要指手划脚,而应努力站在作者的立场上,设想自己在与作者共同创作。假如你退缩不前,有所保留并且一开始就批评指责,你就在妨碍自己从你所读的书中得到最大的益处, 然而,如果你能尽量敞开思想,那么,书中开头几句迂回曲折的话里所包含的几乎难以觉察的细微的迹象和暗示,就会把你引到一个与众不同的人物的面前去。如果你深入下去,如果你去认识这个人物,你很快就会领悟作者正在给你或试图给你某些明确得多的东西。倘若我们首先考虑怎样读小说,那么,一部小说中的三十二章就是企图创造出象一座建筑物那样既有一定的形式而各部分又受到控制的东西,不过词句要比砖块难以捉摸,阅读的过程要比看一看更费时、更复杂。理解小说家创作工作的各项要素的捷径也许并不是阅读,而是写作,而是亲自试一试遣词造句中的艰难险阻。那么,回想一下给你留下鲜明印象的某些事——比如,你怎样在大街的拐角处从两个正在交谈着的人身边走过,树在摇曳、灯光在晃动,谈话的语气既喜又悲;这一瞬间似乎包含了一个完整的想象,一个整体的构思。

  作者简介

  维吉尼亚·伍夫(1882-1941),英国小说家,在她的小说里,缩小作者作为叙述者或评论者的作用。她同时也是一位公认的评论家。

  注释

  blur模糊;使模糊不清

  banish 流放,放逐

  preconception偏见

  try to become him:应努力站在作者的立场上。become在这里用作及物动伺、解作"配合"、"适应"。

  steep陡峭的

  acquaint yourself with……,使(你)自己认识(了解)……

  impalpable无形的

  contained从容的

  PART VI WRITING (45 MIN)

  Interview is frequently used by employers as a means to recruit prospective employees. As a result, there have been many arguments for or against the interview as a selection procedure. What is your opinion? Write an essay of about 400 words to state your view.

  In the first part of your writing you should state your main argument, and in the second part you should support your argument with appropriate details. In the last part you should bring what you have written to a natural conclusion or make a summary. You should supply an appropriate title for your essay.

  Marks will be awarded for content, organization, grammar and appropriateness. Failure to follow the above instructions may result in a loss of marks.Write your composition on ANSWER SHEET FOUR.

  参考答案征求意见稿

  提示:现在提供的参考答案仅为考场稿,尚未经准确推敲,正确率应为80%左右,

  正确答案晚些时候公布,欢迎大家把自己的答案写出来讨论,我们将在接受大家的

  反馈意见下完成最终答案。√表示答案已确定。 D 表示网友纠正

  PART I LISTENING COMPREHENSION (30 MIN)

  SECTION A MINI-LECTURE

  1. basic steps 2. raw materials 3. head 4. facts

  5. explain 6. objective 7. purpose 8. ask questions

  9. a manageable size 10. the topic itself

  SECTION B INTERVIEW

  1. C 2. A 3. D 4. B 5. D

  SECTION C NEWS BROADCAST

  6. B 7. C 8. A 9. C 10. B

  PART II READING COMPREHENSION (30 MIN)

  11. B 12. D 13. A 14. C 15. D

  16. B 17. D 18. A 19. A 20. C

  21. A 22. D 23. B 24. A 25. C

  26. B 27. A 28. C 29. B 30. A

  PART III GENERAL KNOWLEDGE (10 MIN)

  31. B 32. B 33. A 34. D 35. D

  36. B 37. C 38. B 39. D 40. B

  PART IV PROOFREADING & ERROR CORRECTION (15 MIN)

  1. 把investing改为invested

  2. 第二题irrespective后加of

  3. 把those 改为 that

  4. the fact 后面加that

  5. eliminate 改为eliminating

  6. 最后一个purely of need 改为on need

  PART V TRANSLATION (60 MIN)

  SECTION A CHINESE TO ENGLISH

  SECTION B ENGLISH TO CHINESE


2005年英语专业四级试题及答案

TEST FOR ENGLISH MAJORSTEST FOR ENGLISH MAJORS (2005)

  -GRADE FOUR-

  TIME LIMIT: 130 MIN

  PART I     DICTATION [15 MIN ]

  Listen to the following passage. Altogether the passage will be read to you four times. During the first reading, which will be read at normal speed, listen and try to understand the meaning. For the second and third readings, the passage will be read sentence by sentence, or phrase by phrase, with intervals of 15 seconds. The last reading will be read at normal speed again and during this time you should check your work. You will then be given 2 minutes to check through your work once more.

  Please write the whole passage on ANSWER SHEET ONE.

  PART II    LISTENING COMPREHENSION [15 MIN]

  In Sections A, B and C you will hear everything once only. Listen carefully and then answer the questions that follow. Mark the correct answer to each question on your answer sheet.

  SECTION A CONVERSATIONS

  In this section you will hear several conversations. Listen to the conversations carefully and then answer the questions that follow.

  Questions 1 to 3 are based on the following conversation. At the end of the conversation, you will be given 15 seconds to answer the questions. Now listen to the conversation.

  1. According to the conversation, Mr Johnson is NOT very strong in

  A. history.

  B. geography.

  C. mathematics.

  D. art.

  2. Mr Johnson thinks that _______ can help him a lot in the job.

  A. logic

  B. writing

  C. history

  D. mathematics

  3. Mr Johnson would like to work as a(n)

  A. adviser.

  B. computer programmer.

  C. product designer.

  D. school teacher.

  Questions 4 to 7 are based on the following conversation. At the end of the conversation, you will be given 20 seconds to answer the questions. Now listen to the conversation.

  4. What is the main purpose of the research?

  A. To make preparations for a new publication.

  B. To learn how couples spend their weekends.

  C. To know how housework is shared.

  D. To investigate what people do at the weekend.

  5. What does the man do on Fridays?

  A. He goes to exercise classes.

  B. He goes sailing.

  C. He goes to the cinema.

  D. He stays at home.

  6. On which day does the couple always go out?

  A. Friday.

  B. Saturday.

  C. Sunday.

  D. Any weekday.

  7. Which personal detail does the man give?

  A. Surname.

  B. First name.

  C. Address.

  D. Age.

  Questions 8 to 10 are based on the following conversation. At the end of the conversation, you will be given 15 seconds to answer the questions. Now listen to the conversation.

  8. Parcel Express needs the following details about the sender EXCEPT

  A. name.

  B. address.

  C. receipt.

  D. phone number.

  9. Parcels must be left open mainly for

  A. customs' check.

  B. security check.

  C. convenience's sake.

  D. the company's sake.

  10. The woman's last inquiry is mainly concerned with

  A. the time needed for sending the parcel.

  B. the flight time to New York.

  C. the parcel destination.

  D. parcel collection.

  SECTION B PASSAGES

  In this section, you will hear several passages. Listen to the passages carefully and then answer the questions that follow.

  Questions 11 to 13 are based on the following passage. At the end of the passage, you will be given 15 seconds to answer the questions. Now listen to the passage.

  11. Where is the train to Nanjing now standing?

  A. At Platform 7.

  B. At Platform 8.

  C. At Platform 9.

  D. At Platform 13.

  12. Which train will now leave at 11:35?

  A. The train to Jinnan.

  B. The train to Zhengzhou.

  C. The train to Tianjin.

  D. The train to Hangzhou.

  13. Which train has now been cancelled?

  A. The train to Jinnan.

  B. The train to Zhengzhou.

  C. The train to Tianjin.

  D. The train to Hangzhou.

  Questions 14 to 17 are based on the following passage. At the end of the

  passage, you will be given 20 seconds to answer the questions. Now listen to the

  passage.

  14. The museum was built in memory of those

  A. who died in wars.

  B. who worked to help victims.

  C. who lost their families in disasters.

  D. who fought in wars.

  15. Henry Durant put forward the idea because he

  A. had once fought in a war in Italy.

  B. had been wounded in a war.

  C. had assisted in treating the wounded.

  D. had seen the casualties and cruelties of war.

  16. Which of the following statements about the symbols is INCORRECT?

  A. Both are used as the organization's official symbols.

  B. Both are used regardless of religious significance.

  C. The red cross was the organization's original symbol.

  D. The red crescent was later adopted for use in certain regions.

  17. How should cheerleading be viewed according to the passage?

  A. It is just a lot of cheering.

  B. It mainly involves yelling.

  C. It mainly involves dancing.

  D. It is competitive in nature.

  Questions 18 to 20 are based on the following passage. At the end of the

  passage, you will be given 15 seconds to answer the questions. Now listen to the

  passage.

  18. How do the cheerleaders perform their jobs?

  A. They set fireworks for their team.

  B. They put on athletic shows.

  C. They run around the spectators.

  D. They yell for people to buy drinks.

  19. Why do the cheerleaders sometimes suffer physical injuries?

  A. Because they try dangerous acts to catch people's attention.

  B. Because they shout and yell so their voice becomes hoarse.

  C. Because they go to the pyramid and the hills to perform.

  D. Because they dance too much every day for practice.

  20. Which of the following statements is NOT true?

  A. The first cheerleaders was a man named John Campbell.

  B. Cheerleaders' contests are only held at the state level.

  C. Before 1930 there were no women cheerleaders.

  D. The first cheerleading occurred in 1898.

  SECTION C NEWS BROAOCAST

  Questions 21 to 22 are based on the following news. At the end of the news item,

  you will be given 10 seconds to answer the questions. Now listen to the news.

  21. How many of the emigrants died after being thrown into the sea?

  A. 15 of them.

  B. 3 of them.

  C. 100 of them.

  D. Dozens of them.

  22. The illegal emigrants came from

  A. Italy.

  B. Africa.

  C. the Mediterranean region.

  D. places unknown.

  Question 23 is based on the following news. At the end of the news item, you

  will be given 5 seconds to answer the question. Now listen to the news.

  23. What does the news item mainly report?

  A. China will send three people into space in a week.

  B. Three Chinese astronauts will spend a week in space.

  C. The Shenzhou VI will be launched next year.

  D. Shenzhou V circled the earth for two days.

  Questions 24 and 25 are based on the following news. At the end of the news item, you will be given 10 seconds to answer the questions. Now listen to the news.

  24. Which of the following had NOT been affected by the wildfires?

  A. Houses.

  B. Land.

  C. Skies.

  D. Cars.

  25. The fires were thought to have been started

  A. purposefully.

  B. accidentally.

  C. on the Mexican border.

  D. in southern California.

  Questions 26 to 28 are based on the following news. At the end of the news item,

  you will be given 15 seconds to answer the question. Now listen to the news.

  26. ________ ranks second among leading tourism nations.

  A. France

  B. The United States

  C. Spain

  D. Italy

  27. It is predicted that by 2020 China will receive _________ visitors.

  A. 77 million

  B. 130 million

  C. 36.8 million

  D. 100 million

  28. According to a Xinhua report, last year saw a _________ per cent increase in the number of Chinese traveling abroad.

  A. 16.6

  B. 30

  C. 100

  D. 37

  Question 29 and 30 are based on the following news. At the end of the news item, you will be given 10 seconds to answer the question. Now listen to the news.

  29. What would happen to the Argentine officers?

  A. They would be arrested by Spanish authorities.

  B. They would be tried in an Argentine court.

  C. They would be sent to Spain for trial.

  D. They would be tortured or murdered.

  30. What accusation would the Argentine officers face?

  A. Violation of human rights.

  B. Involvement in illegal actions.

  C. Planning anti-government activities.

  D. Being part of the military rule.

  PART III CLOZE [15 MIN]

  Decide which of the choices given below would best complete the passage if inserted in the corresponding blanks. Mark the best choice for each blank on your answer sheet.

  A person's home is as much a reflection of his personality as the clothes he wears, the food he eats and the friends with whom he spends his time. Depending on personality, most have in mind a(n) "(31) ______ home". But in general, and especially for the student or new wage earners, there are practical (32) ________ of cash and location on achieving that idea.

  Cash (33) ________, in fact, often means that the only way of (34) _________ when you leave school is to stay at home for a while until things (35) _________ financially. There are obvious (36) ________of living at home-personal laundry is usually (37) _________ done along with the family wash; meals are provided and there will be a well-established circle of friends to (38) _________. And there is (39) _________ the responsibility for paying bills, rates, etc.

  On the other hand, (40) _________ depends on how a family gets on. Do your parents like your friends? You may love your family-(41) _________do you like them? Are you prepared to be (42) __________ when your parents ask where you are going in the evening and what time you expect to be back? If you find that you cannot manage a(n) (43) _________, and that you finally have the money to leave, how do you (44) _________ finding somewhere else to live?

  If you plan to stay in your home area, the possibilities are (45) _________well-known to you already. Friends and the local paper are always (46) _________. If you are going to work in a (47) _________ area, again there are the papers-and the accommodation agencies, (48) _________ these should be approached with (49) _________. Agencies are allowed to charge a fee, usually the (50) ________ of the first week's rent, if you take accommodation they have found for you.

  31. A. ideal    B. perfect    C. imaginary   D. satisfactory

  32. A. deficiencies   B. weaknesses   C. insufficiencies   D. limitations

  33. A. cut   B. shortage   C. lack   D. drain

  34. A. getting over   B. getting in   C. getting back   D. getting along

  35. A. improve   B. enhance   C. develop   D. proceed

  36. A. concerns   b. issues   C. advantages   D. problems

  37. A. still   B. always   C. habitually   D. consequently

  38. A. call in   B. call over   C. call upon   D. call out

  39. A. always   B. rarely    C. little   D. sometimes

  40. A. little    B. enough   C. many   D. much

  41. A. and   B. but   C. still   D. or

  42. A. tolerant   B. hostile   C. indifferent   D. good-tempered

  43. A. agreement   B. consensus   C. compromise   D. deal

  44. A. go about   B. go over   C. go in for   D. go through

  45. A. seldom   B. less   C. probably   D. certainly

  46. A. dependent   B. a good source of information   C. of great value   D.

  reliable

  47. A. familiar   B. cold   C. humid   D. new

  48. A. though   B. while   C. since   D. as

  49. A. enthusiasm   B. hesitation   C. caution   D. concern

  50. A. same   B. equivalent   C. equal   D. simiarity

  PART IV GRAMMAR & VOCABULARY      [15 MIN]

  There are thirty sentences in this section. Beneath each sentence there are four words or phrases marked A, B, C and D. Choose one word or phrase that best completes the sentence.

  Mark your answers on your answer sheet.

  51. If you explained the situation to your solicitor, he ________ able to advise

  you much better than I can.

  A. would be         B. will have been      C. was         D. were

  52. _________, Mr. Wells is scarcely in sympathy with the working class.

  A. Although he is a socialist                 B. Even if he is a socialist

  C. Being a socialist                        D. Since he is a socialist

  53. His remarks were ________ annoy everybody at the meeting.

  A. so as to          B. such as to        C. such to           D. as much as to

  54. James has just arrived, but I didn't know he _________ until yesterday.

  A. will come        B. was coming       C. had been coming   D. came

  55. _________ conscious of my moral obligations as a citizen.

  A. I was and always will be                B. I have to be and always will be

  C. I had been and always will be            D. I have been and always will be

  56. Because fuel supplies are finite and many people are wasteful, we will have to install _________ solar heating device in our home.

  A. some type of                         B. some types of a

  C. some type of a                        D. some types of

  57. I went there in 1984, and that was the only occasion when I ________ the journey in exactly two days.

  A. must take                           B. must have made

  C. was able to make                     D. could make

  58. I know he failed his last test, but really he's _________ stupid.

  A. something but                        B. anything but

  C. nothing but                          D. not but

  59. Do you know Tim's brother? He is _________ than Tim.

  A. much more sportsman                  B. more of a sportsman

  C. more of sportsman                     D. more a sportsman

  60. That was not the first time he ________ us. I think it's high time we

  ________ strong actions against him.

  A. betrayed…take                        B. had betrayed…took

  C. has betrayed…took                     D. has betrayed…take

  61. What's the chance of ________ a general election this year?

  A. there being                           B. there to be

  C. there be                              D. there going to be

  62. The meeting was put off because we __________ a meeting without John.

  A. objected having                        B. were objected to having

  C. objected to have                        D. objected to having

  63. ________ you _______ further problems with your printer, contact your dealer

  for advice.

  A. If, had               B. Have, had          C. Should, have        D. In

  case, had

  64. He asked me to lend him some money, which I agreed to do, ________ that he paid me back the following week.

  A. on occasion      B. on purpose        C. on condition       D. only if

  65. Children who stay away from school do ________ for different reasons.

  A. them        B. /          C. it          D. theirs

  66. A:Why are you staring?

  B:I've never seen ______tree before.

  67. There are still many problem ahead of us, but by his time next year we can see light at the end of the _________.

  A. battle           B. day       C. road         D. tunnel

  68. We realized that he was under great _________, so we took no notice of his bad temper.

  A. excitement      B. stress      C. crisis         D. nervousness

  69. The director tried to get the actors to _________ to the next scene by hand signals.

  A. move on        B. move off    C. move out     D. move along

  70. His ideas are invariably condemned as ________ by his colleagues.

  A. imaginative      B. ingenious   C. impractical    D. theoretical

  71. Thousands of people turned out into the streets to _________ against the local authorities' decision to build a highway across the field.

  A. contradict        B. reform     C. counter       D. protest

  72. The majority of nurses are women, but in the higher ranks of the medical profession women are in a _________.

  A. minority         B. scarcity      C. rarity       D. minimum

  73. Professor Johnson's retirement ________ from next January.

  A. carries into effect                     B. takes effect

  C. has effect                            D. puts into effect

  74. The president explained that the purpose of taxation was to ________ government spending.

  A. finance         B. expand          C. enlarge         D. budget

  75. The heat in summer is no less _________ here in this mountain region.

  A. concentrated     B. extensive        C. intense         D. intensive

  76. Taking photographs is strictly ________ here, as it may damage the precious cave paintings.

  A. forbidden       B. rejected          C. excluded       D. denied

  77. Mr. Brown's condition looks very serious and it is doubtful if he will _________.

  A. pull back      B. pull up         C. pull through         D. pull out

  78. Since the early nineties, the trend in most businesses has been toward on-demand, always-available products and services that suit the customer's _________ rather than the company's.

  A. benefit        B. availability     C. suitability         D. convenience

  79. The priest made the ________ of the cross when he entered the church.

  A. mark         B. signal          C. sign           D. gesture

  80. This spacious room is ________ furnished with just a few articles in it.

  A. lightly         B. sparsely        C. hardly         D. rarely

  PART V   READING COMPREHENSION   [25 MIN ]

  In this section there are four passages followed by questions or unfinished

  statements, each with four suggested answers marked A, B, C and D. Choose the one that you think is the best answer.

  Mark your answers on your answer sheet.

  TEXT A

  81. We can learn from the beginning of the passage that

  A. the author and his brother had done poorly in school.

  B. the author had been very concerned about his school work.

  C. the author had spent much time watching TV after school.

  D. the author had realized how important schooling was.

  82. Which of the following is NOT true about the author‘s family?

  A. He came from a middle-class family.

  B. He came from a single-parent family.

  C. His mother worked as a cleaner.

  D. His mother had received little education.

  83. The mother was ________ to make her two sons switch to reading books.

  A. hesitant

  B. unprepared

  C. reluctant

  D. determined

  84. How did the two boys feel about going to the library at first?

  A. They were afraid.

  B. They were reluctant.

  C. They were indifferent.

  D. They were eager to go.

  85. The author began to love books for the following reasons EXCEPT that

  A. he began to see something in his mind.

  B. he could visualize what he read in his mind.

  C. he could go back to read the books again.

  D. he realized that books offered him new experience.

  TEXT B

  注:本文摘自 《英语学习四十年精选之异域风情 + 国外风情面面观》

  Predicting the future is always risky. But it's probably safe to say that at least a few historians will one day speak of the 20th century as America's " Disney era " . Today, it's certainly difficult to think of any other single thing that represents modern America as powerfully as the company that created Mickey Mouse. Globally, brands like Coca-Cola and McDonalds may be more widely-known, but neither encapsulates 20th-century America in quite the same way as Disney.

  The reasons for Disney's success are varied and numerous, but ultimately the credit belongs to one person - the man who created the cartoon and built the company from nothing, Walt Disney. Ironically, he could not draw particularly well. But he was a genius in plenty of other respects. In business, his greatest skills were his insight and his management ability. After setting himself up in Hollywood, he single-handedly pioneered the concepts of branding and merchandising - something his company still does brilliantly today.

  But what really distinguished Disney was his ability to identify with his audiences. Disney always made sure his films championed the " little guy " , and made him feel proud to be American. This he achieved by creating characters that reflected the hopes and fears of ordinary people. Some celebrated American achievements - Disney's very first cartoon Plane Crazy, featuring a silent Mickey Mouse, was inspired by Charles Lindbergh's flight across the Atlantic.

  Others, like the There Little Pigs and Snow White and the Seven Dwarves, showed how, through hard work and helping one's fellow man, or Americans could survive social and economic crises like the Great Depression.

  Disney's other great virtue was the fact that his company - unlike other big corporations - had a human face. His Hollywood studio - the public heard - operated just like a democracy, where everyone was on firstname terms and had a say in how things should be run. He was also regarded as a great patriot because not only did his cartoons celebrate America, but, during World War II, studios made training films for American soldiers.

  The reality, of course, was less idyllic. As the public would later learn, Disney's patriotism had an unpleasant side. After a strike by cartoonists in 1941, he became convinced that Hollywood had been infiltrated by Communists. He agreed to work for the FBI as a mole, identifying and spying on colleagues whom he suspected were subversives.

  But, apart from his affiliations with the FBI, Disney was more or less the genuine article. A new book, The Magic Kingdom; Walt Disney and the American Way of Life, by Steven Watts, confirms that he was very definitely on the side of ordinary Americans - in the 30s and 40s he voted for Franklin Roosevelt, believing he was a champion of the workers. Also, Disney was not an apologist for the FBI, as some have suggested. In fact, he was always suspicious of large, bureaucratic organizations, as is evidenced in films like That Darned Cat, in which he portrayed FBI agents as bungling incompetents.

  By the time he died in 1966, Walt Disney was an icon like Thomas Edison and the Wright Brothers. To business people and filmmakers, he was a role model; to the public at large, he was " Uncle Walt "- the man who had entertained them all their lives, the man who represented them all their lives, the man who represented all that was good about America.

  86. Walt Disney is believed to possess the following abilities EXCEPT

  A. painting.

  B. creativity

  C. management.

  D. merchandising.

  87. According to the passage, what was the pleasant side of Disney's patriotism?

  A. He sided with ordinary Americans in his films.

  B. He supported America's war efforts in his own way.

  C. He had doubts about large, bureaucratic organizations.

  D. He voted for Franklin Roosevelt in the 30s and 40s.

  88. In the sixth paragraph the sentence "Disney was more or less the genuine article" means that

  A. Disney was a creative and capable person.

  B. Disney once agreed to work for the FBI.

  C. Disney ran his company in a democratic way.

  D. Disney was sympathetic with ordinary people.

  89. The writer's attitude toward Walt Disney can best be described as

  A. sympathetic.

  B. objective.

  C. critical.

  D. skeptical.

  TEXT C

  90. The author indicates at the beginning of the passage that

  A. people listen to music for similar reasons.

  B. reasons for listening to music are varied.

  C. some people don‘t understand music at all.

  D. purposes for listening to music can be specified.

  91. We can infer from the second paragraph that the book from which this excerpt is taken is mainly meant for

  A. listeners.

  B. composers.

  C. musicians.

  D. directors.

  92. According to the passage, enjoying music is not an end in itself because people hope to ________ through listening.

  A. learn more musical devices

  B. know more about composers

  C. communicate more effectively

  D. understand music better

  93. What is the common ground for musical experience to develop?

  A. Material.

  B. Listening.

  C. Sound.

  D. Activity.

  94. The importance of movement in music is explained by comparing it to

  A. a pattern of melody.

  B. a series of drumbeats.

  C. physical movement.

  D. existence and experience.

  TEXT D

  95. What are gut feelings?

  A. They are feelings one is born with.

  B. They are feelings one may be unaware of.

  C. They are feelings of fear and anxiety.

  D. They are feelings felt by sensible people.

  96. According to the author, the importance of knowing one‘s gut feelings is that

  A. one can develop them.

  B. one can call others‘ attention to them.

  C. one may get rid of them.

  D. one may control them.

  97. The word “spice” in paragraph Six is closest in meaning to

  A. add interest to

  B. lengthen.

  C. make dull

  D. bring into existence.

  98. On mood control, the author seems to suggest that we

  A. can control the occurrence of mood.

  B. are often unaware of what mood we are in.

  C. can determine the duration of mood.

  D. lack strategies for controlling moods.

  99. The essence of “reframing” is

  A. to forget the unpleasant situation.

  B. to adopt a positive attitude.

  C. to protect oneself properly.

  d. to avoid road accidents.

  100. What is the best title for the passage?

  A. What is emotional intelligence?

  B. How to develop emotional intelligence.

  C. Strategies for geeting rid of foul moods.

  D. How to control one‘s gut feelings.

  PART VI WRITING     [45 MIN.]

  SECTION A COMPOSITION       [35 MIN]

  The students' Union of your university is planning to hold an arts festival next semester, and they are inviting students to contribute their ideas and suggestions as to how it should be organized or what should be included.

  Write on ANSWER SHEET TWO a composition of about 200 words on the following topic:

  MY IDEA OF A UNIVERSITY ARTS FESTIVAL

  You are to write in three parts.

  In the first part, state specifically what your idea is.

  In the second part, provide one or two reasons to support your idea OR describe your idea.

  In the last part, bring what you have written to a natural conclusion or a summary.

  Marks will be awarded for content, organization, grammar and appropriateness.

  Failure to follow the instructions may result in a loss of marks.

  SECTION B NOTE-WRITING      [10 MIN]

  Write on ANSWER SHEET TWO a note of about 50-60 words based on the following situation:

  You have got two tickets to a concert given by a famous pop band/orchestra.

  Write a note to your friend, Hilda/Mike, describing briefly what it is and inviting her/him to come with you.

  Marks will be awarded for content ,organization, grammar and appropriateness.

2006年英语专业八级真题及答案

PART I LISTENING COMPREHENSION(35MIN)

  SECTION A   MINI-LECTURE

  In this section you sill hear a mini-lecture. You. will hear the lecture ONCE ONLY. While listening, take notes on the important points. Your notes will not be marked, but you will need them to complete a gap-filling task after the mini-lecture. When the lecture is over, you will be given two minutes to check your notes, and another ten minutes to complete the gap-filling task on ANSWER SHEET ONE. Use the blank sheet for note-taking.

  SECTION B   INTERVIEW

  In this section you will hear everything ONCE ONLY. Listen carefully and then answer the questions that follow. Mark the correct answer to each question on your coloured answer sheet.

  Questions 1 to 5 are based on an interview. At the end of the interview you will be given 10 seconds to answer each of the following five questions.

  Now listen to the interview.

  1.        Which of the following statements is TRUE about Miss Green's university days?

  A.      She felt bored.

  B.      She felt lonely.

  C.      She cherished them.

  D.     The subject was easy.

  2.        Which of the following is NOT part of her job with the Department of Employment?

  A.      Doing surveys at workplace.

  B.      Analyzing survey results.

  C.      Designing questionnaires.

  D.     Taking a psychology course.

  3.        According to Miss Green, the main difference between the Department of Employment and the advertising agency lies in

  A.      the nature of work.

  B.      office decoration.

  C.      office location.

  D.     work procedures.

  4.        Why did Miss green want to leave the advertising agency?

  A.      She felt unhappy inside the company.

  B.      She felt work there too demanding.

  C.      She was denied promotion in the company.

  D.     She longed for new opportunities.

  5.        How did Miss Green react to a heavier workload in the new job?

  A.      She was willing and ready.

  B.      She sounded mildly eager.

  C.      She a bit surprised.

  D.     She sounded very reluctant.

  SECTION C   NEWS BROADCAST

  In this section you will hear everything ONCE ONLY. Listen carefully and then answer the questions that follow. Mark the correct answer to each question on your coloured answer sheet.

  Questions 6 and 7 based on the following news. At the end of the news item, you will be given 10 seconds to answer each of the two questions.

  Now listen to the news.

  6.        The man stole the aircraft mainly because he wanted to

  A.      destroy the European Central Bank.

  B.      have an interview with a TV station.

  C.      circle skyscrapers in downtown Frankfurt.

  D.     remember the death of a US astronaut.

  7.        Which of the following statements about the man is TRUE?

  A.      He was a 31-year-old student from Frankfurt.

  B.      He was piloting a two-seat helicopter he had stolen.

  C.      He had talked to air traffic controllers by radio.

  D.     He threatened to land on the European Central Bank.

  Question 8 is based on the following news. At the end of the news item, you will be given 10 seconds to answer the question.

  Now listen to the news.

  8.        The news is mainly about the city government's plan to

  A.      expand and improve the existing subway system.

  B.      build underground malls and parking lots.

  C.      prevent further land subsidence.

  D.     promote advanced technology.

  Questions 9 and 10 are based on the following news. At the end of the news item, you will be given 10 seconds to answer each of the two questions.

  Now listen to the news.

  9.        According to the news, what makes this credit card different from conventional ones is

  A.      that it can hear the owner's voice.

  B.      that it can remember a password.

  C.      that it can identify the owner's voice.

  D.     that it can remember the owner's PIN.

  10.    The newly developed credit card is said to said to have all the following EXCEPT

  A.      switch.

  B.      battery.

  C.      speaker.

  D.     built-in chip.

  参考答案:

  Section A  Mini-lecture

  1.the author

  2.other works

  3.literary trends

  4.grammar,diction or uses of  image

  5.cultural codes

  6.cultural

  7.the reader

  8.social

  9.reader competency

  10. social sructure,traditions of writing or political  cultural influences,etc.

  Section B Interview

  1-5  CDDDA

  Section C  News  Broadcast

  6-10  DCBCA

  PART II   READING COMPREHENSION(30MIN)

  In this section there are four reading passages followed by a total of 20 multiple-choice questions.

  Read the passages and then mark your answers on your coloured answer sheet.

  TEXT A

  The University in transformation, edited by Australian futurists Sohail Inayatullah and Jennifer Gidley, presents some 20 highly varied outlooks on tomorrow's universities by writers representing both Western and mon-Western perspectives. Their essays raise a broad range of issues, questioning nearly every key assumption we have about higher education today.

  The most widely discussed alternative to the traditional campus is the Internet University - a voluntary community to scholars/teachers physically scattered throughout a country or around the world but all linked in cyberspace. A computerized university could have many advantages, such as easy scheduling, efficient delivery of lectures to thousands or even millions of students at once, and ready access for students everywhere to the resources of all the world's great libraries.

  Yet the Internet University poses dangers, too. For example, a line of franchised courseware, produced by a few superstar teachers, marketed under the brand name of a famous institution, and heavily advertised, might eventually come to dominate the global education market, warns sociology professor Peter Manicas of the University of Hawaii at Manoa. Besides enforcing a rigidly standardized curriculum, such a "college education in a box" could undersell the offerings of many traditional brick and mortar institutions, effectively driving then out of business and throwing thousands of career academics out of work, note Australian communications professors David Rooney and Greg Hearn.

  On the other hand, while global connectivity seems highly likely to play some significant role in future higher education, that does not mean greater uniformity in course content - or other dangers - will necessarily follow. Counter-movements are also at work.

  Many in academia, including scholars contributing to this volume, are questioning the fundamental mission of university education. What if, for instance, instead of receiving primarily technical training and building their individual careers, university students and professors could focus their learning and research efforts on existing problems in their local communities and the world? Feminist scholar Ivana Milojevic dares to dream what a university might become "if we believed that child-care workers and teachers in early childhood education should be one of the highest (rather than lowest) paid professionals?"

  Co-editor Jennifer Gidley shows how tomorrow's university faculty, instead of giving lectures and conducting independent research, may take on three new roles. Some would act as brokers, assembling customized degree-credit programmes for individual students by mixing and matching the best course offerings available from institutions all around the world. A second group, mentors, would function much like today's faculty advisers, but are likely to be working with many more students outside their own academic specialty. This would require them to constantly be learning from their students as well as instructing them.

  A third new role for faculty, and in Gidley's view the most challenging and rewarding of all, would be as meaning-makers: charismatic sages and practitioners leading groups of students/colleagues in collaborative efforts to find spiritual as well as rational and technological solutions to specific real-world problems.

  Moreover, there seems little reason to suppose that any one form of university must necessarily drive out all other options. Students may be "enrolled" in courses offered at virtual campuses on the Internet, between -or even during - sessions at a real-world problem-focused institution.

  As co-editor Sohail Inayatullah points out in his introduction, no future is inevitable, and the very act of imagining and thinking through alternative possibilities can directly affect how thoughtfully, creatively and urgently even a dominant technology is adapted and applied. Even in academia, the future belongs to those who care enough to work their visions into practical, sustainable realities.

  11.    When the book reviewer discusses the Internet University,

  A.      he is in favour of it.

  B.      his view is balanced.

  C.      he is slightly critical of it.

  D.     he is strongly critical of it.

  12.    Which of the following is NOT seen as a potential danger of the Internet University?

  A.      Internet-based courses may be less costly than traditional ones.

  B.      Teachers in traditional institutions may lose their jobs.

  C.      internet-based courseware may lack variety in course content.

  D.     The Internet University may produce teachers with a lot of publicity.

  13.    According to the review, what is the fundamental mission of traditional university education?

  A.      Knowledge learning and career building.

  B.      Learning how to solve existing social problems.

  C.      Researching into solutions to current world problems.

  D.     Combining research efforts of teachers and students in learning.

  14. Judging from the Three new roles envisioned for tomorrow's university faculty, university teachers

  A,    are required to conduct more independent research.

  B.   are required to offer more course to their students…… C.   are supposed to assume more demanding duties.

  D.  are supposed to supervise more students in their specialty.

  15.  Which category of writing does the review belong to?

  A.  Narration.

  B. Description

  C. persuasion

  D. Exposition.

  TEXT B

  Every street had a story, every building a memory, Those blessed with wonderful childhoods can drive the streets of their hometowns and happily roll back the years. The rest are pulled home by duty and leave as soon as possible. After Ray Atlee had been in Clanton (his hometown) for fifteen minutes he was anxious to get out.

  The town had changed, but then it hadn't. On the highways leading in, the cheap metal buildings and mobile homes were gathering as tightly as possible next to the roads for maximum visibility. This town had no zoning whatsoever. A landowner could build anything wiih no permit no inspection, no notice to adjoining landowners. nothing. Only hog farms and nuclear reactors required approvals and paperwork. The result was a slash-and-build clutter that got uglier by the year.

  But in the older sections, nearer the square, the town had not changed at all The long shaded streets were as clean and neat as when Kay roamed them on his bike. Most of the houses were still owned by people he knew, or if those folks had passed on the new owners kept the lawns clipped and the shutters painted. Only a few were being neglected. A handful had been abandoned.

  This deep in Bible country, it was still an unwritten rule in the town that little was done on Sundays except go to church, sit on porches, visit neighbours, rest and relax the way God intended.

  It was cloudy, quite cool for May, and as he toured his old turf, killing time until the appointed hour for the family meeting, he tried to dwell on the good memories from Clanton. There was Dizzy Dean Park where he had played little League for the Pirates, and (here was the public pool he'd swum in every summer except 1969 when the city closed it rather than admit black children. There were the churches - Baptist, Methodist, and Presbyterian - facing each other at the intersection of Second and Elm like wary sentries, their steeples competing for height. They were empty now, hut in an hour or so the more faithful would gather for evening services.

  The square was as lifeless as the streets leading to it. With eight thousand people, Clanton was just large enough to have attracted the discount stores that had wiped out so many small towns. But here the people had been faithful to their downtown merchants, and there wasn't s single empty or boarded-up building around the square - no small miracle. The retail shops were mixed in with the banks and law offices and cafes, all closed for the Sabbath.

  He inched through the cemetery and surveyed the Atlee section in the old part, where the tombstones were grander. Some of his ancestors had built monuments for their dead. Ray had always assumed that the family money he'd never seen must have been buried in those graves. He parked and walked to his mother's grave, something he hadn't done in years. She was buried among the Atlees, at the far edge of the family plot because she had barely belonged.

  Soon, in less than an hour, he would be sitting in his father's study, sipping bad instant tea and receiving instructions on exactly how his father would be laid to rest. Many orders were about to be give, many decrees and directions, because his father(who used to be a judge) was a great man and cared deeply about how he was to be remembered.

  Moving again, Ray passed the water tower he'd climbed twice, the second time with the police waiting below. He grimaced at his old high school, a place he'd never visited since he'd left it. Behind it was the football field where his brother Forrest had romped over opponents and almost became famous before getting bounced off the team.

  It was twenty minutes before five, Sunday, May 7. Time for the family meeting.

  16.        From the first paragraph, we get the impression that

  A.      Ray cherished his childhood memories.

  B.      Ray had something urgent to take care of.

  C.      Ray may not have a happy childhood.

  D.     Ray cannot remember his childhood days.

  17.        Which of the following adjectives does NOT describe Ray's hometown?

  A.      Lifeless.

  B.      Religious.

  C.      Traditional.

  D.     Quiet.

  18.        Form the passage we can infer that the relationship between Ray and his parents was

  A.      close.

  B.      remote.

  C.      tense.

  D.     impossible to tell.

  19.        It can be inferred from the passage that Ray's father was all EXCEPT

  A.      considerate.

  B.      punctual.

  C.      thrifty.

  D.     dominant.

  TEXT C

  Campaigning on the Indian frontier is an experience by itself. Neither the landscape nor the people find their counterparts in any other portion of the globe. Valley walls rise steeply five or six thousand feet on every side. The columns crawl through a maze of giant corridors down which fierce snow-fed torrents foam under skies of brass. Amid these scenes of savage brilliancy there dwells a race whose qualities seem to harmonize with their environment. Except at harvest-time, when self-preservation requires a temporary truce, the Pathan tribes are always engaged in private or public war. Every man is a warrior, a politician and a theologian. Every large house is a real feudal fortress made, it is true, only of sun-baked clay, but with battlements, turrets, loopholes, drawbridges, etc. complete. Every village has its defence. Every family cultivates its vendetta; every clan, its feud. The numerous tribes and combinations of tribes all have their accounts to settle with one another. Nothing is ever forgotten, and very few debts are left unpaid. For the purposes of social life, in addition to the convention about harvest-time, a most elaborate code of honour has been established and is on the whole faithfully observed. A man who knew it and observed it faultlessly might pass unarmed from one end of the frontier to another. The slightest technical slip would, however, be fatal. The life of the Pathan is thus full of interest; and his valleys, nourished alike by endless sunshine and abundant water, are fertile enough to yield with little labour the modest material requirements of a sparse population.

  Into this happy world the nineteenth century brought two new facts: the rifle and the British Government. The first was an enormous luxury and blessing; the second, an unmitigated nuisance. The convenience of the rifle was nowhere more appreciated than in the Indian highlands. A weapon which would kill with accuracy at fifteen hundred yards opened a whole new vista of delights to every family or clan which could acquire it. One could actually remain in one's own house and fire at one's neighbour nearly a mile away. One could lie in wait on some high crag, and at hitherto unheard-of ranges hit a horseman far below. Even villages could fire at each other without the trouble of going far from home. Fabulous prices were therefore offered for these glorious products of science. Rifle-thieves scoured all India to reinforce the efforts of the honest smuggler. A steady flow of the coveted weapons spread its genial influence throughout the frontier, and the respect which the Pathan tribesmen entertained for Christian civilization was vastly enhanced.

  The action of the British Government on the other hand was entirely unsatisfactory. The great organizing, advancing, absorbing power to the southward seemed to be little better than a monstrous spoil-sport. If the Pathan made forays into the plains, not only were they driven back (which after all was no more than fair), but a whole series of subsequent interferences took place, followed at intervals by expeditions which toiled laboriously through the valleys, scolding the tribesmen and exacting fines for any damage which they had done. No one would have minded these expeditions if they had simply come, had a fight and then gone away again. In many cases this was their practice under what was called the "butcher and bolt policy" to which the Government of India long adhered. But towards the end of the nineteenth century these intruders began to make roads through many of the valleys, and in particular the great road to Chitral. They sought to ensure the safety of these roads by threats, by forts and by subsidies. There was no objection to the last method so far as it went. But the whole of this tendency to road-making was regarded by the Pathans with profound distaste. All along the road people were expected to keep quiet, not to shoot one another, and above all not to shoot at travellers along the road. It was too much to ask, and a whole series of quarrels took their origin from this source.

  20.   The word debts in "very few debts are left unpaid" in the first paragraph means

  A. loans.  B. accounts   C.killings   D.bargains.

  21.   Which of the following is NOT one of the geographical facts about the Indian frontier?

  A.    Melting snows.     B.  Large population.

  C.    Steep hillsides.     D.  Fertile valleys.

  22.   According to the passage, the Pathans welcomed

  A. the introduction of the rifle.

  B. the spread of British rule.

  C. the extension of luxuries

  D. the spread of trade.

  23. Building roads by the British

  A. put an end to a whole series of quarrels.

  B. prevented the Pathans from earning on feuds.

  C. lessened the subsidies paid to the Pathans.

  D. gave the Pathans a much quieter life.

  24.  A suitable title for the passage would be

  A. Campaigning on the Indian frontier.

  B. Why the Pathans resented the British rule.

  C. The popularity of rifles among the Pathans.

  D. The Pathans at war.

  TEXT D

  "Museum" is a slippery word. It first meant (in Greek) anything consecrated to the Muses: a hill, a shrine, a garden, a festival or even a textbook. Both Plato's Academy and Aristotle's Lyceum had a mouseion, a muses' shrine. Although the Greeks already collected detached works of art, many temples - notably that of Hera at Olympia (before which the Olympic flame is still lit) - had collections of objects, some of which were works of art by well-known masters, while paintings and sculptures in the Alexandrian Museum were incidental to its main purpose.

  The Romans also collected and exhibited art from disbanded temples, as well as mineral specimens, exotic plants, animals; and they plundered sculptures and paintings (mostly Greek) for exhibition. Meanwhile, the Greek word had slipped into Latin by transliteration (though not to signify picture galleries, which were called pinacothecae) and museum still more or less meant "Muses' shrine".

  The inspirational collections of precious and semi-precious objects were kept in larger churches and monasteries - which focused on the gold-enshrined, bejewelled relics of saints and martyrs. Princes, and later merchants, had similar collections, which became the deposits of natural curiosities: large lumps of amber or coral, irregular pearls, unicorn horns, ostrich eggs, fossil bones and so on. They also included coins and gems - often antique engraved ones - as well as, increasingly, paintings and sculptures. As they multiplied and expanded, to supplement them, the skill of the fakers grew increasingly refined.

  At the same time, visitors could admire the very grandest paintings and sculptures in the churches, palaces and castles; they were not "collected" either, but "site-specific", and were considered an integral part both of the fabric of the buildings and of the way of life which went on inside them - and most of the buildings were public ones. However, during the revival of antiquity in the fifteenth century, fragments of antique sculpture were given higher status than the work of any contemporary, so that displays of antiquities would inspire artists to imitation, or even better, to emulation; and so could be considered Muses' shrines in the former sense. The Medici garden near San Marco in Florence, the Belvedere and the Capitol in Rome were the most famous of such early "inspirational" collections. Soon they multiplied, and, gradually, exemplary "modern" works were

  In the seventeenth century, scientific and prestige collecting became so widespread that three or four collectors independently published directories to museums all over the known world. But it was the age of revolutions and industry which produced the next sharp shift in the way the institution was perceived: the fury against royal and church monuments prompted antiquarians to shelter them in asylum-galleries, of which the Musee des Monuments Francais was the most famous. Then, in the first half of the nineteenth century, museum funding took off, allied to the rise of new wealth: London acquired the National Gallery and the British Museum, the Louvre was organized, the Museum-Insel was begun in Berlin, and the Munich galleries were built. In Vienna, the huge Kunsthistorisches and Naturhistorisches Museums took over much of the imperial treasure. Meanwhile, the decline of craftsmanship (and of public taste with it) inspired the creation of "improving" collections. The Victoria and Albert Museum in London was the most famous, as well as perhaps the largest of them.

  25.The sentence "Museum is a slippery word" in the first paragraph means that

  A. the meaning of the word didn't change until after the 15th century.

  B. the meaning of the word had changed over the years.

  C. the Greeks held different concepts from the Romans.

  D. princes and merchants added paintings to their collections.

  26.The idea that museum could mean a mountain or an object originates from

  A. the Romans.     B. Florence.

  C. Olympia.        D. Greek.

  27.   "…… the skill of the fakers grew increasingly refined" in the third paragraph means that

  A. there was a great demand for fakers.   B. fakers grew rapidly in number.

  C. fakers became more skillful.          D. fakers became more polite.

  28.   Painting and sculptures on display in churches in the 15th century were

  A. collected from elsewhere.

  B. made part of the buildings.

  C. donated by people.

  D. bought by churches.

  29.  Modern museums came into existence in order to

  A. protect royal and church treasures.

  B. improve existing collections.

  C. stimulate public interest.

  D. raise more funds.

  30.  Which is the main idea of the passage?

  A. Collection and collectors.

  B. The evolution of museums.

  C. Modern museums and their functions.

  D. The birth of museums.

  11-15  BAACD  16-20 CDBAC  21-25BABAB  26-30 DCBAB

  PART III. 人文知识

  There are ten multiple-choice questions in this section.Choose the best answers to each question.

  Mark your answers on your coloured answer sheet.

  31.The Presidents during the American Civil War was

  A. Andrew Jackson

  B. Abraham Lincoln

  C. Thomas Jefferson

  D. George Washington

  32.The capital of New Zealand is

  A.Christchurch

  B.Auckland

  C.Wellington

  D.Hamilton

  33.Who were the natives of Austrilia before the arrival of the British settlers?

  A.The Aborigines

  B.The Maori

  C.The Indians

  D.The Eskimos

  34.The Prime Minister in Britain is head of

  A.the Shadow Cabinet

  B.the Parliament

  C.the Opposition

  D.the Cabinet

  35.Which of the following writers is a poet of the 20th century?

  A.T.S.Eliot

  B.D.H.Lawrence

  C.Theodore Dreiser

  D.James Joyce

  36.The novel For Whom the Bell Tolls is written by

  A.Scott Fitzgerald

  B.William Faulkner

  C.Eugene O'Neil

  D.Ernest Hemingway

  37._____ is defined as an expression of human emotion which is condensed into fourteen lines

  A.Free verse

  B.Sonnet

  C.Ode

  D.Epigram

  38.What essentially distinguishes semantics and pragmatics is the notion of

  A.reference

  B.meaning

  C.antonymy

  D.context

  39.The words"kid,child,offspring" are examples of

  A.dialectal synonyms

  B.stylistic synonyms

  C.emotive synonyms

  D.collocational synonyms

  40.The distinction between parole and langue was made by

  A.Halliay

  B.Chomsky

  C.Bloomfield

  D.Saussure

  参考答案: 31-35BCADA  36-40 DBDBD

  PART IV 改错参考答案

  1. agreeing-agreed

  2. in which  可有可无

  3. in his disposal- at his disposal

  4.enables-enable

  5.the other English speakers-other English speakers

  6.old-older

  7.seen-understood

  8.take it for granted- take for granted

  9.or-and

  10. the most striking of human achievements

  V. 汉译英及参考译文

  中国民族自古以来从不把人看作高于一切,在哲学文艺方面的表现都反映出人在自然界中与万物占着一个比例较为恰当的地位,而非绝对统治万物的主宰。因此我们的苦闷,基本上比西方人为少为小;因为苦闷的强弱原是随欲望与野心的大小而转移的。农业社会的人比工业社会的人享受差得多;因此欲望也小得多。况中国古代素来以不滞于物,不为物役为最主要的人生哲学。并非我们没有守财奴,但比起莫利哀与巴尔扎克笔下的守财奴与野心家来,就小巫见大巫了。中国民族多数是性情中正和平、淡泊、朴实、比西方人容易满足。

  Chinese people has never thought of human being as the highest creature among everything since ancient times, whose reflection takes a quite approporate proportion with all others in our natural world in both aspects of philosophy and arts, but not as an absolute dominant  ruler. Therefore, our bitterness and depression are basically less than those of westerners, because the intensity of which is growing with the expansion of one's desire and ambition. People in the agriculture society enjoyed far less than people in the industry society, thus their wants are far less either. Besides, ancient Chinese always regard "not confined by material, not driven by material" as the major philosophy. It not means we do not have misers, but in comparison with Mauriat and Balzac's miser and aspirant, that is dwarfed. Chinese people almost characterized by moderation, peacefulness, insecular, plainess, and easier to get satisfied than westerners.

  PART VI WRITING (45 MIN)

  Joseph epstein, a famous american writer,once said"we decide what is important and what is trivial in life we decide that what makes us significant is either what we do or what we refuse todo but no matter how indifferent the universe may be to our choices and decisions, these choices and decisions are ours to make. we decide. we choose.and as we decide and choose, so are our lives formed. in the end, forming our own destiny is what ambition is about

  do you agree or disagree with him? write an eassay of about 400 words entitled: On Ambition

  In the first part of your writing you should state your main argument, and in the second part you should support your argument with appropriate details. In the last part you should bring what you have written to a natural conclusion or make a summary. You should supply an appropriate title for your essay.

  Marks will be awarded for content, organization, grammar and appropriateness. Failure to follow the above instructions may result in a loss of marks.Write your composition on ANSWER SHEET FOUR.

2006年英语专业四级真题及答案

PART Ⅲ CLOZE

  There are many superstitions in Britain, but one of the most ( 31 ) held is that it is unlucky to walk under a ladder even if it means (32) the pavement into a busy street! (33) you must pass under a ladder you can (34) bad luck by crossing your fingers and (35) them crossed until you have seen a dog. (36) , you may lick your finger and (37) a cross on the toe of your shoe, and not look again at the shoe until the (38) has dried.

  Another common (39) is that it is unlucky to open an umbrella in the house-it will either bring (40) to the person who opened it or to the whole (41). Anyone opening an umbrella in fine weather is (42), as it inevitably brings rain!

  The number 13 is said to be unlucky for some, and when the 13th day of the month (43) on a Friday, anyone wishing to avoid a bad event had better stay (44). the worst misfortune that can happen to a person is caused by breaking a mirror, (45) it brings seven years of bad luck! The superstition is supposed to (46) in ancient times, when mirrors were considered to be tools of the gods.

  Black cats are generally considered lucky in Britain, even though they are (47) witchcraft…… it is (48) lucky if a black cat crosses your path-although in America the exact opposite belief prevails.

  Finally, a commonly held superstition is that of touching wood (49) luck. This measure is most often taken if you think you have said something that is tempting fate, such as "my car has never (50) , touch wood?"

  31. A broadly B widely C quickly D speedily

  32. A running from B jumping off C stepping off D keeping from

  33. A If B As C Though D Unless

  34. A erase B remove C avoid D ease

  35. A keep B keeping C kept D to keep

  36. A Consequently B However C Comparatively D Alternatively

  37. A make B print C perform D produce

  38. A label B symbol C mark D cut

  39. A argument B superstition C opinion D idea

  40. A loss B difficulty C tragedy D misfortune

  41. A house B household C home D circle

  42. A unwise B unintelligent C unpopular D unfortunate

  43. A falls B arrives C drops D happens

  44. A away B outdoors C indoors D far

  45. A when B as C if D though

  46. A have originated B be originating C be originated D originate

  47. A concerned about B related with C associated with D connected in

  48. A especially B specially C frequently D rarely

  49. A as B for C in D of

  50. A broken up B broken off C broken away D broken down

  PART Ⅳ GRAMMAR and VOCABULARY

  51. __dull he may be, he is certainly a very successful top executive.

  A Although B whatever C As D However

  52. If only I __play the guitar as well as you!

  A would B could C should D might

  53. The party, __I was the guest of honour, was extremely enjoyable.

  A by which B for which C to which D at which

  54 It's high time we __ cutting down the rainforests.

  A stopped B had to stop C shall stop D stop

  55 The student said there were a few points in the essay he __ impossible to comprehend.

  A has found B was finding C had found D would find

  56 Loudspeakers were fixed in the hall so that everyone__ an opportunity to hear the speech.

  A ought to have B must have C may have D should have

  57 I am surprised__ this city is a dull place to live in.

  A that you should think B by what you are thinking C that you would think D with what you were thinking

  58 Susan is very hardworking, but her pay is not__ for her work.

  A enough good B good enough C as good enough D good as enough

  59 It is imperative that the government __ more investment into the shipbuilding industry.

  A attracts B shall attract C attract D has to

  60 Land belongs to the city; there is __ thing as private ownership of land.

  A no such a B not such C not such a D no such

  61 My daughter has walked eight miles today. We never guessed that she could walk__far.

  A / B such C that D as

  62 The statistics __ that living standards in the area have improved drastically in recent times.

  A proves B is proving C are proving D prove

  63 There are only ten apples left in the baskets, __ the spoilt ones.

  A not counting B not to count C don't count D having not counted

  64 It was __ we had hoped

  A more a success than B a success more than C as much of a success as D a success as much as

  65 There used to be a petrol station near the park, __?

  A didn't it B doesn't there C usedn't it? D didn't there

  66 It is an offence to show __ against people of different races.

  A distinction B difference C separation D discrimination

  67 A great amount of work has gone into __ the Cathedral to its previous splendour.

  A refreshing B restoring C renovating D renewing

  68 The thieves fled with the local police close on their __.

  A backs B necks C toes D heels

  69 The economic recession has meant that job__ is a rare thing.

  A security B safety C protection D secureness

  70 Many people nowadays save money to __ for their old age.

  A cater B supply C provide D equip

  71 The tone of the article __ the writer's mood at the time.

  A reproduced B reflected C imagined D imitated

  72 This is not the right __ to ask for my help; I am far too busy even to listen

  73 The job of a student accommodation officer__ a great many visits to landladies.

  A concerns B offers C asks D involves

  74 Our family doctor's clinic __at the junction of two busy roads.

  A rests B stands C stays D seats

  75 She was so fat that she could only just __ through the door.

  A assemble B appear C squeeze D gather

  76 After the heavy rain, a builder was called to repair the roof, which was __.

  A leaking B trickling C prominent D noticeable

  77 The reception was attended by __ members of the local community.

  A excellent B conspicuous C prominent D noticeable

  78 Share prices on the Stock Exchange plunged sharply in the morning but __slightly in the afternoon.

  A regained B recovered C restored D revived

  79 His brain has worked away on the idea of a universal cure.

  A rich B quick C productive D fertile

  80 The couple has donated a not__ amount of money to the foundation.

  A inconsiderable B inconsiderate C inaccurate D incomparable

  PART Ⅴ READING COMPREHENSION

  TEXT A

  In the case of mobile phones, change is everything. Recent research indicates that the mobile phone is changing not only our culture, but our very bodies as well.

  First. Let's talk about culture. The difference between the mobile phone and its parent, the fixed-line phone, you get whoever answers it.

  This has several implications. The most common one, however, and perhaps the thing that has changed our culture forever, is the "meeting" influence. People no longer need to make firm plans about when and where to meet. Twenty years ago, a Friday night would need to be arranged in advance. You needed enough time to allow everyone to get from their place of work to the first meeting place. Now, however, a night out can be arranged on the run. It is no longer "see you there at 8", but "text me around 8 and we'll see where we all are".

  Texting changes people as well. In their paper, "insights into the Social and Psychological Effects of SMS Text Messaging", two British researchers distinguished between two types of mobile phone users: the "talkers" and the "texters"-those who prefer voice to text message and those who prefer text to voice.

  They found that the mobile phone's individuality and privacy gave texters the ability to express a whole new outer personality. Texters were likely to report that their family would be surprised if they were to read their texts. This suggests that texting allowed texters to present a self-image that differed from the one familiar to those who knew them well.

  Another scientist wrote of the changes that mobiles have brought to body language. There are two kinds that people use while speaking on the phone. There is the "speakeasy": the head is held high, in a self-confident way, chatting away. And there is the "spacemaker": these people focus on themselves and keep out other people.

  Who can blame them? Phone meetings get cancelled or reformed and camera-phones intrude on people's privacy. So, it is understandable if your mobile makes you nervous. But perhaps you needn't worry so much. After all, it is good to talk.

  81 when people plan to meet nowadays, they

  A: arrange the meeting place beforehand

  B. postpone fixing the place till last minute

  C: seldom care about when and where to meet

  D: still love to work out detailed meeting plans.

  82 According to the two British researchers, the social and psychological effect are mostly likely to be seen on

  A: TALKERS

  B; the "speakeasy"

  c. the "spacemaker"

  D. texters

  83 We can infer from the passage that the texts sent by texters are

  A: quite revealing

  B: well written

  c: unacceptable by others

  d; shocking to others

  84 according to the passage ,who is afraid of being heard while talking on the mobile

  a: talkers

  b: the speakeasy

  c :the spacemaker

  d: texters

  85 an appropriate title for the passage might be

  A: the SMS effect

  b: cultural implication of mobile use

  c: change in the use of the mobile

  d: body language and the mobile phone!

  TEXT B

  Over the last 25 years, British society has changed a great deal-or at least many parts of it have. In some ways, however, very little has changed, particularly where attitudes are concerned. Ideas about social class-whether a person is "working-class" or "middle-class"

  -are one area in which changes have been extremely slow.

  In the past, the working-class tended to be paid less than middle-class people, such as teachers and doctors. As a result of this and also of the fact that workers' jobs were generally much less secure, distinct differences in life-styles and attitudes came into existence. The typical working man would collect his wages on Friday evening and then, it was widely believed, having given his wife her "housekeeping", would go out and squander the rest on beer and betting.

  The stereotype of what a middle-class man did with his money was perhaps nearer the truth. He was-and still is - inclined to take a longer-term view. Not only did he regard buying a house of these provided him and his family with security. Only in very few cases did workers have the opportunity (or the education and training) to make such long-term plans.

  Nowadays, a great deal has changed. In a large number of cases factory workers earn as much, if not more, than their middle-class supervisors. Social security and laws to improve century, have made it less necessary than before to worry about "tomorrow". Working-class people seem slowly to be losing the feeling of inferiority they had in the past. In fact there has been a growing tendency in the past few years for the middle-classes to feel slightly ashamed of their position.

  The changes in both life-styles and attitudes are probably most easily seen amongst younger people. They generally tend to share very similar tastes in music and clothes, they spend their money in having a good time, and save for holidays or longer-term plans when necessary. There seems to be much less difference than in precious generations. Nevertheless, we still have a wide gap between the well-paid (whatever the type of job they may have) and the low-paid. As long as this gap exists, there will always be a possibility that new conflicts and jealousies will emerge, or rather that the old conflicts will re-appear, but between different groups.

  86, which of the following is seen as the cause of class differences in the past?

  A: life style and occupation

  B: Attitude and income

  C: income and job security

  D: job security and hobbies

  87 the writer seems to suggest that the description of —— is closer to truth?

  A: middle -class ways of spending money

  B: working-class ways of spending the weekend

  C: working-class drinking habits

  D: middle-class attitudes

  88 according to the passage, which of the following is not a typical feature of the middle -class?

  A: desiring for security

  B: Making long term plans

  C: having priorities in life

  D: saving money

  89 working -class people's sense of security increased as a resulf of all the follwoing factor except?

  A: better social security

  B: more job opportunities

  C: higher living standard

  D: better legal protection.

  90 Which of the following statement is incorrect?

  A: Changes are slowly taking place in all sectors of the British society.

  B: The gap between working -class and middle- class young people is narrowing

  C: different in income will remain but those in occupation will disappear

  D: middle-class people may sometimes feel inferior to working-class people!

  TEXT C

  For several days I saw little of Mr. Rochester. In the morning he seemed much occupied with business, and in the afternoon gentlemen from the neighourhood called and some times stayed to dine with him. When his foot was well enough, he rode out a great deal.

  During this time, all my knowledge of him was limited to occasional meetings about the house, when he would sometimes pass me coldly, and sometimes bow and smile. His changes of manner did not offend me, because I saw that I had nothing to do with the cause of them.

  One evening, several days later, I was invited to talk to Mr. Rochester after dinner. He was sitting in his armchair, and looked not quite so severe, and much less gloomy. There was a smile on his lips, and his eyes were bright, probably with wine. As I was looking at him, he suddenly turned, and asked me, "do you think I'm handsome, Miss Eyre?"

  The answer somehow slipped from my tongue before I realized it: 'No, sir."

  "ah, you really are unusual! You are a quiet, serious little person, but you can be almost rude."

  "Sir, I'm sorry. I should have said that beauty doesn't matter, or something like that,"

  "no, you shouldn't! I see, you criticize my appearance, and then you stab me in the back! You have honesty and feeling. There are not many girls like you. But perhaps I go too fast. Perhaps you have awaful faults to counterbalance your few good points

  I thought to myself that he might have too. He seemed to read my mind, and said quickly," yes, you're right. I have plenty of faults. I went the wrong way when I was twenty-one, and have never found the right path again. I might have been very different. I might have been as good as you, and perhaps wiser. I am not a bad man, take my word for it, but I have done wrong. It wasn't my character, but circumstances which were to blame. Why do I tell you all this? Because you're the sort of person people tell their problems and secrets to, because you're sympathetic and give them hope."

  It seemed he had quite a lot to talk to me. He didn't seem to like to finish the talk quickly, as was the case for the first time.

  "Don't be afraid of me, Miss Eyre." He continued. " you don't relax or laugh very much, perhaps because of the effect Lowood school has had on you. But in time you will be more natural with me, and laugh, and speak freely. You're like a bird in a cage. When you get out of the cage, you'll fly very high. Good night."

  91:at the beginning miss Eyre 's impressions of Mr. Rochester were all except

  A: busy

  B: sociable

  C: friendly

  D: changeable

  92, in "……and all my knowledge him was limited to occasional meetings about the house,…".the word about means

  A: around

  B: on

  C: outside

  D: concerning.

  93. why did Mr. Rochester say" ……and the you stab me in the back!" the (7thpara.

  A: because Jane had intended to kill him with a knife

  B: because Jane had intended to be more critical.

  C: because Jane had regretted having talked to him

  D: because Jane had said something else to correct herself.

  94, from what Mr. Rochest told Miss Eyre, we can conclude that he wanted to

  A: Tell her all his troubles

  B: tell her his life experience.

  C: change her opinion of him

  D change his circumstances

  95, at the end of the passage , Mr. Rochester sounded

  A: rude

  B: cold

  C: friendly

  D: encouraging.

  TEXTD

  The ideal companion machine-the computer- would not only look, feel, and sound friendly but would also be programmed to behave in a pleasant manner. Those qualities that make interaction comfortable, and yet the machine would remain slightly unpredictable and therefore interesting. In its first encounter it might be somewhat hesitant, but as it came to know the user it would progress to a more relaxed and intimate style. The machine would not be a passive participant but would add its own suggestions, information, and opinions; it would sometimes take the initiative in developing or changing the topic and would have a personality of its own.

  Friendships are not made in a day, and the computer would be more acceptable as a friend if it imitated the gradual changes that occur when one person is getting to know another. At an appropriate time it might also express the kind of affection that stimulates attachment and intimacy. The whole process would be accomplished in a subtle way to avoid giving an impression of over-familiarity that would be likely to produce irritation. After experiencing a wealth of powerful, well-timed friendship indicators, the user would be very likely to accept the computer as far more than a machine and might well come to regard it as a friend.

  An artificial relationship of this type would provide many of the benefits that could continue from previous discussions. It would have a familiarity with the user's life as revealed in earlier contact, and it would be understanding and good-humored. The computer's own personality would be lively and impressive, and it would develop in response to that of the user. With features such as these, the machine might indeed become a very attractive social partner.

  96. Which of the following is not a feature of the ideal companion machine?

  A: Active in communication

  B: Attractive in personality.

  C: enjoyable in performance

  D: unpredictable in behaviour

  97. The computer would develop friendships with humans in a (n) ——way.

  A: Quick

  B: unpredictable

  C: productive

  D: inconspicuous.

  98. Which of the following aspects is not mentioned when the passage discusses the benefits of artificial relationships?

  A: Being able to pick up an interesting conversation.

  B: Being sensitive to earlier contact.

  C: Being ready to learn about the person's life

  D: Having a pleasant and adaptable personality.

  99 Throughout the passage, the author is _____in his attitude toward the computer

  A: favorable

  B: critical

  C: vague

  D: hesitant

  100. Which might be the most appropriate title of the passage?

  A: Articial relationships .

  B: How to form intimate relationships

  C: The affectionate machine

  D: Humans and computers

  PART Ⅵ WRITING

  Section A Composition

  Recently a Beijing information company did a survey of student life among more than 700 students in Beijing, Guangzhou, Xi'an, Chengdu, Shanghai, Wuhan, Nanjing, and Shenyang . The results have shown that 67 percent of students think that saving money is a good habit while the rest believe that using tomorrow's money today is better, what do you think?

  Write on answer sheet two a compositions of about 200 words.

  You are to write in three parts.

  In the first part, state specifically what you opinion is

  In the second part, support your opinion with appropriate detail.

  In the last part, bring what you have written to a natural conclusion or a summary.

  You should supply an appropriate title for you composition.

  Marks will be awarded for content, organization, grammar and appropriateness. Failure to follow the instructions may result in a loss of marks.

  Section B Note- writing

  Write an answer sheet two a note of about 50-60 words based on the following situation:

  You have got to know that you classmates, Michael ,is organizing a weekend excursion for the class. And you are thinking of joining the trip, write him a note expressing your interest in the excursion and asking for information on two details related to the excursion.

  Marks will be awarded for content, organization, grammar and appropriateness.

2006年英语专业四级考试参考答案

  听力原文参考

  The internet

  The internet is the most significant progress in the field of communications. Imagine a book that never rend, a library with million floors, or imagine a research project with thousands of scientists working around the clock forever. This is the magic of the internet.

  As the internet has potential for good or bad, one can find where organized information of websites. At the same time, one can also find waste for websites. Most websites are known as different internet of applications. These include online games, check rooms and so on. These applications have great power, too. Sometimes, the power can be so great. That young people may easily become victims to their attraction. So we need to recognize the seriousness of the problem. We must work together to use its power for better rents.

  [环球时代提供的答案]

  PART Ⅲ CLOZE

  31. B widely 32. C stepping off 33. A If 34. C avoid 35. B keeping

  36. D Alternatively 37. A make 38. C mark 39. B superstition

  40. D misfortune 41. B household 42. D unfortunate 43. A falls

  44. C indoors 45. B as 46. A have originated 47. C associated with

  48. A especially 49.B for 50. D broken down

  PART Ⅳ GRAMMAR and VOCABULARY

  51. _D_dull he may be, he is certainly a very successful top executive.

  A Although B whatever C As D However

  52. If only I _B_play the guitar as well as you!

  A would B could C should D might

  53. The party, _D_I was the guest of honour, was extremely enjoyable.

  A by which B for which C to which D at which

  54 It's high time we _A_ cutting down the rainforests.

  A stopped B had to stop C shall stop D stop

  55 The student said there were a few points in the essay he _C_ impossible to comprehend.

  A has found B was finding C had found D would find

  56 Loudspeakers were fixed in the hall so that everyone_C_ an opportunity to hear the speech.

  A ought to have B must have C may have D should have

  57 I am surprised_A_ this city is a dull place to live in.

  A that you should think B by what you are thinking

  C that you would think D with what you were thinking

  58 Susan is very hardworking, but her pay is not_B_ for her work.

  A enough good B good enough C as good enough D good as enough

  59 It is imperative that the government _C_ more investment into the shipbuilding industry.

  A attracts B shall attract C attract D has to

  60 Land belongs to the city; there is _A_ thing as private ownership of land.

  A no such a B not such C not such a D no such

  61 My daughter has walked eight miles today. We never guessed that she could walk_C_far.

  A / B such C that D as

  62 The statistics _D_ that living standards in the area have improved drastically in recent times.

  A proves B is proving C are proving D prove

  63 There are only ten apples left in the baskets, _A_ the spoilt ones.

  A not counting B not to count C don't count D having not counted

  64 It was _A_ we had hoped

  A more a success than B a success more than

  C as much of a success as D a success as much as

  65 There used to be a petrol station near the park, _D_?

  A didn't it B doesn't there C usedn't it? D didn't there

  66 It is an offence to show _D_ against people of different races.

  A distinction B difference C separation D discrimination

  67 A great amount of work has gone into _B_ the Cathedral to its previous splendour.

  A refreshing B restoring C renovating D renewing

  68 The thieves fled with the local police close on their _D_.

  A backs B necks C toes D heels

  69 The economic recession has meant that job_A_ is a rare thing.

  A security B safety C protection D secureness

  70 Many people nowadays save money to _A_ for their old age.

  A cater B supply C provide D equip

  71 The tone of the article _B_ the writer's mood at the time.

  A reproduced B reflected C imagined D imitated

  72 This is not the right _A_ to ask for my help; I am far too busy even to listen

  A moment B situation C imagined D imitated

  73 The job of a student accommodation officer_D_ a great many visits to landladies.

  A concerns B offers C asks D involves

  74 Our family doctor's clinic _B_at the junction of two busy roads.

  A rests B stands C stays D seats

  75 She was so fat that she could only just _C_ through the door.

  A assemble B appear C squeeze D gather

  76 After the heavy rain, a builder was called to repair the roof, which was _A_.

  A leaking B trickling C prominent D noticeable

  77 The reception was attended by _C_ members of the local community.

  A excellent B conspicuous C prominent D noticeable

  78 Share prices on the Stock Exchange plunged sharply in the morning but _D_slightly in the afternoon.

  A regained B recovered C restored D revived

  79 His C brain has worked away on the idea of a universal cure.

  A rich B quick C productive D fertile

  80 The couple has donated a not_A_ amount of money to the foundation.

  A inconsiderable B inconsiderate C inaccurate D incomparable

  PART Ⅴ READING COMPREHENSION

  81 when people plan to meet nowadays, they (B)

  A: arrange the meeting place beforehand

  B. postpone fixing the place till last minute

  C: seldom care about when and where to meet

  D: still love to work out detailed meeting plans.

  82 According to the two British researchers, the social and psychological effect are mostly likely to be seen on (D)

  A: TALKERS

  B; the "speakeasy"

  C. the "spacemaker"

  D. texters

  83 We can infer from the passage that the texts sent by texters are (A)

  A: quite revealing

  B: well written

  C: unacceptable by others

  D; shocking to others

  84 according to the passage , who is afraid of being heard while talking on the mobile (C)

  A: talkers

  B: the speakeasy

  C :the spacemaker

  D: texters

  85 an appropriate title for the passage might be (B)

  A: the SMS effect

  B: cultural implication of mobile use

  C: change in the use of the mobile

  D: body language and the mobile phone!

  86, which of the following is seen as the cause of class differences in the past?(B)

  A: life style and occupation

  B: Attitude and income

  C: income and job security

  D: job security and hobbies

  87 the writer seems to suggest that the description of —— is closer to truth?(A)

  A: middle -class ways of spending money

  B: working-class ways of spending the weekend

  C: working-class drinking habits

  D: middle-class attitudes

  88 according to the passage, which of the following is not a typical feature of the middle -class?(D)

  A: desiring for security

  B: Making long term plans

  C: having priorities in life

  D: saving money

  89 working -class people's sense of security increased as a resulf of all the follwoing factor except?(D)

  A: better social security

  B: more job opportunities

  C: higher living standard

  D: better legal protection.

  90.which of the following statement is incorrect?(A)

  A: Changes are slowly taking place in all sectors of the British society.

  B: The gap between working -class and middle- class young people is narrowing

  C: different in income will remain but those in occupation will disappear

  D: middle-class people may sometimes feel inferior to working-class people!

  91:at the beginning miss Eyre 's impressions of Mr. Rochester were all except (D)

  A: busy

  B: sociable

  C: friendly

  D: changeable

  92, in "……and all my knowledge him was limited to occasional meetings about the house,…".the word about means(A)

  A: around

  B: on

  C: outside

  D: concerning.

  93. why did Mr. Rochester say" ……and the you stab me in the back!" (the 7th paragraph.)(B)

  A: because Jane had intended to kill him with a knife

  B: because Jane had intended to be more critical.

  C: because Jane had regretted having talked to him

  D: because Jane had said something else to correct herself.

  94, from what Mr. Rochest told miss Eyre, we can conclude that he wanted to (B)

  A: Tell her all his troubles

  B: tell her his life experience.

  C: change her opinion of him

  D change his circumstances

  95, at the end of the passage , Mr. Rochester sounded(D)

  A: rude

  B: cold

  C: friendly

  D: encouraging.

  96.which of the following is not a feature of the ideal companion machine? (D)

  A: Active in communication

  B: Attractive in personality.

  C: enjoyable in performance

  D: unpredictable in behaviour

  97. the computer would develop friendships with humans in a (n) ——way. (D)

  A: Quick

  B: unpredictable

  C: productive

  D: inconspicuous.

  98.which of the following aspects is not mentioned when the passage discusses the benefits of artificial relationships? (B)

  A: Being able to pick up an interesting conversation.

  B: Being sensitive to earlier contact.

  C: Being ready to learn about the person's life

  D: Having a pleasant and adaptable personality.

  99 throughout the passage, the author is _____in his attitude toward the computer (A)

  A: favourable

  B: critical

  C: vague

  D: hesitant

  100. which mgiht be the most appropriate title of the passage?(C)

  A: Articial relationshios .

  B: How to form intimate relationships

  C: The affectionate machine

  D: Humans and computers

相关热词搜索: